Vous êtes sur la page 1sur 207

AFP CHIEF OF STAFF LT. GEN. RICARDO R.

VISAYA, AND PHILIPPINE VETERANS AFFAIRS


OFFICE (PVAO) ADMINISTRATOR LT. GEN. ERNESTO G. CAROLINA (RET.), Respondents.

G.R. No. 226116

HEHERSON T. ALVAREZ, JOEL C. LAMANGAN, FRANCIS X. MANGLAPUS, EDILBERTO C. DE


EN BANC JESUS, BELINDA O. CUNANAN, CECILIA GUIDOTE ALVAREZ, REX DEGRACIA LORES, SR.,
ARNOLD MARIE NOEL, CARLOS MANUEL, EDMUND S. TAYAO, DANILO P. OLIVARES, NOEL F.
G.R. No. 225973, November 08, 2016 TRINIDAD, JESUS DELA FUENTE, REBECCA M. QUIJANO, FR. BENIGNO BELTRAN, SVD,
ROBERTO S. VERZOLA, AUGUSTO A. LEGASTO, JR., AND JULIA KRISTINA P.
SATURNINO C. OCAMPO, TRINIDAD H. REPUNO, BIENVENIDO LUMBERA, BONIFACIO P. LEGASTO, Petitioners, v. EXECUTIVE SECRETARY SALVADOR C. MEDIALDEA, DEFENSE
ILAGAN, NERI JAVIER COLMENARES, MARIA CAROLINA P. ARAULLO, M.D., SAMAHAN NG EX- SECRETARY DELFIN LORENZANA, AFP CHIEF OF STAFF LT. GEN. RICARDO R. VISAYA, AFP
DETAINEES LABAN SA DETENSYON AT ARESTO (SELDA), REPRESENTED BY DIONITO DEPUTY CHIEF OF STAFF REAR ADMIRAL ERNESTO C. ENRIQUEZ, AND PHILIPPINE
CABILLAS, CARMENCITA M. FLORENTINO, RODOLFO DEL ROSARIO, FELIX C. DALISAY, AND VETERANS AFFAIRS OFFICE (PVAO) OF THE DND, Respondents.
*
DANILO M. DELAFUENTE, Petitioners, v. REAR ADMIRAL ERNESTO C. ENRIQUEZ (IN HIS
CAPACITY AS THE DEPUTY CHIEF OF STAFF FOR RESERVIST AND RETIREE AFFAIRS, G.R. No. 226117
ARMED FORCES OF THE PHILIPPINES), THE GRAVE SERVICES UNIT (PHILIPPINE ARMY), AND
GENERAL RICARDO R. VISAYA (IN HIS CAPACITY AS THE CHIEF OF STAFF, ARMED FORCES ZAIRA PATRICIA B. BANIAGA, JOHN ARVIN BUENAAGUA, JOANNE ROSE SACE LIM, JUAN
OF THE PHILIPPINES), DEFENSE SECRETARY DELFIN LORENZANA, AND HEIRS OF ANTONIO RAROGAL MAGALANG, Petitioners, v. SECRETARY OF NATIONAL DEFENSE DELFIN
FERDINAND E. MARCOS, REPRESENTED BY HIS SURVIVING SPOUSE IMELDA ROMUALDEZ N. LORENZANA, AFP CHIEF OF STAFF RICARDO R. VISAYA, ADMINISTRATOR OF THE
MARCOS, Respondents. PHILIPPINE VETERANS AFFAIRS OFFICE ERNESTO G. CAROLINA, Respondents.

RENE A.V. SAGUISAG, SR., RENE A.Q. SAGUISAG, JR., RENE A.C. SAGUISAG III, Intervenors. G.R. No. 226120

G.R. No. 225984 ALGAMAR A. LATIPH, Petitioner, v. SECRETARY DELFIN N. LORENZANA, SUED IN HIS
CAPACITY AS SECRETARY OF NATIONAL DEFENSE, LT. GEN. RICARDO R. VISAYA, IN HIS
REP. EDCEL C. LAGMAN, IN HIS PERSONAL AND OFFICIAL CAPACITIES AND AS A MEMBER CAPACITY AS CHIEF OF STAFF OF THE ARMED FORCES OF THE PHILIPPINES AND LT. GEN.
OF CONGRESS AND AS THE HONORARY CHAIRPERSON OF THE FAMILIES OF VICTIMS OF ERNESTO G. CAROLINA (RET.), IN HIS CAPACITY AS ADMINISTRATOR, PHILIPPINE
INVOLUNTARY DISAPPEARANCE (FIND); FAMILIES OF VICTIMS OF INVOLUNTARY VETERANS AFFAIRS OFFICE (PVAO), Respondents.
DISAPPEARANCE (FIND), REPRESENTED BY ITS COCHAIRPERSON, NILDA L. SEVILLA; REP.
TEDDY BRAWNER BAGUILAT, JR.; REP. TOMASITO S. VILLARIN; REP. EDGAR R. ERICE; AND G.R. No. 226294
REP. EMMANUEL A. BILLONES, Petitioners, v. EXECUTIVE SECRETARY SALVADOR C.
MEDIALDEA; DEFENSE SECRETARY DELFIN N. LORENZANA; AFP CHIEF OF STAFF LT. GEN. LEILA M. DE LIMA, IN HER CAPACITY AS SENATOR OF THE REPUBLIC AND AS
RICARDO R. VISAYA; AFP DEPUTY CHIEF OF STAFF REAR ADMIRAL ERNESTO C. ENRIQUEZ; TAXPAYER, Petitioner, v. HON. SALVADOR C. MEDIALDEA, DEFENSE SECRETARY DELFIN
AND HEIRS OF FERDINAND E. MARCOS, REPRESENTED BY HIS SURVIVING SPOUSE IMELDA LORENZANA, AFP CHIEF OF STAFF LT. GEN. RICARDO R. VISAYA, UNDERSECRETARY
ROMUALDEZ MARCOS, Respondents. ERNESTO G. CAROLINA, IN HIS CAPACITY AS PHILIPPINE VETERANS AFFAIRS OFFICE
(PVAO) ADMINISTRATOR AND B/GEN. RESTITUTO L. AGUILAR, IN HIS CAPACITY AS SHRINE
G.R. No. 226097 CURATOR AND CHIEF VETERANS MEMORIAL AND HISTORICAL DIVISION AND HEIRS OF
FERDINAND EDRALIN MARCOS, Respondent.
LORETTA ANN PARGAS-ROSALES, HILDA B. NARCISO, AIDA F. SANTOSMARANAN, JO-ANN
Q. MAGLIPON, ZENAIDA S. MIQUE, FE B. MANGAHAS, MA. CRISTINA P. BAWAGAN, MILA D. DECISION
AGUILAR, MINERVA G. GONZALES, MA. CRISTINA V. RODRIGUEZ, LOUIE G. CRISMO,
FRANCISCO E. RODRIGO, JR., LIWAYWAY D. ARCE, AND ABDULMARI DE LEON IMAO, PERALTA, J.:
JR., Petitioners, v. EXECUTIVE SECRETARY SALVADOR C. MEDIALDEA, DEFENSE SECRETARY
DELFIN LORENZANA, AFP DEPUTY CHIEF OF STAFF REAR ADMIRAL ERNESTO C. ENRIQUEZ,
In law, as much as in life, there is need to find closure. Issues that have lingered and festered for Headquarters, Philippine Army
so long and which unnecessarily divide the people and slow the path to the future have to be Fort Bonifacio, Taguig City
interred. To move on is not to forget the past. It is to focus on the present and the future, leaving Attn: Assistant Chief of Staff for RRA, G9
behind what is better left for history to ultimately decide. The Court finds guidance from the
Constitution and the applicable laws, and in the absence of clear prohibition against the Pursuant to paragraph 2b, SOP Number 8, GHQ, AFP dated 14 July 1992, provide services, honors
exercise of discretion entrusted to the political branches of the Government, the Court must not and other courtesies for the late Former President Ferdinand E. Marcos as indicated:
overextend its readings of what may only be seen as providing tenuous connection to the issue
before it. chanRoblesvirtualLawlibrary [x] Vigil - Provide vigil
[x] Bugler/Drummer
Facts [x] Firing Party
[x] Military Host/Pallbearers
[x] Escort and Transportation
During the campaign period for the 2016 Presidential Election, then candidate Rodrigo R. Duterte [x] Arrival/Departure Honors
(Duterte) publicly announced that he would allow the burial of former President Ferdinand E. Marcos
(Marcos) at the Libingan Ng Mga Bayani (LNMB). He won the May 9, 2016 election, garnering His remains lie in state at Ilocos Norte
16,601,997 votes. At noon of June 30, 2016, he formally assumed his office at the Rizal Hall in the
Malacañan Palace. Interment will take place at the Libingan ng mga Bayani, Ft. Bonifacio, Taguig City. Date: TBAL.

On August 7, 2016, public respondent Secretary of National Defense Delfin N. Lorenzana issued a Provide all necessary military honors accorded for a President
Memorandum to the public respondent Chief of Staff of the Armed Forces of the Philippines (AFP), 2
POC: Administrator, PVAO BY COMMAND OF GENERAL VISAYA
General Ricardo R. Visaya, regarding the interment of Marcos at the LNMB, to
wit:ChanRoblesVirtualawlibrary Dissatisfied with the foregoing issuance, the following were filed by petitioners:
Subject: Interment of the late Former President Ferdinand Marcos at LNMB 3
chanRoblesvirtualLawlibrary1. Petition for Certiorari and Prohibition filed by Saturnino Ocampo and
4
several others, in their capacities as human rights advocates or human rights violations victims as
Reference: Verbal Order of President Rodrigo Duterte on July 11, 2016.
defined under Section 3 (c) of Republic Act (R.A.) No. 10368 (Human Rights Victims Reparation and
Recognition Act of 2013).
In compliance to (sic) the verbal order of the President to implement his election campaign promise to
have the remains of the late former President Ferdinand E. Marcos be interred at the Libingan ng mga 5 6
2. Petition for Certiorari-in-Intervention filed by Rene A.V. Saguisag, Sr. and his son, as members of
Bayani, kindly undertake all the necessary planning and preparations to facilitate the coordination of all 7
the Bar and human rights lawyers, and his grandchild. chanrobleslaw
agencies concerned specially the provisions for ceremonial and security requirements. Coordinate
closely with the Marcos family regarding the date of interment and the transport of the late former 8
3. Petition for Prohibition filed by Representative Edcel C. Lagman, in his personal capacity, as
President's remains from Ilocos Norte to the LNMB.
member of the House of Representatives and as Honorary Chairperson of Families of Victims of
Involuntary Disappearance (FIND), a duly-registered corporation and organization of victims and
The overall OPR for this activity will [be] the PVAO since the LNMB is under its supervision and
families of enforced disappearance, mostly during the martial law regime of the former President
administration. PVAO shall designate the focal person for this activity who shall be the overall overseer 9
Marcos, and several others, in their official capacities as duly-elected Congressmen of the House of
of the event.
Representatives of the Philippines.
1
Submit your Implementing Plan to my office as soon as possible. chanroblesvirtuallawlibrary 10
4. Petition for Prohibition filed by Loretta Ann Pargas-Rosales, former Chairperson of the Commission
11
On August 9, 2016, respondent AFP Rear Admiral Ernesto C. Enriquez issued the following directives on Human Rights, and several others, suing as victims of State-sanctioned human rights violations
to the Philippine Army (PA) Commanding General:ChanRoblesVirtualawlibrary during the martial law regime of Marcos.

12
SUBJECT: Funeral Honors and Service 5. Petition for Mandamus and Prohibition filed by Heherson T. Alvarez, former Senator of the Republic
13
of the Philippines, who fought to oust the dictatorship of Marcos, and several others, as concerned
TO: Commanding General, Philippine Army Filipino citizens and taxpayers.
14 15
6. Petition for Certiorari and Prohibition filed by Zaira Patricia B. Baniaga and several others, as (e) The International Covenant on Civil and Political Rights;
concerned Filipino citizens and taxpayers.
(f) The "Basic Principles and Guidelines on the Right to a Remedy and Reparation for Victims of Gross
16
7. Petition for Certiorari and Prohibition filed by Algamar A. Latiph, former Chairperson of the Regional Violations of International Human Rights Law and Serious Violations of International Humanitarian Law"
Human Rights Commission, Autonomous Region in Muslim Mindanao, by himself and on behalf of of the United Nations (U.N.) General Assembly; and cralawlawlibrary
17
the Moro who are victims of human rights during the martial law regime of Marcos.
(g) The "Updated Set of Principles for Protection and Promotion of Human Rights through Action to
18
8. Petition for Certiorari and Prohibition filed by Leila M. De Lima as member of the Senate of the Combat Impunity" of the U.N. Economic and Social Council;
Republic of the Philippines, public official and concerned citizen.
3. Whether historical facts, laws enacted to recover ill-gotten wealth from the Marcoses and their
Issues cronies, and the pronouncements of the Court on the Marcos regime have nullified his entitlement as a
soldier and former President to interment at the LNMB.

4. Whether the Marcos family is deemed to have waived the burial of the remains of former President
Procedural
Marcos at the LNMB after they entered into an agreement with the Government of the Republic of the
Philippines as to the conditions and procedures by which his remains shall be brought back to and
1. Whether President Duterte's determination to have the remains of Marcos interred at the LNMB interred in the Philippines.
poses a justiciable controversy.
Opinion

2. Whether petitioners have locus standi to file the instant petitions.


The petitions must be dismissed.
3. Whether petitioners violated the doctrines of exhaustion of administrative remedies and hierarchy of
courts. Procedural Grounds

Substantive
Justiciable controversy

1. Whether the respondents Secretary of National Defense and AFP Rear Admiral committed grave It is well settled that no question involving the constitutionality or validity of a law or governmental act
abuse of discretion, amounting to lack or excess of jurisdiction, when they issued the assailed may be heard and decided by the Court unless the following requisites for judicial inquiry are present:
memorandum and directive in compliance with the verbal order of President Duterte to implement his (a) there must be an actual case or controversy calling for the exercise of judicial power; (b) the person
election campaign promise to have the remains of Marcos interred at the LNMB. challenging the act must have the standing to question the validity of the subject act or issuance; (c) the
question of constitutionality must be raised at the earliest opportunity; and (d) the issue of
2. Whether the Issuance and implementation of the assailed memorandum and directive violate the 19
constitutionality must be the very lis mota of the case. In this case, the absence of the first two
Constitution, domestic and international laws, particularly: requisites, which are the most essential, renders the discussion of the last two
20
superfluous. chanrobleslaw
chanRoblesvirtualLawlibrary(a) Sections 2, 11, 13, 23, 26, 27 and 28 of Article II, Section 1 of Article III,
Section 17 of Article VII, Section 1 of Article XI, Section 3(2) of Article XIV, and Section 26 of Article An "actual case or controversy" is one which involves a conflict of legal rights, an assertion of opposite
XVIII of the 1987 Constitution; legal claims, susceptible of judicial resolution as distinguished from a hypothetical or abstract difference
21
or dispute. There must be a contrariety of legal rights that can be interpreted and enforced on the
(b) R.A. No. 289; 22
basis of existing law and jurisprudence. Related to the requisite of an actual case or controversy is the
requisite of "ripeness," which means that something had then been accomplished or performed by
(c) R.A. No. 10368; either branch before a court may come into the picture, and the petitioner must allege the existence of
23
an immediate or threatened injury to itself as a result of the challenged action. Moreover, the limitation
(d) AFP Regulation G 161-375 dated September 11, 2000; on the power of judicial review to actual cases and controversies carries the assurance that the courts
24
will not intrude into areas committed to the other branches of government. Those areas pertain to
questions which, under the Constitution, are to be decided by the people in their sovereign capacity, or As concerned citizens, petitioners are also required to substantiate that the issues raised are of
34
in regard to which full discretionary authority has been delegated to the legislative or executive branch transcendental importance, of overreaching significance to society, or of paramount public interest. In
25
of the government. cralawred As they are concerned with questions of policy and issues dependent cases involving such issues, the imminence and clarity of the threat to fundamental constitutional rights
26 35 36
upon the wisdom, not legality of a particular measure, political questions used to be beyond the ambit outweigh the necessity for prudence. In Marcos v. Manglapus, the majority opinion observed that the
of judicial review. However, the scope of the political question doctrine has been limited by Section 1 of subject controversy was of grave national importance, and that the Court's decision would have a
Article VIII of the 1987 Constitution when it vested in the judiciary the power to determine whether or not profound effect on the political, economic, and other aspects of national life. The ponencia explained
there has been grave abuse of discretion amounting to lack or excess of jurisdiction on the part of any that the case was in a class by itself, unique and could not create precedent because it involved a
branch or instrumentality of the Government. dictator forced out of office and into exile after causing twenty years of political, economic and social
havoc in the country and who, within the short space of three years (from 1986), sought to return to the
The Court agrees with the OSG that President Duterte's decision to have the remains of Marcos Philippines to die.
interred at the LNMB involves a political question that is not a justiciable controversy. In the exercise of
his powers under the Constitution and the Executive Order (E.O.) No. 292 (otherwise known as the At this point in time, the interment of Marcos at a cemetery originally established as a national military
Administrative Code of 1987) to allow the interment of Marcos at the LNMB, which is a land of the public cemetery and declared a national shrine would have no profound effect on the political, economic, and
domain devoted for national military cemetery and military shrine purposes, President Duterte decided a other aspects of our national life considering that more than twenty-seven (27) years since his death
question of policy based on his wisdom that it shall promote national healing and forgiveness. There and thirty (30) years after his ouster have already passed. Significantly, petitioners failed to demonstrate
being no taint of grave abuse in the exercise of such discretion, as discussed below, President Duterte's a clear and imminent threat to their fundamental constitutional rights.
decision on that political question is outside the ambit of judicial review.
As human rights violations victims during the Martial Law regime, some of petitioners decry re-
Locus standi traumatization, historical revisionism, and disregard of their state recognition as heroes. Petitioners'
argument is founded on the wrong premise that the LNMB is the National Pantheon intended by law to
27
Defined as a right of appearance in a court of justice on a given question, locus standi requires that a perpetuate the memory of all Presidents, national heroes and patriots. The history of the LNMB, as will
party alleges such personal stake in the outcome of the controversy as to assure that concrete be discussed further, reveals its nature and purpose as a national military cemetery and national shrine,
adverseness which sharpens the presentation of issues upon which the court depends for illumination under the administration of the AFP.
28
of difficult constitutional questions. Unless a person has sustained or is in imminent danger of
sustaining an injury as a result of an act complained of, such proper party has no Apart from being concerned citizens and taxpayers, petitioners Senator De Lima, and Congressman
29 37
standing. Petitioners, who filed their respective petitions for certiorari, prohibition and mandamus, in Lagman, et al. come before the Court as legislators suing to defend the Constitution and to protect
their capacities as citizens, human rights violations victims, legislators, members of the Bar and appropriated public funds from being used unlawfully. In the absence of a clear showing of any direct
taxpayers, have no legal standing to file such petitions because they failed to show that they have injury to their person or the institution to which they belong, their standing as members of the Congress
38
suffered or will suffer direct and personal injury as a result of the interment of Marcos at the LNMB. cannot be upheld. They do not specifically claim that the official actions complained of, i.e., the
memorandum of the Secretary of National Defense and the directive of the AFP Chief of Staff regarding
39
Taxpayers have been allowed to sue where there is a claim that public funds are illegally disbursed or the interment of Marcos at the LNMB, encroach on their prerogatives as legislators. chanrobleslaw
that public money is being deflected to any improper purpose, or that public funds are wasted through
30
the enforcement of an invalid or unconstitutional law. In this case, what is essentially being assailed is Exhaustion of Administrative Remedies
the wisdom behind the decision of the President to proceed with the interment of Marcos at the LNMB.
As taxpayers, petitioners merely claim illegal disbursement of public funds, without showing that Marcos Petitioners violated the doctrines of exhaustion of administrative remedies and hierarchy of courts.
is disqualified to be interred at the LNMB by either express or implied provision of the Constitution, the Under the doctrine of exhaustion of administrative remedies, before a party is allowed to seek the
laws or jurisprudence. intervention of the court, one should have availed first of all the means of administrative processes
40
available. If resort to a remedy within the administrative machinery can still be made by giving the
31
Petitioners Saguisag, et al., as members of the Bar, are required to allege any direct or potential injury administrative officer concerned every opportunity to decide on a matter that comes within his
which the Integrated Bar of the Philippines, as an institution, or its members may suffer as a jurisdiction, then such remedy should be exhausted first before the court's judicial power can be
32 41
consequence of the act complained of. Suffice it to state that the averments in their petition-in- sought. For reasons of comity and convenience, courts of justice shy away from a dispute until the
intervention failed to disclose such injury, and that their interest in this case is too general and shared system of administrative redress has been completed and complied with, so as to give the
42
by other groups, such that their duty to uphold the rule of law, without more, is inadequate to clothe administrative agency concerned every opportunity to correct its error and dispose of the case. While
33 43
them with requisite legal standing. chanrobleslaw there are exceptions to the doctrine of exhaustion of administrative remedies, petitioners failed to
prove the presence of any of those exceptions.
As the Office of the Solicitor General (OSG) logically reasoned out, while the Constitution is a product of
Contrary to their claim of lack of plain, speedy, adequate remedy in the ordinary course of law, our collective history as a people, its entirety should not be interpreted as providing guiding principles to
petitioners should be faulted for failing to seek reconsideration of the assailed memorandum and just about anything remotely related to the Martial Law period such as the proposed Marcos burial at the
directive before the Secretary of National Defense. The Secretary of National Defense should be given LNMB.
opportunity to correct himself, if warranted, considering that AFP Regulations G 161-375 was issued
58
upon his order. Questions on the implementation and interpretation thereof demand the exercise of Tañada v. Angara already ruled that the provisions in Article II of the Constitution are not self-
sound administrative discretion, requiring the special knowledge, experience and services of his office executing. Thus:ChanRoblesVirtualawlibrary
to determine technical and intricate matters of fact. If petitioners would still be dissatisfied with the
decision of the Secretary, they could elevate the matter before the Office of the President which has By its very title, Article II of the Constitution is a "declaration of principles and state policies." The
44 counterpart of this article in the 1935 Constitution is called the "basic political creed of the nation" by
control and supervision over the Department of National Defense (DND). chanrobleslaw
Dean Vicente Sinco. These principles in Article II are not intended to be self executing principles ready
Hierarchy of Courts for enforcement through the courts. They are used by the judiciary as aids or as guides in the exercise
of its power of judicial review, and by the legislature in its enactment of laws. As held in the leading case
In the same vein, while direct resort to the Court through petitions for the extraordinary writs of Kilosbayan, Incorporated vs. Morato, the principles and state policies enumerated in Article II x x x
45 are not "self-executing provisions, the disregard of which can give rise to a cause of action in the courts.
of certiorari, prohibition and mandamus are allowed under exceptional cases, which are lacking in this
case, petitioners cannot simply brush aside the doctrine of hierarchy of courts that requires such They do not embody judicially enforceable constitutional rights but guidelines for legislation."
petitions to be filed first with the proper Regional Trial Court (RTC). The RTC is not just a trier of facts,
but can also resolve questions of law in the exercise of its original and concurrent jurisdiction over In the same light, we held in Basco vs. Pagcor that broad constitutional principles need legislative
petitions for certiorari, prohibition and mandamus, and has the power to issue restraining order and enactments to implement them x x x.
injunction when proven necessary.
xxx
In fine, the petitions at bar should be dismissed on procedural grounds alone. Even if We decide the
case based on the merits, the petitions should still be denied. The reasons for denying a cause of action to an alleged infringement of broad constitutional principles
are sourced from basic considerations of due process and the lack of judicial authority to wade "into the
59
Substantive Grounds uncharted ocean of social and economic policy making." chanroblesvirtuallawlibrary

In the same vein, Sec. 1 of Art. XI of the Constitution is not a self-executing provision considering that a
There is grave abuse of discretion when an act is (1) done contrary to the Constitution, the law or law should be passed by the Congress to clearly define and effectuate the principle embodied therein.
jurisprudence or (2) executed whimsically, capriciously or arbitrarily, out of malice, ill will or personal As a matter of fact, pursuant thereto, Congress enacted R.A. No. 6713 ("Code of Conduct and Ethical
46
bias. None is present in this case. Standards for Public Officials and Employees"), R.A. No. 6770 ("The Ombudsman Act of 1989"), R.A.
No. 7080 (An Act Defining and Penalizing the Crime of Plunder), and Republic Act No. 9485 ("Anti-Red
I Tape Act of 2007"). To complement these statutes, the Executive Branch has issued various orders,
The President's decision to bury Marcos at the LNMB is in accordance with the Constitution, the memoranda, and instructions relative to the norms of behavior/code of conduct/ethical standards of
law or jurisprudence officials and employees; workflow charts/public transactions; rules and policies on gifts and benefits;
whistle blowing and reporting; and client feedback program.

Petitioners argue that the burial of Marcos at the LNMB should not be allowed because it has the effect Petitioners' reliance on Sec. 3(2) of Art. XIV and Sec. 26 of Art. XVIII of the Constitution is also
of not just rewriting history as to the Filipino people's act of revolting against an authoritarian ruler but misplaced. Sec. 3(2) of Art. XIV refers to the constitutional duty of educational institutions in teaching
also condoning the abuses committed during the Martial Law, thereby violating the letter and spirit of the values of patriotism and nationalism and respect for human rights, while Sec. 26 of Art. XVIII is a
the 1987 Constitution, which is a "post-dictatorship charter" and a "human rights constitution." For them, transitory provision on sequestration or freeze orders in relation to the recovery of Marcos' ill-gotten
the ratification of the Constitution serves as a clear condemnation of Marcos' alleged "heroism." To wealth. Clearly, with respect to these provisions, there is no direct or indirect prohibition to Marcos'
47 48 49 50 51 52 53
support their case, petitioners invoke Sections 2, 11, 13, 23, 26, 27 and 28 of Article II, Sec. interment at the LNMB.
54 55 56 57
17 of Art. VII, Sec. 3(2) of Art. XIV, Sec. 1 of Art. XI, and Sec. 26 of Art. XVIII of the Constitution.
The second sentence of Sec. 17 of Art. VII pertaining to the duty of the President to "ensure that the
There is no merit to the contention.
laws be faithfully executed," which is identical to Sec. 1, Title I, Book III of the Administrative Code of the creation of the Board on National Pantheon. This is indicative of the legislative will not to pursue, at
60
1987, is likewise not violated by public respondents. Being the Chief Executive, the President the moment, the establishment of a singular interment place for the mortal remains of all Presidents of
represents the government as a whole and sees to it that all laws are enforced by the officials and the Philippines, national heroes, and patriots. Perhaps, the Manila North Cemetery, the Manila South
61
employees of his or her department. Under the Faithful Execution Clause, the President has the Cemetery, and other equally distinguished private cemeteries already serve the noble purpose but
62
power to take "necessary and proper steps" to carry into execution the law. The mandate is self- without cost to the limited funds of the government.
executory by virtue of its being inherently executive in nature and is intimately related to the other
63
executive functions. It is best construed as an imposed obligation, not a separate grant of Even if the Court treats R.A. No. 289 as relevant to the issue, still, petitioners' allegations must fail. To
64
power. The provision simply underscores the rule of law and, corollarily, the cardinal principle that the apply the standard that the LNMB is reserved only for the "decent and the brave" or "hero" would be
65
President is not above the laws but is obliged to obey and execute them. chanrobleslaw violative of public policy as it will put into question the validity of the burial of each and every mortal
remains resting therein, and infringe upon the principle of separation of powers since the allocation of
Consistent with President Duterte's mandate under Sec. 17, Art. VII of the Constitution, the burial of plots at the LNMB is based on the grant of authority to the President under existing laws and
Marcos at the LNMB does not contravene R.A. No. 289, R.A. No. 10368, and the international human regulations. Also, the Court shares the view of the OSG that the proposed interment is not equivalent to
rights laws cited by petitioners. the consecration of Marcos' mortal remains. The act in itself does not confer upon him the status of a
"hero." Despite its name, which is actually a misnomer, the purpose of the LNMB, both from legal and
66
A. On R.A. No. 289 chanrobleslaw historical perspectives, has neither been to confer to the people buried there the title of "hero" nor to
require that only those interred therein should be treated as a "hero." Lastly, petitioners' repeated
For the perpetuation of their memory and for the inspiration and emulation of this generation and of reference to a "hero's burial" and "state honors," without showing proof as to what kind of burial or
generations still unborn, R.A. No. 289 authorized the construction of a National Pantheon as the burial honors that will be accorded to the remains of Marcos, is speculative until the specifics of the interment
67
place of the mortal remains of all the Presidents of the Philippines, national heroes and patriots. It also have been finalized by public respondents.
68
provided for the creation of a Board on National Pantheon to implement the law. chanrobleslaw
70
B. On R.A. No. 10368 chanrobleslaw
On May 12, 1953, President Elpidio R. Quirino approved the site of the National Pantheon at East
69
Avenue, Quezon City. On December 23, 1953, he issued Proclamation No. 431 to formally "withdraw For petitioners, R.A. No. 10368 modified AFP Regulations G 161-375 by implicitly disqualifying Marcos'
from sale or settlement and reserve as a site for the construction of the National Pantheon a certain burial at the LNMB because the legislature, which is a co-equal branch of the government, has
parcel of land located in Quezon City." However, on July 5, 1954, President Magsaysay issued statutorily declared his tyranny as a deposed dictator and has recognized the heroism and sacrifices of
71
Proclamation No. 42 revoking Proclamation Nos. 422 and 431, both series of 1953, and reserving the the Human Rights Violations Victims (HRVVs) under his regime. They insist that the intended act of
parcels of land embraced therein for national park purposes to be known as Quezon Memorial Park. public respondents damages and makes mockery of the mandatory teaching of Martial Law atrocities
and of the lives and sacrifices of its victims. They contend that "reparation" under R.A. No. 10368
It is asserted that Sec. 1 of R.A. No 289 provides for the legal standard by which a person's mortal is non-judicial in nature but a political action of the State through the Legislative and Executive branches
remains may be interred at the LNMB, and that AFP Regulations G 161-375 merely implements the law by providing administrative relief for the compensation, recognition, and memorialization of human
and should not violate its spirit and intent. Petitioners claim that it is known, both here and abroad, that rights victims.
Marcos' acts and deed - the gross human rights violations, the massive corruption and plunder of
government coffers, and his military record that is fraught with myths, factual inconsistencies, and lies - We beg to disagree.
are neither worthy of perpetuation in our memory nor serve as a source of inspiration and emulation of
the present and future generations. They maintain that public respondents are not members of the Certainly, R.A. No. 10368 recognizes the heroism and sacrifices of all Filipinos who were victims of
Board on National Pantheon, which is authorized by the law to cause the burial at the LNMB of the summary execution, torture, enforced or involuntary disappearance, and other gross human rights
deceased Presidents of the Philippines, national heroes, and patriots. violations committed from September 21, 1972 to February 25, 1986. To restore their honor and dignity,
72
the State acknowledges its moral and legal obligation to provide reparation to said victims and/or their
Petitioners are mistaken. Both in their pleadings and during the oral arguments, they miserably failed to families for the deaths, injuries, sufferings, deprivations and damages they experienced.
provide legal and historical bases as to their supposition that the LNMB and the National Pantheon are
one and the same. This is not at all unexpected because the LNMB is distinct and separate from the In restoring the rights and upholding the dignity of HRVVs, which is part of the right to an effective
burial place envisioned in R.A. No 289. The parcel of land subject matter of President Quirino's remedy, R.A. No. 10368 entitles them to monetary and non-monetary reparation. Any HRVV qualified
73
Proclamation No. 431, which was later on revoked by President Magsaysay's Proclamation No. 42, is under the law shall receive a monetary reparation, which is tax-free and without prejudice to the
different from that covered by Marcos' Proclamation No. 208. The National Pantheon does not exist at receipt of any other sum from any other person or entity in any case involving human rights
74
present. To date, the Congress has deemed it wise not to appropriate any funds for its construction or violations. Anent the non-monetary reparation, the Department of Health (DOH), the Department of
Social Welfare and Development (DSWD), the Department of Education (DepEd), the Commission on must be made before the inference of implied repeal may be drawn, for inconsistency is never
Higher Education (CHED), the Technical Education and Skills Development Authority (TESDA), and presumed. There must be a showing of repugnance clear and convincing in character. The language
such other government agencies are required to render the necessary services for the HRVVs and/or used in the later statute must be such as to render it irreconcilable with what had been formerly
their families, as may be determined by the Human Rights Victims' Claims Board (Board) pursuant to enacted. An inconsistency that falls short of that standard does not suffice. x x
75 84
the provisions of the law. chanrobleslaw x chanroblesvirtuallawlibrary

Additionally, R.A. No. 10368 requires the recognition of the violations committed against the HRVVs, C. On International Human Rights Laws
regardless of whether they opt to seek reparation or not. This is manifested by enshrining their names in
76 Petitioners argue that the burial of Marcos at the LNMB will violate the rights of the HRVVs to "full" and
the Roll of Human Rights Violations Victims (Roll) prepared by the Board. The Roll may be displayed
77 "effective" reparation, which is provided under the International Covenant on Civil and Political
in government agencies designated by the HRVV Memorial Commission (Commission). Also, a
85
Memorial/Museum/Library shall be established and a compendium of their sacrifices shall be prepared Rights (ICCPR), the Basic Principles and Guidelines on the Right to a Remedy and Reparation for
78 Victims of Gross Violations of International Human Rights Law and Serious Violations of International
and may be readily viewed and accessed in the internet. The Commission is created primarily for the
86
establishment, restoration, preservation and conservation of the Memorial/Museum/ Humanitarian Law adopted by the U.N. General Assembly on December 16, 2005, and the Updated
79 Set of Principles for the Protection and Promotion of Human Rights Through Action to Combat
Library/Compendium. chanrobleslaw
87
Impunity dated February 8, 2005 by the U.N. Economic and Social Council.
80
To memorialize the HRVVs, the Implementing Rules and Regulations of R.A. No. 10368 further
88
mandates that: (1) the database prepared by the Board derived from the processing of claims shall be We do not think so. The ICCPR, as well as the U.N. principles on reparation and to combat impunity,
turned over to the Commission for archival purposes, and made accessible for the promotion of human call for the enactment of legislative measures, establishment of national programmes, and provision for
rights to all government agencies and instrumentalities in order to prevent recurrence of similar abuses, administrative and judicial recourse, in accordance with the country's constitutional processes, that are
81 necessary to give effect to human rights embodied in treaties, covenants and other international laws.
encourage continuing reforms and contribute to ending impunity; (2) the lessons learned from Martial
Law atrocities and the lives and sacrifices of HRVVs shall be included in the basic and higher education The U.N. principles on reparation expressly states:ChanRoblesVirtualawlibrary
curricula, as well as in continuing adult learning, prioritizing those most prone to commit human rights
82
violations; and (3) the Commission shall publish only those stories of HRVVs who have given prior Emphasizing that the Basic Principles and Guidelines contained herein do not entail new international
83
informed consent. chanrobleslaw or domestic legal obligations but identify mechanisms, modalities, procedures and methods for the
implementation of existing legal obligations under international human rights law and international
This Court cannot subscribe to petitioners' logic that the beneficial provisions of R.A. No. 10368 are not humanitarian law which are complementary though different as to their norms[.][Emphasis supplied]
exclusive as it includes the prohibition on Marcos' burial at the LNMB. It would be undue to extend the
The Philippines is more than compliant with its international obligations. When the Filipinos regained
law beyond what it actually contemplates. With its victim-oriented perspective, our legislators could
their democratic institutions after the successful People Power Revolution that culminated on February
have easily inserted a provision specifically proscribing Marcos' interment at the LNMB as a "reparation"
25, 1986, the three branches of the government have done their fair share to respect, protect and fulfill
for the HRVVs, but they did not. As it is, the law is silent and should remain to be so. This Court cannot
the country's human rights obligations, to wit:
read into the law what is simply not there. It is irregular, if not unconstitutional, for Us to presume the
legislative will by supplying material details into the law. That would be tantamount to judicial legislation.
chanRoblesvirtualLawlibraryThe 1987 Constitution contains provisions that promote and protect human
rights and social justice.
Considering the foregoing, the enforcement of the HRVVs' rights under R.A. No 10368 will surely not be
impaired by the interment of Marcos at the LNMB. As opined by the OSG, the assailed act has no 89 90
As to judicial remedies, aside from the writs of habeas corpus, amparo, and habeas data, the
causal connection and legal relation to the law. The subject memorandum and directive of public 91
Supreme Court promulgated on March 1, 2007 Administrative Order No. 25-2007, which provides
respondents do not and cannot interfere with the statutory powers and functions of the Board and the
rules on cases involving extra-judicial killings of political ideologists and members of the media. The
Commission. More importantly, the HRVVs' entitlements to the benefits provided for by R.A. No 10368
provision of the Basic Principles and Guidelines on the prevention of the victim's re-traumatization
and other domestic laws are not curtailed. It must be emphasized that R.A. No. 10368 does not amend
applies in the course of legal and administrative procedures designed to provide justice and
or repeal, whether express or implied, the provisions of the Administrative Code or AFP Regulations G 92
reparation. chanrobleslaw
161-375:ChanRoblesVirtualawlibrary

It is a well-settled rule of statutory construction that repeals by implication are not favored. In order to On the part of the Executive Branch, it issued a number of administrative and executive orders. Notable
effect a repeal by implication, the later statute must be so irreconcilably inconsistent and repugnant with of which are the following:
the existing law that they cannot be made to reconcile and stand together. The clearest case possible
A.O. No. 370 dated December 10, 1997 (Creating the Inter-Agency Coordinating Committee on Human Republic Act No. 7438 (An Act Defining Certain Rights of Person Arrested, Detained or Under
Rights) Custodia/Investigation as well as the Duties of the Arresting, Detaining and Investigating Officers and
Providing Penalties for Violations Thereof)
E.O. No. 118 dated July 5, 1999 (Providing for the Creation of a National Committee on the Culture of
Peace) Republic Act No. 8371 (The Indigenous Peoples' Rights Act of 1997)

E.O. No. 134 dated July 31, 1999 (Declaring August 12, 1999 and Every 12th Day of August Thereafter Republic Act No. 9201 (National Human Rights Consciousness Week Act of 2002)
as International Humanitarian Law Day)
Republic Act No. 9208 (Anti-Trafficking in Persons Act of 2003)
E.O. No. 404 dated January 24, 2005 (Creating the Government of the Republic of the Philippines
Monitoring Committee [GRPMC] on Human Rights and International Humanitarian Law) Republic Act No. 9262 (Anti-Violence Against Women and Their Children Act of 2004)

A.O. No. 157 dated August 21, 2006 (Creating an Independent Commission to Address Media and Republic Act No. 9344 (Juvenile Justice and Welfare Act of 2006)
Activist Killings)
Republic Act No. 9372 (Human Security Act of 2007)
A.O. No. 163 dated December 8, 2006 (Strengthening and Increasing the Membership of the
Presidential Human Rights Committee, and Expanding Further the Functions of Said Republic Act No. 9710 (The Magna Carta of Women)
93
Committee) chanrobleslaw
Republic Act No. 9745 (Anti-Torture Act of 2009)
A.O. No. 181 dated July 3, 2007 (Directing the Cooperation and Coordination Between the National
Republic Act No. 9851 (Philippine Act on Crimes Against International Humanitarian Law, Genocide,
Prosecution Service and Other Concerned Agencies of Government for the Successful Investigation
and Other Crimes Against Humanity)
and Prosecution of Political and Media Killings)
Republic Act No. 10121 (Philippine Disaster Risk Reduction and Management Act of 2010)
A.O. No. 197 dated September 25, 2007 (DND and AFP Coordination with PHRC Sub-committee on
Killings and Disappearances) Republic Act No. 10168 (The Terrorism Financing Prevention and Suppression Act of 2012)
A.O. No. 211 dated November 26, 2007 (Creating a Task Force Against Political Violence) Republic Act No. 10353 (Anti-Enforced or Involuntary Disappearance Act of 2012)
A.O. No. 249 dated December 10, 2008 (Further Strengthening Government Policies, Plans, and Republic Act No. 10364 (Expanded Anti-Trafficking In Persons Act of 2012)
Programs for the Effective Promotion and Protection of Human Rights on the Occasion of the 60th
Anniversary of the Universal Declaration of Human Rights) Republic Act No. 10368 (Human Rights Victims Reparation And Recognition Act of 2013)

E.O. No. 847 dated November 23, 2009 (Creating the Church-Police-Military-Liaison Committee to Republic Act No. 10530 (The Red Cross and Other Emblems Act of 2013)
Formulate and Implement a Comprehensive Program to Establish Strong Partnership Between the
State and the Church on Matters Concerning Peace and Order and Human Rights) Contrary to petitioners' postulation, our nation's history will not be instantly revised by a single resolve of
President Duterte, acting through the public respondents, to bury Marcos at the LNMB. Whether
A.O. No. 35 dated November 22, 2012 (Creating the Inter-Agency Committee on Extra-Legal Killings, petitioners admit it or not, the lessons of Martial Law are already engraved, albeit in varying degrees, in
Enforced Disappearances, Torture and Other Grave Violations of the Right to Life, Liberty and Security the hearts and minds of the present generation of Filipinos. As to the unborn, it must be said that the
of Persons) preservation and popularization of our history is not the sole responsibility of the Chief Executive; it is a
joint and collective endeavor of every freedom-loving citizen of this country.
A.O. No. 1 dated October 11, 2016 (Creating the Presidential Task Force on Violations of the Right to
Life, Liberty and Security of the Members of the Media) Notably, complementing the statutory powers and functions of the Human Rights Victims' Claims Board
and the HRVV Memorial Commission in the memorialization of HRVVs, the National Historical
Finally, the Congress passed the following laws affecting human rights: 94
Commission of the Philippines (NHCP), formerly known as the National Historical Institute (NHI), is
mandated to act as the primary government agency responsible for history and is authorized to
95
determine all factual matters relating to official Philippine history. Among others, it is tasked to: (a)
conduct and support all kinds of research relating to Philippine national and local history; (b) develop
119
educational materials in various media, implement historical educational activities for the popularization As one of the cultural agencies attached to the NCAA, the NHCP manages, maintains and
of Philippine history, and disseminate, information regarding Philippine historical events, dates, places administers national shrines, monuments, historical sites, edifices and landmarks of significant historico-
120
and personages; and (c) actively engage in the settlement or resolution of controversies or issues cultural value. In particular, the NHCP Board has the power to approve the declaration of historic
96
relative to historical personages, places, dates and events. Under R.A. Nos. 10066 (National Cultural structures and sites, such as national shrines, monuments, landmarks and heritage houses and to
97
Heritage Act of 2009) and 10086 (Strengthening Peoples' Nationalism Through Philippine History determine the manner of their identification, maintenance, restoration, conservation, preservation and
98 121
Act), the declared State policy is to conserve, develop, promote, and popularize the nation's historical protection. chanrobleslaw
99
and cultural heritage and resources. Towards this end, means shall be provided to strengthen
people's nationalism, love of country, respect for its heroes and pride for the people's accomplishments Excluded, however, from the jurisdiction of the NHCP are the military memorials and battle monuments
by reinforcing the importance of Philippine national and local history in daily life with the end in view of declared as national shrines, which have been under the administration, maintenance and development
100
raising social consciousness. Utmost priority shall be given not only with the research on history but of the Philippine Veterans Affairs Office (PVAO) of the DND. Among the military shrines are: Mt. Samat
101 122 123
also its popularization. chanrobleslaw National Shrine in Pilar, Bataan; Kiangan War Memorial Shrine in Linda, Kiangan, Ifugao; Capas
124
National Shrine in Capas, Tarlac; Ricarte National Shrine in Malasin, Batac, Ilocos
II. 125 126
Norte; Balantang Memorial Cemetery National Shrine in Jaro, Iloilo; Balete Pass National Shrine in
The President's decision to bury Marcos at the LNMB is not done whimsically, capriciously or 127
Sta. Fe, Nueva Vizcaya; USAFIP, NL Military Shrine and Park in Bessang Pass, Cervantes, Ilocos
arbitrarily, out of malice, ill will or personal bias 128 129
Sur; and the LNMB in Taguig City, Metro Manila. chanrobleslaw

B. The Libingan Ng Mga Bayani


Petitioners contend that the interment of Marcos at the LNMB will desecrate it as a sacred and hallowed
place and a revered national shrine where the mortal remains of our country's great men and women At the end of World War II, the entire nation was left mourning for the death of thousands of Filipinos.
are interred for the inspiration and emulation of the present generation and generations to come. They Several places served as grounds for the war dead, such as the Republic Memorial Cemetery, the
erred. Bataan Memorial Cemetery, and other places throughout the country. The Republic Memorial
Cemetery, in particular, was established in May 1947 as a fitting tribute and final resting place of Filipino
A. National Shrines military personnel who died in World War II.

As one of the cultural properties of the Philippines, national historical shrines (or historical shrines) refer On October 23, 1954, President Ramon D. Magsaysay, Sr. issued E.O. No. 77, which ordered "the
to sites or structures hallowed and revered for their history or association as declared by the remains of the war dead interred at the Bataan Memorial Cemetery, Bataan Province, and at other
102
NHCP. The national shrines created by law and presidential issuance include, among others: Fort places in the Philippines, be transferred to, and reinterred at, the Republic Memorial Cemetery at Fort
103 104
Santiago (Dambana ng Kalayaan) in Manila; all battlefield areas in Corregidor and Bataan; the site Wm Mckinley, Rizal Province" so as to minimize the expenses for the maintenance and upkeep, and to
105
of First Mass in the Philippines in Magallanes, Limasawa, Leyte; Aguinaldo Shrine or Freedom Shrine make the remains accessible to the widows, parents, children, relatives, and friends.
106 107
in Kawit, Cavite; Fort San Antonio Abad National Shrine in Malate, Manila; Tirad Pass National
108 109 110
Shrine in Ilocos Sur; Ricarte Shrine and Aglipay Shrine in Batac, Ilocos Norte; Liberty Shrine in On October 27, 1954, President Magsaysay issued Proclamation No. 86, which changed the name of
111
Lapu-Lapu, Cebu; "Red Beach" or the landing point of General Douglas MacArthur and the liberating Republic Memorial Cemetery to Libingan Ng Mga Bayani to symbolize "the cause for which our soldiers
112
forces in Baras, Palo, Leyte; Dapitan City as a National Shrine City in Zamboanga Del 130
have died" and to "truly express the nations esteem and reverence for her war dead." chanrobleslaw
113 114
Norte; General Leandro Locsin Fullon National Shrine in Hamtic, Antique; and Mabini Shrine in
115
Polytechnic University of the Philippines - Mabini Campus, Sta. Mesa, Manila. As sites of the birth, On July 12, 1957, President Carlos P. Garcia issued Proclamation No. 423, which reserved for military
exile, imprisonment, detention or death of great and eminent leaders of the nation, it is the policy of the purposes, under the administration of the AFP Chief of Staff, the land where LNMB is located. The
116
Government to hold and keep the national shrines as sacred and hallowed place. P.O. No. LNMB was part of a military reservation site then known as Fort Wm McKinley (now known as Fort
117
105 strictly prohibits and punishes by imprisonment and/or fine the desecration of national shrines by Andres Bonifacio).
disturbing their peace and serenity through digging, excavating, defacing, causing unnecessary noise,
and committing unbecoming acts within their premises. R.A. No. 10066 also makes it punishable to On May 28, 1967, Marcos issued Proclamation No. 208, which excluded the LNMB from the Fort
intentionally modify, alter, or destroy the original features of, or undertake construction or real estate Bonifacio military reservation and reserved the LNMB for national shrine purposes under the
development in any national shrine, monument, landmark and other historic edifices and structures, administration of the National Shrines Commission (NSC) under the DND.
declared, classified, and marked by the NHCP as such, without the prior written permission from the
118
National Commission for Culture and the Arts (NCAA). chanrobleslaw On September 24, 1972, Marcos, in the exercise of his powers as the AFP Commander-in-Chief, and
pursuant to Proclamation No. 1081 dated September 21, 1972, and General Order No. 1 dated
September 22, 1972, as amended, issued Presidential Decree (P.D.) No. 1 which reorganized the Black Stone Walls - Erected on opposite sides of the main entrance road leading to the Tomb of the
Executive Branch of the National Government through the adoption of the Integrated Reorganization Unknown Soldiers and just near the Heroes Memorial are two 12-foot high black stone walls which bear
Plan (IRP). Section 7, Article XV, Chapter I, Part XII thereof abolished the NSC and its functions the words, "I do not know the dignity of his birth, but I do know the glory of his death." that General
together with applicable appropriations, records, equipment, property and such personnel as may be Douglas MacArthur made during his sentimental journey to the Philippines in 1961.
necessary were transferred to the NHI under the Department of Education (DEC). The NHI was
responsible for promoting and preserving the Philippine cultural heritage by undertaking, inter alia, Defenders of Bataan and Corregidor Memorial Pylon - Inaugurated on April 5, 1977 by Secretary
studies on Philippine history and national heroes and maintaining national shrines and Renato S. De Villa in memory of the defenders of Bataan and Corregidor during World War II. This
131 monument is dedicated as an eternal acknowledgment of their valor and sacrifice in defense of the
monuments. chanrobleslaw
Philippines.
Pending the organization of the DEC, the functions relative to the administration, maintenance and
development of national shrines tentatively integrated into the PVAO in July 1973. Korean Memorial Pylon - A towering monument honoring the 112 Filipino officers and men who, as
members of the Philippine Expeditionary Forces to Korea (PEFTOK), perished during the Korean War.
On January 26, 1977, President Marcos issued P.D. No. 1076. Section 7, Article XV, Chapter I, Part XII
Vietnam Veterans Memorial Pylon - Dedicated to the members of the Philippine contingents and
of the IRP was repealed on the grounds that "the administration, maintenance and development of
Philippine civic action groups to Vietnam (PHILCON-V and PHILCAG-V) who served as medical, dental,
national shrines consisting of military memorials or battle monuments can be more effectively
engineering construction, community and psychological workers, and security complement. They
accomplished if they are removed from the [DEC] and transferred to the [DND] by reason of the latter s
offered tremendous sacrifices as they alleviated human suffering in war-ravaged Vietnam from 1964-
greater capabilities and resources" and that "the functions of the [DND] are more closely related and
1971. Inscribed on the memorial pylon are the words: "To build and not to destroy, to bring the
relevant to the charter or significance of said national shrines." Henceforth, the PVAO through the
Vietnamese people happiness and not sorrow, to develop goodwill and not hatred."
Military Shrines Service (MSS), which was created to perform the functions of the abolished NSC -
would administer, maintain and develop military memorials and battle monuments proclaimed as Philippine World War II Guerillas Pylon - Erected by the Veterans Federation of the Philippines as a
national shrines. testimony to the indomitable spirit and bravery of the Filipino guerillas of World War II who refused to be
cowed into submission and carried on the fight for freedom against an enemy with vastly superior arms
On July 25, 1987, President Corazon C. Aquino issued the Administrative Code. The Code retains and under almost insurmountable odds. Their hardship and sufferings, as well as their defeats and
132
PVAO under the supervision and control of the Secretary of National Defense. Among others, PVAO victories, are enshrined in this memorial.
134
133
shall administer, develop and maintain military shrines. With the approval of PVAO Rationalization
Plan on June 29, 2010, pursuant to E.O. No. 366 dated October 4, 2004, MSS was renamed to 135
Contrary to the dissent, P.D. No. 105 does not apply to the LNMB. Despite the fact that P.D. No. 208
Veterans Memorial and Historical Division, under the supervision and control of PVAO, which is 136
predated P.D. No. 105, the LNMB was not expressly included in the national shrines enumerated in
presently tasked with the management and development of military shrines and the perpetuation of the 137
the latter. The proposition that the LNMB is implicitly covered in the catchall phrase "and others which
heroic deeds of our nation's veterans. may be proclaimed in the future as National Shrines" is erroneous because:

As a national military shrine, the main features, structures, and facilities of the LNMB are as follows: chanRoblesvirtualLawlibrary(1) As stated, Marcos issued P.D. No. 208 prior to P.D. No. 105.

Tomb of the Unknown Soldiers - The main structure constructed at the center of the cemetery where 138
(2) Following the canon of statutory construction known as ejusdem generis, the LNMB is not a site
wreath laying ceremonies are held when Philippine government officials and foreign dignitaries visit the "of the birth, exile, imprisonment, detention or death of great and eminent leaders of the nation." What
LNMB. The following inscription is found on the tomb: "Here lies a Filipino soldier whose name is known P.D. No. 105 contemplates are the following national shrines: Fort Santiago ("Dambana ng Kalayaan"),
only to God." Behind the tomb are three marble pillars representing the three main island groups of the all battlefield areas in Corregidor and Bataan, the site of First Mass in the Philippines, Aguinaldo Shrine
Philippines - Luzon, Visayas and Mindanao. Buried here were the remains of 39,000 Filipino soldiers or Freedom Shrine, Fort San Antonio Abad National Shrine, Tirad Pass National Shrine, Ricarte Shrine,
who were originally buried in Camp O'Donnell Concentration Camp and Fort Santiago, Intramuros, Aglipay Shrine, Liberty Shrine, "Red Beach" or the landing point of General Douglas MacArthur and the
Manila. liberating forces, Dapitan City, General Leandro Locsin Fullon National Shrine, and Mabini Shrine.
Excluded are the military memorials and battle monuments declared as national shrines under the
Heroes Memorial Gate - A structure shaped in the form of a large concrete tripod with a stairway
PVAO, such as: Mt. Samat National Shrine, Kiangan War Memorial Shrine, Capas National Shrine,
leading to an upper view deck and a metal sculpture at the center. This is the first imposing structure
Ricarte National Shrine, Balantang Memorial Cemetery National Shrine, Balete Pass National Shrine;
one sees upon entering the grounds of the cemetery complex.
USAFIP, NL Military Shrine and Park, and the LNMB.

(3) Since its establishment, the LNMB has been a military shrine under the jurisdiction of the PVAO.
While P.D. No. 1 dated September 24, 1972 transferred the administration, maintenance and
development of national shrines to the NHI under the DEC, it never actually materialized. Pending the The LNMB is considered as a national shrine for military memorials. The PVAO, which is empowered to
organization of the DEC, its functions relative to national shrines were tentatively integrated into the administer, develop, and maintain military shrines, is under the supervision and control of the DND. The
PVAO in July 1973. Eventually, on January 26, 1977, Marcos issued P.D. No. 1076. The PVAO, DND, in turn, is under the Office of the President.
through the MSS, was tasked to administer, maintain, and develop military memorials and battle
monuments proclaimed as national shrines. The reasons being that "the administration, maintenance The presidential power of control over the Executive Branch of Government is a self-executing provision
and development of national shrines consisting of military memorials or battle monuments can be more of the Constitution and does not require statutory implementation, nor may its exercise be limited, much
147
effectively accomplished if they are removed from the [DEC] and transferred to the [DND] by reason of less withdrawn, by the legislature. This is why President Duterte is not bound by the alleged 1992
148
the latter's greater capabilities and resources" and that "the functions of the [DND] are more closely Agreement between former President Ramos and the Marcos family to have the remains of Marcos
related and relevant to the charter or significance of said national shrines." interred in Batac, Ilocos Norte. As the incumbent President, he is free to amend, revoke or rescind
political agreements entered into by his predecessors, and to determine policies which he considers,
The foregoing interpretation is neither narrow and myopic nor downright error. Instead, it is consistent based on informed judgment and presumed wisdom, will be most effective in carrying out his mandate.
with the letter and intent of P.D. No. 105.
Moreover, under the Administrative Code, the President has the power to reserve for public use and for
Assuming that P.D. No. 105 is applicable, the descriptive words "sacred and hallowed" refer to the specific public purposes any of the lands of the public domain and that the reserved land shall remain
149
LNMB as a place and not to each and every mortal remains interred therein. Hence, the burial of subject to the specific public purpose indicated until otherwise provided by law or proclamation. At
Marcos at the LNMB does not diminish said cemetery as a revered and respected ground. Neither does present, there is no law or executive issuance specifically excluding the land in which the LNMB is
it negate the presumed individual or collective "heroism" of the men and women buried or will be buried located from the use it was originally intended by the past Presidents. The allotment of a cemetery plot
150 151
therein. The "nations esteem and reverence for her war dead," as originally contemplated by President at the LNMB for Marcos as a former President and Commander-in-Chief, a legislator, a Secretary
152 153 154 155
Magsaysay in issuing Proclamation No. 86, still stands unaffected. That being said, the interment of of National Defense, a military personnel, a veteran, and a Medal of Valor awardee, whether
Marcos, therefore, does not constitute a violation of the physical, historical, and cultural integrity of the recognizing his contributions or simply his status as such, satisfies the public use requirement. The
LNMB as a national military shrine. disbursement of public funds to cover the expenses incidental to the burial is granted to compensate
156
him for valuable public services rendered. Likewise, President Duterte's determination to have
At this juncture, reference should be made to Arlington National Cemetery (Arlington), which is identical Marcos' remains interred at the LNMB was inspired by his desire for national healing and reconciliation.
to the LNMB in terms of its prominence in the U.S. It is not amiss to point that our armed forces have Presumption of regularity in the performance of official duty prevails over petitioners' highly disputed
been patterned after the U.S. and that its military code produced a salutary effect in the Philippines' factual allegation that, in the guise of exercising a presidential prerogative, the Chief Executive is
139
military justice system. Hence, relevant military rules, regulations, and practices of the U.S. have actually motivated by utang na loob (debt of gratitude) and bayad utang (payback) to the Marcoses. As
persuasive, if not the same, effect in this jurisdiction. the purpose is not self-evident, petitioners have the burden of proof to establish the factual basis of their
claim. They failed. Even so, this Court cannot take cognizance of factual issues since We are not a trier
140
As one of the U.S. Army national military cemeteries, the Arlington is under the jurisdiction of the of facts.
141
Department of the Army. The Secretary of the U.S. Army has the responsibility to develop, operate,
manage, administer, oversee, and fund the Army national military cemeteries in a manner and to C. AFP Regulations on the LNMB
standards that fully honor the service and sacrifices of the deceased members of the armed forces
buried or inurned therein, and shall prescribe such regulations and policies as may be necessary to A review of the regulations issued by the AFP Chief of Staff as to who may and may not be interred at
142
administer the cemeteries. In addition, the Secretary of the U.S. Army is empowered to appoint an the LNMB underscores the nature and purpose of the LNMB as an active military cemetery/grave site.
advisory committee, which shall make periodic reports and recommendations as well as advise the
Secretary with respect to the administration of the cemetery, the erection of memorials at the cemetery, On May 13, 1947, the Chief of Staff of the Philippine Army, by the direction of the President and by
143
and master planning for the cemetery. chanrobleslaw order of the Secretary of National Defense, issued General Orders No. 111, which constituted and
activated, as of said date, the Graves Registration Platoon as a unit of the Philippine Army.
Similar to the Philippines, the U.S. national cemeteries are established as national shrines in tribute to
144
the gallant dead who have served in the U.S. Armed Forces. The areas are protected, managed and On February 2, 1960, the AFP Chief of Staff, by order of the Secretary of National Defense, issued AFP
145
administered as suitable and dignified burial grounds and as significant cultural resources. As such, Regulations G 161-371 (Administrative and Special Staff Services, Grave Registration Service), which
the authorization of activities that take place therein is limited to those that are consistent with provided that the following may be interred in the LNMB: (a) World War II dead of the AFP and
applicable legislation and that are compatible with maintaining their solemn commemorative and historic recognized guerillas; (b) Current dead of the AFP; (c) Retired military personnel of the AFP; (d)
146
character. chanrobleslaw Remains of former members of the AFP who died while in the active service and in the Retired List of
the AFP now interred at different cemeteries and other places throughout the Philippines or the National Defense; and (i) Former Presidents, Secretaries of Defense, CSAFP, Generals/Flag Officers,
Secretary of National Defense; and (e) Others upon approval of the Congress of the Philippines, the Dignitaries, Statesmen, National Artists, widows of former Presidents, Secretaries of National Defense
President of the Philippines or the Secretary of National Defense. The regulation also stated that the and Chief of Staff. The remains of the following were not allowed to be interred in the LNMB: (a)
AFP Quartermaster General will be responsible for, among other matters, the efficient operation of the Personnel who were dishonorably separated/reverted/discharged from the service; and (b) Authorized
Graves Registration Service; the interment, disinterment and reinterment of the dead mentioned above; personnel who were convicted by final judgment of an offense involving moral turpitude. Like AFP
and preservation of military cemeteries, national cemeteries, and memorials. Regulations G 161-373, it stated that the Quartermaster General shall be responsible for the allocation
of specific section/areas for the deceased persons, whereas the Commanding Officer of the
On July 31, 1973, the AFP Chief of Staff, by order of the Secretary of National Defense, issued AFP Quartermaster Graves Registration Unit shall be charged with the preparation of grave sites,
Regulations G 161-372 (Administration and Operation of AFP Graves Registration Installations), which supervision of burials, and the registration of graves.
superseded AFP Regulations G 161-371. It provided that the following may be interred in the LNMB: (a)
Deceased Veterans of the Philippine Revolution of 1896/World War I; (b) Deceased World War II Finally, on September 11, 2000, the AFP Chief of Staff, by the order of the Secretary of National
members of the AFP and recognized guerillas; (c) Deceased military personnel of the AFP who died Defense, issued AFP Regulations G 161-375 (Allocation of Cemetery Plots at the Libingan Ng Mga
while in the active duty; (d) Deceased retired military personnel of the AFP; (e) Deceased military Bayani), which superseded AFP Regulations G 161-374. The regulation stated that the Chief of Staff
personnel of the AFP interred at different cemeteries and other places outside the LNMB; and (f) Such shall be responsible for the issuance of interment directive for all active military personnel for interment,
remains of persons as the Commander-in-Chief of the AFP may direct. The remains of the following authorized personnel (such as those former members of the AFP who laterally entered or joined the
were not allowed to be interred in the LNMB: (a) The spouse of an active, or retired, deceased military Philippine Coast Guard [PCG] and the Philippine National Police [PNP]), and retirees, veterans and
personnel, recognized guerillas who himself/herself is not a military personnel; and (b) AFP personnel reservists enumerated therein. The Quartermaster General is tasked to exercise over-all supervision in
who were retireable but separated/reverted/discharged for cause, or joined and aided the enemy of the the implementation of the regulation and the Commander ASCOM, PA through the Commanding Officer
Republic of the Philippines, or were convicted of capital or other criminal offenses, involving moral of Grave Services Unit is charged with the registration of the deceased/graves, the allocation of specific
turpitude. The regulation also stated that the Quartermaster General shall be responsible for, among section/area at the LNMB for interment of deceased, the preparation of grave sites, and the supervision
other matters, the efficient operation of the AFP graves registration installations; the interment, of burials.
disinterment and reinterment of deceased military personnel mentioned above; and the preservation of
military cemeteries, proper marking and official recording of graves therein. Under AFP Regulations G 161-375, the following are eligible for interment at the LNMB: (a) Medal of
Valor Awardees; (b) Presidents or Commanders-in-Chief, AFP; (c) Secretaries of National Defense; (d)
On April 9, 1986, AFP Chief of Staff Fidel V. Ramos, by order of National Defense Minister, issued AFP Chiefs of Staff, AFP; (e) General/Flag Officers of the AFP; (f) Active and retired military personnel of the
Regulations G 161-373 (Allocation of Cemetery Plots at the Libingan Ng Mga Bayani), which AFP to include active draftees and trainees who died in line of duty, active reservists and CAFGU Active
superseded AFP Regulations G 161-372. It enumerated a list of deceased person who may be interred Auxiliary (CAA) who died in combat operations or combat related activities; (g) Former members of the
at the LNMB, namely: (a) Medal of Valor Awardees; (b) Presidents or Commanders-in-Chief, AFP; (c) AFP who laterally entered or joined the PCG and the PNP; (h) Veterans of Philippine Revolution of
Ministers of National Defense; (d) Chiefs of Staff, AFP; (e) General/Flag Officers of the AFP; (f) Active 1890, WWI, WWII and recognized guerillas; (i) Government Dignitaries, Statesmen, National Artists and
and retired military personnel of the AFP; (g) Veterans of Philippine Revolution of 1896, WWI, WWII and other deceased persons whose interment or reinterment has been approved by the Commander-in-
recognized guerillas; and (h) Government Dignitaries, Statesmen, National Artist and other deceased Chief, Congress or the Secretary of National Defense; and G) Former Presidents, Secretaries of
persons whose interment or reinterment has been approved by the Commander-in-Chief, Batasang Defense, Dignitaries, Statesmen, National Artists, widows of Former Presidents, Secretaries of National
Pambansa or the Minister of National Defense. The regulation also stated that the Quartermaster Defense and Chief of Staff. Similar to AFP Regulations G 161-374, the following are not qualified to be
General shall be responsible for the allocation of specific section/areas for the said deceased persons, interred in the LNMB: (a) Personnel who were dishonorably separated/reverted/discharged from the
while the Commanding Officer of the Quartermaster Graves Registration Company shall be charged service; and (b) Authorized personnel who were convicted by final judgment of an offense involving
with the preparation of grave sites, supervision of burials at LNMB and the registration of graves. moral turpitude.

On March 27, 1998, the AFP Chief of Staff, by order of the Secretary of National Defense, issued AFP In the absence of any executive issuance or law to the contrary, the AFP Regulations G 161-375
Regulations G 161-374 (Allocation of Cemetery Plots at the Libingan Ng Mga Bayani), which remains to be the sole authority in determining who are entitled and disqualified to be interred at the
superseded AFP Regulations G 161-373. It provided that the following may be interred in the LNMB: (a) LNMB. Interestingly, even if they were empowered to do so, former Presidents Corazon C. Aquino and
Medal of Valor Awardees; (b) Presidents or Commanders-inChief, AFP; (c) Secretaries of National Benigno Simeon C. Aquino III, who were themselves aggrieved at the Martial Law, did not revise the
Defense; (d) Chiefs of Staff, AFP; (e) General/Flag Officers of the AFP; (f) Active and retired military rules by expressly prohibiting the burial of Marcos at the LNMB. The validity of AFP Regulations G 161-
personnel of the AFP; (g) Veterans of Philippine Revolution of 1890, WWI, WWII and recognized 375 must, therefor, be sustained for having been issued by the AFP Chief of Staff acting under the
guerillas; (h) Government Dignitaries, Statesmen, National Artists and other deceased persons whose direction of the Secretary of National Defense, who is the alter ego of the President.
interment or reinterment has been approved by the Commander-in-Chief, Congress or Secretary of
x x x In Joson v. Torres, we explained the concept of the alter ego principle or the doctrine of qualified (4) Any veteran who received an honorable discharge from the Armed Forces prior to October 1, 1949,
political agency and its limit in this wise: who was discharged for a permanent physical disability, who served on active duty (other than for
training), and who would have been eligible for retirement under the provisions of 10 U.S.C. 1201 had
chanRoblesvirtualLawlibraryUnder this doctrine, which recognizes the establishment of a single the statute been in effect on the date of separation.
executive, all executive and administrative organizations are adjuncts of the Executive Department, the
heads of the various executive departments are assistants and agents of the Chief Executive, (5) Any veteran awarded one of the following decorations:
and, except in cases where the Chief Executive is required by the Constitution or law to act in
163
person or the exigencies of the situation demand that he act personally, the multifarious executive chanRoblesvirtualLawlibrary(i) Medal of Honor; chanrobleslaw
and administrative functions of the Chief Executive are performed by and through the executive
departments, and the acts of the Secretaries of such departments, performed and promulgated in the (ii) Distinguished Service Cross, Air Force Cross, or Navy Cross;
regular course of business, are, unless disapproved or reprobated by the Chief Executive presumptively
157
the acts of the Chief Executive. (Emphasis ours, citation omitted.) chanroblesvirtuallawlibrary (iii) Distinguished Service Medal;

It has been held that an administrative regulation adopted pursuant to law has the force and effect of (iv) Silver Star; or
law and, until set aside, is binding upon executive and administrative agencies, including the President
158
as the chief executor of laws. chanrobleslaw (v) Purple Heart.

1. Qualification under the AFP Regulations (6) Any veteran who served on active duty (other than active duty for training) and who held any of the
following positions:
AFP Regulations G 161-375 should not be stricken down in the absence of clear and unmistakable
showing that it has been issued with grave abuse of discretion amounting to lack or excess of chanRoblesvirtualLawlibrary(i) President or Vice President of the United States;
jurisdiction. Neither could it be considered ultra vires for purportedly providing incomplete, whimsical,
and capricious standards for qualification for burial at the LNMB. (ii) Elected member of the U.S. Congress;

To compare, We again refer to the U.S. Army regulations on Arlington. In the U.S., the Secretary of the (iii) Chief Justice of the Supreme Court of the United States or Associate Justice of the Supreme Court
Army, with the approval of the Secretary of Defense, determines eligibility for interment or inurnment in of the United States;
159 160
the Army national military cemeteries. Effective October 26, 2016, the rule is as
follows:ChanRoblesVirtualawlibrary 164 165
(iv) A position listed, at the time the person held the position, in 5 U.S.C. 5312 or 5313 (Levels I
and II of the Executive Schedule); or
Only those who qualify as a primarily eligible person or a derivatively eligible person are eligible for
161
interment in Arlington National Cemetery, unless otherwise prohibited as provided for in §§ 553.19 - (v) Chief of Mission of a Category 4, 5, or post if the Department of State classified that post as a
162
553.20, provided that the last period of active duty of the service member or veteran ended with an Category 4, 5, or 5+ post during the person's tenure as Chief of Mission.
honorable discharge.
(7) Any former prisoner of war who, while a prisoner of war, served honorably in the active military
(a) Primarily eligible persons. The following are primarily eligible persons for purposes of interment: service, and who died on or after November 30, 1993.

chanRoblesvirtualLawlibrary(1) Any service member who dies on active duty in the U.S. Armed Forces (b) Derivatively eligible persons. The following individuals are derivatively eligible persons for purposes
(except those service members serving on active duty for training only), if the General Courts Martial of interment who may be interred if space is available in the gravesite of the primarily eligible person:
Convening Authority grants a certificate of honorable service.
chanRoblesvirtualLawlibrary(1) The spouse of a primarily eligible person who is or will be interred in
(2) Any veteran retired from a Reserve component who served a period of active duty (other than for Arlington National Cemetery. A former spouse of a primarily eligible person is not eligible for interment
training), is carried on the official retired list, and is entitled to receive military retired pay. in Arlington National Cemetery under this paragraph.

(3) Any veteran retired from active military service and entitled to receive military retired pay. (2) The spouse of an active duty service member or an eligible veteran, who was:
chanRoblesvirtualLawlibrary(i) Lost or buried at sea, temporarily interred overseas due to action by the "bayani" in the LNMB has become a misnomer since while a symbolism of heroism may attach to the
Government, or officially determined to be missing in action; LNMB as a national shrine for military memorial, the same does not automatically attach to its feature
as a military cemetery and to those who were already laid or will be laid therein. As stated, the purpose
(ii) Buried in a U.S. military cemetery maintained by the American Battle Monuments Commission; or of the LNMB, both from the legal and historical perspectives, has neither been to confer to the people
buried there the title of "hero" nor to require that only those interred therein should be treated as a
(iii) Interred in Arlington National Cemetery as part of a group burial (the derivatively eligible spouse "hero." In fact, the privilege of internment at the LNMB has been loosen up through the years. Since
may not be buried in the group burial gravesite). 1986, the list of eligible includes not only those who rendered active military service or military-related
activities but also non-military personnel who were recognized for their significant contributions to the
(3) The parents of a minor child or a permanently dependent adult child, whose remains were interred in Philippine society (such as government dignitaries, statesmen, national artists, and other deceased
Arlington National Cemetery based on the eligibility of a parent at the time of the child's death, unless persons whose interment or reinterment has been approved by the Commander-in-Chief, Congress or
eligibility of the non-service connected parent is lost through divorce from the primarily eligible parent. Secretary of National Defense). In 1998, the widows of former Presidents, Secretaries of National
Defense and Chief of Staff were added to the list. Whether or not the extension of burial privilege to
(4) An honorably discharged veteran who does not qualify as a primarily eligible person, if the veteran civilians is unwarranted and should be restricted in order to be consistent with the original purpose of
will be buried in the same gravesite as an already interred primarily eligible person who is a close the LNMB is immaterial and irrelevant to the issue at bar since it is indubitable that Marcos had
relative, where the interment meets the following conditions: rendered significant active military service and military-related activities.

chanRoblesvirtualLawlibrary(i) The veteran is without minor or unmarried adult dependent children; Petitioners did not dispute that Marcos was a former President and Commander-in-Chief, a legislator, a
Secretary of National Defense, a military personnel, a veteran, and a Medal of Valor awardee. For his
(ii) The veteran will not occupy space reserved for the spouse, a minor child, or a permanently alleged human rights abuses and corrupt practices, we may disregard Marcos as a President and
dependent adult child; Commander-in-Chief, but we cannot deny him the right to be acknowledged based on the other
positions he held or the awards he received. In this sense, We agree with the proposition that Marcos
(iii) All other close relatives of the primarily eligible person concur with the interment of the veteran with should be viewed and judged in his totality as a person. While he was not all good, he was not pure evil
the primarily eligible person by signing a notarized statement; either. Certainly, just a human who erred like us.

(iv) The veteran's spouse waives any entitlement to interment in Arlington National Cemetery, where Our laws give high regard to Marcos as a Medal of Valor awardee and a veteran. R.A. No.
171
such entitlement might be based on the veteran's interment in Arlington National Cemetery. The 9049 declares the policy of the State "to consistently honor its military heroes in order to strengthen
172
Executive Director may set aside the spouse's waiver, provided space is available in the same the patriotic spirit and nationalist consciousness of the military." For the "supreme self-sacrifice and
173
gravesite, and all close relatives of the primarily eligible person concur; distinctive acts of heroism and gallantry," a Medal of Valor awardee or his/her
dependents/heirs/beneficiaries are entitled to the following social services and financial rewards:
(v) Any cost of moving, recasketing, or revaulting the remains will be paid from private funds.
Tax-exempt lifetime monthly gratuity of Twenty Thousand Pesos (P20,000.00), which is separate and
There is a separate list of eligible with respect to the inurnment of cremated remains in the distinct from any salary or pension that the awardee currently receives or will receive from the
166 167 168 174
Columbarium, interment of cremated remains in the Unmarked Area, and group burial. As a government of the Philippines; chanrobleslaw
national military cemetery, eligibility standards for interment, inurnment, or memorialization in Arlington
169 Precedence in employment in government agencies or government-owned or controlled corporation, if
are based on honorable military service. Exceptions to the eligibility standards for new graves,
which are rarely granted, are for those persons who have made significant contributions that directly the job qualifications or requirements are met;
170
and substantially benefited the U.S. military. chanrobleslaw
Priority in the approval of the awardee's housing application under existing housing programs of the
Judging from the foregoing, it is glaring that the U.S. Army regulations on Arlington and the AFP government;
Regulations G 161-375 on the LNMB, as a general rule, recognize and reward the military services or
Priority in the acquisition of public lands under the Public Land Act and preferential right in the lease of
military related activities of the deceased. Compared with the latter, however, the former is actually less
pasture lands and exploitation of natural resources;
generous in granting the privilege of interment since only the spouse or parent, under certain conditions,
may be allowed "if space is available in the gravesite of the primarily eligible person." Privilege of obtaining loans in an aggregate amount not exceeding Five Hundred Thousand Pesos
(P500,000.00) from governmentowned or controlled financial institutions without having to put up any
It is not contrary to the "well-established custom," as the dissent described it, to argue that the word collateral or constitute any pledge or mortgage to secure the payment of the loan;
Twenty (20%) percent discount from all establishments relative to utilization of transportation services,
hotels and similar lodging establishments, restaurants, recreation and sport centers and purchase of Furthermore, according to petitioners, to limit the application of the disqualifying provisions of AFP
medicine anywhere in the country; Regulations G 161-375 only to soldiers would be unfair (since, unlike Presidents, soldiers have an
additional cause for disqualification) and lead to absurd results (because soldiers who were
Twenty (20%) percent discount on admission fees charged by theaters, cinema houses and concert dishonorably discharged would be disqualified for acts that are less atrocious than that committed by
halls, circuses, carnivals and other similar places of culture, leisure and amusement; Marcos). Also, the AFP regulations would place Marcos in the same class as the other Philippine
Presidents when in fact he is a class of his own, sui generis. The other Presidents were never removed
Free medical and dental services and consultation in hospital and clinics anywhere in the country; by People Power Revolution and were never subject of laws declaring them to have committed human
rights violations. Thus, the intended burial would be an act of similarly treating persons who are
Exemption from the payment of tuition and matriculation fees in public or private schools, universities,
differently situated.
colleges and other educational institutions in any pre-school, baccalaureate or post graduate courses
such as or including course leading to the degree of Doctor of Medicine (MD), Bachelor of Laws (LLB),
Despite all these ostensibly persuasive arguments, the fact remains that Marcos was not convicted by
and Bachelor of Science in Nursing (BSN) or allied and similar courses; and cralawlawlibrary
final judgment of any offense involving moral turpitude. No less than the 1987 Constitution mandates
If interested and qualified, a quota is given to join the cadet corps of the Philippine Military Academy or that a person shall not be held to answer for a criminal offense without due process of law and that, "[i]n
otherwise priority for direct commission, call to active duty (CAD) and/or enlistment in regular force of all criminal prosecutions, the accused shall be presum innocent until the contrary is proved, and shall
the AFP. enjoy the right to be heard by himself and counsel, to be informed of the nature and cause of the
accusation against him, to have a speedy, impartial, and public trial, to meet the witnesses face to face,
On the other hand, in recognizing their patriotic services in times of war and peace for the cause of and to have compulsory process to secure the attendance of witnesses and the production of evidence
179
freedom and democracy; for the attainment of national unity, independence, and socioeconomic in his behalf." Even the U.N. principles on reparation and to combat impunity cited by petitioners
175 176 unequivocally guarantee the rights of the accused, providing that:ChanRoblesVirtualawlibrary
advancement; and for the maintenance of peace and order, R.A. No. 6948, as amended, grants
177
our veterans and their dependents or survivors with pension (old age, disability, total administrative
XIII. Rights of others
disability, and death) and non-pension (burial, education, hospitalization, and medical care and
treatment) benefits as well as provisions from the local governments. Under the law, the benefits may
be withheld if the Commission on Human Rights certifies to the AFP General Headquarters that the 27. Nothing in this document is to be construed as derogating from internationally or nationally
veteran has been found guilty by final judgment of a gross human rights violation while in the protected rights of others, in particular the right of an accused person to benefit from applicable
178
service, but this factor shall not be considered taken against his next of kin. chanrobleslaw standards of due process.

xxx
2. Disqualification under the AFP Regulations
PRINCIPLE 9. GUARANTEES FOR PERSONS IMPLICATED
Aside from being eligible for burial at the LNMB, Marcos possessed none of the disqualifications stated
in AFP Regulations G 161-375. He was neither convicted by final judgment of the offense involving
moral turpitude nor dishonorably separated/reverted/discharged from active military service. Before a commission identifies perpetrators in its report, the individuals concerned shall be entitled to
the following guarantees:
Petitioners, however, protest that a narrow interpretation of the AFP regulations disregards historical
context and the rule on statutory construction. They urge the Court to construe statutes not literally but chanRoblesvirtualLawlibrary(a) The commission must try to corroborate information implicating
according to their spirit and reason. individuals before they are named publicly;

It is argued that Marcos committed offenses involving moral turpitude for his gross human rights (b) The individuals implicated shall be afforded an opportunity to provide a statement setting forth their
violations, massive graft and corruption, and dubious military records, as found by foreign and local version of the facts either at a hearing convened by the commission while conducting its investigation or
courts as well as administrative agencies. By going into exile, he deliberately evaded liability for his through submission of a document equivalent to a right of reply for inclusion in the commission's file.
actions. And by allowing death to overtake him, he inevitably escaped the prospect of facing
accountability for his crimes. They also contend that his removal in the 1986 popular uprising is a clear To note, in the U.S., a person found to have committed a Federal or State capital crime (i.e., a crime
sign of his discharge from the AFP. The People Power Revolution was the direct exercise of the which a sentence of imprisonment for life or death penalty may be imposed) but who has not been
Filipinos' power to overthrow an illegitimate and oppressive regime. As a sovereign act, it necessarily convicted by reason of not being available for trial due to death or flight to avoid prosecution, may be
includes the power to adjudge him as dishonorably discharged from the AFP. ineligible for interment, inurnment, or memorialization in an Army national military cemetery.
180
Nevertheless, such ineligibility must still observe the procedures specified in § 553.21. chanrobleslaw service. The fact that the President is the Commander-in-Chief of the AFP under the 1987 Constitution
only enshrines the principle of supremacy of civilian authority over the military. Not being a military
The various cases cited by petitiOners, which were decided with finality by courts here and abroad, person who may be prosecuted before the court martial, the President can hardly be deemed
have no bearing in this case since they are merely civil in nature; hence, cannot and do not establish "dishonorably separated/reverted/discharged from the service" as contemplated by AFP Regulations G
moral turpitude. 161-375. Dishonorable discharge through a successful revolution is an extra-constitutional and direct
sovereign act of the people which is beyond the ambit of judicial review, let alone a mere administrative
Also, the equal protection clause is not violated. Generally, there is no property right to safeguard regulation.
because even if one is eligible to be buried at the LNMB, such fact would only give him or her
the privilege to be interred therein. Unless there is a favorable recommendation from the Commander-- It is undeniable that former President Marcos was forced out of office by the people through the so-
in-Chief, the Congress or the Secretary of National Defense, no right can be said to have ripen. Until called EDSA Revolution. Said political act of the people should not be automatically given a particular
then, such inchoate right is not legally demandable and enforceable. legal meaning other than its obvious consequence- that of ousting him as president. To do otherwise
would lead the Court to the treacherous and perilous path of having to make choices from multifarious
Assuming that there is a property right to protect, the requisites of equal protection clause are not inferences or theories arising from the various acts of the people. It is not the function of the Court, for
181
met. In this case, there is a real and substantial distinction between a military personnel and a former instance, to divine the exact implications or significance of the number of votes obtained in elections, or
182
President. The conditions of dishonorable discharge under the Articles of War attach only to the the message from the number of participants in public assemblies. If the Court is not to fall into the
members of the military. There is also no substantial distinction between Marcos and the three pitfalls of getting embroiled in political and oftentimes emotional, if not acrimonious, debates, it must
Philippine Presidents buried at the LNMB (Presidents Quirino, Garcia, and Macapagal). All of them remain steadfast in abiding by its recognized guiding stars - clear constitutional and legal rules - not by
were not convicted of a crime involving moral turpitude. In addition, the classification between a military the uncertain, ambiguous and confusing messages from the actions of the people.
personnel and a former President is germane to the purposes of Proclamation No. 208 and P.D. No.
1076. While the LNMB is a national shrine for militarymemorials, it is also an active military cemetery Conclusion
that recognizes the status or position held by the persons interred therein.

Likewise, Marcos was honorably discharged from military service. PVAO expressly recognized him as a In sum, there is no clear constitutional or legal basis to hold that there was a grave abuse of discretion
retired veteran pursuant to R.A. No. 6948, as amended. Petitioners have not shown that he was amounting to lack or excess of jurisdiction which would justify the Court to interpose its authority to
dishonorably discharged from military service under AFP Circular 17, Series of 1987 (Administrative check and override an act entrusted to the judgment of another branch. Truly, the President's discretion
Discharge Prior to Expiration of Term of Enlistment) for violating Articles 94, 95 and 97 of the Articles of is not totally unfettered. "Discretion is not a freespirited stallion that runs and roams wherever it pleases
183 184
War. The NHCP study is incomplete with respect to his entire military career as it failed to cite and but is reined in to keep it from straying. In its classic formulation, 'discretion is not unconfined and
186
include the official records of the AFP. vagrant' but 'canalized within banks that keep it from overflowing.'" At bar, President Duterte, through
the public respondents, acted within the bounds of the law and jurisprudence. Notwithstanding the call
With respect to the phrase "[p]ersonnel who were dishonorably separated/reverted/discharged from the of human rights advocates, the Court must uphold what is legal and just. And that is not to deny Marcos
service," the same should be viewed in light of the definition provided by AFP Regulations G 161-375 to of his rightful place at the LNMB. For even the Framers of our Constitution intend that full respect for
the term "active service" which is "[s]ervice rendered by a military person as a Commissioned Officer, human rights is available at any stage of a person's development, from the time he or she becomes a
187
enlisted man/woman, probationary officer, trainee or draftee in the Armed Forces of the person to the time he or she leaves this earth. chanrobleslaw
Philippines and service rendered by him/her as a civilian official or employee in the Philippine
Government prior to the date of his/her separation or retirement from the Armed Forces of the There are certain things that are better left for history - not this Court - to adjudge. The Court could only
Philippines, for which military and/or civilian service he/she shall have received pay from the Philippine do so much in accordance with the clearly established rules and principles. Beyond that, it is ultimately
Government, and/or such others as may be hereafter be prescribed by law as active service (PD 1638, for the people themselves, as the sovereign, to decide, a task that may require the better perspective
185
as amended)." To my mind, the word "service" should be construed as that rendered by a military that the passage of time provides. In the meantime, the country must mov'e on and let this issue
person in the AFP, including civil service, from the time of his/her commission, enlistment, probation, rest.
training or drafting, up to the date of his/her separation or retirement from the AFP. Civil service after
honorable separation and retirement from the AFP is outside the context of "service" under AFP WHEREFORE, PREMISES CONSIDERED, the petitions are DISMISSED. Necessarily, the Status Quo
Regulations G 161-375. Ante Order is hereby LIFTED.

Hence, it cannot be conveniently claimed that Marcos' ouster from the presidency during the EDSA
Revolution is tantamount to his dishonorable separation, reversion or discharge from the military
Republic of the Philippines OF THE ORDER OF NATIONAL ARTISTS ON RESPONDENTS GUIDOTE-ALVAREZ, CAPARAS,
SUPREME COURT MORENO AND MANOSA, Respondents.
Manila
DECISION
EN BANC
LEONARDO-DE CASTRO, J.:
G.R. No. 189028 July 16, 2013
Art has traditionally been viewed as the expression of everything that is true, good and beautiful. As
NATIONAL ARTIST FOR LITERATURE VIRGILIO ALMARIO, NATIONAL ARTIST FOR such, it is perceived to evoke and produce a spirit of harmony. Art is also considered as a civilizing
LITERATURE BIENVENIDO LUMBERA, NATIONAL ARTIST FOR VISUAL ARTS (PAINTING) force, a catalyst of nation-building. The notion of art and artists as privileged expressions of national
BENEDICTO CABRERA, NATIONAL ARTIST FOR VISUAL ARTS (SCULPTURE) NAPOLEON culture helped shape the grand narratives of the nation and shared symbols of the people. The artist
ABUEVA, NATIONAL ARTIST FOR VISUAL ARTS (PAINTING AND SCULPTURE) ARTURO LUZ, does not simply express his/her own individual inspiration but articulates the deeper aspirations of
2
NATIONAL ARTIST FOR PRODUCTION DESIGN SALVADOR BERNAL, UNIVERSITY history and the soul of the people. The law recognizes this role and views art as something that
3
PROFESSOR EMERITUS GEMINO ABAD, DEAN MARVIC M.V.F. LEONEN (UP COLLEGE OF "reflects and shapes values, beliefs, aspirations, thereby defining a people’s national identity." If unduly
LAW), DEAN DANILO SILVESTRE (UP COLLEGE OF ARCHITECTURE), DEAN ROLAND politicized, however, art and artists could stir controversy and may even cause discord, as what
TOLENTINO (UP COLLEGE OF MASS COMMUNICATION), PROF. JOSE DALISAY, DR. ANTON happened in this case.
JUAN, DR. ALEXANDER CORTEZ, DR. JOSE NEIL GARCIA, DR. PEDRO JUN CRUZ REYES,
PROF. JOSE CLAUDIO GUERRERO, PROF. MICHAEL M. COROZA, PROF. GERARD LICO, PROF. The Antecedents
VERNE DE LA PENA, PROF. MARIAN ABUAN, PROF. THEODORE O. TE, DR. CRISTINA
PANTOJA-HIDALGO, PROF. JOSE WENDELL CAPILI, PROF. SIR ANRIAL TIATCO, PROF. History of the Order of National Artists
NICOLO DEL CASTILLO, PROF. HORACIO DUMANLIG, PROF. DANTON REMOTO, PROF. 4
On April 27, 1972, former President Ferdinand E. Marcos issued Proclamation No. 1001 and, upon
PRISCELINA PATAJOLEGASTO, PROF. BELEN CALINGACION, PROF. AMIEL Y. LEONARDIA,
recommendation of the Board of Trustees of the Cultural Center of the Philippines (CCP), created the
PROF. VIM NADERA, PROF. MARILYN CANTA, PROF. CECILIA DELA PAZ, ROF. CHARLSON
category of Award and Decoration of National Artist to be awarded to Filipinos who have made distinct
ONG, PROF. CLOD MARLON YAMBAO, PROF. KENNETH JAMANDRE, PROF. JETHRO
contributions to arts and letters. In the same issuance, Fernando Amorsolo was declared as the first
JOAQUIN, ATTY. F.D. NICOLAS B. PICHAY, ATTY. ROSE BEATRIX ANGELES, MR. FERNANDO
National Artist.
JOSEF, MS. SUSAN S. LARA, MR. ALFRED YUSON, MS. JING PANGANIBANMENDOZA, MR.
ROMULO BAQUIRAN, JR., MR. CARLJOE JAVIER, MS. REBECCA T. ANONUEVO, MR. JP 5
On May 15, 1973, Proclamation No. 1144 was issued. It amended Proclamation No. 1001 "by creating
ANTHONY D. CUNADA, MS. LEAH NAVARRO, MR. MARK MEILLY, MR. VERGEL O. SANTOS,
a National Artists Awards Committee" that would "administer the conferment of the category of National
MR. GIL OLEA MENDOZA, MR. EDGAR C. SAMAR, MS. CHRISTINE BELLEN, MR. ANGELO R. Artist" upon deserving Filipino artists. The Committee, composed of members of the Board of Trustees
LACUESTA, MS. ANNA MARIA KATIGBAKLACUESTA, MR. LEX LEDESMA, MS. KELLY of the CCP, was tasked to "draft the rules to guide its deliberations in the choice of National Artists, to
PERIQUET, MS. CARLA PACIS, MR. J. ALBERT GAMBOA, MR. CESAR EVANGELISTA BUENDIA,
the end that those who have created a body of work in the arts and letters capable of withstanding the
MR. PAOLO ALCAZAREN, MR. ALWYN C. JAVIER, MR. RAYMOND MAGNO GARLITOS, MS.
test of time will be so recognized."
GANG BADOY, MR. LESLIE BOCOBO, MS. FRANCES BRETANA, MS. JUDITH TORRES, MS.
JANNETTE PINZON, MS. JUNE POTICAR-DALISAY, MS. CAMILLE DE LA ROSA, MR. JAMES The authority of the National Artists Awards Committee to administer the conferment of the National
LADIORAY, MR. RENATO CONSTANTINO, JR., and CONCERNED ARTISTS OF THE PHILIPPINES 6
Artist Award was again reiterated in Presidential Decree No. 208 issued on June 7, 1973.
(CAP), Petitioners,
vs. On April 3, 1992, Republic Act No. 7356, otherwise known as the Law Creating the National
THE EXECUTIVE SECRETARY, THE SECRETARY OF THE DEPARTMENT OF BUDGET AND Commission for Culture and the Arts, was signed into law. It established the National Commission for
MANAGEMENT, THE CULTURAL CENTER OF THE PHILIPPINES, THE NATIONAL COMMISSION Culture and the Arts (NCCA) and gave it an extensive mandate over the development, promotion and
ON CULTURE AND THE ARTS, MS. CECILE GUIDOTE-ALVAREZ, MR. CARLO MAGNO JOSE preservation of the Filipino national culture and arts and the Filipino cultural heritage. The NCCA was
1
CAPARAS, MR. JOSE MORENO, MR. FRANCISCO MANOSA, AND ALL PERSONS, PUBLIC AND tasked with the following:
PRIVATE, ACTING UNDER THEIR INSTRUCTIONS, DIRECTION, CONTROL AND SUPERVISION
IN RELATION TO THE CONFERMENT OF THE ORDER OF THE NATIONAL ARTIST AND THE Sec. 8. The Commission. – A National Commission for Culture and Arts is hereby created to formulate
RELEASE OF FUNDS IN RELATION TO THE CONFERMENT OF THE HONORS AND PRIVILEGES policies for the development of culture and arts; implement these policies in coordination with affiliated
cultural agencies; coordinate the implementation of programs of these affiliated agencies; administer
the National Endowment Fund for Culture and Arts (NEFCA); encourage artistic creation within a
climate of artistic freedom; develop and promote the Filipino national culture and arts; and preserve (b) should have extensive knowledge in his field(s) and his views on Philippine art and culture must be
Filipino cultural heritage. The Commission shall be an independent agency. It shall render an annual national in perspective;
report of its activities and achievements to the President and to Congress.
(c) should be a recognized authority in the study or research of Philippine art and culture;
Among the specific mandates of the NCCA under Republic Act No. 7356 is to "extend recognition of
artistic achievement through awards, grants and services to artists and cultural groups which contribute (d) must be willing to devote sufficient time and effort to the work of the Council;
7
significantly to the Filipino’s cultural legacy." In connection with this mandate, the NCCA is vested with
the power to "advise the President on matters pertaining to culture and the arts, including the creation of (e) must be willing to sign a non-disclosure statement in order to safeguard the confidentiality of the
a special decoration or award, for persons who have significantly contributed to the development and deliberations;
8
promotion of Philippine culture and arts."
(f) must not have been convicted with finality of any crime by a court of justice or dismissed for cause by
As both the CCP Board of Trustees and the NCCA have been mandated by law to promote, develop any organization, whether public or private.
and protect the Philippine national culture and the arts, and authorized to give awards to deserving
4.7. The National Artist Award Council of Experts shall be composed of a maximum of seven (7)
Filipino artists, the two bodies decided to team up and jointly administer the National Artists
9 members each of the seven (7) areas/disciplines. The living National Artists will automatically become
Award. Thereafter, they reviewed the guidelines for the nomination, selection and administration of the
members in addition to the forty-nine (49) selected members. These members will constitute the first
National Artists Award. Pursuant to their respective powers to draft and promulgate rules, regulations
deliberation panel and will be invited to evaluate the nominations and materials submitted by the
and measures to guide them in their deliberations in the choice of National Artists, the CCP and NCCA
10 Special Research Group.
adopted the following revised guidelines in September 2007 :
4.8. Any member of the Council of Experts who is nominated or related to a nominee up to the fourth
4. ADMINISTRATION OF THE AWARD
degree of consanguinity or affinity shall inhibit himself/herself from the deliberation process. Likewise,
4.1. The National Commission for Culture and the Arts (NCCA) shall plan, organize and implement the any member may decline to participate in the deliberation for any reason or may be removed for just
Order of National Artists in coordination with the Cultural Center of the Philippines (CCP). cause upon recommendation to the NCCA Board by at least two thirds (2/3) of the members; in which
case, the National Artist Award Secretariat shall again select the replacements for those who decline or
4.2. It shall enlist the support and cooperation of private sector experts from the various fields of art to resigned until the first deliberation panel is completed.
ensure that the awards are implemented in a successful and impartial manner.
4.9. The list of nominated members of the National Artist Award Council of Experts shall be reviewed by
4.3. The National Artist Award Secretariat shall commission art experts to form a Special Research the National Artist Award Secretariat as needed, for purposes of adding new members or replacements.
Group who shall verify information submitted on nominees and provide essential data.
4.10. The members of the National Artist Award Council of Experts shall serve for a fixed term of three
They shall be selected for their specialization and familiarity with the works and accomplishments of (3) years.
nominated artists.
5. CRITERIA FOR SELECTION
4.4. The Special Research Group shall be composed of ten (10) to twenty (20) members who have
expertise in one or more fields or disciplines. The Order of National Artists shall be given to:

4.5. The National Artist Award Council of Experts shall be created before or during the nomination 5.1 Living artists who are Filipino citizens at the time of nomination, as well as those who died after the
period. It is tasked to screen nominees and recommend to the NCCA and CCP Boards the candidates establishment of the award in 1972 but were Filipino citizens at the time of their death.
for the Order of National Artists. It shall be composed of highly regarded peers, scholars, (including
5.2 Artists who through the content and form of their works have contributed in building a Filipino sense
cultural philosophers and historians), academicians, researchers, art critics, and other knowledgeable
of nationhood.
individuals. A wider age-range of experts who would have first-hand knowledge of achievements of
nominees shall be considered. 5.3. Artists who have pioneered in a mode of creative expression or style, thus, earning distinction and
making an impact on succeeding generations of artists.
4.6. The selection of the members of the National Artist Award Council of Experts shall be based on the
following criteria: 5.4. Artists who have created a substantial and significant body of works and/or consistently displayed
excellence in the practice of their art form thus enriching artistic expression or style.
(a) should have achieved authority, credibility and track record in his field(s) of expertise;
5.5 Artists who enjoy broad acceptance through: The cover letter shall be accompanied by a Board Resolution approving the nominee concerned with
the said resolution signed by the organization President and duly certified by the Board Secretary.
5.5.1. prestigious national and/or international recognition, such as the Gawad CCP Para sa Sining,
CCP Thirteen Artists Award and NCCA Alab ng Haraya 6.6.2. A duly accomplished nomination form;

5.5.2. critical acclaim and/or reviews of their works 6.6.3. A detailed curriculum vitae of the nominee;

5.5.3. respect and esteem from peers. 6.6.4. A list of the nominee’s significant works categorized according to the criteria;

6. NOMINATION PROCEDURE 6.6.5. The latest photograph (color or black and white) of the nominee, either 5" x 7" or 8" x 11";

6.1. The National Artist Award Secretariat shall announce the opening of nominations through media 6.6.6. Pertinent information materials on the nominee’s significant works (on CDs, VCDs and DVDs);
releases and letters to qualified organizations.
6.6.7. Copies of published reviews;
6.2. Candidates may be nominated under one or more of the following categories:
6.6.8. Any other document that may be required.
6.2.1. Dance – choreography, direction and/or performance.
6.7. Nominations received beyond the announced deadline for the submission of nominations shall not
6.2.2. Music – composition, direction, and/or performance. be considered.

6.2.3. Theater – direction, performance and/or production design. 6.8. The National Artist Award Secretariat shall announce the opening of nominations through media
releases.
6.2.4. Visual Arts – painting, sculpture, printmaking, photography, installation art, mixed media works,
illustration, comics/komiks, graphic arts, performance art and/or imaging. 6.9. All inquiries and nominations shall be submitted to

6.2.5. Literature – poetry, fiction (short story, novel and play); non-fiction (essay, journalism, literary The NATIONAL ARTIST AWARD SECRETARIAT
criticism and historical literature).
Office of the Artistic Director Cultural Center of the Philippines Roxas Boulevard, 1300 Pasay City or
6.2.6. Film and Broadcast Arts – direction, writing, production design, cinematography, editing, camera The NATIONAL ARTIST AWARD SECRETARIAT Office of the Deputy Executive Director National
work, and/or performance. Commission for Culture and the Arts 633 General Luna Street, Intramuros, Manila

6.2.7. Architecture, Design and Allied Arts – architecture design, interior design, industrial arts design, 7. SCREENING AND SELECTION PROCESS
landscape architecture and fashion design.
7.1. The National Artist Award Secretariat shall pre-screen the nominees based on technical guideline
6.3. Nominations for the Order of National Artists may be submitted by government and non- items 5.1, 6.2, 6.3, 6.4, 6.5 and 6.6. The pre-screening shall not be based on the accomplishments and
government cultural organizations and educational institutions, as well as private foundations and merits of the nominee.
councils.
7.2. The Special Research Group shall accomplish its task within six (6) months. The main objective is
6.4. Members of the Special Research Group, as well as agencies attached to the NCCA and CCP shall to verify the validity of the data, and evaluate the quality, true value and significance of works according
not submit nominations. to the criteria. It shall come up with the updated and comprehensive profiles of nominees reflecting their
most outstanding achievements.
6.5. NCCA and CCP Board members and consultants and NCCA and CCP officers and staff are
automatically disqualified from being nominated. 7.3. The National Artist Award Secretariat will meet to review the list of nominees for oversights.
Consequently, deserving nominees shall be added to the list.
6.6. Nominations shall be accepted only when these are submitted in writing and with proper supporting
documentation, as follows: 7.4. The first deliberation panel (Council of Experts) shall be intra-disciplinary. The panelists shall be
grouped according to their respective fields of expertise or disciplines to shortlist the nominees in their
6.6.1. A cover letter signed by the head or designated representative of the nominating organization. disciplines or categories for presentation to the second deliberation panel.
7.5. The second deliberation panel shall be composed of a different set of experts from the first another layer to the selection process to involve and allow the participation of more members of the arts
deliberation panel three (3) experts each of the seven (7) areas/discipline and may include members and culture sector of the Philippines in the selection of who may be proclaimed a National Artist.
from varying backgrounds such as critics and academicians. The achievements of each shortlisted
nominee shall be presented by one designated member of Council of Experts. Then panel deliberates On September 19, 2003, Executive Order No. 236, s. 2003, entitled Establishing the Honors Code of
and ranks the shortlisted nominees according to the order of precedence following the set criteria of the the Philippines to Create an Order of Precedence of Honors Conferred and for Other Purposes, was
Order of National Artists. In extreme cases, the Second Deliberation may add new names to the lists. issued. The National Artists Award was renamed the Order of National Artists and raised to the level of
a Cultural Order, fourth in precedence among the orders and decorations that comprise the Honors of
7.6. The second deliberation panel may recommend not to give award in any category if no nominee is the Philippines. Executive Order No. 236, s. 2003, recognizes the vital role of the NCCA and the CCP in
found deserving. The number of awardees shall also depend on the availability of funds. All decisions identifying Filipinos who have made distinct contributions to arts and letters and states that the National
and recommendations shall be in writing. Artist recognition is conferred "upon the recommendation of the Cultural Center of the Philippines and
12
the National Commission for Culture and the Arts." Executive Order No. 236, s. 2003, further created
7.7. The recommendations from the Second Deliberation Panel of the National Artist Award Council of a Committee on Honors to "assist the President in evaluating nominations for recipients of
Experts shall then be presented to the joint boards of NCCA and CCP for final selection. The presentors 13
Honors," including the Order of National Artists, and presidential awards. The Committee on Honors
shall prepare their presentation in writing together with an audio-visual presentation or powerpoint has been allowed to "authorize relevant department or government agencies to maintain Honors and/or
presentation. Written interpellations/opinions will be accepted from selected critics. The review shall be 14
Awards Committees to process nominations for Honors and/or Presidential Awards." In this
based on the ranking done by the Second Deliberation. The voting shall be across disciplines. The 15
connection, Section 2.4(A) of the Implementing Rules and Regulations of Executive Order No. 236, s.
National Artists will be given the option whether to vote on all categories or on his/her particular 2003, states:
discipline.
2.4: Awards Committees
7.8. Proxy votes will not be allowed in the Selection Process. Designation of permanent representatives
of agencies should be made at the outset to make them regular Board members of NCCA and thus, There shall be two types of awards committees: the Committee on Honors and the various awards
may be allowed to cast votes. committees in the various units of the government service.

7.9. The list of awardees shall be submitted to the President of the Republic of the Philippines for A. The Committee on Honors
confirmation, proclamation and conferral.
The Committee on Honors serves as a National Awards Committee. It is composed of the following:
8. PRESENTATION OF THE AWARDS
The Executive Secretary, Chairman
8.1. The Order of National Artists shall not be conferred more frequently than every three (3) years.
The Secretary of Foreign Affairs, Vice-Chairman
8.2. The Order of National Artists shall be conferred by the President of the Philippines on June 11 or
any appropriate date in fitting ceremonies to be organized by the National Artist Secretariat. Head, Presidential Management Staff, member

8.3. The medallion of the Order of National Artists and citation shall be given to the honoree during the Presidential Assistant for Historical Affairs, member
conferment ceremony. The cash award of ₱100,000.00 in cheque shall be given immediately after the
ceremony or at another time and place as requested by the honoree. Chief of Presidential Protocol, member

8.4. A posthumous conferral consisting of the medallion and citation shall be given to the family or legal Chief of Protocol, DFA, member
heir/s of the honoree. The cash award of ₱75,000.00 in cheque shall be given to the honoree’s legal
All nominations from the various awards committees must be submitted to the Committee on Honors via
heir/s or a representative designated by the family immediately after the ceremony or at another time
the Chancellery of Philippine Orders and State Decorations. The Chancellery shall process nominations
and place as requested by the family. (Emphases supplied.)
for the consideration of the Committee on Honors. The Committee on Honors shall screen and
In 1996, the NCCA and the CCP created a National Artist Award Secretariat composed of the NCCA recommend these nominations to the President.
Executive Director as Chairperson, the CCP President as Vice-Chairperson, and the NCCA Deputy
The Committee on Honors shall, as a general rule, serve as a screening committee to ensure that
Executive Director, the CCP Vice-President/Artistic Director, the NCCA National Artist Award Officer
nominations received from the various awards committees meet two tests: that there has not been an
and the CCP National Artist Award Officer as members. They also centralized with the NCCA all
financial resources and management for the administration of the National Artists Award. They added
abuse of discretion in making the nomination, and that the nominee is in good standing. Should a May 6, 2009
nomination meet these criteria, a recommendation to the President for conferment shall be made.
Her Excellency GLORIA MACAPAGAL-ARROYO
The President of the Philippines takes the recommendations of the Committee on Honors in the highest President of the Philippines
consideration when making the final decision on the conferment of awards. (Emphasis supplied.) Malacañan Palace, Manila

Executive Order No. 435, s. 2005, entitled Amending Section 5(IV) of Executive Order No. 236 Entitled Subject: 2009 Order of National Artist Awardees
"Establishing the Honors Code of the Philippines to Create an Order of Precedence of Honors
Conferred and for Other Purposes" was subsequently issued on June 8, 2005. It amended the wording Dear President Arroyo:
of Executive Order No. 236, s. 2003, on the Order of National Artists and clarified that the NCCA and
the CCP "shall advise the President on the conferment of the Order of National Artists." We are respectfully submitting a recommendation of the NCCA Board of Trustees and CCP Board of
Trustees for the Proclamation of the following as 2009 Order of National Artists:
Controversy Surrounding the 2009
Order of National Artists 1. Mr. MANUEL CONDE+ (Posthumous) – Film and Broadcast Arts

Petitioners alleged that on January 30, 2007, a joint meeting of the NCCA Board of Commissioners and 2. Dr. RAMON SANTOS – Music
the CCP Board of Trustees was held to discuss, among others, the evaluation of the 2009 Order of
3. Mr. LAZARO FRANCISCO+ (Posthumous) – Literature
National Artists and the convening of the National Artist Award Secretariat. The nomination period was
set for September 2007 to December 31, 2007, which was later extended to February 28, 2008. The 4. Mr. FEDERICO AGUILAR-ALCUAZ – Visual Arts
16
pre-screening of nominations was held from January to March 2008.
17 18
The above persons were identified by experts in the various fields of arts and culture, including living
On April 3, 2009, the First Deliberation Panel met. A total of 87 nominees were considered during National Artists. An intensive selection process was observed following established practice. In the past,
19
the deliberation and a preliminary shortlist of 32 names was compiled. awards were presented by the President at a Ceremony held at the Malacañan Palace followed by a
program called "Parangal" at the Cultural Center of the Philippines. We also propose to continue with
On April 23, 2009, the Second Deliberation Panel purportedly composed of an entirely new set of
20 past practice of celebrating the life and works of the four (4) Order of National Artists through an exhibit
Council of Experts met and shortlisted 13 out of the 32 names in the preliminary shortlist. On May 6,
that will open and a commemorative publication that will be released on the day of the proclamation.
2009, the final deliberation was conducted by the 30-member Final Deliberation Panel comprised of the
21
CCP Board of Trustees and the NCCA Board of Commissioners and the living National Artists. From We respectfully suggest, subject to Her Excellency’s availability, that the Proclamation be on June 11,
22
the 13 names in the second shortlist, a final list of four names was agreed upon. The final list, 2009, if possible at the Malacañan Palace.
according to rank, follows:
Thank you for your kind attention.
Name Art Field/Category Number of Votes
Very respectfully yours,

Manuel Conde (+) Film and Broadcast Arts (Film) 26 (Sgd.)


VILMA L. LABRADOR
Ramon Santos Music 19 Chairman
National Commission for Culture and the Arts

Lazaro Francisco (+) Literature 15 (Sgd.)


NESTOR O. JARDIN
Federico Aguilar-Alcuaz Visual Arts 15 President and Artistic Director
24
Cultural Center of the Philippines
On May 6, 2009, a letter, signed jointly by the Chairperson of the NCCA, Undersecretary Vilma According to respondents, the aforementioned letter was referred by the Office of the President to the
Labrador, and the President and Artistic Director of the CCP, Mr. Nestor Jardin, was sent to the Committee on Honors. Meanwhile, the Office of the President allegedly received nominations from
23
President. The letter stated, thus:
various sectors, cultural groups and individuals strongly endorsing private respondents Cecile Guidote- The cultural workers, academics and CAP claim to be Filipinos who are deeply concerned with the
Alvarez, Carlo Magno Jose Caparas, Francisco Mañosa and Jose Moreno. The Committee on Honors preservation of the country’s rich cultural and artistic heritage. As taxpayers, they are concerned about
33
purportedly processed these nominations and invited resource persons to validate the qualifications and the use of public monies for illegal appointments or spurious acts of discretion.
25
credentials of the nominees.
All of the petitioners claim that former President Macapagal-Arroyo gravely abused her discretion in
The Committee on Honors thereafter submitted a memorandum to then President Gloria Macapagal- disregarding the results of the rigorous screening and selection process for the Order of National Artists
Arroyo recommending the conferment of the Order of National Artists on the four recommendees of the and in substituting her own choice for those of the Deliberation Panels. According to petitioners, the
NCCA and the CCP Boards, as well as on private respondents Guidote-Alvarez, Caparas, Mañosa and President’s discretion to name National Artists is not absolute but limited. In particular, her discretion on
Moreno. Acting on this recommendation, Proclamation No. 1823 declaring Manuel Conde a National the matter cannot be exercised in the absence of or against the recommendation of the NCCA and the
Artist was issued on June 30, 2009. Subsequently, on July 6, 2009, Proclamation Nos. 1824 to 1829 CCP. In adding the names of respondents Caparas, Guidote-Alvarez, Mañosa and Moreno while
were issued declaring Lazaro Francisco, Federico AguilarAlcuaz and private respondents Guidote- dropping Dr. Santos from the list of conferees, the President’s own choices constituted the majority of
Alvarez, Caparas, Mañosa and Moreno, respectively, as National Artists. This was subsequently the awardees in utter disregard of the choices of the NCCA and the CCP and the arts and culture
26
announced to the public by then Executive Secretary Eduardo Ermita on July 29, 2009. community which were arrived at after a long and rigorous process of screening and deliberation.
Moreover, the name of Dr. Santos as National Artist for Music was deleted from the final list submitted
Convinced that, by law, it is the exclusive province of the NCCA Board of Commissioners and the CCP by the NCCA and the CCP Boards without clearly indicating the basis thereof. For petitioners, the
Board of Trustees to select those who will be conferred the Order of National Artists and to set the President’s discretion to name National Artists cannot be exercised to defeat the recommendations
standard for entry into that select group, petitioners instituted this petition for prohibition, certiorari and made by the CCP and NCCA Boards after a long and rigorous screening process and with the benefit of
injunction (with prayer for restraining order) praying that the Order of National Artists be conferred on expertise and experience. The addition of four names to the final list submitted by the Boards of the
Dr. Santos and that the conferment of the Order of National Artists on respondents Guidote-Alvarez, CCP and the NCCA and the deletion of one name from the said list constituted a substitution of
Caparas, Mañosa and Moreno be enjoined and declared to have been rendered in grave abuse of judgment by the President and a unilateral reconsideration without clear justification of the decision of
27 34
discretion. the First, Second and Final Deliberation Panels composed of experts.
28 29
In a Resolution dated August 25, 2009, the Court issued a status quo order enjoining "public Petitioners further argue that the choice of respondent GuidoteAlvarez was illegal and unethical
respondents" "from conferring the rank and title of the Order of National Artists on private respondents; because, as the then Executive Director of the NCCA and presidential adviser on culture and arts, she
from releasing the cash awards that accompany such conferment and recognition; and from holding the 35
was disqualified from even being nominated. Moreover, such action on the part of the former
acknowledgment ceremonies for recognition of the private respondents as National Artists." President constituted grave abuse of discretion as it gave preferential treatment to respondent Guidote-
Alvarez by naming the latter a National Artist despite her not having been nominated and, thus, not
What is the nature and scope of the power of the President to confer the Order of the National Artists subjected to the screening process provided by the rules for selection to the Order of National Artists.
and how should it be exercised? This is the essential issue presented in this case. It will determine Her inclusion in the list by the President represented a clear and manifest favor given by the President
whether the proclamation of respondents as National Artists is valid. Preliminary procedural issues on in that she was exempted from the process that all other artists have to undergo. According to
30
the standing of the petitioners and the propriety of the remedies taken, however, call for resolution as petitioners, it may be said that the President used a different procedure to qualify respondent Guidote-
a prerequisite to the discussion of the main question. Alvarez. This was clearly grave abuse of discretion for being manifest and undue bias violative of the
36
equal protection clause.
Contention of the Parties
Respondent Caparas refutes the contention of the petitioning National Artists and insists that there
A perusal of the pleadings submitted by the petitioners reveals that they are an aggrupation of at least
could be no prejudice to the latter. They remain to be National Artists and continue to receive the
three groups, the National Artists, cultural workers and academics, and the Concerned Artists of the
emoluments, benefits and other privileges pertaining to them by virtue of that honor. On the other hand,
Philippines (CAP). The National Artists assert an "actual as well as legal interest in maintaining the
31 all the other petitioners failed to show any material and personal injury or harm caused to them by the
reputation of the Order of National Artists." In particular, they invoke their right to due process not to
conferment of the Order of National Artists on respondents Guidote-Alvarez, Caparas, Mañosa and
have the honor they have been conferred with diminished by the irregular and questionable conferment
Moreno. The rule on standing may not be relaxed in favor of the petitioners as no question of
of the award on respondents Guidote-Alvarez, Caparas, Mañosa and Moreno. For petitioners, this 37
constitutionality has been raised and no issue of transcendental importance is involved.
would adversely affect their right to live a meaningful life as it detracts not only from their right to enjoy
32
their honor as a fruit of their lifelong labor but also from the respect of their peers. Respondent Caparas further argues that the remedies of prohibition and injunction are improper as the
act sought to be enjoined – the declaration of respondents Guidote-Alvarez, Caparas, Mañosa and
Moreno as National Artists – had already been consummated. In particular, respondent Caparas was
38
already proclaimed National Artist through Proclamation No. 1827 issued on July 6, 2009.
On the merits, respondent Caparas contends that no grave abuse of discretion attended his The OSG also assailed the former President’s choice of respondent Guidote-Alvarez for being contrary
54
proclamation as National Artist. The former President considered the respective recommendations of to Republic Act No. 7356. Section 11 of the said law provides:
the NCCA and the CCP Boards and of the Committee on Honors in eventually declaring him (Caparas)
as National Artist. The function of the NCCA and the CCP Boards is simply to advise the President. The Sec. 11. Membership Restrictions. – During his/her term as member of the Commission, a
award of the Order of National Artists is the exclusive prerogative of the President who is not bound in Commissioner shall not be eligible for any grant, or such other financial aid from the Commission as an
any way by the recommendation of the NCCA and the CCP Boards. The implementing rules and individual: Provided, however, That he/she may compete for grants and awards on the same level as
regulations or guidelines of the NCCA cannot restrict or limit the exclusive power of the President to other artists one (1) year after his/her term shall have expired.
39
select the recipients of the Order of National Artists.
The omission of the word "award" in the first portion of the above provision appears to be unintentional
40 as shown by the proviso which states that a member may compete for grants and awards only one year
For her part, in a letter dated March 11, 2010, respondent Guidote-Alvarez manifested that she was
waiving her right to file her comment on the petition and submitted herself to the Court’s discretion and after his or her term shall have expired. As such, respondent Guidote-Alvarez is restricted and
55
wisdom. disqualified from being conferred the 2009 Order of National Artists.

Respondent Mañosa manifested that his creations speak for themselves as his contribution to Filipino The Court’s Ruling
cultural heritage and his worthiness to receive the award. Nonetheless, he expressed his conviction that
the Order of National Artists is not a right but a privilege that he would willingly relinquish should he be Standing of the Petitioners
41
found not worthy of it.
Standing is the determination of whether a specific person is the proper party to bring a matter to the
56
Respondent Moreno did not file any pleading despite being given several opportunities to do so. Hence, court for adjudication. The gist of the question of standing is whether a party alleges such personal
the Court dispensed with his pleadings.
42 stake in the outcome of the controversy as to assure that concrete adverseness which sharpens the
presentation of issues upon which the court depends for illumination of difficult constitutional
57
In a Resolution dated July 12, 2011, this Court gave due course to the petition and required the parties questions.
43
to file their respective memoranda. Respondent Caparas filed his memorandum on September 8,
44 45
2011, the CCP filed its memorandum on September 19, 2011, respondent Mañosa on September The parties who assail the constitutionality or legality of a statute or an official act must have a direct
46
20, 2011, and the Office of the Solicitor General filed a manifestation stating that it is adopting its and personal interest. They must show not only that the law or any governmental act is invalid, but also
47
comment as its memorandum on September 21, 2011. Respondent Moreno failed to file a that they sustained or are in immediate danger of sustaining some direct injury as a result of its
48
Memorandum, hence, the Court resolved to dispense with the same. Petitioners filed their enforcement, and not merely that they suffer thereby in some indefinite way. They must show that they
Memorandum on May 14, 2012.
49 have been or are about to be denied some right or privilege to which they are lawfully entitled or that
they are about to be subjected to some burdens or penalties by reason of the statute or act complained
58
On the other hand, the original position of the Office of the Solicitor General (OSG) was similar to that of of.
50 51
respondent Caparas. In a subsequent manifestation, however, the OSG stated that the current
Board of Commissioners of the NCCA agree with the petitioners that the President cannot honor as a In this case, we find that the petitioning National Artists will be denied some right or privilege to which
National Artist one who was not recommended by the joint Boards of the NCCA and the CCP. The they are entitled as members of the Order of National Artists as a result of the conferment of the award
implementing rules and regulations of Executive Order No. 236, s. 2003, recognized the binding on respondents Guidote-Alvarez, Caparas, Mañosa and Moreno. In particular, they will be denied the
character of the recommendation of the NCCA and the CCP Boards and limited the authority of the privilege of exclusive membership in the Order of National Artists.
Committee on Honors to the determination that (1) there has been no grave abuse of discretion on the 59
In accordance with Section 2(a) of Executive Order No. 236, s. 2003, the Order of National Artists is
part of the NCCA and the CCP Boards in making the nomination, and (2) the nominee is in good
"an exclusive association of honored individuals." To ensure the exclusivity of the membership in the
standing. Where a nomination meets the said two criteria, a recommendation to the President to confer
52 Order, a rigid nomination and screening process has been established with different sets of renowned
the award shall be made.
artists and respected art critics invited to sit as the Council of Experts for the First and Second
The OSG further argued that, while the President exercises control over the NCCA and the CCP, the Deliberation Panels. Moreover, all living National Artists are given a voice on who should be included in
President has the duty to faithfully execute the laws, including the NCCA-CCP guidelines for selection their exclusive club as they automatically become members of the Final Deliberation Panel that will vote
of National Artists and the implementing rules of Executive Order No. 236, s. 2003. Moreover, the laws on who should be included in the final list to be submitted to the President for conferment of the Order
recognize the expertise of the NCCA and the CCP in the arts and tasked them to screen and select the of National Artists. To allow the untrammeled discretion and authority of the President to confer the
artists to be conferred the Order of National Artists. Their mandate is clear and exclusive as no other Order of National Artists without regard to the stringent screening and rigorous selection process
agency possesses such expertise.
53 established by the NCCA and the CCP will diminish, if not negate, the exclusive nature of the said
Order. It will unduly subject the selection and conferment of the Order of National Artists to politics It has been held that the remedies of prohibition and injunction are preventive and, as such, cannot be
66
rather than to principles and procedures. It will subvert the transparent and rigorous process and allow availed of to restrain an act that is already fait accompli. Where the act sought to be prohibited or
67
entry to the exclusive Order of National Artists through a secret backdoor of lobbying, back channeling enjoined has already been accomplished or consummated, prohibition or injunction becomes moot.
and political accommodation.
Nevertheless, even if the principal issue is already moot, this Court may still resolve its merits for the
Among the other petitioners, Prof. Gemino Abad presents a unique valid personal and substantial future guidance of both bench and bar. Courts will decide a question otherwise moot and academic if it
68
interest. Like respondents Caparas, Mañosa and Moreno, he was among the 87 nominees for the 2009 is "capable of repetition, yet evading review."
Order of National Artists. Like respondent Moreno, he made it to the preliminary shortlist. As he did not
69
make it to the second shortlist, he was not considered by the Final Deliberation Panel, more so by the It is an opportune time for the Court to assert its role as republican schoolmaster, a teacher in a vital
70
former President. national seminar. There are times when the controversy is of such character that, to prevent its
recurrence and to assure respect for constitutional limitations, this Court must pass on the merits of a
71
It should be recalled too that respondent Guidote-Alvarez was disqualified to be nominated for being the case. This is one such case. More than being a teaching moment, this is not the first time that the
72
Executive Director of the NCCA at that time while respondents Mañosa and Caparas did not make it to Order of National Artists was conferred in the manner that is being assailed in this case. If not
the preliminary shortlist and respondent Moreno was not included in the second shortlist. Yet, the four of addressed here and now, there is great probability that the central question involved in this case will
them were treated differently and considered favorably when they were exempted from the rigorous haunt us again in the future. Every President may invoke absolute presidential prerogative and thrust
screening process of the NCCA and the CCP and conferred the Order of National Artists. The upon us National Artists after his or her own heart, in total disregard of the advise of the CCP and the
Committee on Honors and the former President effectively treated respondents Guidote-Alvarez, NCCA and the voice of the community of artists, resulting to repeated episodes of indignation and
Caparas, Mañosa and Moreno as a preferred class. The special treatment accorded to respondents uproar from the artists and the public.
60
Guidote-Alvarez, Caparas, Mañosa and Moreno fails to pass rational scrutiny. No real and substantial
distinction between respondents and petitioner Abad has been shown that would justify deviating from Furthermore, if not corrected, such an act would give rise to mischief and dangerous precedent
the laws, guidelines and established procedures, and placing respondents in an exceptional position. whereby those in the corridors of power could avoid judicial intervention and review by merely speedily
73
The undue classification was not germane to the purpose of the law. Instead, it contradicted the law and and stealthily completing the commission of an illegality.
well-established guidelines, rules and regulations meant to carry the law into effect. While petitioner
61
Abad cannot claim entitlement to the Order of National Artists, he is entitled to be given an equal In any event, the present petition is also for certiorari and there is no procedural bar for the Court to
opportunity to vie for that honor. In view of the foregoing, there was a violation of petitioner Abad’s right pass upon the question of whether the proclamations of respondents Guidote-Alvarez, Caparas,
to equal protection, an interest that is substantial enough to confer him standing in this case. Mañosa and Moreno as National Artists were attended by grave abuse of presidential discretion.

As regards the other concerned artists and academics as well as the CAP, their claim of deep concern Limits of the President’s Discretion
for the preservation of the country’s rich cultural and artistic heritage, while laudable, falls short of the
The respective powers of the CCP Board of Trustees and of the NCCA Board of Commissioners with
injury in fact requirement of standing. Their assertion constitutes a generalized grievance shared in a
62 respect to the conferment of the Order of National Artists are clear. They jointly administer the said
substantially equal measure by all or a large class of citizens. Nor can they take refuge in their status
award and, upon their recommendation or advice, the President confers the Order of National Artists.
as taxpayers as the case does not involve any illegal appropriation or taxation. A taxpayer’s suit is
63
proper only when there is an exercise of the spending or taxing power of the Congress. To "recommend" and to "advise" are synonymous. To "recommend" is "to advise or counsel." To
74

"advise" is "to give an opinion or counsel, or recommend a plan or course of action; also to give notice.
Nonetheless, as a reading of the petition shows that it has advanced an issue which deserves the 75
To encourage, inform or acquaint." "Advise" imports that it is discretionary or optional with the person
attention of this Court in view of its seriousness, novelty and weight as precedent, it behooves the Court 76
64 addressed whether he will act on such advice or not. This has been clearly explained in Cojuangco,
to relax the rules on standing and to resolve the issue presented before it. Moreover, this issue is of 77
65 Jr. v. Atty. Palma :
paramount interest, which further justifies a liberal stance on standing.
The "power to recommend" includes the power to give "advice, exhortation or indorsement, which is
Propriety of the Remedies
essentially persuasive in character, not binding upon the party to whom it is made." (Emphasis
The present action is a petition for prohibition, certiorari, injunction, restraining order and all other legal, supplied.)
just and equitable reliefs.
Thus, in the matter of the conferment of the Order of National Artists, the President may or may not
adopt the recommendation or advice of the NCCA and the CCP Boards. In other words, the advice of
the NCCA and the CCP is subject to the President’s discretion.
Nevertheless, the President’s discretion on the matter is not totally unfettered, nor the role of the NCCA Management and Archives Office. However, they shall continue operating under their respective
and the CCP Boards meaningless. charters or as provided by law where provisions therein are not inconsistent with the provisions of this
Act. They shall serve as the national repository and/or showcase, as the case may be, of the best of
Discretion is not a free-spirited stallion that runs and roams wherever it pleases but is reined in to keep Philippine culture and arts. For this purpose, these agencies shall submit periodic reports, including
it from straying. In its classic formulation, "discretion is not unconfined and vagrant" but "canalized recommendations to the [NCCA]. (Emphasis supplied.)
78
within banks that keep it from overflowing."
On the other hand, the NCCA has been given the following mandate in connection with the conferment
The President’s power must be exercised in accordance with existing laws. Section 17, Article VII of the of cultural or arts awards:
Constitution prescribes faithful execution of the laws by the President:
Sec. 12. Mandate. – The Commission is hereby mandated to formulate and implement policies and
Sec. 17. The President shall have control of all the executive departments, bureaus and offices. He plans in accordance with the principles stated in Title 1 of this Act.
shall ensure that the laws be faithfully executed. (Emphasis supplied.)
(a) To encourage the continuing and balanced development of a pluralistic culture by the people
The President’s discretion in the conferment of the Order of National Artists should be exercised in themselves, it shall:
accordance with the duty to faithfully execute the relevant laws. The faithful execution clause is best
79
construed as an obligation imposed on the President, not a separate grant of power. It simply xxxx
underscores the rule of law and, corollarily, the cardinal principle that the President is not above the
80 (4) extend recognition of artistic achievement through awards, grants and services to artists and cultural
laws but is obliged to obey and execute them. This is precisely why the law provides that
"administrative or executive acts, orders and regulations shall be valid only when they are not contrary groups which contribute significantly to the Filipino’s cultural legacy;
81
to the laws or the Constitution."
xxxx
In this connection, the powers granted to the NCCA and the CCP Boards in connection with the
conferment of the Order of National Artists by executive issuances were institutionalized by two laws, Sec. 13. Powers and Functions. – To carry out its mandate, the Commission shall exercise the following
namely, Presidential Decree No. 208 dated June 7, 1973 and Republic Act No. 7356. In particular, powers and functions:
Proclamation No. 1144 dated May 15, 1973 constituted the CCP Board as the National Artists Awards
xxxx
Committee and tasked it to "administer the conferment of the category of National Artist" upon
deserving Filipino artists with the mandate to "draft the rules to guide its deliberations in the choice of (j) advise the President on matters pertaining to culture and the arts, including the creation of a special
National Artists": decoration or award, for persons who have significantly contributed to the development and promotion
of Philippine culture and arts;
Proclamation No. 1001 dated April 27, 1972, creating the Award and Decoration of National Artist, is
hereby amended by creating a National Artists Awards Committee, hereinafter to administer the (k) promulgate rules, regulations and undertake any and all measures as may be necessary to
conferment of the category of National Artist upon those deserving thereof. The Committee, which shall implement this Act. (Emphases supplied.)
be composed of members of the Board of Trustees of the Cultural Center of the Philippines, shall
organize itself immediately and shall draft the rules to guide its deliberations in the choice of National By virtue of their respective statutory mandates in connection with the conferment of the National Artist
Artists, to the end that those who have created a body of work in the arts and in letters capable of Award, the NCCA and the CCP decided to work together and jointly administer the National Artist
withstanding the test of time will be so recognized. (Emphases supplied.) Award. They reviewed the guidelines for the nomination, selection and administration of the National
Artist Award, created a National Artist Award Secretariat, centralized all financial resources and
The authority of the CCP Board of Trustees as National Artists Awards Committee was reiterated in management for the administration of the National Artist Award, and added another layer to the
Presidential Decree No. 208 dated June 7, 1973. selection process so that more members of the arts and culture sector of the Philippines may be
involved and participate in the selection of National Artists.
The function of the CCP Board of Trustees as National Artists Awards Committee has been recognized
under Republic Act No. 7356: We have held that an administrative regulation adopted pursuant to law has the force and effect of
82
law. Thus, the rules, guidelines and policies regarding the Order of National Artists jointly issued by
Sec. 18. The National Cultural Agencies. – The [NCCA] shall coordinate with the national cultural
the CCP Board of Trustees and the NCCA pursuant to their respective statutory mandates have the
agencies including but not limited to the Cultural Center of the Philippines, the Institute of Philippine
force and effect of law. Until set aside, they are binding upon executive and administrative
Languages, the National Historical Institute, the National Library, the National Museum, the Records
83
agencies, including the President himself/herself as chief executor of laws. In this connection, Section Pursuant to the above provision of the implementing rules of Executive Order No. 236, s. 2003, the
84
2.5(A) of the Implementing Rules and Regulations of Executive Order No. 236, s. 2003 provides: authority of the Committee on Honors is limited to determining whether the nominations submitted by a
particular awards committee, in this case, the joint NCCA and CCP Boards, have been tainted by abuse
2.5: General Guidelines for Awards Committees of discretion, and whether the nominees are in good standing. Should the nominations meet these two
criteria, the Committee on Honors shall make a recommendation to the President for conferment of the
A. National Orders of Cultural and Scientific Merit Order of National Artists.

The existing modalities of the NCCA for selecting recipients for the Order of National Artists, and the In view of the various stages of deliberation in the selection process and as a consequence of his/her
Gawad sa Manlilikha ng Bayan, and of the NAST for selecting recipients of the Order of National duty to faithfully enforce the relevant laws, the discretion of the President in the matter of the Order of
Scientists, shall remain in force. (Emphases supplied.) National Artists is confined to the names submitted to him/her by the NCCA and the CCP Boards. This
means that the President could not have considered conferment of the Order of National Artists on any
Section 2.4(A) of the same implementing rules further states:
person not considered and recommended by the NCCA and the CCP Boards. That is the proper import
2.4: Awards Committees of the provision of Executive Order No. 435, s. 2005, that the NCCA and the CCP "shall advise the
President on the conferment of the Order of National Artists." Applying this to the instant case, the
There shall be two types of awards committees: the Committee on Honors and the various awards former President could not have properly considered respondents Guidote-Alvarez, Caparas, Mañosa
committees in the various units of the government service. and Moreno, as their names were not recommended by the NCCA and the CCP Boards. Otherwise, not
only will the stringent selection and meticulous screening process be rendered futile, the respective
A. The Committee on Honors mandates of the NCCA and the CCP Board of Trustees under relevant laws to administer the
conferment of Order of National Artists, draft the rules and regulations to guide its deliberations,
The Committee on Honors serves as a National Awards Committee. It is composed of the following: formulate and implement policies and plans, and undertake any and all necessary measures in that
regard will also become meaningless.
The Executive Secretary, Chairman
Furthermore, with respect to respondent Guidote-Alvarez who was the Executive Director of the NCCA
The Secretary of Foreign Affairs, Vice-Chairman at that time, the Guidelines expressly provides:

Head, Presidential Management Staff, member 6.5 NCCA and CCP Board members and consultants and NCCA and CCP officers and staff are
85
automatically disqualified from being nominated.
Presidential Assistant for Historical Affairs, member
Respondent Guidote-Alvarez could not have even been nominated, hence, she was not qualified to be
Chief of Presidential Protocol, member
considered and conferred the Order of National Artists at that time. The President’s discretion on the
matter does not extend to removing a legal impediment or overriding a legal restriction.
Chief of Protocol, DFA, member
From the foregoing, the advice or recommendation of the NCCA and the CCP Boards as to the
All nominations from the various awards committees must be submitted to the Committee on Honors via
conferment of the Order of National Artists on Conde, Dr. Santos, Francisco and Alcuaz was not binding
the Chancellery of Philippine Orders and State Decorations. The Chancellery shall process nominations
on the former President but only discretionary or optional for her whether or not to act on such advice or
for the consideration of the Committee on Honors. The Committee on Honors shall screen and
recommendation. Also, by virtue of the power of control, the President had the authority to alter or
recommend these nominations to the President.
modify or nullify or set aside such recommendation or advice. It was well within the President’s power
The Committee on Honors shall, as a general rule, serve as a screening committee to ensure that and discretion to proclaim all, or some or even none of the recommendees of the CCP and the NCCA
nominations received from the various awards committees meet two tests: that there has not been an Boards, without having to justify his or her action. Thus, the exclusion of Santos did not constitute grave
abuse of discretion in making the nomination, and that the nominee is in good standing. Should a abuse of discretion on the part of the former President.
nomination meet these criteria, a recommendation to the President for conferment shall be made.
The conferment of the Order of National Artists on respondents Guidote-Alvarez, Caparas, Mañosa and
The President of the Philippines takes the recommendations of the Committee on Honors in the highest Moreno was an entirely different matter.
consideration when making the final decision on the conferment of awards. (Emphasis supplied.)
There is grave abuse of discretion when an act is (1) done contrary to the Constitution, the law or
jurisprudence or (2) executed whimsically, capriciously or arbitrarily, out of malice, ill will or personal
86
bias.
87
There was a violation of the equal protection clause of the Constitution when the former President
gave preferential treatment to respondents Guidote-Alvarez, Caparas, Mañosa and
Moreno.1âwphi1 The former President’s constitutional duty to faithfully execute the laws and observe
the rules, guidelines and policies of the NCCA and the CCP as to the selection of the nominees for
conferment of the Order of National Artists proscribed her from having a free and uninhibited hand in
the conferment of the said award. The manifest disregard of the rules, guidelines and processes of the
NCCA and the CCP was an arbitrary act that unduly favored respondents Guidote-Alvarez, Caparas,
Mañosa and Moreno. The conferment of the Order of National Artists on said respondents was
therefore made with grave abuse of discretion and should be set aside.

While the Court invalidates today the proclamation of respondents Guidote-Alvarez, Caparas, Mañosa
and Moreno as National Artists, such action should not be taken as a pronouncement on whether they
are worthy to be conferred that honor. Only the President, upon the advise of the NCCA and the CCP
Boards, may determine that. The Court simply declares that, as the former President committed grave
abuse of discretion in issuing Proclamation Nos. 1826 to 1829 dated July 6, 2009, the said
proclamations are invalid. However, nothing in this Decision should be read as a disqualification on the
part of respondents Guidote-Alvarez, Caparas, Mañosa and Moreno to be considered for the honor of
National Artist in the future, subject to compliance with the laws, rules and regulations governing said
award.

WHEREFORE, the petition is hereby GRANTED in PART. Proclamation Nos. 1826 to 1829 dated July
6, 2009 proclaiming respondents Cecile Guidote-Alvarez, Carlo Magno Jose Caparas, Francisco
Mañosa, and Jose Moreno, respectively, as National Artists are declared INVALID and

SET ASIDE for having been issued with grave abuse of discretion.

SO ORDERED.

TERESITA J. LEONARDO-DE CASTRO


Associate Justice
Republic of the Philippines WHEREAS, under law and jurisprudence, the President of the Philippines has broad powers to
SUPREME COURT reorganize the offices under her supervision and control;
Manila
NOW[,] THEREFORE[,] I, Gloria Macapagal-Arroyo, by the powers vested in me as President of the
EN BANC Republic of the Philippines, do hereby order:

G.R. No. 166052 August 29, 2007 SECTION 1. The Department of Agrarian Reform is hereby transformed into the Department of Land
Reform. It shall be responsible for all land reform in the country, including agrarian reform, urban land
ANAK MINDANAO PARTY-LIST GROUP, as represented by Rep. Mujiv S. Hataman, and reform, and ancestral domain reform.
MAMALO DESCENDANTS ORGANIZATION, INC., as represented by its Chairman Romy
Pardi, Petitioners, SECTION 2. The PCUP is hereby placed under the supervision and control of the Department of Land
vs. Reform. The Chairman of the PCUP shall be ex-officio Undersecretary of the Department of Land
THE EXECUTIVE SECRETARY, THE HON. EDUARDO R. ERMITA, and THE SECRETARY OF Reform for Urban Land Reform.
AGRARIAN/LAND REFORM, THE HON. RENE C. VILLA, Respondents.
SECTION 3. The NCIP is hereby placed under the supervision and control of the Department of Land
DECISION Reform. The Chairman of the NCIP shall be ex-officio Undersecretary of the Department of Land
Reform for Ancestral Domain Reform.
CARPIO MORALES, J.:
SECTION 4. The PCUP and the NCIP shall have access to the services provided by the Department’s
Petitioners Anak Mindanao Party-List Group (AMIN) and Mamalo Descendants Organization, Inc. Finance, Management and Administrative Office; Policy, Planning and Legal Affairs Office, Field
(MDOI) assail the constitutionality of Executive Order (E.O.) Nos. 364 and 379, both issued in 2004, via Operations and Support Services Office, and all other offices of the Department of Land Reform.
the present Petition for Certiorari and Prohibition with prayer for injunctive relief.
SECTION 5. All previous issuances that conflict with this Executive Order are hereby repealed or
E.O. No. 364, which President Gloria Macapagal-Arroyo issued on September 27, 2004, reads: modified accordingly.

EXECUTIVE ORDER NO. 364 SECTION 6. This Executive Order takes effect immediately. (Emphasis and underscoring supplied)

TRANSFORMING THE DEPARTMENT OF AGRARIAN REFORM INTO THE DEPARTMENT OF E.O. No. 379, which amended E.O. No. 364 a month later or on October 26, 2004, reads:
LAND REFORM
EXECUTIVE ORDER NO. 379
WHEREAS, one of the five reform packages of the Arroyo administration is Social Justice and Basic
[N]eeds; AMENDING EXECUTIVE ORDER NO. 364 ENTITLED TRANSFORMING THE DEPARTMENT OF
AGRARIAN REFORM INTO THE DEPARTMENT OF LAND REFORM
WHEREAS, one of the five anti-poverty measures for social justice is asset reform;
WHEREAS, Republic Act No. 8371 created the National Commission on Indigenous Peoples;
WHEREAS, asset reforms covers [sic] agrarian reform, urban land reform, and ancestral domain
reform; WHEREAS, pursuant to the Administrative Code of 1987, the President has the continuing authority to
reorganize the administrative structure of the National Government.
WHEREAS, urban land reform is a concern of the Presidential Commission [for] the Urban Poor
(PCUP) and ancestral domain reform is a concern of the National Commission on Indigenous Peoples NOW, THEREFORE, I, GLORIA MACAPAGAL-ARROYO, President of the Republic of the Philippines,
(NCIP); by virtue of the powers vested in me by the Constitution and existing laws, do hereby order:

WHEREAS, another of the five reform packages of the Arroyo administration is Anti-Corruption and Section 1. Amending Section 3 of Executive Order No. 364. Section 3 of Executive Order No. 364,
Good Government; dated September 27, 2004 shall now read as follows:

WHEREAS, one of the Good Government reforms of the Arroyo administration is rationalizing the "Section 3. The National Commission on Indigenous Peoples (NCIP) shall be an attached agency of the
bureaucracy by consolidating related functions into one department; Department of Land Reform."
Section 2. Compensation. The Chairperson shall suffer no diminution in rank and salary. The OSG questions, however, the standing of MDOI, a registered people’s organization
of Teduray and Lambangiantribesfolk of (North) Upi and South Upi in the province of Maguindanao.
Section 3. Repealing Clause. All executive issuances, rules and regulations or parts thereof which are
inconsistent with this Executive Order are hereby revoked, amended or modified accordingly. As co-petitioner, MDOI alleges that it is concerned with the negative impact of NCIP’s becoming an
attached agency of the DAR on the processing of ancestral domain claims. It fears that transferring the
Section 4. Effectivity. This Executive Order shall take effect immediately. (Emphasis and underscoring NCIP to the DAR would affect the processing of ancestral domain claims filed by its members.
in the original)
Locus standi or legal standing has been defined as a personal and substantial interest in a case such
Petitioners contend that the two presidential issuances are unconstitutional for violating: that the party has sustained or will sustain direct injury as a result of the governmental act that is being
challenged. The gist of the question of standing is whether a party alleges such personal stake in the
- THE CONSTITUTIONAL PRINCIPLES OF SEPARATION OF POWERS AND OF THE RULE OF outcome of the controversy as to assure that concrete adverseness which sharpens the presentation of
LAW[;] issues upon which the court depends for illumination of difficult constitutional questions.
10

- THE CONSTITUTIONAL SCHEME AND POLICIES FOR AGRARIAN REFORM, URBAN LAND It has been held that a party who assails the constitutionality of a statute must have a direct and
REFORM, INDIGENOUS PEOPLES’ RIGHTS AND ANCESTRAL DOMAIN[; AND] personal interest. It must show not only that the law or any governmental act is invalid, but also that it
sustained or is in immediate danger of sustaining some direct injury as a result of its enforcement, and
- THE CONSTITUTIONAL RIGHT OF THE PEOPLE AND THEIR ORGANIZATIONS TO EFFECTIVE
not merely that it suffers thereby in some indefinite way. It must show that it has been or is about to be
AND REASONABLE PARTICIPATION IN DECISION-MAKING, INCLUDING THROUGH ADEQUATE
1 denied some right or privilege to which it is lawfully entitled or that it is about to be subjected to some
CONSULTATION[.] 11
burdens or penalties by reason of the statute or act complained of.
By Resolution of December 6, 2005, this Court gave due course to the Petition and required the
For a concerned party to be allowed to raise a constitutional question, it must show that (1) it has
submission of memoranda, with which petitioners and respondents complied on March 24, 2006 and
personally suffered some actual or threatened injury as a result of the allegedly illegal conduct of the
April 11, 2006, respectively.
government, (2) the injury is fairly traceable to the challenged action, and (3) the injury is likely to be
12
The issue on the transformation of the Department of Agrarian Reform (DAR) into the Department of redressed by a favorable action.
Land Reform (DLR) became moot and academic, however, the department having reverted to its former 13
2 An examination of MDOI’s nebulous claims of "negative impact" and "probable setbacks" shows that
name by virtue of E.O. No. 456 which was issued on August 23, 2005.
they are too abstract to be considered judicially cognizable. And the line of causation it proffers between
3
The Court is thus left with the sole issue of the legality of placing the Presidential Commission for the the challenged action and alleged injury is too attenuated.
Urban Poor (PCUP) under the supervision and control of the DAR, and the National Commission on
Vague propositions that the implementation of the assailed orders will work injustice and violate the
Indigenous Peoples (NCIP) under the DAR as an attached agency.
rights of its members cannot clothe MDOI with the requisite standing. Neither would its status as a
14
Before inquiring into the validity of the reorganization, petitioners’ locus standi or legal standing, inter "people’s organization" vest it with the legal standing to assail the validity of the executive orders.
4
alia, becomes a preliminary question. 15
La Bugal-B’laan Tribal Association, Inc. v. Ramos, which MDOI cites in support of its claim to legal
The Office of the Solicitor General (OSG), on behalf of respondents, concedes that AMIN has the
5 standing, is inapplicable as it is not similarly situated with the therein petitioners who alleged personal
6
requisite legal standing to file this suit as member of Congress. and substantial injury resulting from the mining activities permitted by the assailed statute. And so
16
is Cruz v. Secretary of Environment and Natural Resources, for the indigenous peoples’ leaders and
Petitioners find it impermissible for the Executive to intrude into the domain of the Legislature. They organizations were not the petitioners therein, who necessarily had to satisfy the locus
posit that an act of the Executive which injures the institution of Congress causes a derivative but standi requirement, but were intervenors who sought and were allowed to be impleaded, not to assail
7 but to defend the constitutionality of the statute.
nonetheless substantial injury, which can be questioned by a member of Congress. They add that to
the extent that the powers of Congress are impaired, so is the power of each member thereof, since his
office confers a right to participate in the exercise of the powers of that institution.
8 Moreover, MDOI raises no issue of transcendental importance to justify a relaxation of the rule on legal
standing. To be accorded standing on the ground of transcendental importance, Senate of the
17
Indeed, a member of the House of Representatives has standing to maintain inviolate the prerogatives, Philippines v. Ermita requires that the following elements must be established: (1) the public character
9 of the funds or other assets involved in the case, (2) the presence of a clear case of disregard of a
powers and privileges vested by the Constitution in his office.
constitutional or statutory prohibition by the public respondent agency or instrumentality of government,
and (3) the lack of any other party with a more direct and specific interest in raising the questions being the Constitution has lodged it elsewhere. In fine, except as limited by the Constitution, either expressly
raised. The presence of these elements MDOI failed to establish, much less allege. or impliedly, legislative power embraces all subjects and extends to matters of general concern or
common interest.
18
Francisco, Jr. v. Fernando more specifically declares that the transcendental importance of the issues
raised must relate to the merits of the petition. While Congress is vested with the power to enact laws, the President executes the laws. The executive
power is vested in the President. It is generally defined as the power to enforce and administer the laws.
This Court, not being a venue for the ventilation of generalized grievances, must thus deny adjudication It is the power of carrying the laws into practical operation and enforcing their due observance.
of the matters raised by MDOI.
As head of the Executive Department, the President is the Chief Executive. He represents the
Now, on AMIN’s position. AMIN charges the Executive Department with transgression of the principle of government as a whole and sees to it that all laws are enforced by the officials and employees of his
separation of powers. department. He has control over the executive department, bureaus and offices. This means that he
has the authority to assume directly the functions of the executive department, bureau and office, or
Under the principle of separation of powers, Congress, the President, and the Judiciary may not interfere with the discretion of its officials. Corollary to the power of control, the President also has the
encroach on fields allocated to each of them. The legislature is generally limited to the enactment of duty of supervising and enforcement of laws for the maintenance of general peace and public order.
laws, the executive to the enforcement of laws, and the judiciary to their interpretation and application to Thus, he is granted administrative power over bureaus and offices under his control to enable him to
cases and controversies. The principle presupposes mutual respect by and between the executive, 25
discharge his duties effectively. (Italics omitted, underscoring supplied)
legislative and judicial departments of the government and calls for them to be left alone to discharge
19
their duties as they see fit. The Constitution’s express grant of the power of control in the President justifies an executive action to
26
20
carry out reorganization measures under a broad authority of law.
AMIN contends that since the DAR, PCUP and NCIP were created by statutes, they can only be
transformed, merged or attached by statutes, not by mere executive orders. In enacting a statute, the legislature is presumed to have deliberated with full knowledge of all existing
27
21
laws and jurisprudence on the subject. It is thus reasonable to conclude that in passing a statute
While AMIN concedes that the executive power is vested in the President who, as Chief Executive, which places an agency under the Office of the President, it was in accordance with existing laws and
22
holds the power of control of all the executive departments, bureaus, and offices, it posits that this jurisprudence on the President’s power to reorganize.
broad power of control including the power to reorganize is qualified and limited, for it cannot be
23
exercised in a manner contrary to law, citing the constitutional duty of the President to ensure that the In establishing an executive department, bureau or office, the legislature necessarily ordains an
laws, including those creating the agencies, be faithfully executed. executive agency’s position in the scheme of administrative structure. Such determination is
28
primary, but subject to the President’s continuing authority to reorganize the administrative structure.
AMIN cites the naming of the PCUP as a presidential commission to be clearly an extension of the As far as bureaus, agencies or offices in the executive department are concerned, the power of control
President, and the creation of the NCIP as an "independent agency under the Office of the may justify the President to deactivate the functions of a particular office. Or a law may expressly grant
24
President." It thus argues that since the legislature had seen fit to create these agencies at separate 29
the President the broad authority to carry out reorganization measures. The Administrative Code of
times and with distinct mandates, the President should respect that legislative disposition. 1987 is one such law:
30

In fine, AMIN contends that any reorganization of these administrative agencies should be the subject of SEC. 30. Functions of Agencies under the Office of the President.– Agencies under the Office of the
a statute. President shall continue to operate and function in accordance with their respective charters or laws
creating them, except as otherwise provided in this Code or by law.
AMIN’s position fails to impress.
SEC. 31. Continuing Authority of the President to Reorganize his Office.– The President, subject to the
The Constitution confers, by express provision, the power of control over executive departments,
policy in the Executive Office and in order to achieve simplicity, economy and efficiency, shall
bureaus and offices in the President alone. And it lays down a limitation on the legislative power.
have continuing authority to reorganize the administrative structure of the Office of the President. For
The line that delineates the Legislative and Executive power is not indistinct. Legislative power is "the this purpose, he may take any of the following actions:
authority, under the Constitution, to make laws, and to alter and repeal them." The Constitution, as the
(1) Restructure the internal organization of the Office of the President Proper, including the immediate
will of the people in their original, sovereign and unlimited capacity, has vested this power in the
Offices, the Presidential Special Assistants/Advisers System and the Common Staff Support System, by
Congress of the Philippines. The grant of legislative power to Congress is broad, general and
abolishing, consolidating, or merging units thereof or transferring functions from one unit to another;
comprehensive. The legislative body possesses plenary power for all purposes of civil government. Any
power, deemed to be legislative by usage and tradition, is necessarily possessed by Congress, unless
(2) Transfer any function under the Office of the President to any other Department or Agency as well The characterization of the NCIP as an independent agency under the Office of the President does not
as transfer functions to the Office of the President from other Departments and Agencies; and remove said body from the President’s control and supervision with respect to its performance of
administrative functions. So it has been opined:
(3) Transfer any agency under the Office of the President to any other department or agency as well as
31
transfer agencies to the Office of the President from other departments or agencies. (Italics in the That Congress did not intend to place the NCIP under the control of the President in all instances is
original; emphasis and underscoring supplied) evident in the IPRA itself, which provides that the decisions of the NCIP in the exercise of its quasi-
judicial functions shall be appealable to the Court of Appeals, like those of the National Labor Relations
In carrying out the laws into practical operation, the President is best equipped to assess whether an Commission (NLRC) and the Securities and Exchange Commission (SEC). Nevertheless, the
executive agency ought to continue operating in accordance with its charter or the law creating it. This NCIP, although independent to a certain degree, was placed by Congress "under the office of the
is not to say that the legislature is incapable of making a similar assessment and appropriate action President" and, as such, is still subject to the President’s power of control and supervision granted
within its plenary power. The Administrative Code of 1987 merely underscores the need to provide the under Section 17, Article VII of the Constitution with respect to its performance of administrative
President with suitable solutions to situations on hand to meet the exigencies of the service that may 40
functions[.] (Underscoring supplied)
call for the exercise of the power of control.
In transferring the NCIP to the DAR as an attached agency, the President effectively tempered the
x x x The law grants the President this power in recognition of the recurring need of every President to exercise of presidential authority and considerably recognized that degree of independence.
reorganize his office "to achieve simplicity, economy and efficiency." The Office of the President is the
nerve center of the Executive Branch. To remain effective and efficient, the Office of the President must The Administrative Code of 1987 categorizes administrative relationships into (1) supervision and
41
be capable of being shaped and reshaped by the President in the manner he deems fit to carry out his control, (2) administrative supervision, and (3) attachment. With respect to the third category, it has
directives and policies. After all, the Office of the President is the command post of the President. This been held that an attached agency has a larger measure of independence from the Department to
is the rationale behind the President’s continuing authority to reorganize the administrative structure of which it is attached than one which is under departmental supervision and control or administrative
32
the Office of the President. supervision. This is borne out by the "lateral relationship" between the Department and the attached
42
agency. The attachment is merely for "policy and program coordination." Indeed, the essential
33 43
The Office of the President consists of the Office of the President proper and the agencies under it. It autonomous character of a board is not negated by its attachment to a commission.
34
is not disputed that PCUP and NCIP were formed as agencies under the Office of the President. The
"Agencies under the Office of the President" refer to those offices placed under the chairmanship of the AMIN argues, however, that there is an anachronism of sorts because there can be no policy and
President, those under the supervision and control of the President, those under the administrative program coordination between conceptually different areas of reform. It claims that the new framework
supervision of the Office of the President, those attached to the Office for policy and program subsuming agrarian reform, urban land reform and ancestral domain reform is fundamentally incoherent
44
coordination, and those that are not placed by law or order creating them under any special in view of the widely different contexts. And it posits that it is a substantive transformation or
35
department. reorientation that runs contrary to the constitutional scheme and policies.
45
As thus provided by law, the President may transfer any agency under the Office of the President to any AMIN goes on to proffer the concept of "ordering the law" which, so it alleges, can be said of the
other department or agency, subject to the policy in the Executive Office and in order to achieve Constitution’s distinct treatment of these three areas, as reflected in separate provisions in different
36 46
simplicity, economy and efficiency. Gauged against these guidelines, the challenged executive orders parts of the Constitution. It argues that the Constitution did not intend an over-arching concept of
may not be said to have been issued with grave abuse of discretion or in violation of the rule of law. agrarian reform to encompass the two other areas, and that how the law is ordered in a certain way
should not be undermined by mere executive orders in the guise of administrative efficiency.
The references in E.O. 364 to asset reform as an anti-poverty measure for social justice and to
37
rationalization of the bureaucracy in furtherance of good government encapsulate a portion of the The Court is not persuaded.
existing "policy in the Executive Office." As averred by the OSG, the President saw it fit to streamline
38 The interplay of various areas of reform in the promotion of social justice is not something implausible or
the agencies so as not to hinder the delivery of crucial social reforms.
47
unlikely. Their interlocking nature cuts across labels and works against a rigid pigeonholing of
The consolidation of functions in E.O. 364 aims to attain the objectives of "simplicity, economy and executive tasks among the members of the President’s official family. Notably, the Constitution inhibited
efficiency" as gathered from the provision granting PCUP and NCIP access to the range of services from identifying and compartmentalizing the composition of the Cabinet. In vesting executive power in
39
provided by the DAR’s technical offices and support systems. one person rather than in a plural executive, the evident intention was to invest the power holder with
48
energy.
AMIN takes premium on the severed treatment of these reform areas in marked provisions of the Finally, a word on the last ground proffered for declaring the unconstitutionality of the assailed
57
Constitution. It is a precept, however, that inferences drawn from title, chapter or section headings are issuances ─ that they violate Section 16, Article XIII of the Constitution on the people’s right to
49 50
entitled to very little weight. And so must reliance on sub-headings, or the lack thereof, to support a participate in decision-making through adequate consultation mechanisms.
strained deduction be given the weight of helium.
The framers of the Constitution recognized that the consultation mechanisms were already operating
51
Secondary aids may be consulted to remove, not to create doubt. AMIN’s thesis unsettles, more than without the State’s action by law, such that the role of the State would be mere facilitation, not
settles the order of things in construing the Constitution. Its interpretation fails to clearly establish that necessarily creation of these consultation mechanisms. The State provides the support, but eventually it
the so-called "ordering" or arrangement of provisions in the Constitution was consciously adopted to is the people, properly organized in their associations, who can assert the right and pursue the
imply a signification in terms of government hierarchy from where a constitutional mandate can per objective. Penalty for failure on the part of the government to consult could only be reflected in the ballot
58
se be derived or asserted. It fails to demonstrate that the "ordering" or layout was not simply a matter of box and would not nullify government action.
style in constitutional drafting but one of intention in government structuring. With its inherent ambiguity,
the proposed interpretation cannot be made a basis for declaring a law or governmental act WHEREFORE, the petition is DISMISSED. Executive Order Nos. 364 and 379 issued on September
unconstitutional. 27, 2004 and October 26, 2004, respectively, are declared not unconstitutional.

A law has in its favor the presumption of constitutionality. For it to be nullified, it must be shown that SO ORDERED.
there is a clear and unequivocal breach of the Constitution. The ground for nullity must be clear and
52
beyond reasonable doubt. Any reasonable doubt should, following the universal rule of legal CONCHITA CARPIO MORALES
hermeneutics, be resolved in favor of the constitutionality of a law.
53 Associate Justice

54
Ople v. Torres on which AMIN relies is unavailing. In that case, an administrative order involved a
system of identification that required a "delicate adjustment of various contending state policies"
properly lodged in the legislative arena. It was declared unconstitutional for dealing with a subject that
should be covered by law and for violating the right to privacy.

In the present case, AMIN glaringly failed to show how the reorganization by executive fiat would
hamper the exercise of citizen’s rights and privileges. It rested on the ambiguous conclusion that the
reorganization jeopardizes economic, social and cultural rights. It intimated, without expounding, that
the agendum behind the issuances is to weaken the indigenous peoples’ rights in favor of the mining
industry. And it raised concerns about the possible retrogression in DAR’s performance as the added
workload may impede the implementation of the comprehensive agrarian reform program.lavvphil

AMIN has not shown, however, that by placing the NCIP as an attached agency of the DAR, the
President altered the nature and dynamics of the jurisdiction and adjudicatory functions of the NCIP
concerning all claims and disputes involving rights of indigenous cultural communities and

indigenous peoples. Nor has it been shown, nay alleged, that the reorganization was made in bad
55
faith.

As for the other arguments raised by AMIN which pertain to the wisdom or soundness of the executive
decision, the Court finds it unnecessary to pass upon them. The raging debate on the most fitting
framework in the delivery of social services is endless in the political arena. It is not the business of this
Court to join in the fray. Courts have no judicial power to review cases involving political questions and,
as a rule, will desist from taking cognizance of speculative or hypothetical cases, advisory opinions and
56
cases that have become moot.
Republic of the Philippines On July 28, 2000, the Secretary of Health again issued Department Circular No. 221, Series of 2000,
SUPREME COURT stating that the Department will start implementing the Rationalization and Streamlining Plan by a
Manila process of selection, placement or matching of personnel to the approved organizational chart and the
3
list of the approved plantilla items. The Secretary also issued Administrative Order (A.O.) No. 94,
FIRST DIVISION Series of 2000, which set the implementing guidelines for the restructuring process on personnel
selection and placement, retirement and/or voluntary resignation. A.O. No. 94 outlined the general
G.R. No. 160093 July 31, 2007 guidelines for the selection and placement of employees adopting the procedures and standards set
4
forth in R.A. No. 6656 or the "Rules on Governmental Reorganization," Civil Service Rules and
MALARIA EMPLOYEES AND WORKERS ASSOCIATION OF THE PHILIPPINES, INC. (MEWAP),
Regulations, Sections 76 to 78 of the GAA for the Year 2000, and Section 42 of E.O. No. 292.
represented by its National President, DR. RAMON A. SULLA, and MEWAP DOH Central Office
Chapter President, DR. GRACELA FIDELA MINA-RAMOS, and PRISCILLA CARILLO, and On August 29, 2000, the Secretary of Health issued Department Memorandum No. 157, Series of
HERMINIO JAVIER, petitioners, 2000, viz.:
vs.
THE HONORABLE EXECUTIVE SECRETARY ALBERTO ROMULO, (substituting the former Pursuant to the Notice of Organization, Staffing and Compensation Action (NOSCA) approved by the
Executive Secretary Renato de Villa), THE HONORABLE SECRETARY OF HEALTH MANUEL DBM on 8 July 2000 and Memorandum Circular No. 62 issued by the Presidential Committee on
DAYRIT and THE HONORABLE SECRETARY OF BUDGET AND MANAGEMENT EMILIA T. Effective Governance (PCEG) on 17 July 2000, Implementing E.O. 102 dated 24 May 1999, the
BONCODIN, respondents. following approved Placement List of DOH Personnel is hereby disseminated for your information and
guidance.
DECISION
All personnel are hereby directed to report to their new assignments on or before 2 October 2000
PUNO, CJ.: pending processing of new appointments, required clearances and other pertinent documents.

At bar is a Petition for Review on Certiorari of the Decision of the Court of Appeals in CA-G.R. SP No. All Heads of Office/Unit in the Department of Health are hereby directed to facilitate the implementation
1
65475 dated September 12, 2003 which upheld the validity of Executive Order (E.O.) No. 102, the law of E.O. 102, to include[,] among others, the transfer or movement of personnel, properties, records and
Redirecting the Functions and Operations of the Department of Health. Then President Joseph E. documents to appropriate office/unit and device other necessary means to minimize disruption of office
Estrada issued E.O. No. 102 on May 24, 1999 pursuant to Section 20, Chapter 7, Title I, Book III of E.O. functions and delivery of health services.
No. 292, otherwise known as the Administrative Code of 1987, and Sections 78 and 80 of Republic Act
(R.A.) No. 8522, also known as the General Appropriations Act (GAA) of 1998. E.O. No. 102 provided Appeals, oversights, issues and concerns of personnel related to this Placement List shall be made in
for structural changes and redirected the functions and operations of the Department of Health. writing using the Appeals Form (available at the Administrative Service) addressed to the Appeals
Committee chaired by Dr. Gerardo Bayugo. All Appeals Forms shall be submitted to the Re-Engineering
On October 19, 1999, the President issued E.O. No. 165 "Directing the Formulation of an Institutional Secretariat xxx not later than 18 September 2000.
5

Strengthening and Streamlining Program for the Executive Branch" which created the Presidential
Committee on Executive Governance (PCEG) composed of the Executive Secretary as chair and the Petitioner Malaria Employees and Workers Association of the Philippines, Inc. (MEWAP) is a union of
Secretary of the Department of Budget and Management (DBM) as co-chair. affected employees in the Malaria Control Service of the Department of Health. MEWAP filed a
complaint, docketed as Civil Case No. 00-98793, with the Regional Trial Court of Manila seeking to
The DBM, on July 8, 2000, issued the Notice of Organization, Staffing and Compensation Action nullify Department Memorandum No. 157, the NOSCA and the Placement List of Department of Health
(NOSCA). On July 17, 2000, the PCEG likewise issued Memorandum Circular (M.C.) No. 62, entitled Personnel and other issuances implementing E.O. No. 102.
"Implementing Executive Order No. 102, Series of 1999 Redirecting the Functions and Operations of
2
the Department of Health." M.C. No. 62 directed the rationalization and streamlining of the said On May 2, 2001, while the civil case was pending at the Regional Trial Court of Manila, Branch 22,
Department. petitioners filed with this Court a petition for certiorari under Rule 65 of the Rules of Court. Petitioners
sought to nullify E.O. No. 102 for being issued with grave abuse of discretion amounting to lack or
On July 24, 2000, the Secretary of Health issued Department Memorandum No. 136, Series of 2000, excess of jurisdiction as it allegedly violates certain provisions of E.O. No. 292 and R.A. No. 8522. The
ordering the Undersecretary, Assistant Secretaries, Bureau or Service Directors and Program petition was referred to the Court of Appeals which dismissed the same in its assailed Decision. Hence,
Managers of the Department of Health to direct all employees under their respective offices to this appeal where petitioners ask for a re-examination of the pertinent pronouncements of this Court
accomplish and submit the Personal Information Sheet due to the approval of the Department of Health that uphold the authority of the President to reorganize a department, bureau or office in the executive
– Rationalization and Streamlining Plan. department. Petitioners raise the following issues, viz.:
1. Whether Sections 78 and 80 of the General Provision of Republic Act No. 8522, otherwise known as The President’s power to reorganize the executive branch is also an exercise of his residual powers
the General Appropriation[s] Act of 1998[,] empower former President Joseph E. Estrada to reorganize under Section 20, Title I, Book III of E.O. No. 292 which grants the President broad organization powers
structurally and functionally the Department of Health. to implement reorganization measures, viz.:

2. Whether Section 20, Chapter I, title i, Book III of the Administrative Code of 1987 provides legal basis SEC. 20. Residual Powers. – Unless Congress provides otherwise, the President shall exercise such
in reorganizing the Department of Health. other powers and functions vested in the President which are provided for under the laws and
which are not specifically enumerated above, or which are not delegated by the President in
(A) Whether Presidential Decree No. 1416, as amended by Presidential Decree No. 1772, has been 10
accordance with law.
repealed.
We explained the nature of the President’s residual powers under this section in the case of Larin v.
3. Whether the President has authority under Section 17, Article VIII of the Constitution to effect a 11
Executive Secretary, viz.:
reorganization of a department under the executive branch.
This provision speaks of such other powers vested in the President under the law. What law then
4. Whether there has been abuse of discretion amounting to lack or excess of jurisdiction on the part of gives him the power to reorganize? It is Presidential Decree No. 1772 which amended
former President Joseph E. Estrada in issuing Executive Order No. 102, Redirecting the functions and Presidential Decree No. 1416. These decrees expressly grant the President of the Philippines the
operations of the Department of Health. continuing authority to reorganize the national government, which includes the power to group,
6
consolidate bureaus and agencies, to abolish offices, to transfer functions, to create and
5. Whether Executive Order No. 102 is null and void. classify functions, services and activities and to standardize salaries and materials. The validity
of these two decrees [is] unquestionable. The 1987 Constitution clearly provides that "all laws, decrees,
We deny the petition.
executive orders, proclamations, letters of instructions and other executive issuances not inconsistent
with this Constitution shall remain operative until amended, repealed or revoked." So far, there is yet
The President has the authority to carry out a reorganization of the Department of Health under the 12
no law amending or repealing said decrees.
Constitution and statutory laws. This authority is an adjunct of his power of control under Article VII,
Sections 1 and 17 of the 1987 Constitution, viz.:
The pertinent provisions of Presidential Decree No. 1416, as amended by Presidential Decree No.
1772, clearly support the President’s continuing power to reorganize the executive branch, viz.:
Section 1. The executive power shall be vested in the President of the Philippines.

Section 17. The President shall have control of all the executive departments, bureaus and offices. He 1. The President of the Philippines shall have continuing authority to reorganize the National
shall ensure that the laws be faithfully executed. Government. In exercising this authority, the President shall be guided by generally acceptable
principles of good government and responsive national development, including but not limited to the
7 following guidelines for a more efficient, effective, economical and development-oriented governmental
In Canonizado v. Aguirre, we held that reorganization "involves the reduction of
personnel, consolidation of offices, or abolition thereof by reason of economy or redundancy of framework:
functions." It alters the existing structure of government offices or units therein, including the lines of
8
control, authority and responsibility between them. While the power to abolish an office is generally xxx
lodged with the legislature, the authority of the President to reorganize the executive branch, which may
b) Abolish departments, offices, agencies or functions which may not be necessary, or create those
include such abolition, is permissible under our present laws, viz.:
which are necessary, for the efficient conduct of government functions, services and activities;
The general rule has always been that the power to abolish a public office is lodged with the
c) Transfer functions, appropriations, equipment, properties, records and personnel from one
legislature. This proceeds from the legal precept that the power to create includes the power to destroy.
department, bureau, office, agency or instrumentality to another;
A public office is either created by the Constitution, by statute, or by authority of law. Thus, except
where the office was created by the Constitution itself, it may be abolished by the same legislature that d) Create, classify, combine, split, and abolish positions;
brought it into existence.
e) Standardize salaries, materials, and equipment;
The exception, however, is that as far as bureaus, agencies or offices in the executive department are
concerned, the President’s power of control may justify him to inactivate the functions of a particular f) Create, abolish, group, consolidate, merge, or integrate entities, agencies, instrumentalities, and units
9
office, or certain laws may grant him the broad authority to carry out reorganization measures. of the National Government, as well as expand, amend, change, or otherwise modify their powers,
functions, and authorities, including, with respect to government-owned or controlled corporations, their Thus, the realignment of the appropriations to implement the reorganization of the Department of Health
corporate life, capitalization, and other relevant aspects of their charters; under E.O. No. 102 is illegal.

g) Take such other related actions as may be necessary to carry out the purposes and objectives of this Again, petitioners’ construction of the law is unduly restrictive. This Court has consistently held
16 17
Decree. in Larin and Buklod ng Kawanihang EIIB v. Zamora that the corresponding pertinent provisions in
the GAA in these subject cases authorize the President to effect organizational changes in the
Petitioners argue that the residual powers of the President under Section 20, Title I, Book III of E.O. No. department or agency concerned.
292 refer only to the Office of the President and not to the departments, bureaus or offices within the
executive branch. They invoke Section 31, Chapter 10, Title III, Book III of the same law, viz.: Be that as it may, the President must exercise good faith in carrying out the reorganization of any
branch or agency of the executive department. Reorganization is effected in good faith if it is for the
Section 31. Continuing Authority of the President to Reorganize his Office. – The President, subject to 18 19
purpose of economy or to make bureaucracy more efficient. R.A. No. 6656 provides for the
the policy in the Executive Office and in order to achieve simplicity, economy and efficiency, shall have circumstances which may be considered as evidence of bad faith in the removal of civil service
continuing authority to reorganize the administrative structure of the Office of the President. x x x employees made as a result of reorganization, to wit: (a) where there is a significant increase in the
number of positions in the new staffing pattern of the department or agency concerned; (b) where an
The interpretation of petitioners is illogically restrictive and lacks legal basis. The residual powers office is abolished and another performing substantially the same functions is created; (c)where
granted to the President under Section 20, Title I, Book III are too broad to be construed as having incumbents are replaced by those less qualified in terms of status of appointment, performance and
a sole application to the Office of the President. As correctly stated by respondents, there is nothing in merit; (d)where there is a classification of offices in the department or agency concerned and the
E.O. No. 292 which provides that the continuing authority should apply only to the Office of the reclassified offices perform substantially the same functions as the original offices; and (e) where the
13
President. If such was the intent of the law, the same should have been expressly stated. To adopt the removal violates the order of separation.
argument of petitioners would result to two conflicting provisions in one statute. It is a basic canon of
statutory construction that in interpreting a statute, care should be taken that every part thereof be given We agree with the ruling of the Court of Appeals that the President did not commit bad faith in the
effect, on the theory that it was enacted as an integrated measure and not as a hodge-podge of questioned reorganization, viz.:
conflicting provisions. The rule is that a construction that would render a provision inoperative should be
avoided; instead, apparently inconsistent provisions should be reconciled whenever possible as parts of In this particular case, there is no showing that the reorganization undertaking in the [Department of
14
a coordinated and harmonious whole. Health] had violated this requirement, nor [are] there adequate allegations to that effect. It is only
alleged that the petitioners were directly affected by the reorganization ordered under E.O. [No.] 102.
In fact, as pointed out by respondents, the President’s power to reorganize the executive department Absent is any showing that bad faith attended the actual implementation of the said presidential
15
even finds further basis under Sections 78 and 80 of R.A. No. 8522, viz.: issuance.

Section 78. Organizational Changes – Unless otherwise provided by law or directed by the President of IN VIEW WHEREOF, the petition is DENIED. The assailed Decision of the Court of Appeals in CA-G.R.
the Philippines, no organizational unit or changes in key positions in any department or agency shall be SP No. 65475 dated September 12, 2003 is AFFIRMED.
authorized in their respective organizational structure and funded from appropriations provided by this
Act. Costs against petitioners.

Section 80. Scaling Down and Phase-out of Activities of Agencies within the Executive Branch – The SO ORDERED.
heads of departments, bureaus, offices and agencies are hereby directed to identify their respective
activities which are no longer essential in the delivery of public services and which may be scaled down, Sandoval-Gutierrez, Corona, Azcuna, Quisumbing, Garcia, JJ., concur.
phased-out or abolished subject to Civil Service rules and regulations. Said activities shall be reported
to the Office of the President through the Department of Budget and Management and to the Chairman,
Committee on Appropriations of the House of Representatives and the Chairman, Committee on
Finance of the Senate. Actual scaling down, phase-out or abolition of the activities shall be effected
pursuant to Circulars or Orders issued for the purpose by the Office of the President.

Petitioners contend that Section 78 refers only to changes in "organizational units" or "key positions" in
any department or agency, while Section 80 refers merely to scaling down and phasing out of
"activities" within the executive department. They argue that neither section authorizes reorganization.
After preliminary investigation, Pasig City Prosecutor Noel Paz issued a Resolution, dismissing
petitioner's complaint in this wise:

The publication being a bona fide communication on matters of public concern, and made without
malice, we find the respondent entitled to the protection of the rule on privileged matters under Article
354 of the Revised Penal Code.

Petitioner appealed to the Department of Justice (DOJ). Acting on the appeal, Chief State Prosecutor
Jovencito Zuño issued a Resolution (Zuño Resolution), setting aside the findings of the City Prosecutor
and directing the latter to file an Information for libel against respondent. Accordingly, in the Regional
SECOND DIVISION
Trial Court (RTC) of Pasig City, an Information for libel was filed against respondent, thereat docketed
G.R. No. 140423 July 14, 2006 as Criminal Case No. 114517.

JOSE LUIS ANGEL B. OROSA, petitioner, Adversely affected, respondent appealed to the Secretary of Justice. On October 28, 1998, then Justice
vs. Secretary Serafin Cuevas reversed the Zuño Resolution and directed the City Prosecutor of Pasig to
ALBERTO C. ROA, respondent. withdraw the Information earlier filed with the RTC. In compliance therewith, a "Motion to Withdraw
Information" was accordingly filed in court by the Pasig City Prosecution Office.
DECISION
Petitioner seasonably moved for a reconsideration but his motion was denied by the Secretary of
GARCIA, J.: Justice in his Resolution of May 12, 1999.

1 2
Assailed and sought to be set aside in this petition for review is the Resolution dated July 8, 1999 of Therefrom, petitioner went to the CA on a petition for review under Rule 43 of the 1997 Rules of Civil
the Court of Appeals (CA) in CA-G.R. SP No. 53190, dismissing the petition for review under Rule 43 of Procedure, docketed as CA-G.R. No. SP No. 53190.
the 1997 Rules of Civil Procedure thereat filed by the herein petitioner from an adverse resolution of the
Secretary of Justice. As stated at the outset hereof, the CA, in the herein assailed Resolution dated July 8, 1999, dismissed
petitioner's petition for review. Partly says the CA in its dismissal Resolution:
The petition is casts against the following factual backdrop:
The Pasig City Prosecution Office and the Department of Justice are not among the quasi-judicial
On November 27, 1996, petitioner, a dentist by profession, filed with the Pasig City Prosecution Office a agencies included in Section 1 of Rule 43 whose final orders or resolutions are subject to review by the
complaint-affidavit charging respondent Alberto C. Roa, likewise a dentist, with the crime of libel. The Court of Appeals.
complaint, docketed in said office as I.S. No. 96-5442, stemmed from an article entitled "Truth vs.
Rumors: Questions against Dr. Orosa" written by respondent and published in the March-April 1996 The Supreme Court in its Resolution En Banc dated April 8, 1997, approving the 1997 Rules of Civil
issue of the Dental Trading Post, a bi-monthly publication of the Dental Exchange Co., Inc. In gist, the Procedure in Bar Matter No. 803, did not include final orders or resolutions issued by these agencies as
article delved into the possibility of a father, who happened to be an examiner in a licensure appealable under Rule 43. The Court of Appeals is therefore not at liberty to supply the omissions in the
3
examination for dentistry where his sons were examinees, manipulating the examinations or the results Rule, that would constitute an encroachment on the rule making power of the Supreme Court.
thereof to enable his children to top the same.
With his motion for reconsideration having been denied by the CA in its subsequent Resolution of
In his complaint-affidavit, petitioner alleged that the article in question is defamatory as it besmirched his October 14, 1999, petitioner is now with this Court on his submission that the appellate court erred:
honor and reputation as a dentist and as the topnotcher in the dental board examinations held in May
I
1994.
XXX IN HOLDING THAT THE RESOLUTIONS OF THE DEPARTMENT OF JUSTICE ARE NOT
Respondent denied the accusation, claiming that the article constitutes a "fair and accurate report on a
REVIEWABLE BY IT UNDER RULE 43 OF THE 1997 RULES OF CIVIL PROCEDURE.
matter of both public and social concern." He averred that the article in question was not written with
malice but with a sincere desire to contribute to the improvement of the integrity of professional
II
examinations.
XXX IN FINDING THE PETITION IN CA G.R. SP NO. 53190 [WAS] PREMATURELY FILED.
III Bureau of Patents, Trademarks and Technology Transfer, National Electrification Administration,
Energy Regulatory Board, National Telecommunications Commission, Department of Agrarian Reform
XXX IN HOLDING THAT THE RESOLUTIONS OF THE DEPARTMENT OF JUSTICE ASSAILED IN under Republic Act No. 6657, Government Service and Insurance System, Employees' Compensation
CA G.R. SP NO. 53190 ARE NOT REVIEWABLE UNDER RULE 65 (sic) OF THE 1997 RULES OF Commission, Agricultural Inventions Board, Insurance Commission, Philippine Atomic Energy
CIVIL PROCEDURE SINCE THESE RESOLUTIONS WERE ISSUED BY THE SECRETARY OF Commission, Board of Investments, Construction Industry Arbitration Commission, and voluntary
JUSTICE IN THE EXERCISE OF HIS POWER OF CONTROL AND SUPERVISION OVER arbitrators authorized by law.
PROSECUTORS.
As may be noted, the DOJ is not among the agencies expressly enumerated under Section 1 of Rule
IV 43, albeit any suggestion that it does not perform quasi-judicial functions may have to be rejected.
However, its absence from the list of agencies mentioned thereunder does not, by this fact alone,
XXX IN NOT RESOLVING THE PETITION IN CA G.R. SP NO. 53190 ON THE MERITS. already imply its exclusion from the coverage of said Rule. This is because said Section 1 uses the
phrase "among these agencies," thereby implying that the enumeration made is not exclusive of the
V
agencies therein listed.
XXX IN NOT REVERSING THE ASSAILED RESOLUTION OF THE DEPARTMENT OF JUSTICE IN
There is compelling reason to believe, however, that the exclusion of the DOJ from the list is deliberate,
CA G.R. SP NO. 53190 ON THE FOLLOWING GROUNDS: 4
being in consonance with the constitutional power of control lodged in the President over executive
a. RESPONDENT'S APPEAL FROM THE RESOLUTION OF THE DEPARTMENT OF JUSTICE, departments, bureaus and offices. This power of control, which even Congress cannot limit, let alone
THROUGH THE CHIEF STATE PROSECUTOR, DATED JANUARY 22, 1998, WAS FATALLY withdraw, means the power of the Chief Executive to review, alter, modify, nullify, or set aside what a
DEFECTIVE. subordinate, e.g., members of the Cabinet and heads of line agencies, had done in the performance of
5
their duties and to substitute the judgment of the former for that of the latter.
b. RESPONDENT'S ARTICLE WAS DEFAMATORY.
Being thus under the control of the President, the Secretary of Justice, or, to be precise, his decision is
c. MALICE ATTENDED THE PUBLICATION OF RESPONDENT'S ARTICLE. subject to review of the former. In fine, recourse from the decision of the Secretary of Justice should be
to the President, instead of the CA, under the established principle of exhaustion of administrative
d. RESPONDENT'S ARTICLE WAS NOT PROTECTED BY THE MANTLE OF PRIVILEGED MATTER. remedies. The thrust of the rule on exhaustion of administrative remedies is that if an appeal or remedy
obtains or is available within the administrative machinery, this should be resorted to before resort can
6 7
As the Court sees it, the petition commends for its consideration the issue of whether or not a petition be made to the courts. Immediate recourse to the court would be premature and precipitate; subject
for review under Rule 43 of the 1997 Rules of Civil Procedure is a proper mode of appeal from a to defined exception, a case is susceptible of dismissal for lack of cause of action should a party fail to
8
resolution of the Secretary of Justice directing the prosecutor to withdraw an information in a criminal exhaust administrative remedies. Notably, Section 1, supra, of Rule 43 includes the Office of the
case. President in the agencies named therein, thereby accentuating the fact that appeals from rulings of
department heads must first be taken to and resolved by that office before any appellate recourse may
It is petitioner's thesis that Rule 43 was intended to apply to all quasi-judicial agencies exercising quasi- be resorted to.
judicial functions. Upon this premise, petitioner submits that resolutions of the DOJ in the exercise of its
quasi-judicial functions are properly appealable to the CA via a petition for review under Rule 43, adding Given the above perspective, the question of whether or not a preliminary investigation is a quasi-
that the quasi-judicial bodies enumerated under said Rule are not exclusive. judicial proceeding, as petitioner posits, or whether or not the Secretary of Justice performs quasi-
judicial functions when he reviews the findings of a state or city prosecutor is of little moment. The Court
Petitioner's above posture, while valid to a point, will not carry the day for him. 9
wishes, however, to draw attention to what it said in Santos v. Go where the Court, citing Bautista v.
10
Court of Appeals, stated:
Rule 43 governs all appeals from the Court of Tax Appeals and quasi-judicial bodies to the CA. Section
1 thereof provides: [t]he prosecutor in a preliminary investigation does not determine the guilt or innocence of the accused.
He does not exercise adjudication nor rule-making functions. Preliminary investigation is merely
Section 1. Scope.― This Rule shall apply to appeals from judgments or final orders of the Court of Tax
inquisitorial, and is often the only means of discovering the persons who may be reasonably charged
Appeals, and from awards, judgments, final orders or resolutions of or authorized by any quasi-judicial
with a crime and to enable the fiscal [prosecutor] to prepare his complaint or information. It is not a trial
agency in the exercise of its quasi-judicial functions. Among these agencies are the Civil Service
of the case on the merits and has no purpose except that of determining whether a crime has been
Commission, Central Board of Assessment Appeals, Securities and Exchange Commission, Office of
committed and whether there is probable cause to believe that the accused is guilty thereof. While the
the President, Land Registration Authority, Social Security Commission, Civil Aeronautics Board,
fiscal [prosecutor] makes that determination, he cannot be said to be acting as a quasi-court, for it is the
courts, ultimately that pass judgment on the accused, not the fiscal [prosecutor]. (Words in bracket ours)

While now perhaps anti-climactic to delve into, the ensuing holdings of the appellate court are worth
quoting:

The petition is premature. The Information charging respondent with the crime of libel, docketed as
Criminal Case No. 114517, is now with Branch 155 of the Regional Trial Court in Pasig City. Thus
understood, the said trial court has now the control of the case. The remedy of petitioner is to reiterate
the reasons or grounds alleged in his present petition by way of an appropriate opposition to the Pasig
City Prosecution Office's "Motion to Withdraw Information" dated November 5, 1998, filed in compliance
with the assailed directive of the Secretary of Justice. Having control of the case, the trial court can look
into the claim of petitioner. This will enable the trial court to rule on the matter first without the precipitate
intervention of this Court. In other words, this is a prerequisite to the elevation of the case to this
11
Court.

In view of the foregoing disquisition, the Court deems it unnecessary to address the other issues raised
in the petition.

WHEREFORE, the instant petition is DENIED and the assailed resolution of the Court of Appeals
is AFFIRMED.

SO ORDERED.

Puno, Chairperson, Sandoval-Gutierrez, Corona, Azcuna, J.J., concur.


Republic of the Philippines The first case is G.R. No. 192935, a special civil action for prohibition instituted by petitioner Louis
SUPREME COURT Biraogo (Biraogo) in his capacity as a citizen and taxpayer. Biraogo assails Executive Order No. 1 for
6
Manila being violative of the legislative power of Congress under Section 1, Article VI of the Constitution as it
usurps the constitutional authority of the legislature to create a public office and to appropriate funds
EN BANC 7
therefor.

G.R. No. 192935 December 7, 2010 The second case, G.R. No. 193036, is a special civil action for certiorari and prohibition filed by
petitioners Edcel C. Lagman, Rodolfo B. Albano Jr., Simeon A. Datumanong, and Orlando B. Fua, Sr.
LOUIS "BAROK" C. BIRAOGO, Petitioner, (petitioners-legislators) as incumbent members of the House of Representatives.
vs.
THE PHILIPPINE TRUTH COMMISSION OF 2010, Respondent. The genesis of the foregoing cases can be traced to the events prior to the historic May 2010 elections,
when then Senator Benigno Simeon Aquino III declared his staunch condemnation of graft and
x - - - - - - - - - - - - - - - - - - - - - - -x corruption with his slogan, "Kung walang corrupt, walang mahirap." The Filipino people, convinced of
his sincerity and of his ability to carry out this noble objective, catapulted the good senator to the
G.R. No. 193036
presidency.
REP. EDCEL C. LAGMAN, REP. RODOLFO B. ALBANO, JR., REP. SIMEON A.
To transform his campaign slogan into reality, President Aquino found a need for a special body to
DATUMANONG, and REP. ORLANDO B. FUA, SR., Petitioners,
investigate reported cases of graft and corruption allegedly committed during the previous
vs.
administration.
EXECUTIVE SECRETARY PAQUITO N. OCHOA, JR. and DEPARTMENT OF BUDGET AND
MANAGEMENT SECRETARY FLORENCIO B. ABAD, Respondents. Thus, at the dawn of his administration, the President on July 30, 2010, signed Executive Order No. 1
establishing the Philippine Truth Commission of 2010 (Truth Commission). Pertinent provisions of said
DECISION
executive order read:
MENDOZA, J.:
EXECUTIVE ORDER NO. 1
When the judiciary mediates to allocate constitutional boundaries, it does not assert any superiority over CREATING THE PHILIPPINE TRUTH COMMISSION OF 2010
the other departments; it does not in reality nullify or invalidate an act of the legislature, but only asserts
WHEREAS, Article XI, Section 1 of the 1987 Constitution of the Philippines solemnly enshrines the
the solemn and sacred obligation assigned to it by the Constitution to determine conflicting claims of
principle that a public office is a public trust and mandates that public officers and employees, who are
authority under the Constitution and to establish for the parties in an actual controversy the rights which
servants of the people, must at all times be accountable to the latter, serve them with utmost
that instrument secures and guarantees to them.
responsibility, integrity, loyalty and efficiency, act with patriotism and justice, and lead modest lives;
1
--- Justice Jose P. Laurel
WHEREAS, corruption is among the most despicable acts of defiance of this principle and notorious
The role of the Constitution cannot be overlooked. It is through the Constitution that the fundamental violation of this mandate;
powers of government are established, limited and defined, and by which these powers are distributed
2 WHEREAS, corruption is an evil and scourge which seriously affects the political, economic, and social
among the several departments. The Constitution is the basic and paramount law to which all other
life of a nation; in a very special way it inflicts untold misfortune and misery on the poor, the
laws must conform and to which all persons, including the highest officials of the land, must
3 marginalized and underprivileged sector of society;
defer. Constitutional doctrines must remain steadfast no matter what may be the tides of time. It cannot
be simply made to sway and accommodate the call of situations and much more tailor itself to the WHEREAS, corruption in the Philippines has reached very alarming levels, and undermined the
4
whims and caprices of government and the people who run it. people’s trust and confidence in the Government and its institutions;
5
For consideration before the Court are two consolidated cases both of which essentially assail the WHEREAS, there is an urgent call for the determination of the truth regarding certain reports of large
validity and constitutionality of Executive Order No. 1, dated July 30, 2010, entitled "Creating the scale graft and corruption in the government and to put a closure to them by the filing of the appropriate
Philippine Truth Commission of 2010." cases against those involved, if warranted, and to deter others from committing the evil, restore the
people’s faith and confidence in the Government and in their public servants;
WHEREAS, the President’s battlecry during his campaign for the Presidency in the last elections "kung d) Upon proper request and representation, obtain information from the courts, including the
walang corrupt, walang mahirap" expresses a solemn pledge that if elected, he would end corruption Sandiganbayan and the Office of the Court Administrator, information or documents in respect to
and the evil it breeds; corruption cases filed with the Sandiganbayan or the regular courts, as the case may be;

WHEREAS, there is a need for a separate body dedicated solely to investigating and finding out the e) Invite or subpoena witnesses and take their testimonies and for that purpose, administer oaths or
truth concerning the reported cases of graft and corruption during the previous administration, and affirmations as the case may be;
which will recommend the prosecution of the offenders and secure justice for all;
f) Recommend, in cases where there is a need to utilize any person as a state witness to ensure that
WHEREAS, Book III, Chapter 10, Section 31 of Executive Order No. 292, otherwise known as the the ends of justice be fully served, that such person who qualifies as a state witness under the Revised
Revised Administrative Code of the Philippines, gives the President the continuing authority to Rules of Court of the Philippines be admitted for that purpose;
reorganize the Office of the President.
g) Turn over from time to time, for expeditious prosecution, to the appropriate prosecutorial authorities,
NOW, THEREFORE, I, BENIGNO SIMEON AQUINO III, President of the Republic of the Philippines, by means of a special or interim report and recommendation, all evidence on corruption of public
by virtue of the powers vested in me by law, do hereby order: officers and employees and their private sector co-principals, accomplices or accessories, if any, when
in the course of its investigation the Commission finds that there is reasonable ground to believe that
SECTION 1. Creation of a Commission. – There is hereby created the PHILIPPINE TRUTH they are liable for graft and corruption under pertinent applicable laws;
COMMISSION, hereinafter referred to as the "COMMISSION," which shall primarily seek and find the
truth on, and toward this end, investigate reports of graft and corruption of such scale and magnitude h) Call upon any government investigative or prosecutorial agency such as the Department of Justice or
that shock and offend the moral and ethical sensibilities of the people, committed by public officers and any of the agencies under it, and the Presidential Anti-Graft Commission, for such assistance and
employees, their co-principals, accomplices and accessories from the private sector, if any, during the cooperation as it may require in the discharge of its functions and duties;
previous administration; and thereafter recommend the appropriate action or measure to be taken
thereon to ensure that the full measure of justice shall be served without fear or favor. i) Engage or contract the services of resource persons, professionals and other personnel determined
by it as necessary to carry out its mandate;
The Commission shall be composed of a Chairman and four (4) members who will act as an
independent collegial body. j) Promulgate its rules and regulations or rules of procedure it deems necessary to effectively and
efficiently carry out the objectives of this Executive Order and to ensure the orderly conduct of its
SECTION 2. Powers and Functions. – The Commission, which shall have all the powers of an investigations, proceedings and hearings, including the presentation of evidence;
investigative body under Section 37, Chapter 9, Book I of the Administrative Code of 1987, is primarily
tasked to conduct a thorough fact-finding investigation of reported cases of graft and corruption referred k) Exercise such other acts incident to or are appropriate and necessary in connection with the
to in Section 1, involving third level public officers and higher, their co-principals, accomplices and objectives and purposes of this Order.
accessories from the private sector, if any, during the previous administration and thereafter submit its
finding and recommendations to the President, Congress and the Ombudsman. SECTION 3. Staffing Requirements. – x x x.

In particular, it shall: SECTION 4. Detail of Employees. – x x x.

a) Identify and determine the reported cases of such graft and corruption which it will investigate; SECTION 5. Engagement of Experts. – x x x

b) Collect, receive, review and evaluate evidence related to or regarding the cases of large scale SECTION 6. Conduct of Proceedings. – x x x.
corruption which it has chosen to investigate, and to this end require any agency, official or employee of
SECTION 7. Right to Counsel of Witnesses/Resource Persons. – x x x.
the Executive Branch, including government-owned or controlled corporations, to produce documents,
books, records and other papers; SECTION 8. Protection of Witnesses/Resource Persons. – x x x.
c) Upon proper request or representation, obtain information and documents from the Senate and the SECTION 9. Refusal to Obey Subpoena, Take Oath or Give Testimony. – Any government official or
House of Representatives records of investigations conducted by committees thereof relating to matters personnel who, without lawful excuse, fails to appear upon subpoena issued by the Commission or
or subjects being investigated by the Commission; who, appearing before the Commission refuses to take oath or affirmation, give testimony or produce
documents for inspection, when required, shall be subject to administrative disciplinary action. Any To accomplish its task, the PTC shall have all the powers of an investigative body under Section 37,
private person who does the same may be dealt with in accordance with law. Chapter 9, Book I of the Administrative Code of 1987. It is not, however, a quasi-judicial body as it
cannot adjudicate, arbitrate, resolve, settle, or render awards in disputes between contending parties.
SECTION 10. Duty to Extend Assistance to the Commission. – x x x. All it can do is gather, collect and assess evidence of graft and corruption and make recommendations.
It may have subpoena powers but it has no power to cite people in contempt, much less order their
SECTION 11. Budget for the Commission. – The Office of the President shall provide the necessary arrest. Although it is a fact-finding body, it cannot determine from such facts if probable cause exists as
funds for the Commission to ensure that it can exercise its powers, execute its functions, and perform to warrant the filing of an information in our courts of law. Needless to state, it cannot impose criminal,
its duties and responsibilities as effectively, efficiently, and expeditiously as possible. civil or administrative penalties or sanctions.
SECTION 12. Office. – x x x. The PTC is different from the truth commissions in other countries which have been created as official,
transitory and non-judicial fact-finding bodies "to establish the facts and context of serious violations of
SECTION 13. Furniture/Equipment. – x x x. 9
human rights or of international humanitarian law in a country’s past." They are usually established by
SECTION 14. Term of the Commission. – The Commission shall accomplish its mission on or before states emerging from periods of internal unrest, civil strife or authoritarianism to serve as mechanisms
December 31, 2012. for transitional justice.

SECTION 15. Publication of Final Report. – x x x. Truth commissions have been described as bodies that share the following characteristics: (1) they
examine only past events; (2) they investigate patterns of abuse committed over a period of time, as
SECTION 16. Transfer of Records and Facilities of the Commission. – x x x. opposed to a particular event; (3) they are temporary bodies that finish their work with the submission of
a report containing conclusions and recommendations; and (4) they are officially sanctioned, authorized
10
SECTION 17. Special Provision Concerning Mandate. If and when in the judgment of the President or empowered by the State. "Commission’s members are usually empowered to conduct research,
there is a need to expand the mandate of the Commission as defined in Section 1 hereof to include the support victims, and propose policy recommendations to prevent recurrence of crimes. Through their
investigation of cases and instances of graft and corruption during the prior administrations, such investigations, the commissions may aim to discover and learn more about past abuses, or formally
mandate may be so extended accordingly by way of a supplemental Executive Order. acknowledge them. They may aim to prepare the way for prosecutions and recommend institutional
11
reforms."
SECTION 18. Separability Clause. If any provision of this Order is declared unconstitutional, the same
shall not affect the validity and effectivity of the other provisions hereof. Thus, their main goals range from retribution to reconciliation. The Nuremburg and Tokyo war crime
tribunals are examples of a retributory or vindicatory body set up to try and punish those responsible for
SECTION 19. Effectivity. – This Executive Order shall take effect immediately. crimes against humanity. A form of a reconciliatory tribunal is the Truth and Reconciliation Commission
of South Africa, the principal function of which was to heal the wounds of past violence and to prevent
DONE in the City of Manila, Philippines, this 30th day of July 2010. future conflict by providing a cathartic experience for victims.

(SGD.) BENIGNO S. AQUINO III The PTC is a far cry from South Africa’s model. The latter placed more emphasis on reconciliation than
By the President: on judicial retribution, while the marching order of the PTC is the identification and punishment of
12
perpetrators. As one writer puts it:
(SGD.) PAQUITO N. OCHOA, JR.
Executive Secretary The order ruled out reconciliation. It translated the Draconian code spelled out by Aquino in his
inaugural speech: "To those who talk about reconciliation, if they mean that they would like us to simply
Nature of the Truth Commission
forget about the wrongs that they have committed in the past, we have this to say: There can be no
reconciliation without justice. When we allow crimes to go unpunished, we give consent to their
As can be gleaned from the above-quoted provisions, the Philippine Truth Commission (PTC) is a mere
occurring over and over again."
ad hoc body formed under the Office of the President with the primary task to investigate reports of graft
and corruption committed by third-level public officers and employees, their co-principals, accomplices
The Thrusts of the Petitions
and accessories during the previous administration, and thereafter to submit its finding and
recommendations to the President, Congress and the Ombudsman. Though it has been described as Barely a month after the issuance of Executive Order No. 1, the petitioners asked the Court to declare it
an "independent collegial body," it is essentially an entity within the Office of the President Proper and unconstitutional and to enjoin the PTC from performing its functions. A perusal of the arguments of the
8
subject to his control. Doubtless, it constitutes a public office, as an ad hoc body is one.
petitioners in both cases shows that they are essentially the same. The petitioners-legislators 2] E.O. No. 1 does not usurp the power of Congress to appropriate funds because there is no
summarized them in the following manner: appropriation but a mere allocation of funds already appropriated by Congress.

(a) E.O. No. 1 violates the separation of powers as it arrogates the power of the Congress to create a 3] The Truth Commission does not duplicate or supersede the functions of the Office of the
public office and appropriate funds for its operation. Ombudsman (Ombudsman) and the Department of Justice (DOJ), because it is a fact-finding body and
not a quasi-judicial body and its functions do not duplicate, supplant or erode the latter’s jurisdiction.
(b) The provision of Book III, Chapter 10, Section 31 of the Administrative Code of 1987 cannot
legitimize E.O. No. 1 because the delegated authority of the President to structurally reorganize the 4] The Truth Commission does not violate the equal protection clause because it was validly created for
Office of the President to achieve economy, simplicity and efficiency does not include the power to laudable purposes.
create an entirely new public office which was hitherto inexistent like the "Truth Commission."
The OSG then points to the continued existence and validity of other executive orders and presidential
(c) E.O. No. 1 illegally amended the Constitution and pertinent statutes when it vested the "Truth issuances creating similar bodies to justify the creation of the PTC such as Presidential Complaint and
Commission" with quasi-judicial powers duplicating, if not superseding, those of the Office of the Action Commission (PCAC) by President Ramon B. Magsaysay, Presidential Committee on
Ombudsman created under the 1987 Constitution and the Department of Justice created under the Administrative Performance Efficiency (PCAPE) by President Carlos P. Garcia and Presidential Agency
18
Administrative Code of 1987. on Reform and Government Operations (PARGO)by President Ferdinand E. Marcos.

(d) E.O. No. 1 violates the equal protection clause as it selectively targets for investigation and From the petitions, pleadings, transcripts, and memoranda, the following are the principal issues to be
prosecution officials and personnel of the previous administration as if corruption is their peculiar resolved:
species even as it excludes those of the other administrations, past and present, who may be indictable.
1. Whether or not the petitioners have the legal standing to file their respective petitions and question
(e) The creation of the "Philippine Truth Commission of 2010" violates the consistent and general Executive Order No. 1;
international practice of four decades wherein States constitute truth commissions to exclusively
investigate human rights violations, which customary practice forms part of the generally accepted 2. Whether or not Executive Order No. 1 violates the principle of separation of powers by usurping the
principles of international law which the Philippines is mandated to adhere to pursuant to the powers of Congress to create and to appropriate funds for public offices, agencies and commissions;
Declaration of Principles enshrined in the Constitution.
3. Whether or not Executive Order No. 1 supplants the powers of the Ombudsman and the DOJ;
(f) The creation of the "Truth Commission" is an exercise in futility, an adventure in partisan hostility, a
launching pad for trial/conviction by publicity and a mere populist propaganda to mistakenly impress the 4. Whether or not Executive Order No. 1 violates the equal protection clause; and
people that widespread poverty will altogether vanish if corruption is eliminated without even addressing
5. Whether or not petitioners are entitled to injunctive relief.
the other major causes of poverty.
Essential requisites for judicial review
(g) The mere fact that previous commissions were not constitutionally challenged is of no moment
because neither laches nor estoppel can bar an eventual question on the constitutionality and validity of Before proceeding to resolve the issue of the constitutionality of Executive Order No. 1, the Court needs
13
an executive issuance or even a statute." to ascertain whether the requisites for a valid exercise of its power of judicial review are present.
14
In their Consolidated Comment, the respondents, through the Office of the Solicitor Like almost all powers conferred by the Constitution, the power of judicial review is subject to
General (OSG), essentially questioned the legal standing of petitioners and defended the assailed limitations, to wit: (1) there must be an actual case or controversy calling for the exercise of judicial
executive order with the following arguments: power; (2) the person challenging the act must have the standing to question the validity of the subject
act or issuance; otherwise stated, he must have a personal and substantial interest in the case such
1] E.O. No. 1 does not arrogate the powers of Congress to create a public office because the
that he has sustained, or will sustain, direct injury as a result of its enforcement; (3) the question of
President’s executive power and power of control necessarily include the inherent power to conduct
constitutionality must be raised at the earliest opportunity; and (4) the issue of constitutionality must be
investigations to ensure that laws are faithfully executed and that, in any event, the Constitution, 19
15 16 the very lis mota of the case.
Revised Administrative Code of 1987 (E.O. No. 292), Presidential Decree (P.D.) No. 1416 (as
17
amended by P.D. No. 1772), R.A. No. 9970, and settled jurisprudence that authorize the President to Among all these limitations, only the legal standing of the petitioners has been put at issue.
create or form such bodies.
Legal Standing of the Petitioners
The OSG attacks the legal personality of the petitioners-legislators to file their petition for failure to "stranger," or in the category of a "citizen," or ‘taxpayer." In either case, he has to adequately show that
demonstrate their personal stake in the outcome of the case. It argues that the petitioners have not he is entitled to seek judicial protection. In other words, he has to make out a sufficient interest in the
shown that they have sustained or are in danger of sustaining any personal injury attributable to the vindication of the public order and the securing of relief as a "citizen" or "taxpayer.
creation of the PTC. Not claiming to be the subject of the commission’s investigations, petitioners will
20 Case law in most jurisdictions now allows both "citizen" and "taxpayer" standing in public actions. The
not sustain injury in its creation or as a result of its proceedings.
distinction was first laid down in Beauchamp v. Silk, where it was held that the plaintiff in a taxpayer’s
The Court disagrees with the OSG in questioning the legal standing of the petitioners-legislators to suit is in a different category from the plaintiff in a citizen’s suit. In the former, the plaintiff is affected by
assail Executive Order No. 1. Evidently, their petition primarily invokes usurpation of the power of the the expenditure of public funds, while in the latter, he is but the mere instrument of the public concern.
Congress as a body to which they belong as members. This certainly justifies their resolve to take the As held by the New York Supreme Court in People ex rel Case v. Collins: "In matter of mere public
cudgels for Congress as an institution and present the complaints on the usurpation of their power and right, however…the people are the real parties…It is at least the right, if not the duty, of every citizen to
rights as members of the legislature before the Court. As held in Philippine Constitution Association v. interfere and see that a public offence be properly pursued and punished, and that a public grievance
21
Enriquez, be remedied." With respect to taxpayer’s suits, Terr v. Jordan held that "the right of a citizen and a
taxpayer to maintain an action in courts to restrain the unlawful use of public funds to his injury cannot
To the extent the powers of Congress are impaired, so is the power of each member thereof, since his be denied."
office confers a right to participate in the exercise of the powers of that institution.
However, to prevent just about any person from seeking judicial interference in any official policy or act
An act of the Executive which injures the institution of Congress causes a derivative but nonetheless with which he disagreed with, and thus hinders the activities of governmental agencies engaged in
substantial injury, which can be questioned by a member of Congress. In such a case, any member of public service, the United State Supreme Court laid down the more stringent "direct injury" test in Ex
Congress can have a resort to the courts. Parte Levitt, later reaffirmed in Tileston v. Ullman. The same Court ruled that for a private individual to
invoke the judicial power to determine the validity of an executive or legislative action, he must show
Indeed, legislators have a legal standing to see to it that the prerogative, powers and privileges vested that he has sustained a direct injury as a result of that action, and it is not sufficient that he has
by the Constitution in their office remain inviolate. Thus, they are allowed to question the validity of any a general interest common to all members of the public.
22
official action which, to their mind, infringes on their prerogatives as legislators.
This Court adopted the "direct injury" test in our jurisdiction. In People v. Vera, it held that the person
With regard to Biraogo, the OSG argues that, as a taxpayer, he has no standing to question the creation who impugns the validity of a statute must have "a personal and substantial interest in the case
23
of the PTC and the budget for its operations. It emphasizes that the funds to be used for the creation such that he has sustained, or will sustain direct injury as a result." The Vera doctrine was upheld
and operation of the commission are to be taken from those funds already appropriated by Congress. in a litany of cases, such as, Custodio v. President of the Senate, Manila Race Horse Trainers’
Thus, the allocation and disbursement of funds for the commission will not entail congressional action Association v. De la Fuente, Pascual v. Secretary of Public Works and Anti-Chinese League of the
but will simply be an exercise of the President’s power over contingent funds. Philippines v. Felix. [Emphases included. Citations omitted]

As correctly pointed out by the OSG, Biraogo has not shown that he sustained, or is in danger of Notwithstanding, the Court leans on the doctrine that "the rule on standing is a matter of procedure,
sustaining, any personal and direct injury attributable to the implementation of Executive Order No. 1. hence, can be relaxed for nontraditional plaintiffs like ordinary citizens, taxpayers, and legislators when
Nowhere in his petition is an assertion of a clear right that may justify his clamor for the Court to the public interest so requires, such as when the matter is of transcendental importance, of
exercise judicial power and to wield the axe over presidential issuances in defense of the Constitution. overreaching significance to society, or of paramount public interest."
25
24
The case of David v. Arroyo explained the deep-seated rules on locus standi. Thus:
26
Thus, in Coconut Oil Refiners Association, Inc. v. Torres, the Court held that in cases of paramount
Locus standi is defined as "a right of appearance in a court of justice on a given question." In private importance where serious constitutional questions are involved, the standing requirements may be
suits, standing is governed by the "real-parties-in interest" rule as contained in Section 2, Rule 3 of the relaxed and a suit may be allowed to prosper even where there is no direct injury to the party claiming
1997 Rules of Civil Procedure, as amended. It provides that "every action must be prosecuted or 27
the right of judicial review. In the first Emergency Powers Cases, ordinary citizens and taxpayers were
defended in the name of the real party in interest." Accordingly, the "real-party-in interest" is "the allowed to question the constitutionality of several executive orders although they had only an indirect
party who stands to be benefited or injured by the judgment in the suit or the party entitled to the avails and general interest shared in common with the public.
of the suit." Succinctly put, the plaintiff’s standing is based on his own right to the relief sought.
28
The OSG claims that the determinants of transcendental importance laid down in CREBA v. ERC and
The difficulty of determining locus standi arises in public suits. Here, the plaintiff who asserts a "public 29
Meralco are non-existent in this case. The Court, however, finds reason in Biraogo’s assertion that the
right" in assailing an allegedly illegal official action, does so as a representative of the general public. He petition covers matters of transcendental importance to justify the exercise of jurisdiction by the Court.
may be a person who is affected no differently from any other person. He could be suing as a There are constitutional issues in the petition which deserve the attention of this Court in view of their
40
seriousness, novelty and weight as precedents. Where the issues are of transcendental and paramount department, bureau and office, or interfere with the discretion of his officials. The power of the
importance not only to the public but also to the Bench and the Bar, they should be resolved for the President to investigate is not limited to the exercise of his power of control over his subordinates in the
30
guidance of all. Undoubtedly, the Filipino people are more than interested to know the status of the executive branch, but extends further in the exercise of his other powers, such as his power to discipline
41 42
President’s first effort to bring about a promised change to the country. The Court takes cognizance of subordinates, his power for rule making, adjudication and licensing purposes and in order to be
43
the petition not due to overwhelming political undertones that clothe the issue in the eyes of the public, informed on matters which he is entitled to know.
but because the Court stands firm in its oath to perform its constitutional duty to settle legal
44
controversies with overreaching significance to society. The OSG also cites the recent case of Banda v. Ermita, where it was held that the President has the
power to reorganize the offices and agencies in the executive department in line with his constitutionally
Power of the President to Create the Truth Commission granted power of control and by virtue of a valid delegation of the legislative power to reorganize
executive offices under existing statutes.
In his memorandum in G.R. No. 192935, Biraogo asserts that the Truth Commission is a public office
31
and not merely an adjunct body of the Office of the President. Thus, in order that the President may Thus, the OSG concludes that the power of control necessarily includes the power to create offices. For
create a public office he must be empowered by the Constitution, a statute or an authorization vested in the OSG, the President may create the PTC in order to, among others, put a closure to the reported
32 45
him by law. According to petitioner, such power cannot be presumed since there is no provision in the large scale graft and corruption in the government.
33
Constitution or any specific law that authorizes the President to create a truth commission. He adds
that Section 31 of the Administrative Code of 1987, granting the President the continuing authority to The question, therefore, before the Court is this: Does the creation of the PTC fall within the ambit of the
reorganize his office, cannot serve as basis for the creation of a truth commission considering the power to reorganize as expressed in Section 31 of the Revised Administrative Code? Section 31
aforesaid provision merely uses verbs such as "reorganize," "transfer," "consolidate," "merge," and contemplates "reorganization" as limited by the following functional and structural lines: (1) restructuring
34 the internal organization of the Office of the President Proper by abolishing, consolidating or merging
"abolish." Insofar as it vests in the President the plenary power to reorganize the Office of the
President to the extent of creating a public office, Section 31 is inconsistent with the principle of units thereof or transferring functions from one unit to another; (2) transferring any function under the
separation of powers enshrined in the Constitution and must be deemed repealed upon the effectivity Office of the President to any other Department/Agency or vice versa; or (3) transferring any agency
35 under the Office of the President to any other Department/Agency or vice versa. Clearly, the provision
thereof.
refers to reduction of personnel, consolidation of offices, or abolition thereof by reason of economy or
Similarly, in G.R. No. 193036, petitioners-legislators argue that the creation of a public office lies within redundancy of functions. These point to situations where a body or an office is already existent but a
the province of Congress and not with the executive branch of government. They maintain that the modification or alteration thereof has to be effected. The creation of an office is nowhere mentioned,
delegated authority of the President to reorganize under Section 31 of the Revised Administrative Code: much less envisioned in said provision. Accordingly, the answer to the question is in the negative.
1) does not permit the President to create a public office, much less a truth commission; 2) is limited to
the reorganization of the administrative structure of the Office of the President; 3) is limited to the To say that the PTC is borne out of a restructuring of the Office of the President under Section 31 is a
restructuring of the internal organs of the Office of the President Proper, transfer of functions and misplaced supposition, even in the plainest meaning attributable to the term "restructure"– an "alteration
36 of an existing structure." Evidently, the PTC was not part of the structure of the Office of the President
transfer of agencies; and 4) only to achieve simplicity, economy and efficiency. Such continuing
authority of the President to reorganize his office is limited, and by issuing Executive Order No. 1, the prior to the enactment of Executive Order No. 1. As held in Buklod ng Kawaning EIIB v. Hon. Executive
46
President overstepped the limits of this delegated authority. Secretary,

The OSG counters that there is nothing exclusively legislative about the creation by the President of a But of course, the list of legal basis authorizing the President to reorganize any department or agency in
fact-finding body such as a truth commission. Pointing to numerous offices created by past presidents, it the executive branch does not have to end here. We must not lose sight of the very source of the power
argues that the authority of the President to create public offices within the Office of the President – that which constitutes an express grant of power. Under Section 31, Book III of Executive Order No.
37 292 (otherwise known as the Administrative Code of 1987), "the President, subject to the policy in the
Proper has long been recognized. According to the OSG, the Executive, just like the other two
branches of government, possesses the inherent authority to create fact-finding committees to assist it Executive Office and in order to achieve simplicity, economy and efficiency, shall have the continuing
in the performance of its constitutionally mandated functions and in the exercise of its administrative authority to reorganize the administrative structure of the Office of the President." For this purpose, he
38 may transfer the functions of other Departments or Agencies to the Office of the President. In
functions. This power, as the OSG explains it, is but an adjunct of the plenary powers wielded by the
President under Section 1 and his power of control under Section 17, both of Article VII of the Canonizado v. Aguirre [323 SCRA 312 (2000)], we ruled that reorganization "involves the reduction of
39 personnel, consolidation of offices, or abolition thereof by reason of economy or redundancy of
Constitution.
functions." It takes place when there is an alteration of the existing structure of government offices or
It contends that the President is necessarily vested with the power to conduct fact-finding investigations, units therein, including the lines of control, authority and responsibility between them. The EIIB is a
pursuant to his duty to ensure that all laws are enforced by public officials and employees of his bureau attached to the Department of Finance. It falls under the Office of the President. Hence, it is
department and in the exercise of his authority to assume directly the functions of the executive subject to the President’s continuing authority to reorganize. [Emphasis Supplied]
50
In the same vein, the creation of the PTC is not justified by the President’s power of control. Control is SOLICITOR GENERAL CADIZ: Yes, Your Honor.
essentially the power to alter or modify or nullify or set aside what a subordinate officer had done in the
47 While the power to create a truth commission cannot pass muster on the basis of P.D. No. 1416 as
performance of his duties and to substitute the judgment of the former with that of the latter. Clearly,
the power of control is entirely different from the power to create public offices. The former is inherent in amended by P.D. No. 1772, the creation of the PTC finds justification under Section 17, Article VII of the
the Executive, while the latter finds basis from either a valid delegation from Congress, or his inherent Constitution, imposing upon the President the duty to ensure that the laws are faithfully executed.
duty to faithfully execute the laws. Section 17 reads:

The question is this, is there a valid delegation of power from Congress, empowering the President to Section 17. The President shall have control of all the executive departments, bureaus, and offices. He
create a public office? shall ensure that the laws be faithfully executed. (Emphasis supplied).

According to the OSG, the power to create a truth commission pursuant to the above provision finds As correctly pointed out by the respondents, the allocation of power in the three principal branches of
48
statutory basis under P.D. 1416, as amended by P.D. No. 1772. The said law granted the President government is a grant of all powers inherent in them. The President’s power to conduct investigations to
the continuing authority to reorganize the national government, including the power to group, aid him in ensuring the faithful execution of laws – in this case, fundamental laws on public
consolidate bureaus and agencies, to abolish offices, to transfer functions, to create and classify accountability and transparency – is inherent in the President’s powers as the Chief Executive. That the
functions, services and activities, transfer appropriations, and to standardize salaries and materials. authority of the President to conduct investigations and to create bodies to execute this power is not
This decree, in relation to Section 20, Title I, Book III of E.O. 292 has been invoked in several cases explicitly mentioned in the Constitution or in statutes does not mean that he is bereft of such
49 51 52
such as Larin v. Executive Secretary. authority. As explained in the landmark case of Marcos v. Manglapus:

The Court, however, declines to recognize P.D. No. 1416 as a justification for the President to create a x x x. The 1987 Constitution, however, brought back the presidential system of government and
public office. Said decree is already stale, anachronistic and inoperable. P.D. No. 1416 was a restored the separation of legislative, executive and judicial powers by their actual distribution among
delegation to then President Marcos of the authority to reorganize the administrative structure of the three distinct branches of government with provision for checks and balances.
national government including the power to create offices and transfer appropriations pursuant to one of
the purposes of the decree, embodied in its last "Whereas" clause: It would not be accurate, however, to state that "executive power" is the power to enforce the laws, for
the President is head of state as well as head of government and whatever powers inhere in such
WHEREAS, the transition towards the parliamentary form of government will necessitate flexibility in positions pertain to the office unless the Constitution itself withholds it. Furthermore, the Constitution
the organization of the national government. itself provides that the execution of the laws is only one of the powers of the President. It also grants the
President other powers that do not involve the execution of any provision of law, e.g., his power over
Clearly, as it was only for the purpose of providing manageability and resiliency during the interim, P.D. the country's foreign relations.
No. 1416, as amended by P.D. No. 1772, became functus oficio upon the convening of the First
Congress, as expressly provided in Section 6, Article XVIII of the 1987 Constitution. In fact, even the On these premises, we hold the view that although the 1987 Constitution imposes limitations on the
Solicitor General agrees with this view. Thus: exercise of specific powers of the President, it maintains intact what is traditionally considered as within
the scope of "executive power." Corollarily, the powers of the President cannot be said to be limited only
ASSOCIATE JUSTICE CARPIO: Because P.D. 1416 was enacted was the last whereas clause of P.D. to the specific powers enumerated in the Constitution. In other words, executive power is more than the
1416 says "it was enacted to prepare the transition from presidential to parliamentary. Now, in a sum of specific powers so enumerated.
parliamentary form of government, the legislative and executive powers are fused, correct?
It has been advanced that whatever power inherent in the government that is neither legislative nor
SOLICITOR GENERAL CADIZ: Yes, Your Honor. judicial has to be executive. x x x.

ASSOCIATE JUSTICE CARPIO: That is why, that P.D. 1416 was issued. Now would you agree with me Indeed, the Executive is given much leeway in ensuring that our laws are faithfully executed. As stated
that P.D. 1416 should not be considered effective anymore upon the promulgation, adoption, ratification above, the powers of the President are not limited to those specific powers under the
53
of the 1987 Constitution. Constitution. One of the recognized powers of the President granted pursuant to this constitutionally-
mandated duty is the power to create ad hoc committees. This flows from the obvious need to ascertain
SOLICITOR GENERAL CADIZ: Not the whole of P.D. [No.] 1416, Your Honor. facts and determine if laws have been faithfully executed. Thus, in Department of Health v.
54
Camposano, the authority of the President to issue Administrative Order No. 298, creating an
ASSOCIATE JUSTICE CARPIO: The power of the President to reorganize the entire National investigative committee to look into the administrative charges filed against the employees of the
Government is deemed repealed, at least, upon the adoption of the 1987 Constitution, correct. Department of Health for the anomalous purchase of medicines was upheld. In said case, it was ruled:
The Chief Executive’s power to create the Ad hoc Investigating Committee cannot be doubted. "Investigate," commonly understood, means to examine, explore, inquire or delve or probe into,
Having been constitutionally granted full control of the Executive Department, to which respondents research on, study. The dictionary definition of "investigate" is "to observe or study closely: inquire into
belong, the President has the obligation to ensure that all executive officials and employees faithfully systematically: "to search or inquire into: x x to subject to an official probe x x: to conduct an official
comply with the law. With AO 298 as mandate, the legality of the investigation is sustained. Such inquiry." The purpose of investigation, of course, is to discover, to find out, to learn, obtain information.
validity is not affected by the fact that the investigating team and the PCAGC had the same Nowhere included or intimated is the notion of settling, deciding or resolving a controversy involved in
composition, or that the former used the offices and facilities of the latter in conducting the inquiry. the facts inquired into by application of the law to the facts established by the inquiry.
[Emphasis supplied]
The legal meaning of "investigate" is essentially the same: "(t)o follow up step by step by patient inquiry
It should be stressed that the purpose of allowing ad hoc investigating bodies to exist is to allow an or observation. To trace or track; to search into; to examine and inquire into with care and accuracy; to
inquiry into matters which the President is entitled to know so that he can be properly advised and find out by careful inquisition; examination; the taking of evidence; a legal inquiry;" "to inquire; to make
guided in the performance of his duties relative to the execution and enforcement of the laws of the an investigation," "investigation" being in turn described as "(a)n administrative function, the exercise of
land. And if history is to be revisited, this was also the objective of the investigative bodies created in which ordinarily does not require a hearing. 2 Am J2d Adm L Sec. 257; x x an inquiry, judicial or
the past like the PCAC, PCAPE, PARGO, the Feliciano Commission, the Melo Commission and the otherwise, for the discovery and collection of facts concerning a certain matter or matters."
Zenarosa Commission. There being no changes in the government structure, the Court is not inclined to
declare such executive power as non-existent just because the direction of the political winds have "Adjudicate," commonly or popularly understood, means to adjudge, arbitrate, judge, decide, determine,
changed. resolve, rule on, settle. The dictionary defines the term as "to settle finally (the rights and duties of the
parties to a court case) on the merits of issues raised: x x to pass judgment on: settle judicially: x x act
On the charge that Executive Order No. 1 transgresses the power of Congress to appropriate funds for as judge." And "adjudge" means "to decide or rule upon as a judge or with judicial or quasi-judicial
the operation of a public office, suffice it to say that there will be no appropriation but only an allotment powers: x x to award or grant judicially in a case of controversy x x."
or allocations of existing funds already appropriated. Accordingly, there is no usurpation on the part of
the Executive of the power of Congress to appropriate funds. Further, there is no need to specify the In the legal sense, "adjudicate" means: "To settle in the exercise of judicial authority. To determine
amount to be earmarked for the operation of the commission because, in the words of the Solicitor finally. Synonymous with adjudge in its strictest sense;" and "adjudge" means: "To pass on judicially, to
General, "whatever funds the Congress has provided for the Office of the President will be the very decide, settle or decree, or to sentence or condemn. x x. Implies a judicial determination of a fact, and
55 the entry of a judgment." [Italics included. Citations Omitted]
source of the funds for the commission." Moreover, since the amount that would be allocated to the
PTC shall be subject to existing auditing rules and regulations, there is no impropriety in the funding.
Fact-finding is not adjudication and it cannot be likened to the judicial function of a court of justice, or
Power of the Truth Commission to Investigate even a quasi-judicial agency or office. The function of receiving evidence and ascertaining therefrom the
facts of a controversy is not a judicial function. To be considered as such, the act of receiving evidence
The President’s power to conduct investigations to ensure that laws are faithfully executed is well and arriving at factual conclusions in a controversy must be accompanied by the authority of applying
recognized. It flows from the faithful-execution clause of the Constitution under Article VII, Section 17 the law to the factual conclusions to the end that the controversy may be decided or resolved
56
thereof. As the Chief Executive, the president represents the government as a whole and sees to it authoritatively, finally and definitively, subject to appeals or modes of review as may be provided by
60 61
that all laws are enforced by the officials and employees of his department. He has the authority to law. Even respondents themselves admit that the commission is bereft of any quasi-judicial power.
57
directly assume the functions of the executive department.
Contrary to petitioners’ apprehension, the PTC will not supplant the Ombudsman or the DOJ or erode
Invoking this authority, the President constituted the PTC to primarily investigate reports of graft and their respective powers. If at all, the investigative function of the commission will complement those of
corruption and to recommend the appropriate action. As previously stated, no quasi-judicial powers the two offices. As pointed out by the Solicitor General, the recommendation to prosecute is but a
62
have been vested in the said body as it cannot adjudicate rights of persons who come before it. It has consequence of the overall task of the commission to conduct a fact-finding investigation." The actual
been said that "Quasi-judicial powers involve the power to hear and determine questions of fact to prosecution of suspected offenders, much less adjudication on the merits of the charges against
63
which the legislative policy is to apply and to decide in accordance with the standards laid down by law them, is certainly not a function given to the commission. The phrase, "when in the course of its
58
itself in enforcing and administering the same law." In simpler terms, judicial discretion is involved in investigation," under Section 2(g), highlights this fact and gives credence to a contrary interpretation
the exercise of these quasi-judicial power, such that it is exclusively vested in the judiciary and must be from that of the petitioners. The function of determining probable cause for the filing of the appropriate
64
clearly authorized by the legislature in the case of administrative agencies. complaints before the courts remains to be with the DOJ and the Ombudsman.

The distinction between the power to investigate and the power to adjudicate was delineated by the At any rate, the Ombudsman’s power to investigate under R.A. No. 6770 is not exclusive but is shared
59 65
Court in Cariño v. Commission on Human Rights. Thus: with other similarly authorized government agencies. Thus, in the case of Ombudsman v. Galicia, it
was written:
This power of investigation granted to the Ombudsman by the 1987 Constitution and The Ombudsman The petitioners assail Executive Order No. 1 because it is violative of this constitutional safeguard. They
Act is not exclusive but is shared with other similarly authorized government agencies such as the contend that it does not apply equally to all members of the same class such that the intent of singling
PCGG and judges of municipal trial courts and municipal circuit trial courts. The power to conduct out the "previous administration" as its sole object makes the PTC an "adventure in partisan
66
preliminary investigation on charges against public employees and officials is likewise concurrently hostility." Thus, in order to be accorded with validity, the commission must also cover reports of graft
67
shared with the Department of Justice. Despite the passage of the Local Government Code in 1991, the and corruption in virtually all administrations previous to that of former President Arroyo.
Ombudsman retains concurrent jurisdiction with the Office of the President and the local Sanggunians
to investigate complaints against local elective officials. [Emphasis supplied]. The petitioners argue that the search for truth behind the reported cases of graft and corruption must
encompass acts committed not only during the administration of former President Arroyo but also during
68
Also, Executive Order No. 1 cannot contravene the power of the Ombudsman to investigate criminal prior administrations where the "same magnitude of controversies and anomalies" were reported to
cases under Section 15 (1) of R.A. No. 6770, which states: have been committed against the Filipino people. They assail the classification formulated by the
respondents as it does not fall under the recognized exceptions because first, "there is no substantial
(1) Investigate and prosecute on its own or on complaint by any person, any act or omission of any distinction between the group of officials targeted for investigation by Executive Order No. 1 and other
public officer or employee, office or agency, when such act or omission appears to be illegal, unjust, groups or persons who abused their public office for personal gain; and second, the selective
improper or inefficient. It has primary jurisdiction over cases cognizable by the Sandiganbayan and, in 69
classification is not germane to the purpose of Executive Order No. 1 to end corruption." In order to
the exercise of its primary jurisdiction, it may take over, at any stage, from any investigatory agency of attain constitutional permission, the petitioners advocate that the commission should deal with "graft
government, the investigation of such cases. [Emphases supplied] and grafters prior and subsequent to the Arroyo administration with the strong arm of the law with equal
70
force."
The act of investigation by the Ombudsman as enunciated above contemplates the conduct of a
preliminary investigation or the determination of the existence of probable cause. This is categorically Position of respondents
out of the PTC’s sphere of functions. Its power to investigate is limited to obtaining facts so that it can
advise and guide the President in the performance of his duties relative to the execution and According to respondents, while Executive Order No. 1 identifies the "previous administration" as the
enforcement of the laws of the land. In this regard, the PTC commits no act of usurpation of the initial subject of the investigation, following Section 17 thereof, the PTC will not confine itself to cases of
71
Ombudsman’s primordial duties. large scale graft and corruption solely during the said administration. Assuming arguendo that the
commission would confine its proceedings to officials of the previous administration, the petitioners
The same holds true with respect to the DOJ. Its authority under Section 3 (2), Chapter 1, Title III, Book argue that no offense is committed against the equal protection clause for "the segregation of the
IV in the Revised Administrative Code is by no means exclusive and, thus, can be shared with a body transactions of public officers during the previous administration as possible subjects of investigation is
likewise tasked to investigate the commission of crimes. a valid classification based on substantial distinctions and is germane to the evils which the Executive
72
Order seeks to correct." To distinguish the Arroyo administration from past administrations, it recited
Finally, nowhere in Executive Order No. 1 can it be inferred that the findings of the PTC are to be the following:
accorded conclusiveness. Much like its predecessors, the Davide Commission, the Feliciano
Commission and the Zenarosa Commission, its findings would, at best, be recommendatory in nature. First. E.O. No. 1 was issued in view of widespread reports of large scale graft and corruption in the
And being so, the Ombudsman and the DOJ have a wider degree of latitude to decide whether or not to previous administration which have eroded public confidence in public institutions. There is, therefore,
reject the recommendation. These offices, therefore, are not deprived of their mandated duties but will an urgent call for the determination of the truth regarding certain reports of large scale graft and
instead be aided by the reports of the PTC for possible indictments for violations of graft laws. corruption in the government and to put a closure to them by the filing of the appropriate cases against
those involved, if warranted, and to deter others from committing the evil, restore the people’s faith and
Violation of the Equal Protection Clause confidence in the Government and in their public servants.

Although the purpose of the Truth Commission falls within the investigative power of the President, the Second. The segregation of the preceding administration as the object of fact-finding is warranted by
Court finds difficulty in upholding the constitutionality of Executive Order No. 1 in view of its apparent the reality that unlike with administrations long gone, the current administration will most likely bear the
transgression of the equal protection clause enshrined in Section 1, Article III (Bill of Rights) of the 1987 immediate consequence of the policies of the previous administration.
Constitution. Section 1 reads:
Third. The classification of the previous administration as a separate class for investigation lies in the
Section 1. No person shall be deprived of life, liberty, or property without due process of law, nor shall reality that the evidence of possible criminal activity, the evidence that could lead to recovery of public
any person be denied the equal protection of the laws. monies illegally dissipated, the policy lessons to be learned to ensure that anti-corruption laws are
faithfully executed, are more easily established in the regime that immediately precede the current
administration.
Fourth. Many administrations subject the transactions of their predecessors to investigations to provide the class are not similarly treated, both as to rights conferred and obligations imposed. It is not
closure to issues that are pivotal to national life or even as a routine measure of due diligence and good necessary that the classification be made with absolute symmetry, in the sense that the members of the
housekeeping by a nascent administration like the Presidential Commission on Good Government class should possess the same characteristics in equal degree. Substantial similarity will suffice; and as
(PCGG), created by the late President Corazon C. Aquino under Executive Order No. 1 to pursue the long as this is achieved, all those covered by the classification are to be treated equally. The mere fact
recovery of ill-gotten wealth of her predecessor former President Ferdinand Marcos and his cronies, that an individual belonging to a class differs from the other members, as long as that class is
84
and the Saguisag Commission created by former President Joseph Estrada under Administrative Order substantially distinguishable from all others, does not justify the non-application of the law to him."
No, 53, to form an ad-hoc and independent citizens’ committee to investigate all the facts and
circumstances surrounding "Philippine Centennial projects" of his predecessor, former President Fidel The classification must not be based on existing circumstances only, or so constituted as to preclude
73 addition to the number included in the class. It must be of such a nature as to embrace all those who
V. Ramos. [Emphases supplied]
may thereafter be in similar circumstances and conditions. It must not leave out or "underinclude" those
Concept of the Equal Protection Clause that should otherwise fall into a certain classification. As elucidated in Victoriano v. Elizalde Rope
85 86
Workers' Union and reiterated in a long line of cases,
One of the basic principles on which this government was founded is that of the equality of right which is
embodied in Section 1, Article III of the 1987 Constitution. The equal protection of the laws is embraced The guaranty of equal protection of the laws is not a guaranty of equality in the application of the laws
in the concept of due process, as every unfair discrimination offends the requirements of justice and fair upon all citizens of the state. It is not, therefore, a requirement, in order to avoid the constitutional
play. It has been embodied in a separate clause, however, to provide for a more specific guaranty prohibition against inequality, that every man, woman and child should be affected alike by a statute.
against any form of undue favoritism or hostility from the government. Arbitrariness in general may be Equality of operation of statutes does not mean indiscriminate operation on persons merely as such, but
challenged on the basis of the due process clause. But if the particular act assailed partakes of an on persons according to the circumstances surrounding them. It guarantees equality, not identity of
74
unwarranted partiality or prejudice, the sharper weapon to cut it down is the equal protection clause. rights. The Constitution does not require that things which are different in fact be treated in law as
though they were the same. The equal protection clause does not forbid discrimination as to things that
"According to a long line of decisions, equal protection simply requires that all persons or things are different. It does not prohibit legislation which is limited either in the object to which it is directed or
75
similarly situated should be treated alike, both as to rights conferred and responsibilities imposed." It by the territory within which it is to operate.
76
"requires public bodies and institutions to treat similarly situated individuals in a similar manner." "The
purpose of the equal protection clause is to secure every person within a state’s jurisdiction against The equal protection of the laws clause of the Constitution allows classification. Classification in law, as
intentional and arbitrary discrimination, whether occasioned by the express terms of a statue or by its in the other departments of knowledge or practice, is the grouping of things in speculation or practice
77
improper execution through the state’s duly constituted authorities." "In other words, the concept of because they agree with one another in certain particulars. A law is not invalid because of simple
equal justice under the law requires the state to govern impartially, and it may not draw distinctions inequality. The very idea of classification is that of inequality, so that it goes without saying that the
78
between individuals solely on differences that are irrelevant to a legitimate governmental objective." mere fact of inequality in no manner determines the matter of constitutionality. All that is required of a
valid classification is that it be reasonable, which means that the classification should be based on
79
The equal protection clause is aimed at all official state actions, not just those of the legislature. Its substantial distinctions which make for real differences, that it must be germane to the purpose of the
inhibitions cover all the departments of the government including the political and executive law; that it must not be limited to existing conditions only; and that it must apply equally to each member
departments, and extend to all actions of a state denying equal protection of the laws, through whatever of the class. This Court has held that the standard is satisfied if the classification or distinction is based
80
agency or whatever guise is taken. on a reasonable foundation or rational basis and is not palpably arbitrary. [Citations omitted]

It, however, does not require the universal application of the laws to all persons or things without Applying these precepts to this case, Executive Order No. 1 should be struck down as violative of the
distinction. What it simply requires is equality among equals as determined according to a valid equal protection clause. The clear mandate of the envisioned truth commission is to investigate and find
classification. Indeed, the equal protection clause permits classification. Such classification, however, to out the truth "concerning the reported cases of graft and corruption during the previous
be valid must pass the test of reasonableness. The test has four requisites: (1) The classification rests 87
administration" only. The intent to single out the previous administration is plain, patent and manifest.
on substantial distinctions; (2) It is germane to the purpose of the law; (3) It is not limited to existing Mention of it has been made in at least three portions of the questioned executive order. Specifically,
conditions only; and these are:
81
(4) It applies equally to all members of the same class. "Superficial differences do not make for a valid WHEREAS, there is a need for a separate body dedicated solely to investigating and finding out the
82
classification." truth concerning the reported cases of graft and corruption during the previous administration, and
which will recommend the prosecution of the offenders and secure justice for all;
For a classification to meet the requirements of constitutionality, it must include or embrace all persons
83
who naturally belong to the class. "The classification will be regarded as invalid if all the members of
SECTION 1. Creation of a Commission. – There is hereby created the PHILIPPINE TRUTH investigate all past administrations. While reasonable prioritization is permitted, it should not be
COMMISSION, hereinafter referred to as the "COMMISSION," which shall primarily seek and find the arbitrary lest it be struck down for being unconstitutional. In the often quoted language of Yick Wo v.
92
truth on, and toward this end, investigate reports of graft and corruption of such scale and magnitude Hopkins,
that shock and offend the moral and ethical sensibilities of the people, committed by public officers and
employees, their co-principals, accomplices and accessories from the private sector, if any, during the Though the law itself be fair on its face and impartial in appearance, yet, if applied and administered by
previous administration; and thereafter recommend the appropriate action or measure to be taken public authority with an evil eye and an unequal hand, so as practically to make unjust and illegal
thereon to ensure that the full measure of justice shall be served without fear or favor. discriminations between persons in similar circumstances, material to their rights, the denial of equal
justice is still within the prohibition of the constitution. [Emphasis supplied]
SECTION 2. Powers and Functions. – The Commission, which shall have all the powers of an
investigative body under Section 37, Chapter 9, Book I of the Administrative Code of 1987, is primarily It could be argued that considering that the PTC is an ad hoc body, its scope is limited. The Court,
tasked to conduct a thorough fact-finding investigation of reported cases of graft and corruption referred however, is of the considered view that although its focus is restricted, the constitutional guarantee of
to in Section 1, involving third level public officers and higher, their co-principals, accomplices and equal protection under the laws should not in any way be circumvented. The Constitution is the
accessories from the private sector, if any, during the previous administration and thereafter submit its fundamental and paramount law of the nation to which all other laws must conform and in accordance
93
finding and recommendations to the President, Congress and the Ombudsman. [Emphases supplied] with which all private rights determined and all public authority administered. Laws that do not conform
94
to the Constitution should be stricken down for being unconstitutional. While the thrust of the PTC is
In this regard, it must be borne in mind that the Arroyo administration is but just a member of a class, specific, that is, for investigation of acts of graft and corruption, Executive Order No. 1, to survive, must
that is, a class of past administrations. It is not a class of its own. Not to include past administrations be read together with the provisions of the Constitution. To exclude the earlier administrations in the
similarly situated constitutes arbitrariness which the equal protection clause cannot sanction. Such guise of "substantial distinctions" would only confirm the petitioners’ lament that the subject executive
95
discriminating differentiation clearly reverberates to label the commission as a vehicle for vindictiveness order is only an "adventure in partisan hostility." In the case of US v. Cyprian, it was written: "A rather
and selective retribution. limited number of such classifications have routinely been held or assumed to be arbitrary; those
include: race, national origin, gender, political activity or membership in a political party, union activity or
Though the OSG enumerates several differences between the Arroyo administration and other past membership in a labor union, or more generally the exercise of first amendment rights."
administrations, these distinctions are not substantial enough to merit the restriction of the investigation
to the "previous administration" only. The reports of widespread corruption in the Arroyo administration To reiterate, in order for a classification to meet the requirements of constitutionality, it must include or
96
cannot be taken as basis for distinguishing said administration from earlier administrations which were embrace all persons who naturally belong to the class. "Such a classification must not be based on
also blemished by similar widespread reports of impropriety. They are not inherent in, and do not inure existing circumstances only, or so constituted as to preclude additions to the number included within a
solely to, the Arroyo administration. As Justice Isagani Cruz put it, "Superficial differences do not make class, but must be of such a nature as to embrace all those who may thereafter be in similar
88
for a valid classification." circumstances and conditions. Furthermore, all who are in situations and circumstances which are
relative to the discriminatory legislation and which are indistinguishable from those of the members of
The public needs to be enlightened why Executive Order No. 1 chooses to limit the scope of the the class must be brought under the influence of the law and treated by it in the same way as are the
intended investigation to the previous administration only. The OSG ventures to opine that "to include 97
members of the class."
other past administrations, at this point, may unnecessarily overburden the commission and lead it to
89
lose its effectiveness." The reason given is specious. It is without doubt irrelevant to the legitimate and The Court is not unaware that "mere underinclusiveness is not fatal to the validity of a law under the
90 98
noble objective of the PTC to stamp out or "end corruption and the evil it breeds." equal protection clause." "Legislation is not unconstitutional merely because it is not all-embracing and
99
does not include all the evils within its reach." It has been written that a regulation challenged under
The probability that there would be difficulty in unearthing evidence or that the earlier reports involving the equal protection clause is not devoid of a rational predicate simply because it happens to be
the earlier administrations were already inquired into is beside the point. Obviously, deceased 100
incomplete. In several instances, the underinclusiveness was not considered a valid reason to strike
presidents and cases which have already prescribed can no longer be the subjects of inquiry by the down a law or regulation where the purpose can be attained in future legislations or regulations. These
PTC. Neither is the PTC expected to conduct simultaneous investigations of previous administrations, 101
cases refer to the "step by step" process. "With regard to equal protection claims, a legislature does
given the body’s limited time and resources. "The law does not require the impossible" (Lex non cogit not run the risk of losing the entire remedial scheme simply because it fails, through inadvertence or
91 102
ad impossibilia). otherwise, to cover every evil that might conceivably have been attacked."

Given the foregoing physical and legal impossibility, the Court logically recognizes the unfeasibility of In Executive Order No. 1, however, there is no inadvertence. That the previous administration was
investigating almost a century’s worth of graft cases. However, the fact remains that Executive Order picked out was deliberate and intentional as can be gleaned from the fact that it was underscored at
No. 1 suffers from arbitrary classification. The PTC, to be true to its mandate of searching for the truth, least three times in the assailed executive order. It must be noted that Executive Order No. 1 does not
must not exclude the other past administrations. The PTC must, at least, have the authority to even mention any particular act, event or report to be focused on unlike the investigative commissions
created in the past. "The equal protection clause is violated by purposeful and intentional instructions, ordinances, and other regulations. These provisions, however, have been fertile grounds of
103
discrimination." conflict between the Supreme Court, on one hand, and the two co-equal bodies of government, on the
other. Many times the Court has been accused of asserting superiority over the other departments.
To disprove petitioners’ contention that there is deliberate discrimination, the OSG clarifies that the
commission does not only confine itself to cases of large scale graft and corruption committed during To answer this accusation, the words of Justice Laurel would be a good source of enlightenment, to wit:
104
the previous administration. The OSG points to Section 17 of Executive Order No. 1, which provides: "And when the judiciary mediates to allocate constitutional boundaries, it does not assert any superiority
over the other departments; it does not in reality nullify or invalidate an act of the legislature, but only
SECTION 17. Special Provision Concerning Mandate. If and when in the judgment of the President asserts the solemn and sacred obligation assigned to it by the Constitution to determine conflicting
there is a need to expand the mandate of the Commission as defined in Section 1 hereof to include the claims of authority under the Constitution and to establish for the parties in an actual controversy the
investigation of cases and instances of graft and corruption during the prior administrations, such 107
rights which that instrument secures and guarantees to them."
mandate may be so extended accordingly by way of a supplemental Executive Order.
Thus, the Court, in exercising its power of judicial review, is not imposing its own will upon a co-equal
The Court is not convinced. Although Section 17 allows the President the discretion to expand the body but rather simply making sure that any act of government is done in consonance with the
scope of investigations of the PTC so as to include the acts of graft and corruption committed in other authorities and rights allocated to it by the Constitution. And, if after said review, the Court finds no
past administrations, it does not guarantee that they would be covered in the future. Such expanded constitutional violations of any sort, then, it has no more authority of proscribing the actions under
mandate of the commission will still depend on the whim and caprice of the President. If he would review. Otherwise, the Court will not be deterred to pronounce said act as void and unconstitutional.
decide not to include them, the section would then be meaningless. This will only fortify the fears of the
petitioners that the Executive Order No. 1 was "crafted to tailor-fit the prosecution of officials and It cannot be denied that most government actions are inspired with noble intentions, all geared towards
105
personalities of the Arroyo administration." the betterment of the nation and its people. But then again, it is important to remember this ethical
principle: "The end does not justify the means." No matter how noble and worthy of admiration the
The Court tried to seek guidance from the pronouncement in the case of Virata v. purpose of an act, but if the means to be employed in accomplishing it is simply irreconcilable with
106 108
Sandiganbayan, that the "PCGG Charter (composed of Executive Orders Nos. 1, 2 and 14) does not constitutional parameters, then it cannot still be allowed. The Court cannot just turn a blind eye and
violate the equal protection clause." The decision, however, was devoid of any discussion on how such simply let it pass. It will continue to uphold the Constitution and its enshrined principles.
conclusory statement was arrived at, the principal issue in said case being only the sufficiency of a
cause of action. "The Constitution must ever remain supreme. All must bow to the mandate of this law. Expediency must
109
not be allowed to sap its strength nor greed for power debase its rectitude."
A final word
Lest it be misunderstood, this is not the death knell for a truth commission as nobly envisioned by the
The issue that seems to take center stage at present is - whether or not the Supreme Court, in the present administration. Perhaps a revision of the executive issuance so as to include the earlier past
exercise of its constitutionally mandated power of Judicial Review with respect to recent initiatives of the administrations would allow it to pass the test of reasonableness and not be an affront to the
legislature and the executive department, is exercising undue interference. Is the Highest Tribunal, Constitution. Of all the branches of the government, it is the judiciary which is the most interested in
which is expected to be the protector of the Constitution, itself guilty of violating fundamental tenets like knowing the truth and so it will not allow itself to be a hindrance or obstacle to its attainment. It must,
the doctrine of separation of powers? Time and again, this issue has been addressed by the Court, but however, be emphasized that the search for the truth must be within constitutional bounds for "ours is
it seems that the present political situation calls for it to once again explain the legal basis of its action 110
still a government of laws and not of men."
lest it continually be accused of being a hindrance to the nation’s thrust to progress.
WHEREFORE, the petitions are GRANTED. Executive Order No. 1 is hereby declared
The Philippine Supreme Court, according to Article VIII, Section 1 of the 1987 Constitution, is vested UNCONSTITUTIONAL insofar as it is violative of the equal protection clause of the Constitution.
with Judicial Power that "includes the duty of the courts of justice to settle actual controversies involving
rights which are legally demandable and enforceable, and to determine whether or not there has been a As also prayed for, the respondents are hereby ordered to cease and desist from carrying out the
grave of abuse of discretion amounting to lack or excess of jurisdiction on the part of any branch or provisions of Executive Order No. 1.
instrumentality of the government."
SO ORDERED.
Furthermore, in Section 4(2) thereof, it is vested with the power of judicial review which is the power to
declare a treaty, international or executive agreement, law, presidential decree, proclamation, order, JOSE CATRAL MENDOZA
instruction, ordinance, or regulation unconstitutional. This power also includes the duty to rule on the Associate Justice
constitutionality of the application, or operation of presidential decrees, proclamations, orders,
FIRST DIVISION Respondents, employees of the DENR Region XII who are members of the employees association,
COURAGE, represented by their Acting President, Baguindanai A. Karim, filed with the Regional Trial
[G.R. No. 149724. August 19, 2003] Court of Cotabato, a petition for nullity of orders with prayer for preliminary injunction.

DEPARTMENT OF ENVIRONMENT AND NATURAL RESOURCES, represented herein by its On December 8, 1999, the trial court issued a temporary restraining order enjoining petitioner from
Secretary, HEHERSON T. ALVAREZ, petitioner, vs. DENR REGION 12 EMPLOYEES, represented implementing the assailed Memorandum. The dispositive portion of the Order reads:
by BAGUIDALI KARIM, Acting President of COURAGE (DENR Region 12 Chapter), respondents.
WHEREFORE, defendants DENR Secretary Antonio H. Cerilles and Regional Executive Director Israel
DECISION C. Gaddi are hereby ordered to cease and desist from doing the act complained of, namely, to stop the
transfer of DENR [Region] 12 offices from Cotabato City to Korandal (Marbel), South Cotabato.
YNARES-SANTIAGO, J.:
[1]
xxx xxx xxx.
This is a petition for review assailing the Resolutions dated May 31, 2000 of the Court of Appeals
which dismissed the petition for certiorari in CA-G.R. SP No. 58896, and its Resolution dated August [5]
SO ORDERED.
[2]
20, 2001 , which denied the motion for reconsideration.
Petitioner filed a Motion for Reconsideration with Motion to Dismiss, raising the following grounds:
The facts are as follows:
I.
On November 15, 1999, Regional Executive Director of the Department of Environment and Natural
[3]
Resources for Region XII, Israel C. Gaddi, issued a Memorandum directing the immediate transfer of The power to transfer the Regional Office of the Department of Environment and Natural Resources
the DENR XII Regional Offices from Cotabato City to Koronadal (formerly Marbel), South Cotabato. The (DENR) is executive in nature.
Memorandum was issued pursuant to DENR Administrative Order No. 99-14, issued by then DENR
Secretary Antonio H. Cerilles, which reads in part: II.

Subject: Providing for the Redefinition of Functions and Realignment of Administrative Units in The decision to transfer the Regional Office is based on Executive Order No. 429, which reorganized
the Regional and Field Offices: Region XII.

Pursuant to Executive Order No. 192, dated June 10, 1987 and as an interim administrative III.
arrangement to improve the efficiency and effectiveness of the Department of Environment and Natural
Resources (DENR) in delivering its services pending approval of the government-wide reorganization The validity of EO 429 has been affirmed by the Honorable Supreme Court in the Case of Chiongbian
by Congress, the following redefinition of functions and realignment of administrative units in the vs. Orbos (1995) 245 SCRA 255.
regional and field offices are hereby promulgated:
IV.
Section 1. Realignment of Administrative Units:
Since the power to reorganize the Administrative Regions is Executive in Nature citing Chiongbian, the
[6]
The DENR hereby adopts a policy to establish at least one Community Environment and Natural Honorable Court has no jurisdiction to entertain this petition.
Resources Office (CENRO) or Administrative Unit per Congressional District except in the Autonomous
On January 14, 2000, the trial court rendered judgment, the dispositive portion of which reads:
Region of Muslim Mindanao (ARMM) and the National Capital Region (NCR). The Regional Executive
Directors (REDs) are hereby authorized to realign/relocate existing CENROs and implement this policy CONSEQUENTLY, order is hereby issued ordering the respondents herein to cease and desist from
in accordance with the attached distribution list per region which forms part of this Order. Likewise, the enforcing their Memorandum Order dated November 15, 1999 relative to the transfer of the DENR
following realignment and administrative arrangements are hereby adopted: Regional Offices from Region 12 to Region 11 at Koronadal, South Cotabato for being bereft of legal
basis and issued with grave abuse of discretion amounting to lack or excess of jurisdiction on their part,
xxxxxxxxx
and they are further ordered to return back the seat of the DENR Regional Offices 12 to Cotabato City.
1.6. The supervision of the Provinces of South Cotabato and Sarangani shall be transferred from [7]
[4] SO ORDERED.
Region XI to XII.
Petitioners motion for reconsideration was denied in an Order dated April 10, 2000. A petition This Court is fully aware that procedural rules are not to be simply disregarded for these prescribed
for certiorari under Rule 65 was filed before the Court of Appeals, docketed as CA-G.R. SP No. procedures ensure an orderly and speedy administration of justice. However, it is equally true that
58896. The petition was dismissed outright for: (1) failure to submit a written explanation why personal litigation is not merely a game of technicalities. Time and again, courts have been guided by the
service was not done on the adverse party; (2) failure to attach affidavit of service; (3) failure to indicate principle that the rules of procedure are not to be applied in a very rigid and technical manner, as rules
[11]
the material dates when copies of the orders of the lower court were received; (4) failure to attach of procedure are used only to help secure and not to override substantial justice. Thus, if the
certified true copy of the order denying petitioners motion for reconsideration; (5) for improper application of the Rules would tend to frustrate rather than promote justice, it is always within the power
[12]
verification, the same being based on petitioners knowledge and belief, and (6) wrong remedy of of this Court to suspend the rules, or except a particular case from its operation.
[8]
certiorari under Rule 65 to substitute a lost appeal.
Despite the presence of procedural flaws, we find it necessary to address the issues because of the
[9]
The motion for reconsideration was denied in a resolution dated August 20, 2001. Hence, this petition demands of public interest, including the need for stability in the public service and the serious
based on the following assignment of errors: implications this case may cause on the effective administration of the executive department. Although
no appeal was made within the reglementary period to appeal, nevertheless, the departure from the
I general rule that the extraordinary writ of certiorari cannot be a substitute for the lost remedy of appeal
is justified because the execution of the assailed decision would amount to an oppressive exercise of
RULES OF PROCEDURE CAN NOT BE USED TO DEFEAT THE ENDS OF SUBSTANTIAL JUSTICE judicial authority.
[13]

II Petitioner maintains that the assailed DAO-99-14 and the implementing memorandum were valid and
that the trial court should have taken judicial notice of Republic Act No. 6734, otherwise known as An
THE DECISION OF THE LOWER COURT DATED 14 JANUARY 2000 WHICH WAS AFFIRMED IN
Organic Act for the Autonomous Region in Muslim Mindanao, and its implementing Executive Order
THE QUESTIONED RESOLUTIONS OF THE COURT OF APPEALS DATED 31 MAY 2000 AND 20 [14]
429, as the legal bases for the issuance of the assailed DAO-99-14. Moreover, the validity of R.A.
AUGUST 2001 IS PATENTLY ILLEGAL AND SHOULD BE NULLIFIED, CONSIDERING THAT: [15]
No. 6734 and E.O. 429 were upheld in the case of Chiongbian v. Orbos. Thus, the respondents
A. RESPONDENTS HAVE NO CAUSE OF ACTION AGAINST PETITIONER AS THEY HAVE NO cannot, by means of an injunction, force the DENR XII Regional Offices to remain in Cotabato City, as
RIGHT TO CAUSE THE DENR REGION 12 OFFICE TO REMAIN IN COTABATO CITY. the exercise of the authority to transfer the same is executive in nature.

B. THE STATE DID NOT GIVE ITS CONSENT TO BE SUED. It is apropos to reiterate the elementary doctrine of qualified political agency, thus:

C. THE DECISION OF THE LOWER COURT DATED 14 JANUARY 2000 IS CONTRARY TO THE Under this doctrine, which recognizes the establishment of a single executive, all executive and
RULE OF PRESUMPTION OF REGULARITY IN THE PERFORMANCE OF OFFICIAL FUNCTIONS. administrative organizations are adjuncts of the Executive Department, the heads of the various
executive departments are assistants and agents of the Chief Executive, and, except in cases where
D. IN ANY EVENT, THE DECISION OF THE LOWER COURT DATED 14 JANUARY 2000 IS the Chief Executive is required by the Constitution or law to act in person or the exigencies of the
CONTRARY TO THE LETTER AND INTENT OF EXECUTIVE ORDER NO. 429 AND REPUBLIC ACT situation demand that he act personally, the multifarious executive and administrative functions of the
NO. 6734. Chief Executive are performed by and through the executive departments, and the acts of the
Secretaries of such departments, performed and promulgated in the regular course of business, are,
E. THE DETERMINATION OF THE PROPRIETY AND PRACTICALITY OF THE TRANSFER OF unless disapproved or reprobated by the Chief Executive, presumptively the acts of the Chief
[10] [16]
REGIONAL OFFICES IS INHERENTLY EXECUTIVE, AND THEREFORE, NON-JUSTICIABLE. Executive.

In essence, petitioner argues that the trial court erred in enjoining it from causing the transfer of the This doctrine is corollary to the control power of the President as provided for under Article VII, Section
DENR XII Regional Offices, considering that it was done pursuant to DENR Administrative Order 99-14. 17 of the 1987 Constitution, which reads:

The issues to be resolved in this petition are: (1) Whether DAO-99-14 and the Memorandum Sec. 17. The President shall have control of all the executive departments, bureaus, and offices. He
implementing the same were valid; and (2) Whether the DENR Secretary has the authority to shall ensure that the laws be faithfully executed.
reorganize the DENR.
However, as head of the Executive Department, the President cannot be expected to exercise his
Prefatorily, petitioner prays for a liberal application of procedural rules considering the greater interest of control (and supervisory) powers personally all the time. He may delegate some of his powers to the
justice. Cabinet members except when he is required by the Constitution to act in person or the exigencies of
[17]
the situation demand that he acts personally.
[18]
In Buklod ng Kawaning EIIB v. Zamora, this Court upheld the continuing authority of the President to A court shall take judicial notice, without the introduction of evidence, of the existence and territorial
carry out the reorganization in any branch or agency of the executive department. Such authority extent of states, their political history, forms of government and symbols of nationality, the law of
[19]
includes the creation, alteration or abolition of public offices. The Chief Executives authority to nations, the admiralty and maritime courts of the world and their seals, the political constitution and
reorganize the National Government finds basis in Book III, Section 20 of E.O. No. 292, otherwise history of the Philippines, the official acts of the legislative, executive and judicial departments of
known as the Administrative Code of 1987, viz: the Philippines, the laws of nature, the measure of time, and the geographical divisions. (Emphasis
supplied)
Section 20. Residual Powers. Unless Congress provides otherwise, the President shall exercise such
other powers and functions vested in the President which are provided for under the laws and which are Article XIX, Section 13 of R.A. No. 6734 provides:
not specifically enumerated above or which are not delegated by the President in accordance with law.
SECTION 13. The creation of the Autonomous Region in Muslim Mindanao shall take effect when
[20]
Further, in Larin v. Executive Secretary, this Court had occasion to rule: approved by a majority of the votes cast by the constituent units provided in paragraph (2) of Sec. 1 of
Article II of this Act in a plebiscite which shall be held not earlier than ninety (90) days or later than one
This provision speaks of such other powers vested in the President under the law. What law then gives hundred twenty (120) days after the approval of this Act: Provided, That only the provinces and cities
him the power to reorganize? It is Presidential Decree No. 1772 which amended Presidential Decree voting favorably in such plebiscite shall be included in the Autonomous Region in Muslim Mindanao.
No. 1416. These decrees expressly grant the President of the Philippines the continuing authority to The provinces and cities which in the plebiscite do not vote for inclusion in the Autonomous Region
reorganize the national government, which includes the power to group, consolidate bureaus and shall remain in the existing administrative regions: Provided, however, That the President may, by
agencies, to abolish offices, to transfer functions, to create and classify functions, services and activities administrative determination, merge the existing regions.
and to standardize salaries and materials. The validity of these two decrees is unquestionable. The
1987 Constitution clearly provides that all laws, decrees, executive orders, proclamations, letters of Pursuant to the authority granted by the aforequoted provision, then President Corazon C. Aquino
instructions and other executive issuances not inconsistent with this Constitution shall remain operative issued on October 12, 1990 E.O. 429, Providing for the Reorganization of the Administrative Regions in
until amended, repealed or revoked. So far, there is yet no law amending or repealing said decrees. Mindanao. Section 4 thereof provides:

Applying the doctrine of qualified political agency, the power of the President to reorganize the National SECTION 4. REGION XII, to be known as CENTRAL MINDANAO, shall include the following provinces
Government may validly be delegated to his cabinet members exercising control over a particular and cities:
[21]
executive department. Thus, in DOTC Secretary v. Mabalot, we held that the President through his
duly constituted political agent and alter ego, the DOTC Secretary may legally and validly decree the Provinces
reorganization of the Department, particularly the establishment of DOTC-CAR as the LTFRB Regional
Office at the Cordillera Administrative Region, with the concomitant transfer and performance of public Sultan Kudarat
functions and responsibilities appurtenant to a regional office of the LTFRB.
Cotabato
Similarly, in the case at bar, the DENR Secretary can validly reorganize the DENR by ordering the
South Cotabato
transfer of the DENR XII Regional Offices from Cotabato City to Koronadal, South Cotabato.The
exercise of this authority by the DENR Secretary, as an alter ego, is presumed to be the acts of the Cities
President for the latter had not expressly repudiated the same.
Cotabato
The trial court should have taken judicial notice of R.A. No. 6734, as implemented by E.O. No. 429, as
legal basis of the Presidents power to reorganize the executive department, specifically those General Santos
administrative regions which did not vote for their inclusion in the ARMM. It is axiomatic that a court has
the mandate to apply relevant statutes and jurisprudence in determining whether the allegations in a The Municipality of Koronadal (Marinduque) in South Cotabato shall serve as the regional center.
complaint establish a cause of action. While it focuses on the complaint, a court clearly cannot
[22]
disregard decisions material to the proper appreciation of the questions before it. In resolving the In Chiongbian v. Orbos, this Court stressed the rule that the power of the President to reorganize the
motion to dismiss, the trial court should have taken cognizance of the official acts of the legislative, administrative regions carries with it the power to determine the regional centers. In identifying the
executive, and judicial departments because they are proper subjects of mandatory judicial notice as regional centers, the President purposely intended the effective delivery of the field services of
[23]
provided by Section 1 of Rule 129 of the Rules of Court, to wit: government agencies. The same intention can be gleaned from the preamble of the assailed DAO-
99-14 which the DENR sought to achieve, that is, to improve the efficiency and effectiveness of the
DENR in delivering its services.
It may be true that the transfer of the offices may not be timely considering that: (1) there are no
buildings yet to house the regional offices in Koronadal, (2) the transfer falls on the month of Ramadan,
(3) the children of the affected employees are already enrolled in schools in Cotabato City, (4) the
Regional Development Council was not consulted, and (5) the Sangguniang Panglungsond, through a
resolution, requested the DENR Secretary to reconsider the orders. However, these concern issues
addressed to the wisdom of the transfer rather than to its legality. It is basic in our form of government
that the judiciary cannot inquire into the wisdom or expediency of the acts of the executive or the
[24]
legislative department, for each department is supreme and independent of the others, and each is
devoid of authority not only to encroach upon the powers or field of action assigned to any of the other
department, but also to inquire into or pass upon the advisability or wisdom of the acts performed,
[25]
measures taken or decisions made by the other departments.

The Supreme Court should not be thought of as having been tasked with the awesome responsibility of
overseeing the entire bureaucracy. Unless there is a clear showing of constitutional infirmity or grave
abuse of discretion amounting to lack or excess of jurisdiction, the Courts exercise of the judicial power,
pervasive and limitless it may seem to be, still must succumb to the paramount doctrine of separation of
[26]
powers. After a careful review of the records of the case, we find that this jurisprudential element of
abuse of discretion has not been shown to exist.

WHEREFORE, in view of the foregoing, the petition for review is GRANTED. The resolutions of the
Court of Appeals in CA-G.R. SP No. 58896 dated May 31, 2000 and August 20, 2001, as well as the
decision dated January 14, 2000 of the Regional Trial Court of Cotabato City, Branch 15, in Civil Case
No 389, are REVERSED and SET ASIDE. The permanent injunction, which enjoined the petitioner from
enforcing the Memorandum Order of the DENR XII Regional Executive Director, is LIFTED.

SO ORDERED.

Vitug, (Acting Chairman), Carpio, and Azcuna, JJ., concur.

Davide, Jr., C.J., (Chairman), abroad, on official business.


Republic of the Philippines The Toll Regulatory Board (TRB) was created on 31 March 1977 by Presidential Decree No. (P.D.)
1
SUPREME COURT 1112 in order to supervise and regulate, on behalf of the government, the collection of toll fees and the
Manila operation of toll facilities by the private sector.
2
FIRST DIVISION On the same date, P.D. 1113 was issued granting to the Construction and Development Corporation of
the Philippines (now Philippine National Construction Corporation or PNCC) the right, privilege, and
G.R. No. 181293 February 23, 2015 authority to construct, operate, and maintain toll facilities in the North and South Luzon Toll
Expressways for a period of 30 years starting 1 May1977.
ANA THERESIA "RISA" HONTIVEROS-BARAQUEL, DANIEL L. EDRALIN, VICTOR M.
GONZALES, SR., JOSE APOLLO R. ADO, RENE D. SORIANO, ALLIANCE OF PROGRESSIVE 3
TRB and PNCC later entered into a Toll Operation Agreement, which prescribed the operating
LABOR, BUKLURAN NG MANGGAGAWANG PILIPINO, LAHING PILIPINO MULTIPURPOSE conditions of the right granted to PNCC under P.D. 1113.
TRANSPORT SERVICE COOPERATIVE, PNCC SKYWAY CORPORATION EMPLOYEES UNION
4
(PSCEU), and PNCC TRAFFIC MANAGEMENT & SECURITY DEPARTMENT WORKERS P.D. 1113 was amended by P.D. 1894, which granted PNCC the right, privilege, and authority to
ORGANIZATION (PTMSDWO), Petitioners, construct, maintain, and operate the North Luzon, South Luzon and Metro Manila Expressways,
vs. together with the toll facilities appurtenant thereto. The term of 30 years provided under P. D. 1113
TOLL REGULATORY BOARD, THE SECRETARY OF THE DEPARTMENT OF TRANSPORTATION starting from 1 May 1977 remained the same for the North and the South Luzon Expressways, while the
AND COMMUNICATIONS (DOTC), PNCC SKYWAY CORPORATION, PHILIPPINE NATIONAL franchise granted for the Metro Manila Expressway (MME) provided a term of 30 years commencing
CONSTRUCTION CORPORATION, SKYWAY O & M CORPORATION, and CITRA METRO MANILA from the date of completion of the project.
TOLLWAYS CORP.,Respondents.
5
On 22 September 1993, PNCC entered into an agreement with PT Citra Lamtoro Gung Persada
DECISION (CITRA), a limited liability company organized and established under the laws of the Republic of
Indonesia, whereby the latter committed to provide PNCC with a pre-feasibility study on the proposed
SERENO, CJ: 6
MME project. The agreement was supplemented on 14 February 1994 with a related undertaking on
the part of CITRA. CITRA was to provide a preliminary feasibility study on the Metro Manila Skyways
This is an original petition for certiorari and prohibition under Rule 65 of the Rules of Court, with a (MMS) project, a system of elevated roadway networks passing through the heart of the Metropolitan
prayer for the issuance of a writ of preliminary injunction and/or temporary restraining order, seeking the Manila area. In order to accelerate the actual implementation of both the MME and the MMS projects,
annulment of the following: 7
PNCC and CITRA entered into a second agreement. Through that agreement, CITRA committed to
finance and undertake the preparation, updating, and revalidation of previous studies on the
1. The Amendment to the Supplemental Toll Operation Agreement executed on 18 July 2007 between
construction, operation, and maintenance of the projects.
the Republic of the Philippines, the Philippine National Construction Corporation, and Citra Metro
Manila Tollways Corporation; As a result of the feasibility and related studies, PNCC and CITRA submitted, through the TRB, a Joint
8
Investment Proposal (JIP) to the Republic of the Philippines. The JIP embodied the implementation
2. The Memorandum dated 20 July 2007 of the Secretary of Transportation and Communications,
schedule for the financing, design and construction of the MMS in three stages: the South Metro Manila
approving the Amendment to the Supplemental Toll Operation Agreement; 9
Skyway, the North Metro Manila Skyway, and the Central Metro Manila Skyway.
3. The Memorandum of Agreement executed on 21 December 2007 between the Philippine National
The TRB reviewed, evaluated and approved the JIP, particularly as it related to Stage 1, Phases 1 and
Construction Corporation, PNCC Skyway Corporation, and Citra Metro Manila Tollways Corporation;
2; and Stage 2, Phase 1 of the South Metro Manila Skyway.
and
10
On 30 August 1995, PNCC and CITRA entered into a Business and Joint Venture Agreement and
4. The Toll Operation Certificate issued by the Toll Regulatory Board on 28 December 2007 in favor of
created the Citra Metro Manila Tollways Corporation (CMMTC). CMMTC was a joint venture corporation
Skyway O & M Corporation.
organized under Philippine laws to serve as a channel through which CITRA shall participate in the
The annulment of the above is sought for being unconstitutional, contrary to law, and grossly construction and development of the project.
disadvantageous to the government. Petitioners also seek to prohibit Skyway O & M Corporation from
On 27 November 1995, the Republic of the Philippines - through the TRB - as Grantor, CMMTC as
assuming operations and maintenance responsibilities over the Skyway toll facilities. ANTECEDENT
Investor, and PNCC as Operator executed a Supplemental Toll Operation Agreement
FACTS 11
(STOA) covering Stage 1, Phases 1 and 2; and Stage 2, Phase 1 of the South Metro Manila Skyway.
24
Under the STOA, the design and construction of the project roads became the primary and exclusive On 23 January 2008, the RTC issued an Order denying the prayer for the issuance of a temporary
privilege and responsibility of CMMTC. The operation and maintenance of the project roads became the restraining order and/or writ of preliminary injunction. According to the RTC, petitioners were seeking to
25
primary and exclusive privilege and responsibility of the PNCC Skyway Corporation (PSC), a wholly enjoin a national government infrastructure project. Under Republic Act No. (R.A.) 8975, lower courts
owned subsidiary of PNCC, which undertook and performed the latter's obligations under the STOA. are prohibited from issuing a temporary restraining order or preliminary injunction against the
government - or any person or entity acting under the government's direction - to restrain the execution,
CMMTC completed the design and construction of Stage 1 of the South Metro Manila Skyway, which implementation, or operation of any such contract or project. Furthermore, the RTC ruled that it could no
12
was operated and maintained by PSC. longer issue a temporary restraining order or preliminary injunction, considering that the act sought to
26
be restrained had already been consummated. The AS TOA, the MOA, and the assumption of the toll
On 18 July 2007, the Republic of the Philippines, through the TRB, CMMTC, and PNCC executed the operations by SOMCO took effect at 10:00 p.m. on 31 December 2007, while petitioners PSCEU and
13
assailed Amendment to the Supplemental Toll Operation Agreement (ASTOA). The ASTOA PTMSDWO sought to prohibit their implementation only on 3 January 2008.
incorporated the amendments, revisions, and modifications necessary to cover the design and
construction of Stage 2 of the South Metro Manila Skyway. Also under the ASTOA, Skyway 0 & M In view of its denial of the ancillary prayer, the RTC required defendants to file their respective Answers
Corporation (SOMCO) replaced PSC in performing the operations and maintenance of Stage 1 of the 27
to the Amended Complaint.
South Metro Manila Skyway.
On 28 January 2008, petitioners PSCEU and PTMSDWO filed a Notice of Dismissal with Urgent Ex-
14 28
Pursuant to the authority granted to him under Executive Order No. (E.O.) 497 dated 24 January Parte Motion for the Issuance of Order Confirming the Dismissal, considering that no Answers had yet
2006, Department of Transportation and Communications (DOTC) Secretary Leandro Mendoza 29
been filed. On the basis thereof, the R TC dismissed the case without prejudice on 29 January 2008.
15
approved the ASTOA through the challenged Memorandum dated 20 July 2007.
30
On 4 February 2008, petitioners filed the instant Petition before this Court. On 13 February 2008, we
On 21 December 2007, PNCC, PSC, and CMMTC entered into the assailed Memorandum of 31
required respondents to comment on the same.
16
Agreement (MOA) providing for the successful and seamless assumption by SOMCO of the operations
32 33
and maintenance of Stage 1 of the South Metro Manila Skyway. Under the MOA, PSC received the Meanwhile, defendants PNCC and PSC filed their respective Motions for Partial Reconsideration of
amount of ₱320 million which was used for the settlement of its liabilities arising from the consequent the Order of the R TC dismissing the case without prejudice. Both argued that the RTC should have
retrenchment or separation of its affected employees. dismissed the case with prejudice. They pointed out that petitioners PSCEU and PTMSDWO had acted
in bad faith by filing the complaint before the RTC, despite the pendency of a labor case over which the
17
The TRB issued the challenged Toll Operation Certificate (TOC) to SOM CO on 28 December 2007, Secretary of Labor and Employment had assumed jurisdiction. Defendant CMMTC joined PNCC and
authorizing the latter to operate and maintain Stage 1 of the South Metro Manila Skyway effective 10:00 34
PSC in moving for a partial reconsideration of the RTC Order.
p.m. on 31December2007.
35
The RTC denied the Motions for Partial Reconsideration in an Order dated 13 June 2008.
Meanwhile, on 28 December 2007, petitioner PNCC Traffic Management and Security Department
36 37 38 39 40
Workers Organization (PTMSDWO) filed a Notice of Strike against PSC on the ground of unfair labor Before this Court, SOMCO, PSC, PNCC, CMMTC, and TRB filed their respective Comments on
18 19
practice, specifically union busting. The Secretary of Labor and Employment assumed jurisdiction the Petition.
over the dispute in an Order dated 31 December 2007 and set the initial hearing of the case on 2
20 THE PARTIES' POSITIONS
January 2008.

On 3 January 2008, petitioners PTMSDWO and PNCC Skyway Corporation Employees Union (PSCEU) Petitioners argue that the franchise for toll operations was exclusively vested by P.D. 1113 in PNCC,
filed before the Regional Trial Court of Parañaque City, Branch 258 (RTC), a complaint against which exercised the powers under its franchise through PSC in accordance with the STOA. By agreeing
respondents TRB, PNCC, PSC, CMMTC, and SOMCO. The complaint was for injunction and to the arrangement whereby SOMCO would replace PSC in the toll operations and management,
prohibition with a prayer for a writ of preliminary injunction and/or a temporary restraining order, and PNCC seriously breached the terms and conditions of its undertaking under the franchise and
sought to prohibit the implementation of the AS TOA and the MOA, as well as the assumption of the toll effectively abdicated its rights and privileges in favor of SOMCO.
21
operations by SOMCO. Petitioners PSCEU and PTMSDWO also sought the subsequent nullification
of the ASTOA and the MOA for being contrary to law and for being grossly disadvantageous to the Furthermore, the TOC granted to SOMCO was highly irregular and contrary to law, because 1) it did not
41
22 23
government. They later filed an Amended Complaint dated 8 January 2008, additionally praying that indicate the conditions that shall be imposed on SOMCO as provided under P.D. 1112; 2) none of the
PSC be allowed to continue the toll operations. With the exception of TRB, all defendants therein filed requirements on public bidding, negotiations, or even publication was complied with before the issuance
their Opposition. of the TOC to SOMCO; 3) applying the stricter "grandfather rule," SOMCO does not qualify as a facility
42 43
operator as defined under R.A. 6957, as amended by R.A. 7718; and 4) there were no public notices
and hearings conducted wherein all legitimate issues and concerns about the transfer of the toll On the merits of the arguments in the petition, respondents argue that nothing in the ASTOA, the
operations would have been properly ventilated. approval thereof by the DOTC Secretary, the MOA, or the TOC was violative of the Constitution. It is
argued that the authority to operate a public utility can be granted by administrative agencies when
Petitioners also claim that the approval by the DOTC Secretary of the AS TOA could not take the place 56
authorized by law. Under P.D. 1112, the TRB is empowered to grant authority and enter into contracts
44 45 57
of the presidential approval required under P.D. 1113 and P.D. 1894 concerning the franchise for the construction, operation, and maintenance of a toll facility, such as the ASTOA in this case.
granted to PNCC. Also, the ASTOA was an amendment, not to the legislative franchise of PNCC, but to the STOA
58
previously executed between the Republic of the Philippines through the TRB, PNCC, and CMMTC. In
Finally, petitioners claim that the assumption of the toll operations by SOM CO was grossly 59
fact, PNCC's franchise was never sold, transferred, or otherwise assigned to SOMCO in the same
disadvantageous to the government, because 1) for a measly capital investment of ₱2.5 million, way that PSC's previous assumption of the operation and maintenance of the South Metro Manila
SOMCO stands to earn ₱400 million in gross revenues based on official and historical records; 2) with Skyway did not amount to a sale, transfer or assignment of PNCC's franchise.
60

its measly capital, SOMCO would not be able to cover the direct overhead for personal services in the
amount of ₱226 million as borne out by Commission on Audit reports; 3) the net revenue from toll There can be no valid objection to the approval of the ASTOA by the DOTC Secretary, because he was
operations would go to private shareholders of SOMCO, whereas all earnings of PSC when it was still 61
authorized by the President to do so by virtue of E.O. 497. Also, the phrase "subject to the approval of
in charge of the toll operations went to PNCC - the mother company whose earnings, as an "acquired- the President of the Philippines" in P.D. 1112 and 1113 does not in any way mean that the presidential
asset corporation," formed part of the public treasury; 4) the new arrangement would result in the poor approval must be obtained prior to the execution of a contract, or that the approval be made personally
delivery of toll services by SOMCO, which had no proven track record; 5) PSC received only ₱320 62
by the President. The presidential approval may be obtained under the doctrine of qualified political
million as settlement for the transfer of toll operations to SOMCO. 63
agency.

All respondents counter that petitioners do not have the requisite legal standing to file the petition. Respondents argue that there is no merit in the claim that the TOC granted to SOMCO was highly
According to respondents, petitioner Hontiveros-Baraquel filed the instant petition as a legislator in her irregular and contrary to law. First, the TOC clearly states that the toll operation and maintenance by
capacity as party-list representative of Akbayan. As such, she was only allowed to sue to question the SOMCO shall be regulated by the Republic of the Philippines in accordance with P.D. 1112, the STOA,
46
validity of any official action when it infringed on her prerogative as a legislator. Presently, she has the toll operations and maintenance rules and regulations, and lawful orders, instructions, and
47 64
cited no such prerogative, power, or privilege that is adversely affected by the assailed acts. conditions that may be imposed from time to time. Second, there is no need to comply with the public
bidding and negotiation requirements, because the South Metro Manila Skyway is an ongoing project,
While suing as citizens, the individual petitioners have not shown any personal or substantial interest in 65
not a new one. Furthermore, the STOA, which was the basis for the ASTOA, was concluded way
the case indicating that they sustained or will sustain direct injury as a result of the implementation of 66
before the effectivity of R.A. 9184 in 2003.
67
48
the assailed acts. The maintenance of the suit by petitioners as taxpayers has no merit either because
49 68
the assailed acts do not involve the disbursement of public funds. Finally, the bringing of the suit by Third, SOMCO is a Filipino corporation with substantial 72% Filipino ownership. Fourth, the law
petitioners as people's organizations does not automatically confer legal standing, especially since requires prior notice and hearing only in an administrative body's exercise of quasi-judicial
50 69
petitioner-organizations do not even allege that they represent their members, nor do they cite any functions. In this case, the transfer of the toll operations and maintenance to SOM CO was a
51 70
particular constitutional provision that has been violated or disregarded by the assailed acts. In fact, contractual arrangement entered into in accordance with law.
the suit raises only issues of contract law, and none of the petitioners is a party or is privy to the
52
assailed agreements and issuances. Finally, the assumption of the toll operation and maintenance by SOMCO is not disadvantageous to the
government. Petitioners belittle the ₱2.5 million capitalization of SOMCO, considering that PSC's
Respondents also argue that petitioners violate the hierarchy of courts. In particular, it is alleged that 71
capitalization at the time it was incorporated was merely ₱500,000.
while lower courts are prohibited from issuing temporary restraining orders or preliminary injunctions
against national government projects under R.A. 8975, the law does not preclude them from assuming Respondents claim that under the ASTOA, PNCC shall get a direct share in the toll revenues without
jurisdiction over complaints that seek the nullification of a national government project as ultimate any corollary obligation, unlike the arrangement in the STOA whereby PNCC's 10% share in the toll
53 72
relief. revenues was intended primarily for the toll operation and maintenance by PSC.

As a final procedural challenge to the petition, respondents aver that petitioners are guilty of forum Finally, respondents assert that there is no reason to fear that the assumption by SOMCO would result
shopping. When petitioners filed the instant petition, the case before the R TC seeking similar reliefs in poor delivery of toll services. CITRA and the other shareholders of SOMCO are entities with
73
was still pending, as respondents PNCC, PSC and CMMTC had moved for the partial reconsideration of experience and proven track record in toll operations. Also, SOM CO hired or absorbed more than 300
54 74
the RTC's Order of dismissal within the reglementary period. Furthermore, the instant case and the PSC employees, who brought with them their work expertise and experience.
one before the RTC were filed while petitioners' labor grievances seeking similar reliefs were also being
55 ISSUES
heard before the Department of Labor and Employment.
The instant case shall be resolved on the basis of the following issues: indirectly from the state through a duly designated agency, and to this extent, the power to grant
franchises has frequently been delegated, even to agencies other than those of a legislative nature. In
Procedural: pursuance of this, it has been held that privileges conferred by grant by local authorities as agents for
the state constitute as much a legislative franchise as though the grant had been made by an act of the
I. Whether petitioners have standing; Legislature.
81

II. Whether petitioners are guilty of forum-shopping; It is thus clear that Congress does not have the sole authority to grant franchises for the operation of
public utilities. Considering the foregoing, we find that the petition raises no issue of constitutional
Substantive:
import. More particularly, no legislative prerogative, power, or privilege has been impaired. Hence,
III. Whether the TRB has the power to grant authority to operate a toll facility; legislators have no standing to file the instant petition, for they are only allowed to sue to question the
82
validity of any official action when it infringes on their prerogatives as members of Congress. Standing
IV. Whether the TOC issued to SOMCO was valid; is accorded to them only if there is an unmistakable showing that the challenged official act affects or
83
impairs their rights and prerogatives as legislators.
V. Whether the approval of the ASTOA by the DOTC Secretary was valid; and
84
In line with our ruling in Kilosbayan, Inc. v. Morato, the rule concerning a real party in interest - which
VI. Whether the assumption of toll operations by SOMCO is disadvantageous to the government. is applicable to private litigation – rather than the liberal rule on standing, should be applied to
petitioners.
OUR RULING
A real party in interest is one who stands to be benefited or injured by the judgment in the suit, or the
85
I party entitled to the avails of the suit. One's interest must be personal and not one based on a desire
86
to vindicate the constitutional right of some third and unrelated party. The purposes of the rule are to
Not all petitioners have personality to sue. prevent the prosecution of actions by persons without any right or title to or interest in the case; to
require that the actual party entitled to legal relief be the one to prosecute the action; to avoid a
Standing is a constitutional law concept allowing suits to be brought not necessarily by parties
multiplicity of suits; and to discourage litigation and keep it within certain bounds, pursuant to sound
personally injured by the operation of a law or official action, but by concerned citizens, taxpayers, or 87
75 public policy.
voters who sue in the public interest. Determining the standing of concerned citizens, taxpayers, or
76
voters requires a partial consideration of the substantive merit of the constitutional question, or at least At bottom, what is being questioned in the petition is the relinquishment by PSC of the toll operations in
77
a preliminary estimate thereof. favor of SOMCO, effectively leading to the cessation of the former' s business. In this case, we find that
among petitioners, the only real parties in interest are the labor unions PSCEU and PTMSDWO.
In this case, petitioners raise the power of Congress to grant franchises as a constitutional question.
They allege that the execution of the ASTOA and the MOA, the approval of the AS TOA by the DOTC PSCEU and PTMSDWO filed the petition not as a representative suit on behalf of their members who
Secretary and the issuance of the TOC infringed on the constitutional power of Congress, which has the are rank-and-file employees of PSC, but as people's organizations "invested with a public duty to
sole authority to grant franchises for the operation of public utilities. This Court has had a few occasions 88 89
defend the rule of law." PSCEU and PTMSDWO cite Kilosbayan v. Ermita as authority to support
to rule that a franchise from Congress is not required before each and every public utility may their standing to file the instant suit.
78
operate. Unless there is a law that specifically requires a franchise for the operation of a public utility,
particular agencies in the executive branch may issue authorizations and licenses for the operation of It is well to point out that the Court, in Ermita, accorded standing to people's organizations to file the
79
certain classes of public utilities. In the instant case, there is no law that states that a legislative suit, because the matter involved therein was the qualification of a person to be appointed as a member
90
franchise is necessary for the operation of toll facilities. of this Court -"an issue of utmost and far-reaching constitutional importance." As discussed, the
80
instant petition raises no genuine constitutional issues.
In PAL v. Civil Aeronautics Board, this Court enunciated:
Nevertheless, for a different reason, we accord standing to PSCEU and PTMSDWO to file the instant
Congress has granted certain administrative agencies the power to grant licenses for, or to authorize suit. With the transfer of toll operations to SOMCO and the resulting cessation of PSC's business comes
the operation of certain public utilities. With the growing complexity of modem life, the multiplication of the retrenchment and separation of all its employees. The existence of petitioner labor unions would
the subjects of governmental regulation, and the increased difficulty of administering the laws, there is a terminate with the dissolution of its employer and the separation of its members. This is why the petition
constantly growing tendency towards the delegation of greater powers by the legislature, and towards also prays that this Court issue an order "that would smoothly preserve the toll operations services of
the approval of the practice by the courts. It is generally recognized that a franchise may be derived respondent PNCC and/or respondent PSC under its legislative franchise."
91
We have recognized that the right of self-preservation is inherent in every labor union or any RTC's Order of dismissal without prejudice was still pending. Section 1, Rule 17 of the Rules of Court
92
organization for that matter. Thus, PSCEU and PTMSDWO, as real parties in interest, have the states:
personality to question the assumption of the toll operations by SOMCO.
SECTION 1. Dismissal upon notice by plaintiff. - A complaint may be dismissed by the plaintiff by filing a
II notice of dismissal at any time before service of the answer or of a motion for summary judgment. Upon
such notice being filed, the court shall issue an order confirming the dismissal. Unless otherwise stated
PSCEU and PTMSDWO are not guilty of forum-shopping. in the notice, the dismissal is without prejudice, except that a notice operates as an adjudication upon
the merits when filed by a plaintiff who has once dismissed in a competent court an action based on or
Forum shopping refers to the act of availing of several remedies in different courts and/or administrative including the same claim.
agencies, either simultaneously or successively, when these remedies are substantially founded on the
same material facts and circumstances and raise basically the same issues either pending in or already In this case, petitioners PSCEU and PTMSDWO had filed a notice of dismissal of the complaint before
93
resolved by some other court or administrative agency. What is pivotal in determining whether forum the RTC on 28 January 2008, before respondents filed their Answers. The following day, the RTC
shopping exists is the vexation caused to the courts and litigants and the possibility of conflicting issued an order confirming the dismissal. Under the above-cited rule, this confirmation is the only
94
decisions being rendered by different courts and/or administrative agencies upon the same issues. qualification imposed on the right of a party to dismiss the action before the adverse party files an
98
answer. In this case, the dismissal of the action therefore became effective upon that confirmation by
The elements of forum shopping are as follows: a) identity of parties or at least such parties that the RTC despite the subsequent filing of the motions for partial reconsideration.
represent the same interests in both actions; b) identity of rights asserted and the relief prayed for, the
relief founded on the same facts; and c) identity of the two preceding particulars, such that any Thus, when the instant petition was filed on 4 February 2008, the complaint before the RTC was no
95
judgment rendered in one action will amount to res judicata in the other. Respondents argue that longer pending. The complaint was dismissed without prejudice by virtue of the notice of dismissal filed
petitioners PSCEU and PTMSDWO committed forum shopping by filing the complaint for injunction and by petitioners PSCEU and PTMSDWO. Consequently, there was not even any need for petitioners to
prohibition before the RTC during the pendency of NCMB-NCR-NS-12-188-07 entitled In Re: Labor mention the prior filing and dismissal of the complaint in the certificate of non-forum shopping in the
Dispute at PNCC Skyway Corporation. It was a case they also filed, over which the Secretary of Labor 99 100
instant petition, but they did so anyway.
and Employment has assumed jurisdiction.
Parenthetically, in their motions for partial reconsideration, respondents PNCC and PSC insisted that
The case involves a Notice of Strike filed against PSC on the ground of unfair labor practice. While the the dismissal should have been with prejudice, because petitioners allegedly acted in bad faith in filing
96
specific act in question is not specified, the prohibited acts constituting unfair labor practice essentially the notice of dismissal, were guilty of forum shopping, and did not notify respondents of their intention to
97
relate to violations concerning the workers' right to self-organization. When compared with the file a notice of dismissal. With regard to the first and the third allegation, petitioners may ask for
complaint filed with the RTC for injunction and prohibition seeking to prohibit the implementation of the dismissal at any time before the filing of the answer as a matter of right, even if the notice cites "the
ASTOA and the MOA, as well as the assumption of the toll operations by SOM CO for being 101
most ridiculous of grounds for dismissal." As to the second, we have already ruled that there was no
unconstitutional, contrary to law and disadvantageous to the government, it is easily discernible that forum shopping as regards the successive filings of the labor case and the complaint before the RTC.
there is no identity of rights asserted and relief prayed for. These cases are distinct and dissimilar in
their nature and character. II

For the sake of argument, let us assume that, in order to hurt the unions, PSC feigned a cessation of TRB has the power to grant authority to operate a toll facility.
business that led to the retrenchment and separation of all employees. That is an unfair labor practice.
102
In that complaint, the unions cannot be expected to ask for, or the Secretary of Labor and Employment This matter has already been settled by the Court in Francisco, Jr. v. TRB, which ruled thus:
to grant, the annulment of the ASTOA and the MOA and the continuation of toll operations by PSC. The
Secretary would only focus on the legality of the retrenchment and separation, and on the presence or It is abundantly clear that Sections 3 (a) and (e) of P.D. 1112 in relation to Section 4 of P.D. 1894 have
absence of bad faith in PSC's cessation of business. On the other hand, the complaint before the RTC invested the TRB with sufficient power to grant a qualified person or entity with authority to construct,
would require it to focus on the legality of the ASTOA, the MOA and the transfer of toll operations. maintain, and operate a toll facility and to issue the corresponding toll operating permit or TOC.
Ultimately, even if the Secretary of Labor and Employment makes a finding of unfair labor practice, this
Sections 3 (a) and (e) of P.D. 1112 and Section 4 of P.D. 1894 amply provide the power to grant
determination would not amount to res judicata as regards the case before the RTC.
authority to operate toll facilities:
We also reject the claim of respondents that petitioners PSCEU and PTMSDWO committed forum
Section 3. Powers and Duties of the Board. - The Board shall have in addition to its general powers of
shopping by filing the instant petition before this Court while the motion for partial reconsideration of the
administration the following powers and duties:
(a) Subject to the approval of the President of the Philippines, to enter into contracts in behalf of the SECTION 4. The Toll Regulatory Board is hereby given jurisdiction and supervision over the GRANTEE
Republic of the Philippines with persons, natural or juridical, for the construction, operation and with respect to the Expressways, the toll facilities necessarily appurtenant thereto and, subject to the
maintenance of toll facilities such as but not limited to national highways, roads, bridges, and public provisions of Section 8 and 9 hereof, the toll that the GRANTEE will charge the users thereof.
thoroughfares. Said contract shall be open to citizens of the Philippines and/or to corporations or
associations qualified under the Constitution and authorized by law to engage in toll operations; By explicit provision of law, the TRB was given the power to grant administrative franchise for toll facility
103
projects. (Emphases supplied)
xxxx
We cannot abide by the contention of petitioners that the franchise for toll operations was exclusively
(e) To grant authority to operate a toll facility and to issue therefore the necessary "Toll Operation vested in PNCC, which effectively breached its franchise when it transferred the toll operations to
Certificate" subject to such conditions as shall be imposed by the Board including inter alia the SOMCO. First, there is nothing in P.D. 1113 or P.D. 1894 that states that the franchise granted to
following: PNCC is to the exclusion of all others.

(1) That the Operator shall desist from collecting toll upon the expiration of the Toll Operation Second, if we were to go by the theory of petitioners, it is only the operation and maintenance of the toll
Certificate. facilities that is vested with PNCC. This interpretation is contrary to the wording of P.D. 1113 and P.D.
1894 g ranting PNCC the right, privilege and authority to construct, operate and maintain the North
(2) That the entire facility operated as a toll system including all operation and maintenance equipment Luzon, South Luzon and Metro Manila Expressways and their toll facilities.
directly related thereto shall be turned over to the government immediately upon the expiration of the
Toll Operation Certificate. It appears that petitioners have confused the franchise granted under P.D. 1113 and P.D. 1894 with
particular provisions in the STOA. To clarify, the operation and maintenance of the project roads were
(3) That the toll operator shall not lease, transfer, grant the usufruct of, sell or assign the rights or the primary and exclusive privilege and responsibility of PNCC through PSC under the STOA. On the
privileges acquired under the Toll Operation Certificate to any person, firm, company, corporation or other hand, the design and construction of the project roads were the primary and exclusive privilege
other commercial or legal entity, nor merge with any other company or corporation organized for the and responsibility of CMMTC. However, with the execution of the AS TOA, the parties agreed that SOM
same purpose, without the prior approval of the President of the Philippines. In the event of any valid CO shall replace PSC in undertaking the operations and maintenance of the project roads. Thus, the
transfer of the Toll Operation Certificate, the Transferee shall be subject to all the conditions, terms, "exclusivity clause" was a matter of agreement between the parties, which amended it in a later
restrictions and limitations of this Decree as fully and completely and to the same extent as if the Toll contract; it was not a matter provided under the law.
Operation Certificate has been granted to the same person, firm, company, corporation or other
commercial or legal entity. Third, aside from having been granted the power to grant administrative franchises for toll facility
projects, TRB is also empowered to modify, amend, and impose additional conditions on the franchise
(4) That in time of war, rebellion, public peril, emergency, calamity, disaster or disturbance of peace and of PNCC in an appropriate contract, particularly when public interest calls for it. This is provided under
order, the President of the Philippines may cause the total or partial closing of the toll facility or order to Section 3 of P.D. 1113 and Section 6 of P.D. 1894, to wit:
take over thereof by the Government without prejudice to the payment of just compensation.
SECTION 3. This franchise is granted subject to such conditions as may be imposed by the [Toll
(5) That no guarantee, Certificate of Indebtedness, collateral, securities, or bonds shall be issued by Regulatory] Board in an appropriate contract to be executed for this purpose, and with the
any government agency or government-owned or controlled corporation on any financing program of understanding and upon the condition that it shall be subject to amendment, alteration or repeal when
the toll operator in connection with his undertaking under the Toll Operation Certificate. public interest so requires.

(6) The Toll Operation Certificate may be amended, modified or revoked whenever the public interest so xxx
requires.
SECTION 6. This franchise is granted subject to such conditions, consistent with the provisions of this
(a) The Board shall promulgate rules and regulations governing the procedures for the grant of Toll Decree, as may be imposed by the Toll Regulatory Board in the Toll Operation Agreement and such
Certificates. The rights and privileges of a grantee under a Toll Operation Certificate shall be defined by other modifications or amendments that may be made thereto, and with the understanding and upon the
the Board. condition that it shall be subject to amendment or alteration when public interest so dictates.

(b) To issue rules and regulations to carry out the purposes of this Decree. Section 6 of P.D. 1894 specifically mentions the Toll Operation Agreement. The STOA was one such
modification or amendment of the franchise of PNCC. So was the ASTOA, which further modified the
franchise. PNCC cannot be said to have breached its franchise when it transferred the toll operations to
SOMCO. PNCC remained the franchise holder for the construction, operation, and maintenance of the prerogatives and in pursuit of its right of delectus personae. Thus, the subject tollway projects were
project roads; it only opted to partner with investors in the exercise of its franchise leading to the undertaken by companies, which are the product of the joint ventures between PNCC and its chosen
107
organization of companies such as PSC and SOMCO. partners.

Again, considering that PNCC was granted the right, privilege, and authority to construct, operate, and Under the STOA in this case, PNCC partnered with CMMTC in Stages 1 and 2 of the South Metro
maintain the North Luzon, South Luzon, and Metro Manila Expressways and their toll facilities, we have Manila Skyway. The STOA gave birth to PSC, which was put in charge of the operation and
not heard petitioners decrying the "breach" by PNCC of its franchise when it agreed to make CMMTC maintenance of the project roads. The ASTOA had to be executed for Stage 2 to accommodate
responsible for the design and construction of the project roads under the STOA. changes and modifications in the original design. The ASTOA then brought forth the incorporation of
SOMCO to replace PSC in the operations and maintenance of Stage 1 of the South Metro Manila
IV Skyway. Clearly, no public bidding was necessary because PNCC, the franchisee, merely exercised its
management prerogative when it decided to undertake the construction, operation, and maintenance of
The TOC issued to SOMCO was not irregular. the project roads through companies which are products of joint ventures with chosen partners.
104
Petitioners argue that the conditions provided under Section 3(e) of P.D. 1112 were not imposed on Petitioners also insist that SOMCO is not qualified to operate a toll facility, because it does not meet the
SOMCO, because these do not appear on the face of the TOC. Petitioners are mistaken. nationality requirement for a corporation when scrutinized under the "grandfather rule." Other than
advancing this argument, however, petitioners have not shown how SOMCO fails to meet the nationality
The TOC, as a grant of authority from the government, is subject to the latter's control insofar as the
105 requirement for a public utility operator. Petitioners only aver in their petition that 40% of SOMCO is
grant affects or concerns the public. Like all other franchises or licenses issued by the government,
owned by CMMTC, a foreign company, while the rest is owned by the following: a) Toll Road Operation
the TOC is issued subject to terms, conditions, and limitations under existing laws and agreements.
and Maintenance Venture Corporation (TROMVC), almost 40% of which is owned by a Singaporean
This rule especially holds true in this instance since the TRB has the power to issue "the necessary 'Toll
company; b) Asset values Holding Company, Inc. (AHCI), of which almost 40% is Dutch-owned; and c)
Operation Certificate' subject to such conditions as shall be imposed by the Board including inter alia"
Metro Strategic Infrastructure Holdings, Inc. (MSIHI), 40% of which is owned by Metro Pacific
those specified under Section 3(e) of P.D. 1112. Thus, impliedly written into every TOC are the 108
Corporation, whose ownership or nationality was not specified.
conditions prescribed therein.
Section 11, Article XII of the Constitution provides that "[n]o franchise, certificate, or any other form of
In any case, part of the TOC issued to SOMCO reads:
authorization for the operation of a public utility shall be granted except to citizens of the Philippines or
Pursuant to Section 3(e) of Presidential Decree No. 1112 or the Toll Operation Decree, Skyway O & M to corporations or associations organized under the laws of the Philippines at least sixty per centum of
Corporation is hereby given authority to operate and maintain Stage 1 of the South Metro Manila whose capital is owned by such citizens x x x." Clearly, under the Constitution, a corporation at least
Skyway effective as of 10:00 p.m. of 31 December 2007. 60% of whose capital is owned by Filipinos is of Philippine nationality. Considering this constitutional
provision, petitioners' silence on the ownership of the remaining 60% of the corporations cited is very
This authorization is issued upon the clear understanding that the operation and maintenance of Stage telling.
1 of the South Metro Manila Skyway as a toll facility and the collection of toll fees shall be closely
supervised and regulated by the Grantor, by and through the Board of Directors, in accordance with the In order to rebut petitioners' allegations, respondents readily present matrices showing the itemization of
terms and conditions set forth in the STOA, as amended, the rules and regulations duly promulgated by percentage ownerships of the subscribed capital stock of SOMCO, as well as that of TROMVC, AHCI,
the Grantor for toll road operations and maintenance, as well as the lawful orders, instructions and and MSIHI. Respondents attempt to show that all these corporations are of Philippine nationality, with
conditions which the Grantor, through the TRB, may impose from time to time in view of the public 60% of their capital stock owned by Filipino citizens. We need not reproduce the itemization here.
109
nature of the facility. Suffice it to say that in their Consolidated Reply, petitioners did not refute the unanimous claim of
respondents. It is axiomatic that one who alleges a fact has the burden of proving it. On this matter, we
As regards the allegation that none of the requirements for public bidding was observed before the TOC find that petitioners have failed to prove their allegation that SOMCO is not qualified to operate a toll
was issued to SOMCO, this matter was also squarely answered by the Court in Francisco, Jr. v. facility for failure to meet the nationality requirement under the Constitution.
106
TRB, to wit:
Finally, no public notices and hearings were necessary prior to the issuance of the TOC to SOMCO. For
Where, in the instant case, a franchisee undertakes the tollway projects of construction, rehabilitation the same reason that a public bidding is not necessary, PNCC cannot be required to call for public
and expansion of the tollways under its franchise, there is no need for a public bidding. In pursuing the hearings concerning matters within its prerogative. At any rate, we have studied P.D. 1112 and the
projects with the vast resource requirements, the franchisee can partner with other investors, which it Implementing Rules and Regulations Authorizing the Establishment of Toll Facilities and found no
may choose in the exercise of its management prerogatives. In this case, no public bidding is required provision requiring the issuance of public notices and the conduct of public hearings prior to the
upon the franchisee in choosing its partners as such process was done in the exercise of management issuance of a TOC.
V Petitioners are unimpressed. They cite Section 8 of P.D. 1113 and Section 13 of P.D. 1894 as follows:

Approval of the AS TOA by the DOTC Secretary was approval by the President. SECTION 8. The GRANTEE shall not lease, transfer, grant the usufruct of, sell or assign this franchise
nor the rights or privileges acquired hereby, to any person, firm, company, corporation or other
The doctrine of qualified political agency declares that, save in matters on which the Constitution or the commercial or legal entity, nor merge with any other company or corporation without the prior approval
circumstances require the President to act personally, executive and administrative functions are of the President of the Philippines. In the event that this franchise is sold, transferred or assigned, the
exercised through executive departments headed by cabinet secretaries, whose acts are presumptively transferee shall be subject to all the conditions, terms, restrictions and limitations of this Decree as fully
110
the acts of the President unless disapproved by the latter. As explained in Villena v. Executive and completely and to the same extents as if the franchise has been granted to the same person, firm,
111
Secretary, this doctrine is rooted in the Constitution: company, corporation or other commercial or legal entity. (Emphasis supplied)

x x x With reference to the Executive Department of the government, there is one purpose which is SECTION 13. The GRANTEE shall not lease, transfer, grant the usufruct of, sell or assign this franchise
crystal-clear and is readily visible without the projection of judicial searchlight, and that is, the nor the rights or privileges required hereby, to any person, firm, company, corporation or other legal
establishment of a single, not plural, Executive. The first section of Article VII of the Constitution, dealing entity, nor merge with any other company or corporation without the prior approval of the President of
with the Executive Department, begins with the enunciation of the principle that "The executive power the Philippines. In the event that this franchise is sold, transferred or assigned, the transferee shall be
shall be vested in a President of the Philippines." This means that the President of the Philippines is the subject to all the conditions, terms, restrictions and limitations of this Decree as fully and completely and
Executive of the Government of the Philippines, and no other. The heads of the executive departments to the same extent as if the franchise has been granted to the said person, firm, company, corporation
occupy political positions and hold office in an advisory capacity, and, in the language of Thomas or other legal entity. (Emphasis supplied) Petitioners insist that based on the above provisions, it is the
Jefferson, "should be of the President's bosom confidence," and, in the language of Attorney-General President who should give personal approval considering that the power to grant franchises was
Cushing, "are subject to the direction of the President." Without minimizing the importance of the heads exclusively vested in Congress. Hence, to allow the DOTC Secretary to exercise the power of approval
of the various departments, their personality is in reality but the projection of that of the President. would supposedly dilute that legislative prerogative.
Stated otherwise, and as forcibly characterized by Chief Justice Taft of the Supreme Court of the United
States, "each head of a department is, and must be, the President's alter ego in the matters of that The argument of petitioners is founded on the assumption that PNCC in some way leased, transferred,
department where the President is required by law to exercise authority." Secretaries of departments, of granted the usufruct of, sold, or assigned to SOMCO its franchise or the rights or privileges PNCC had
course, exercise certain powers under the law but the law cannot impair or in any way affect the acquired by it. Here lies the error in petitioners' stand. First, as discussed above, the power to grant
constitutional power of control and direction of the President. As a matter of executive policy, they may franchises or issue authorizations for the operation of a public utility is not exclusively exercised by
be granted departmental autonomy as to certain matters but this is by mere concession of the Congress. Second, except where the situation falls within that special class that demands the exclusive
117
executive, in the absence of valid legislation in the particular field. If the President, then, is the authority and personal exercise by the President of constitutionally vested power, the President acts through
in the Executive Department, he assumes the corresponding responsibility. The head of a department is alter egos whose acts are as if the Chief Executive's own.
a man of his confidence; he controls and directs his acts; he appoints him and can remove him at
112 Third, no lease, transfer, grant of usufruct, sale, or assignment of franchise by PNCC or its merger with
pleasure; he is the executive, not any of his secretaries. x x x (Citations omitted)
another company ever took place.
Applying the doctrine of qualified political agency, we have ruled that the Secretary of Environment and
Natural Resources can validly order the transfer of a regional office by virtue of the power of the The creation of the TRB and the grant of franchise to PNCC were made in the light of the recognition on
113 114 the part of the government that the private sector had to be involved as an alternative source of
President to reorganize the national government. In Constantino v. Cuisia, the Court upheld the
authority of the Secretary of Finance to execute debt-relief contracts. The authority emanates from the financing for the pursuance of national infrastructure projects. As the franchise holder for the
power of the President to contract foreign loans under Section 20, Article VII of the Constitution. In construction, maintenance and operation of infrastructure toll facilities, PNCC was equipped with the
115 right and privilege, but not necessarily the means, to undertake the project. This is where joint ventures
Angeles v. Gaite, the Court ruled that there can be no issue with regard to the President's act of
limiting his power to review decisions and orders of the Secretary of Justice, especially since the with private investors become necessary.
116
decision or order was issued by the secretary, the President's "own alter ego."
A joint venture is an association of companies jointly undertaking a commercial endeavor, with all of
118
There can be no question that the act of the secretary is the act of the President, unless repudiated by them contributing assets and sharing risks, profits, and losses. It is hardly distinguishable from a
the latter. In this case, approval of the ASTOA by the DOTC Secretary had the same effect as approval partnership considering that their elements are similar and, thus, generally governed by the law on
119
by the President. The same would be true even without the issuance of E.O. 497, in which the partnership.
President, on 24 January 2006, specifically delegated to the DOTC Secretary the authority to approve
contracts entered into by the TRB. In joint ventures with investor companies, PNCC contributes the franchise it possesses, while the
partner contributes the financing - both necessary for the construction, maintenance, and operation of
the toll facilities. PNCC did not thereby lease, transfer, grant the usufruct of, sell, or assign its franchise million settlement given to PSC, the amount was to be used principally for the payment of its liabilities of
or other rights or privileges. This remains true even though the partnership acquires a distinct and PSC arising from the retrenchment of its employees. We note that under the MOA, the residual assets
122
separate personality from that of the joint venturers or leads to the formation of a new company that is of PSC shall still be offered for sale to CMMTC, subject to valuation. Thus, it would be inaccurate to
the product of such joint venture, such as PSC and SOMCO in this case. say that PSC would receive only ₱320 million for the entire arrangement.

Hence, when we say that the approval by the DOTC Secretary in this case was approval by the It is quite understandable that SOMCO does not yet have a proven track record in toll operations,
President, it was not in connection with the franchise of PNCC, as required under Section 8 of P.D. considering that it was only the ASTOA and the MOA that gave birth to it. We are not prepared to rule
1113 and Section 13 of P.D. 1894. Rather, the approval was in connection with the powers of the TRB that this lack of track record would result in poor delivery of toll services, especially because most of the
to enter into contracts on behalf of the government as provided under Section 3(a) of P.D. 1112, which former employees of PSC have been rehired by SOMCO, an allegation of respondents that was never
states: refuted by petitioners. Neither are we prepared to take the amount of SOMCO's initial capital investment
123
against it, as it is considerably higher than ₱500,000, the authorized capital stock of PSC as of 2002.
SECTION 3. Powers and Duties of the Board. - The Board shall have in addition to its general powers
of administration the following powers and duties: A FINAL NOTE

(a) Subject to the approval of the President of the Philippines, to enter into contracts in behalf of the R.A. 8975 prohibits lower courts from issuing any temporary restraining order, preliminary injunction, or
Republic of the Philippines with persons, natural or juridical, for the construction, operation and preliminary mandatory injunction against the government - or any of its subdivisions, officials or any
maintenance of toll facilities such as but not limited to national highways, roads, bridges, and public person or entity, whether public or private, acting under the government's direction - to restrain, prohibit
thoroughfares. Said contract shall be open to citizens of the Philippines and/or to corporations or or compel acts related to the implementation and completion of government infrastructure projects.
associations qualified under the Constitution and authorized by law to engage in toll operations;
(Emphasis supplied) The rationale for the law is easily discernible. Injunctions and restraining orders tend to derail the
expeditious and efficient implementation and completion of government infrastructure projects; increase
VI construction, maintenance and repair costs; and delay the enjoyment of the social and economic
benefits therefrom. Thus, unless the matter is of extreme urgency involving a constitutional issue,
Petitioners have not shown that the transfer of toll operations to SOM CO was grossly disadvantageous judges of lower courts who shall issue injunctive writs or restraining orders in violation of the law shall
to the government. be administratively liable.

In support of their contention that the transfer of toll operations from PSC to SOMCO was grossly The law is clear that what is prohibited is merely the issuance of provisional orders enjoining the
disadvantageous to the government, petitioners belittle the initial capital investment, private ownership, implementation of a national government project. R.A. 8975 does not bar lower courts from assuming
and track record of SOMCO. jurisdiction over complaints that seek the nullification or implementation of a national government
124
infrastructure project as ultimate relief.
When one uses the term "grossly disadvantageous to the government," the allegations in support
thereof must reflect the meaning accorded to the phrase. "Gross" means glaring, reprehensible, There is no question that the ultimate prayer in the instant case is the nullification of a national
120
culpable, flagrant, and shocking. It requires that the mere allegation shows that the disadvantage on government project considering that the ASTOA involved the design and construction of Stage 2 of the
the part of the government is unmistakable, obvious, and certain. South Metro Manila Skyway, as well as the operation and maintenance of Stage 1 thereof. The prayer
is grounded on the contract's alleged unconstitutionality, violation of the law, and gross disadvantage to
In this case, we find that the allegations of petitioners are nothing more than speculations, the government. Such principal action and relief were within the jurisdiction of the RTC, which acted
apprehensions, and suppositions.1âwphi1 They speculate that with its "measly" capital investment, correctly when it ordered respondents to file their respective answers to the complaint, even while it
SOMCO would not be able to cover the overhead expenses for personal services alone. They fear that denied the prayer for the issuance of a writ of preliminary injunction and/or temporary restraining order
the revenue from toll operations would go to "private pockets" in exchange for a small settlement in observance of R.A. 8975.
amount to be given to PSC. Given that SOMCO has no proven track record, petitioners deduce that its
assumption of the toll operations would lead to poor delivery of toll services to the public. It was therefore error on the part of petitioners to come directly before this Court for the sole reason that
the lower courts will not be able to grant the prayer for the issuance of a writ of preliminary injunction
The aim in the establishment of toll facilities is to draw from private resources the financing of and/or temporary restraining order to enjoin the assumption of toll operations by SOMCO. The error
government infrastructure projects. Naturally, these private investors would want to receive reasonable even takes on a whole new meaning, because SOMCO assumed responsibility for the operations and
return on their investments. Thus, the collection of toll fees for the use of public improvements has been maintenance of the South Metro Manila Skyway at 10:00 p.m. on 31 December 2007. On the other
121
authorized, subject to supervision and regulation by the national government. As regards the ₱320
hand, the complaint before the RTC seeking to enjoin the assumption by SOMCO was filed only on 3
January 2008, while the instant petition was filed on 4 February 2008.
125
As we held in Aznar Brothers Realty, Inc. v. CA, injunction does not lie when the act sought to be
enjoined has already become a fait accompli or an accomplished or consummated act.

Parties must observe the hierarchy of courts before seeking relief from this Court. Observance thereof
minimizes the imposition on the already limited time of this Court and prevents delay, intended or
126
otherwise, in the adjudication of cases. We do not appreciate the litigants' practice of directly seeking
recourse before this Court, relying on the gravitas of a personality yet making serious claims without the
proof to support them.

WHEREFORE, the petition is DISMISSED. The prayer for the issuance of a writ of preliminary
injunction and/or temporary restraining order is DENIED.

SO ORDERED.

MARIA LOURDES P.A. SERENO


Chief Justice, Chairperson
asking for a reconsideration of his reassignment, and furnished a copy of the same to the DECS.
However, he subsequently changed his mind and refrained from filing the letter with the Office of
President.
2
On October 19, 1994, the petitioner filed the instant petition."

On October 26, 1994, the Court of Appeals denied private respondent’s prayer for the issuance of a
3
Temporary Restraining Order (TRO).

On November 22, 1994, it set aside its earlier resolution denying the prayer for the issuance of a TRO;
THIRD DIVISION
and thereafter, restrained the petitioners "from implementing the re-assignment of the petitioner [private
respondent herein] from incumbent Schools Division Superintendent of Quezon City to Vocational
G.R. No. 119903 August 15, 2000 4
Schools Superintendent of the Marikina Institute of Science and Technology."
HON. RICARDO T. GLORIA, in his capacity as SECRETARY, AND DIRECTOR NILO L. ROSAS in
On December 21, 1994, the Court of Appeals issued another resolution setting the hearing of the
his capacity as REGIONAL DIRECTOR, DEPARTMENT OF EDUCATION, CULTURE AND
petition for the issuance of a writ of preliminary injunction and enjoining the petitioners from
SPORTS, petitioners,
implementing the reassignment of the private respondent.
vs.
HON. COURT OF APPEALS AND DR. BIENVENIDO A. ICASIANO, respondents.
On March 28, 1995, it issued its assailed decision; holding as follows:
DECISION
"WHEREFORE, for lack of a period or any indication that it is only temporary, the reassignment of the
PURISIMA, J.: petitioner from Schools Division Superintendent, Division of City Schools, Quezon City, to Vocational
Schools Superintendent of the Marikina Institute of Science and Technology pursuant to the
This is a petition for review on certiorari under Rule 45 of the Rules of Court brought by Secretary and Memorandum of Secretary Ricardo T. Gloria to the President of the Philippines dated 10 October 1994,
the Director for the National Capital Region of the Department of Education, Culture and Sports is hereby declared to be violative of petitioner’s right to security of tenure, and the respondents are
1
(DECS), to question the decision of the Court of Appeals in CA-G.R. SP No. 35505. hereby prohibited from implementing the same.
5
The Court of Appeals found the facts as follows: SO ORDERED."

"On June 29, 1989, petitioner [private respondent herein] was appointed Schools Division Petitioners are now before the Court seeking relief from the decision of the appellate court, contending
Superintendent, Division of City Schools, Quezon City, by the then President Corazon C. Aquino. that:

On October 10, 1994, respondent Secretary Gloria recommended to the President of the Philippines I
that the petitioner be reassigned as Superintendent of the MIST [Marikina Institute of Science and
RESPONDENT COURT OF APPEALS HAS ALLOWED ITSELF TO BE INSTRUMENTAL IN PRIVATE
Technology], to fill up the vacuum created by the retirement of its Superintendent, Mr. Bannaoag F.
RESPONDENT’S CIRCUMVENTION OF THE PRESIDENTIAL IMMUNITY FROM SUIT BY GIVING
Lauro, on June 17, 1994.
DUE COURSE AND GRANTING RELIEFS PRAYED FOR IN A SUIT PURPORTEDLY FILED
On October 12, 1994, the President approved the recommendation of Secretary Gloria. AGAINST PETITIONERS BUT ACTUALLY QUESTIONING AN ACT OF THE PRESIDENT.

On October 13, 1994, a copy of the recommendation for petitioner’s reassignment, as approved by the II
President, was transmitted by Secretary Gloria to Director Rosas for implementation.
RESPONDENT COURT OF APPEALS HAS DECIDED A QUESTION OF SUBSTANCE IN A WAY
6
On October 14, 1994, Director Rosas, informed the petitioner of his reassignment, effective October 17, NOT IN ACCORD WITH LAW OR APPLICABLE DECISIONS OF THE SUPREME COURT
1994.
The pivotal issue for resolution here is whether the reassignment of private respondent from School
Petitioner requested respondent Secretary Gloria to reconsider the reassignment, but the latter denied Division Superintendent of Quezon City to Vocational School Superintendent of MIST is violative of his
the request. The petitioner prepared a letter dated October 18, 1994 to the President of the Philippines,
security of tenure? Petitioners maintain that there is no violation of security of tenure involved. Private the said Memorandum to show that the reassignment of private respondent is temporary or would only
respondent maintains otherwise. last until a permanent replacement is found as no period is specified or fixed; which fact evinces an
intention on the part of petitioners to reassign private respondent with no definite period or duration.
In taking favorable action on private respondent’s petition for prohibition, the Court of Appeals Such feature of the reassignment in question is definitely violative of the security of tenure of the private
ratiocinated: respondent. As held in Bentain:

"Notwithstanding the protestations of counsel for the respondents, the reassignment of the petitioner to "Security of tenure is a fundamental and constitutionally guaranteed feature of our civil
MIST appears to be indefinite. No period is fixed. No objective or purpose, from which the service.1âwphi1 The mantle of its protection extends not only to employees removed without cause but
temporariness of the assignment may be inferred, is set. In fact, the recommendation of respondent also to cases of unconsented transfers which are tantamount to illegal removals (Department of
Secretary Gloria to the President that the position of superintendent of MIST ‘will best fit his Education, Culture and Sports vs. Court of Appeals, 183 SCRA 555; Ibanez vs. COMELEC, 19 SCRA
(petitioner’s) qualifications and experience.’ (Exh. ‘C-2’) implies that the proposed reassignment will be 1002; Brillantes vs. Guevarra, 27 SCRA 138).
7
indefinite."
While a temporary transfer or assignment of personnel is permissible even without the employee’s prior
Petitioners theorize that the present petition for prohibition is improper because the same attacks an act consent, it cannot be done when the transfer is a preliminary step toward his removal, or is a scheme to
of the President, in violation of the doctrine of presidential immunity from suit. lure him away from his permanent position, or designed to indirectly terminate his service, or force his
resignation. Such a transfer would in effect circumvent the provision which safeguards the tenure of
Petitioners’ contention is untenable for the simple reason that the petition is directed against petitioners office of those who are in the Civil Service (Sta. Maria vs. Lopez, 31 SCRA 651; Garcia vs. Lejano, 109
and not against the President. The questioned acts are those of petitioners and not of the President. Phil. 116)."
12

Furthermore, presidential decisions may be questioned before the courts where there is grave abuse of
8
discretion or that the President acted without or in excess of jurisdiction. Having found the reassignment of private respondent to the MIST to be violative of his security of
tenure, the order for his reassignment to the MIST cannot be countenanced.
Petitioners’ submission that the petition of private respondent with the Court of Appeals is improper for
failing to show that petitioners constituted themselves into a "court" conducting a "proceeding" and for WHEREFORE, the petition is hereby DENIED, and the Decision of the Court of Appeals in CA-G.R. SP
failing to show that any of the petitioners acted beyond their jurisdiction in the exercise of their judicial or No. 35505 AFFIRMED. No pronouncement as to costs.
ministerial functions, is barren of merit. Private respondent has clearly averred that the petitioners acted
with grave abuse of discretion amounting to lack of jurisdiction and/or excess of jurisdiction in SO ORDERED.
reassigning the private respondent in a way that infringed upon his security of tenure. And petitioners
themselves admitted that their questioned act constituted a ministerial duty, such that they could be Melo, (Chairman), Vitug, Panganiban, and Gonzaga-Reyes, JJ., concur.
subject to charges of insubordination if they did not comply with the presidential order. What is more,
where an administrative department acts with grave abuse of discretion, which is equivalent to a
capricious and whimsical exercise of judgment, or where the power is exercised in an arbitrary or
despotic manner, there is a justification for the courts to set aside the administrative determination thus
9
reached.
10
Petitioners contend that the doctrine enunciated in Bentain vs. Court of Appeals -- that "a
reassignment that is indefinite and results in a reduction in rank, status and salary, is in effect, a
constructive removal from the service" -- does not apply in the present case for the reassignment in
question was merely temporary, lasting only until the appointment of a new Vocational School
Superintendent of MIST.

After a careful study, the Court upholds the finding of the respondent court that the reassignment of
11
petitioner to MIST "appears to be indefinite". The same can be inferred from the Memorandum of
Secretary Gloria for President Fidel V. Ramos to the effect that the reassignment of private respondent
will "best fit his qualifications and experience" being "an expert in vocational and technical education." It
can thus be gleaned that subject reassignment is more than temporary as the private respondent has
been described as fit for the (reassigned) job, being an expert in the field. Besides, there is nothing in
exceeding P1,000.00 in provinces and cities and P700.00 in municipalities may be granted subject to
the following conditions:

a) That the grant is not mandatory on the part of the LGUs;

b) That all contractual and statutory obligations of the LGU including the implementation of R.A. 6758
shall have been fully provided in the budget;

c) That the budgetary requirements/limitations under Section 324 and 325 of R.A. 7160 should be
satisfied and/or complied with; and
EN BANC
d) That the LGU has fully implemented the devolution of functions/personnel in accordance with R.A.
G.R. No. 125350 December 3, 2002 3
7160. " (italics supplied)
HON. RTC JUDGES MERCEDES G. DADOLE (Executive Judge, Branch 28),
xxx xxx xxx
ULRIC R. CAÑETE (Presiding Judge, Branch 25),
AGUSTINE R. VESTIL (Presiding Judge, Branch 56), The said circular likewise provided for its immediate effectivity without need of publication:
HON. MTC JUDGES TEMISTOCLES M. BOHOLST (Presiding Judge, Branch 1),
VICENTE C. FANILAG (Judge Designate, Branch 2), "5.0 EFFECTIVITY
and WILFREDO A. DAGATAN (Presiding Judge, Branch 3), all of Mandaue City, petitioners,
vs. This Circular shall take effect immediately."
COMMISSION ON AUDIT, respondent.
Acting on the DBM directive, the Mandaue City Auditor issued notices of disallowance to herein
DECISION petitioners, namely, Honorable RTC Judges Mercedes G. Dadole, Ulric R. Cañete, Agustin R. Vestil,
Honorable MTC Judges Temistocles M. Boholst, Vicente C. Fanilag and Wilfredo A. Dagatan, in excess
CORONA, J.: of the amount authorized by LBC 55. Beginning October, 1994, the additional monthly allowances of the
1 2
petitioner judges were reduced to P1,000 each. They were also asked to reimburse the amount they
Before us is a petition for certiorari under Rule 64 to annul the decision and resolution , dated received in excess of P1,000 from April to September, 1994.
September 21, 1995 and May 28, 1996, respectively, of the respondent Commission on Audit (COA)
affirming the notices of the Mandaue City Auditor which diminished the monthly additional allowances The petitioner judges filed with the Office of the City Auditor a protest against the notices of
received by the petitioner judges of the Regional Trial Court (RTC) and Municipal Trial Court (MTC) disallowance. But the City Auditor treated the protest as a motion for reconsideration and indorsed the
stationed in Mandaue City. same to the COA Regional Office No. 7. In turn, the COA Regional Office referred the motion to the
head office with a recommendation that the same be denied.
The undisputed facts are as follows:
On September 21, 1995, respondent COA rendered a decision denying petitioners' motion for
In 1986, the RTC and MTC judges of Mandaue City started receiving monthly allowances of P1,260 reconsideration. The COA held that:
each through the yearly appropriation ordinance enacted by the Sangguniang Panlungsod of the said
city. In 1991, Mandaue City increased the amount to P1,500 for each judge. The issue to be resolved in the instant appeal is whether or not the City Ordinance of Mandaue which
provides a higher rate of allowances to the appellant judges may prevail over that fixed by the DBM
On March 15, 1994, the Department of Budget and Management (DBM) issued the disputed Local under Local Budget Circular No. 55 dated March 15, 1994.
Budget Circular No. 55 (LBC 55) which provided that:
xxx xxx xxx
"x x x xxx xxx
Applying the foregoing doctrine, appropriation ordinance of local government units is subject to the
2.3.2. In the light of the authority granted to the local government units under the Local Government organizational, budgetary and compensation policies of budgetary authorities (COA 5th Ind., dated
Code to provide for additional allowances and other benefits to national government officials and March 17, 1994 re: Province of Antique; COA letter dated May 17, 1994 re: Request of Hon. Renato
employees assigned in their locality, such additional allowances in the form of honorarium at rates not Leviste, Cong. 1st Dist. Oriental Mindoro). In this regard, attention is invited to Administrative Order No.
42 issued on March 3, 1993 by the President of the Philippines clarifying the role of DBM in the Hence, this petition for certiorari by the petitioner judges, submitting the following questions for
compensation and classification of local government positions under RA No. 7160 vis-avis the resolution:
provisions of RA No. 6758 in view of the abolition of the JCLGPA. Section 1 of said Administrative
Order provides that: I

"Section 1. The Department of Budget and Management as the lead administrator of RA No. 6758 shall, HAS THE CITY OF MANDAUE STATUTORY AND CONSTITUTIONAL BASIS TO PROVIDE
through its Compensation and Position Classification Bureau, continue to have the following ADDITIONAL ALLOWANCES AND OTHER BENEFITS TO JUDGES STATIONED IN AND ASSIGNED
responsibilities in connection with the implementation of the Local Government Code of 1991: TO THE CITY?

a) Provide guidelines on the classification of local government positions and on the specific rates of pay II
therefore;
CAN AN ADMINISTRATIVE CIRCULAR OR GUIDELINE SUCH AS LOCAL BUDGET CIRCULAR NO.
b) Provide criteria and guidelines for the grant of all allowances and additional forms of compensation to 55 RENDER INOPERATIVE THE POWER OF THE LEGISLATIVE BODY OF A CITY BY SETTING A
local government employees; xxx." (underscoring supplied) LIMIT TO THE EXTENT OF THE EXERCISE OF SUCH POWER?

To operationalize the aforecited presidential directive, DBM issued LBC No. 55, dated March 15, 1994, III
whose effectivity clause provides that:
HAS THE COMMISSION ON AUDIT CORRECTLY INTERPRETED LOCAL BUDGET CIRCULAR NO.
xxx xxx xxx 55 TO INCLUDE MEMBERS OF THE JUDICIARY IN FIXING THE CEILING OF ADDITIONAL
ALLOWANCES AND BENEFITS TO BE PROVIDED TO JUDGES STATIONED IN AND ASSIGNED
"5.0 EFFECTIVITY TO MANDAUE CITY BY THE CITY GOVERNMENT AT P1,000.00 PER MONTH NOTWITHSTANDING
THAT THEY HAVE BEEN RECEIVING ALLOWANCES OF P1,500.00 MONTHLY FOR THE PAST
This Circular shall take effect immediately." FIVE YEARS?

It is a well-settled rule that implementing rules and regulations promulgated by administrative or IV


executive officer in accordance with, and as authorized by law, has the force and effect of law or
partake the nature of a statute (Victorias Milling Co., Inc., vs. Social Security Commission, 114 Phil. IS LOCAL BUDGET CIRCULAR NO. 55 DATED MARCH 15, 1994 ISSUED BY THE DEPARTMENT
555, cited in Agpalo's Statutory Construction, 2nd Ed. P. 16; Justice Cruz's Phil. Political Law, 1984 Ed., OF BUDGET AND MANAGEMENT VALID AND ENFORCEABLE CONSIDERING THAT IT WAS NOT
5
p. 103; Espanol vs. Phil Veterans Administration, 137 SCRA 314; Antique Sawmills Inc. vs. Tayco, 17 DULY PUBLISHED IN ACCODANCE WITH LAW?
SCRA 316).
Petitioner judges argue that LBC 55 is void for infringing on the local autonomy of Mandaue City by
xxx xxx xxx dictating a uniform amount that a local government unit can disburse as additional allowances to judges
stationed therein. They maintain that said circular is not supported by any law and therefore goes
There being no statutory basis to grant additional allowance to judges in excess of P1,000.00 beyond the supervisory powers of the President. They further allege that said circular is void for lack of
chargeable against the local government units where they are stationed, this Commission finds no publication.
substantial grounds or cogent reason to disturb the decision of the City Auditor, Mandaue City,
disallowing in audit the allowances in question. Accordingly, the above-captioned appeal of the MTC On the other hand, the yearly appropriation ordinance providing for additional allowances to judges is
and RTC Judges of Mandaue City, insofar as the same is not covered by Circular Letter No. 91-7, is allowed by Section 458, par. (a)(1)[xi], of RA 7160, otherwise known as the Local Government Code of
hereby dismissed for lack of merit. 1991, which provides that:
4
xxx xxx xxx Sec. 458. Powers, Duties, Functions and Compensation. – (a) The sangguniang panlungsod, as the
legislative body of the city, shall enact ordinances, approve resolutions and appropriate funds for the
On November 27, 1995, Executive Judge Mercedes Gozo-Dadole, for and in behalf of the petitioner general welfare of the city and its inhabitants pursuant to Section 16 of this Code and in the proper
judges, filed a motion for reconsideration of the decision of the COA. In a resolution dated May 28, exercise of the corporate powers of the city as provided for under Section 22 of this Code, and shall:
1996, the COA denied the motion.
(1) Approve ordinances and pass resolutions necessary for an efficient and effective city government,
and in this connection, shall:
xxx xxx xxx On the first issue, we declare LBC 55 to be null and void.
6
(xi) When the finances of the city government allow, provide for additional allowances and other benefits We recognize that, although our Constitution guarantees autonomy to local government units, the
to judges, prosecutors, public elementary and high school teachers, and other national government exercise of local autonomy remains subject to the power of control by Congress and the power of
officials stationed in or assigned to the city; (italics supplied) supervision by the President. Section 4 of Article X of the 1987 Philippine Constitution provides that:

Instead of filing a comment on behalf of respondent COA, the Solicitor General filed a manifestation Sec. 4. The President of the Philippines shall exercise general supervision over local governments. x x
supporting the position of the petitioner judges. The Solicitor General argues that (1) DBM only enjoys x
the power to review and determine whether the disbursements of funds were made in accordance with
7
the ordinance passed by a local government unit while (2) the COA has no more than auditorial In Pimentel vs. Aguirre , we defined the supervisory power of the President and distinguished it from the
visitation powers over local government units pursuant to Section 348 of RA 7160 which provides for power of control exercised by Congress. Thus:
the power to inspect at any time the financial accounts of local government units.
This provision (Section 4 of Article X of the 1987 Philippine Constitution) has been interpreted to
i5
Moreover, the Solicitor General opines that "the DBM and the respondent are only authorized under RA exclude the power of control. In Mondano v. Silvosa, the Court contrasted the President's power of
7160 to promulgate a Budget Operations Manual for local government units, to improve and supervision over local government officials with that of his power of control over executive officials of the
systematize methods, techniques and procedures employed in budget preparation, authorization, national government. It was emphasized that the two terms -- supervision and control -- differed in
execution and accountability" pursuant to Section 354 of RA 7160. The Solicitor General points out that meaning and extent. The Court distinguished them as follows:
LBC 55 was not exercised under any of the aforementioned provisions.
"x x x In administrative law, supervision means overseeing or the power or authority of an officer to see
Respondent COA, on the other hand, insists that the constitutional and statutory authority of a city that subordinate officers perform their duties. If the latter fail or neglect to fulfill them, the former may
government to provide allowances to judges stationed therein is not absolute. Congress may set take such action or step as prescribed by law to make them perform their duties. Control, on the other
limitations on the exercise of autonomy. It is for the President, through the DBM, to check whether these hand, means the power of an officer to alter or modify or nullify or set aside what a subordinate officer
legislative limitations are being followed by the local government units. ha[s] done in the performance of his duties and to substitute the judgment of the former for that of the
ii 6
latter."
One such law imposing a limitation on a local government unit's autonomy is Section 458, par. (a) (1)
iii 7
[xi], of RA 7160, which authorizes the disbursement of additional allowances and other benefits to In Taule v. Santos, we further stated that the Chief Executive wielded no more authority than that of
judges subject to the condition that the finances of the city government should allow the same. Thus, checking whether local governments or their officials were performing their duties as provided by the
DBM is merely enforcing the condition of the law when it sets a uniform maximum amount for the fundamental law and by statutes. He cannot interfere with local governments, so long as they act within
additional allowances that a city government can release to judges stationed therein. the scope of their authority. "Supervisory power, when contrasted with control, is the power of mere
iv 8
oversight over an inferior body; it does not include any restraining authority over such body," we said.
Assuming arguendo that LBC 55 is void, respondent COA maintains that the provisions of the yearly
v9
approved ordinance granting additional allowances to judges are still prohibited by the appropriation In a more recent case, Drilon v. Lim, the difference between control and supervision was further
laws passed by Congress every year. COA argues that Mandaue City gets the funds for the said delineated. Officers in control lay down the rules in the performance or accomplishment of an act. If
additional allowances of judges from the Internal Revenue Allotment (IRA). But the General these rules are not followed, they may, in their discretion, order the act undone or redone by their
Appropriations Acts of 1994 and 1995 do not mention the disbursement of additional allowances to subordinates or even decide to do it themselves. On the other hand, supervision does not cover such
judges as one of the allowable uses of the IRA. Hence, the provisions of said ordinance granting authority. Supervising officials merely see to it that the rules are followed, but they themselves do not
additional allowances, taken from the IRA, to herein petitioner judges are void for being contrary to law. lay down such rules, nor do they have the discretion to modify or replace them. If the rules are not
observed, they may order the work done or redone, but only to conform to such rules. They may not
To resolve the instant petition, there are two issues that we must address: (1) whether LBC 55 of the prescribe their own manner of execution of the act. They have no discretion on this matter except to see
DBM is void for going beyond the supervisory powers of the President and for not having been to it that the rules are followed.
published and (2) whether the yearly appropriation ordinance enacted by the City of Mandaue that
vi10
provides for additional allowances to judges contravenes the annual appropriation laws enacted by Under our present system of government, executive power is vested in the President. The members
Congress. of the Cabinet and other executive officials are merely alter egos. As such, they are subject to the
power of control of the President, at whose will and behest they can be removed from office; or their
vii 11
We rule in favor of the petitioner judges. actions and decisions changed, suspended or reversed. In contrast, the heads of political
subdivisions are elected by the people. Their sovereign powers emanate from the electorate, to whom
they are directly accountable. By constitutional fiat, they are subject to the President's supervision only, publication, a DBM circular that disallowed payment of allowances and other additional compensation to
not control, so long as their acts are exercised within the sphere of their legitimate powers. By the same government officials and employees. In refuting respondent COA's argument that said circular was
token, the President may not withhold or alter any authority or power given them by the Constitution and merely an internal regulation, we ruled that:
the law.
On the need for publication of subject DBM-CCC No. 10, we rule in the affirmative. Following the
Clearly then, the President can only interfere in the affairs and activities of a local government unit if he doctrine enunciated in Tañada v. Tuvera, publication in the Official Gazette or in a newspaper of
or she finds that the latter has acted contrary to law. This is the scope of the President's supervisory general circulation in the Philippines is required since DBM-CCC No. 10 is in the nature of an
powers over local government units. Hence, the President or any of his or her alter egos cannot administrative circular the purpose of which is to enforce or implement an existing law. Stated
interfere in local affairs as long as the concerned local government unit acts within the parameters of differently, to be effective and enforceable, DBM-CCC No. 10 must go through the requisite publication
the law and the Constitution. Any directive therefore by the President or any of his or her alter in the Official Gazette or in a newspaper of general circulation in the Philippines.
egos seeking to alter the wisdom of a law-conforming judgment on local affairs of a local government
unit is a patent nullity because it violates the principle of local autonomy and separation of powers of the In the present case under scrutiny, it is decisively clear that DBM-CCC No. 10, which completely
executive and legislative departments in governing municipal corporations. disallows payment of allowances and other additional compensation to government officials and
employees, starting November 1, 1989, is not a mere interpretative or internal regulation. It is something
Does LBC 55 go beyond the law it seeks to implement? Yes. more than that. And why not, when it tends to deprive government workers of their allowance and
additional compensation sorely needed to keep body and soul together. At the very least, before the
LBC 55 provides that the additional monthly allowances to be given by a local government unit should said circular under attack may be permitted to substantially reduce their income, the
not exceed P1,000 in provinces and cities and P700 in municipalities. Section 458, par. (a)(1)(xi), of RA government officials and employees concerned should be apprised and alerted by the
7160, the law that supposedly serves as the legal basis of LBC 55, allows the grant of additional publication of subject circular in the Official Gazette or in a newspaper of general circulation in
allowances to judges "when the finances of the city government allow." The said provision does not the Philippines – to the end that they be given amplest opportunity to voice out whatever
authorize setting a definite maximum limit to the additional allowances granted to judges. Thus, we opposition they may have, and to ventilate their stance on the matter. This approach is more in
need not belabor the point that the finances of a city government may allow the grant of additional keeping with democratic precepts and rudiments of fairness and transparency. (emphasis
allowances higher than P1,000 if the revenues of the said city government exceed its annual supplied)
expenditures. Thus, to illustrate, a city government with locally generated annual revenues of P40
10
million and expenditures of P35 million can afford to grant additional allowances of more than P1,000 In Philippine International Trading Corporation vs. Commission on Audit , we again declared the same
each to, say, ten judges inasmuch as the finances of the city can afford it. circular as void, for lack of publication, despite the fact that it was re-issued and then submitted for
publication. Emphasizing the importance of publication to the effectivity of a regulation, we therein held
Setting a uniform amount for the grant of additional allowances is an inappropriate way of enforcing the that:
criterion found in Section 458, par. (a)(1)(xi), of RA 7160. The DBM over-stepped its power of
supervision over local government units by imposing a prohibition that did not correspond with the law it It has come to our knowledge that DBM-CCC No. 10 has been re-issued in its entirety and submitted for
sought to implement. In other words, the prohibitory nature of the circular had no legal basis. publication in the Official Gazette per letter to the National Printing Office dated March 9, 1999. Would
the subsequent publication thereof cure the defect and retroact to the time that the above-mentioned
Furthermore, LBC 55 is void on account of its lack of publication, in violation of our ruling in Tañada vs. items were disallowed in audit?
8
Tuvera where we held that:
The answer is in the negative, precisely for the reason that publication is required as a condition
xxx. Administrative rules and regulations must also be published if their purpose is to enforce or precedent to the effectivity of a law to inform the public of the contents of the law or rules and
implement existing law pursuant to a valid delegation. regulations before their rights and interests are affected by the same. From the time the COA
disallowed the expenses in audit up to the filing of herein petition the subject circular remained in legal
Interpretative regulations and those merely internal in nature, that is, regulating only the personnel of an limbo due to its non-publication. As was stated in Tañada v. Tuvera, "prior publication of laws before
administrative agency and the public, need not be published. Neither is publication required of the so- they become effective cannot be dispensed with, for the reason that it would deny the public knowledge
called letters of instruction issued by administrative superiors concerning the rules or guidelines to be of the laws that are supposed to govern it."
11

followed by their subordinates in the performance of their duties.


We now resolve the second issue of whether the yearly appropriation ordinance enacted by Mandaue
Respondent COA claims that publication is not required for LBC 55 inasmuch as it is merely an City providing for fixed allowances for judges contravenes any law and should therefore be struck down
interpretative regulation applicable to the personnel of an LGU. We disagree. In De Jesus vs. as null and void.
9
Commission on Audit where we dealt with the same issue, this Court declared void, for lack of
According to respondent COA, even if LBC 55 were void, the ordinances enacted by Mandaue City Within 90 days from receipt of the copies of the appropriation ordinance, the DBM should have taken
granting additional allowances to the petitioner judges would "still (be) bereft of legal basis for want of a positive action. Otherwise, such ordinance was deemed to have been properly reviewed and deemed to
lawful source of funds considering that the IRA cannot be used for such purposes." Respondent COA have taken effect. Inasmuch as, in the instant case, the DBM did not follow the appropriate procedure
showed that Mandaue City's funds consisted of locally generated revenues and the IRA. From 1989 to for reviewing the subject ordinance of Mandaue City and allowed the 90-day period to lapse, it can no
1995, Mandaue City's yearly expenditures exceeded its locally generated revenues, thus resulting in a longer question the legality of the provisions in the said ordinance granting additional allowances to
deficit. During all those years, it was the IRA that enabled Mandaue City to incur a surplus. Respondent judges stationed in the said city.
avers that Mandaue City used its IRA to pay for said additional allowances and this violated paragraph
12 WHEREFORE, the petition is hereby GRANTED, and the assailed decision and resolution, dated
2 of the Special Provisions, page 1060, of RA 7845 (The General Appropriations Act of 1995) and
paragraph 3 of the Special Provision, page 1225, of RA 7663 (The General Appropriations Act of September 21, 1995 and May 28, 1996, respectively, of the Commission on Audit are hereby set aside.
13
1994) which specifically identified the objects of expenditure of the IRA. Nowhere in said provisions of
the two budgetary laws does it say that the IRA can be used for additional allowances of judges. No costs.
Respondent COA thus argues that the provisions in the ordinance providing for such disbursement are
SO ORDERED.
against the law, considering that the grant of the subject allowances is not within the specified use
allowed by the aforesaid yearly appropriations acts.

We disagree.

Respondent COA failed to prove that Mandaue City used the IRA to spend for the additional allowances
of the judges. There was no evidence submitted by COA showing the breakdown of the expenses of the
city government and the funds used for said expenses. All the COA presented were the amounts
expended, the locally generated revenues, the deficit, the surplus and the IRA received each year.
Aside from these items, no data or figures were presented to show that Mandaue City deducted the
subject allowances from the IRA. In other words, just because Mandaue City's locally generated
revenues were not enough to cover its expenditures, this did not mean that the additional allowances of
petitioner judges were taken from the IRA and not from the city's own revenues.

Moreover, the DBM neither conducted a formal review nor ordered a disapproval of Mandaue City's
appropriation ordinances, in accordance with the procedure outlined by Sections 326 and 327 of RA
7160 which provide that:

Section 326. Review of Appropriation Ordinances of Provinces, Highly Urbanized Cities, Independent
Component Cities, and Municipalities within the Metropolitan Manila Area. The Department of Budget
and Management shall review ordinances authorizing the annual or supplemental appropriations of
provinces, highly-urbanized cities, independent component cities, and municipalities within the
Metropolitan Manila Area in accordance with the immediately succeeding Section.

Section 327. Review of Appropriation Ordinances of Component Cities and Municipalities.- The
sangguninang panlalawigan shall review the ordinance authorizing annual or supplemental
appropriations of component cities and municipalities in the same manner and within the same period
prescribed for the review of other ordinances.

If within ninety (90) days from receipt of copies of such ordinance, the sangguniang
panlalawigan takes no action thereon, the same shall be deemed to have been reviewed in
accordance with law and shall continue to be in full force and effect. (emphasis supplied)
EN BANC undertake such measures as may be allowed by the Constitution and by law to prevent and suppress all
incidents of lawless violence in the named places.
DATU ZALDY UY AMPATUAN, G.R. No. 190259
ANSARUDDIN ADIONG, REGIE Three days later or on November 27, President Arroyo also issued Administrative Order 273 (AO
[2]
SAHALI-GENERALE 273) transferring supervision of the Autonomous Region of Muslim Mindanao (ARMM) from the Office
Petitioners, Present: of the President to the Department of Interior and Local Government (DILG). But, due to issues raised
CORONA, C.J., over the terminology used in AO 273, the President issued Administrative Order 273-A (AO 273-A)
[3]
CARPIO, amending the former, by delegating instead of transferring supervision of the ARMM to the DILG.
CARPIO MORALES,
VELASCO, JR., Claiming that the Presidents issuances encroached on the ARMMs autonomy, petitioners Datu Zaldy
[4]
NACHURA, Uy Ampatuan, Ansaruddin Adiong, and Regie Sahali-Generale, all ARMM officials, filed this petition
LEONARDO-DE CASTRO, for prohibition under Rule 65. They alleged that the proclamation and the orders empowered the DILG
- versus - BRION, Secretary to take over ARMMs operations and seize the regional governments powers, in violation of
PERALTA, the principle of local autonomy under Republic Act 9054 (also known as the Expanded ARMM Act) and
BERSAMIN, the Constitution. The President gave the DILG Secretary the power to exercise, not merely
DEL CASTILLO, administrative supervision, but control over the ARMM since the latter could suspend ARMM officials
[5]
ABAD, and replace them.
VILLARAMA, JR.,
PEREZ, Petitioner ARMM officials claimed that the President had no factual basis for declaring a state of
MENDOZA, and emergency, especially in the Province of Sultan Kudarat and the City of Cotabato, where no critical
SERENO, JJ. violent incidents occurred. The deployment of troops and the taking over of the ARMM constitutes an
[6]
HON. RONALDO PUNO, in his capacity invalid exercise of the Presidents emergency powers. Petitioners asked that Proclamation 1946 as
as Secretary of the Department of Interior well as AOs 273 and 273-A be declared unconstitutional and that respondents DILG Secretary, the
and Local Government and alter-ego of AFP, and the PNP be enjoined from implementing them.
President Gloria Macapagal-Arroyo, [7]
In its comment for the respondents, the Office of the Solicitor General (OSG) insisted that the
and anyone acting in his stead and on
President issued Proclamation 1946, not to deprive the ARMM of its autonomy, but to restore peace
behalf of the President of the Philippines, [8] [9]
and order in subject places. She issued the proclamation pursuant to her calling out power as
ARMED FORCES OF THE PHILIPPINES
Commander-in-Chief under the first sentence of Section 18, Article VII of the Constitution. The
(AFP), or any of their units operating in [10]
determination of the need to exercise this power rests solely on her wisdom. She must use her
the Autonomous Region in Muslim
judgment based on intelligence reports and such best information as are available to her to call out the
Mindanao (ARMM), and PHILIPPINE
armed forces to suppress and prevent lawless violence wherever and whenever these reared their ugly
NATIONAL POLICE, or any of their Promulgated:
heads.
units operating in ARMM,
Respondents. June 7, 2011 On the other hand, the President merely delegated through AOs 273 and 273-A her supervisory powers
over the ARMM to the DILG Secretary who was her alter ego any way. These orders did not authorize a
take over of the ARMM. They did not give him blanket authority to suspend or replace ARMM
x ---------------------------------------------------------------------------------------- x [11] [12]
officials. The delegation was necessary to facilitate the investigation of the mass killings. Further,
DECISION the assailed proclamation and administrative orders did not provide for the exercise of emergency
[13]
powers.
ABAD, J.:
Although normalcy has in the meantime returned to the places subject of this petition, it might be
On November 24, 2009, the day after the gruesome massacre of 57 men and women, including some relevant to rule on the issues raised in this petition since some acts done pursuant to Proclamation
[1] 1946 and AOs 273 and 273-A could impact on the administrative and criminal cases that the
news reporters, then President Gloria Macapagal-Arroyo issued Proclamation 1946, placing the
Provinces of Maguindanao and Sultan Kudarat and the City of Cotabato under a state of emergency. government subsequently filed against those believed affected by such proclamation and orders.
She directed the Armed Forces of the Philippines (AFP) and the Philippine National Police (PNP) to
The Issues Presented Three. The Presidents call on the armed forces to prevent or suppress lawless violence springs from
[17]
the power vested in her under Section 18, Article VII of the Constitution, which provides.
The issues presented in this case are:
SECTION 18. The President shall be the Commander-in-Chief of all armed forces of the
1. Whether or not Proclamation 1946 and AOs 273 and 273-A violate the principle of local autonomy Philippines and whenever it becomes necessary, he may call out such armed forces to prevent
under Section 16, Article X of the Constitution, and Section 1, Article V of the Expanded ARMM Organic or suppress lawless violence, invasion or rebellion. x x x
Act;
While it is true that the Court may inquire into the factual bases for the Presidents exercise of the above
2. Whether or not President Arroyo invalidly exercised emergency powers when she called out the AFP [18]
power, it would generally defer to her judgment on the matter. As the Court acknowledged
and the PNP to prevent and suppress all incidents of lawless violence in Maguindanao, Sultan Kudarat, [19]
in Integrated Bar of the Philippines v. Hon. Zamora, it is clearly to the President that the Constitution
and Cotabato City; and entrusts the determination of the need for calling out the armed forces to prevent and suppress lawless
violence. Unless it is shown that such determination was attended by grave abuse of discretion, the
3. Whether or not the President had factual bases for her actions. Court will accord respect to the Presidents judgment. Thus, the Court said:
The Rulings of the Court If the petitioner fails, by way of proof, to support the assertion that the President acted without
factual basis, then this Court cannot undertake an independent investigation beyond the
We dismiss the petition.
pleadings. The factual necessity of calling out the armed forces is not easily quantifiable and
One. The claim of petitioners that the subject proclamation and administrative orders violate the cannot be objectively established since matters considered for satisfying the same is a
principle of local autonomy is anchored on the allegation that, through them, the President authorized combination of several factors which are not always accessible to the courts. Besides the
absence of textual standards that the court may use to judge necessity, information necessary
the DILG Secretary to take over the operations of the ARMM and assume direct governmental powers
to arrive at such judgment might also prove unmanageable for the courts. Certain pertinent
over the region.
information might be difficult to verify, or wholly unavailable to the courts. In many instances,
But, in the first place, the DILG Secretary did not take over control of the powers of the ARMM. After law the evidence upon which the President might decide that there is a need to call out the armed
enforcement agents took respondent Governor of ARMM into custody for alleged complicity in the forces may be of a nature not constituting technical proof.
Maguindanao massacre, the ARMM Vice-Governor, petitioner Ansaruddin Adiong, assumed the
On the other hand, the President, as Commander-in-Chief has a vast intelligence network to
vacated post on December 10, 2009 pursuant to the rule on succession found in Article VII, Section
[14] gather information, some of which may be classified as highly confidential or affecting the
12, of RA 9054. In turn, Acting Governor Adiong named the then Speaker of the ARMM Regional
[15] security of the state. In the exercise of the power to call, on-the-spot decisions may be
Assembly, petitioner Sahali-Generale, Acting ARMM Vice-Governor. In short, the DILG Secretary did
not take over the administration or operations of the ARMM. imperatively necessary in emergency situations to avert great loss of human lives and mass
destruction of property. Indeed, the decision to call out the military to prevent or suppress
[20]
Two. Petitioners contend that the President unlawfully exercised emergency powers when she ordered lawless violence must be done swiftly and decisively if it were to have any effect at all. x x x.
[16]
the deployment of AFP and PNP personnel in the places mentioned in the proclamation. But such
deployment is not by itself an exercise of emergency powers as understood under Section 23 (2), Here, petitioners failed to show that the declaration of a state of emergency in the Provinces of
Article VI of the Constitution, which provides: Maguindanao, Sultan Kudarat and Cotabato City, as well as the Presidents exercise of the calling out
power had no factual basis. They simply alleged that, since not all areas under the ARMM were placed
SECTION 23. x x x (2) In times of war or other national emergency, the Congress may, by law, under a state of emergency, it follows that the take over of the entire ARMM by the DILG Secretary had
[21]
authorize the President, for a limited period and subject to such restrictions as it may prescribe, no basis too.
to exercise powers necessary and proper to carry out a declared national policy. Unless sooner
withdrawn by resolution of the Congress, such powers shall cease upon the next adjournment But, apart from the fact that there was no such take over to begin with, the OSG also clearly explained
thereof. the factual bases for the Presidents decision to call out the armed forces, as follows:

The Ampatuan and Mangudadatu clans are prominent families engaged in the political control of
The President did not proclaim a national emergency, only a state of emergency in the three places
mentioned. And she did not act pursuant to any law enacted by Congress that authorized her to Maguindanao. It is also a known fact that both families have an arsenal of armed followers who
hold elective positions in various parts of the ARMM and the rest of Mindanao.
exercise extraordinary powers. The calling out of the armed forces to prevent or suppress lawless
violence in such places is a power that the Constitution directly vests in the President. She did not need
a congressional authority to exercise the same.
Considering the fact that the principal victims of the brutal bloodshed are members of the to control the proliferation of loose firearms and dismantle the armed groups that continuously
Mangudadatu family and the main perpetrators of the brutal killings are members and followers threatened the peace and security in the affected places.
of the Ampatuan family, both the military and police had to prepare for and prevent reported
retaliatory actions from the Mangudadatu clan and additional offensive measures from the Notably, the present administration of President Benigno Aquino III has not withdrawn the declaration of
[24]
Ampatuan clan. a state of emergency under Proclamation 1946. It has been reported that the declaration would not
be lifted soon because there is still a need to disband private armies and confiscate loose firearms.
xxxx Apparently, the presence of troops in those places is still necessary to ease fear and tension among the
citizenry and prevent and suppress any violence that may still erupt, despite the passage of more than a
The Ampatuan forces are estimated to be approximately two thousand four hundred (2,400) year from the time of the Maguindanao massacre.
persons, equipped with about two thousand (2,000) firearms, about four hundred (400) of which
have been accounted for. x x x Since petitioners are not able to demonstrate that the proclamation of state of emergency in the subject
places and the calling out of the armed forces to prevent or suppress lawless violence there have
As for the Mangudadatus, they have an estimated one thousand eight hundred (1,800) clearly no factual bases, the Court must respect the Presidents actions.
personnel, with about two hundred (200) firearms. x x x
WHEREFORE, the petition is DISMISSED for lack of merit.
Apart from their own personal forces, both clans have Special Civilian Auxiliary Army (SCAA)
personnel who support them: about five hundred (500) for the Ampatuans and three hundred SO ORDERED.
(300) for the Mangudadatus.
ROBERTO A. ABAD
What could be worse than the armed clash of two warring clans and their armed supporters,
especially in light of intelligence reports on the potential involvement of rebel armed groups Associate Justice
(RAGs).

One RAG was reported to have planned an attack on the forces of Datu Andal Ampatuan, Sr. to
show support and sympathy for the victims. The said attack shall worsen the age-old territorial
dispute between the said RAG and the Ampatuan family.

xxxx

On the other hand, RAG faction which is based in Sultan Kudarat was reported to have received
three million pesos (P3,000,000.00) from Datu Andal Ampatuan, Sr. for the procurement of
ammunition. The said faction is a force to reckon with because the group is well capable of
launching a series of violent activities to divert the attention of the people and the authorities
away from the multiple murder case. x x x

In addition, two other factions of a RAG are likely to support the Mangudadatu family. The
Cotabato-based faction has the strength of about five hundred (500) persons and three hundred
seventy-two (372) firearms while the Sultan Kudarat-based faction has the strength of about four
hundred (400) persons and three hundred (300) firearms and was reported to be moving towards
[22]
Maguindanao to support the Mangudadatu clan in its armed fight against the Ampatuans.

In other words, the imminence of violence and anarchy at the time the President issued Proclamation
1946 was too grave to ignore and she had to act to prevent further bloodshed and hostilities in the
places mentioned. Progress reports also indicated that there was movement in these places of both
[23]
high-powered firearms and armed men sympathetic to the two clans. Thus, to pacify the peoples fears
and stabilize the situation, the President had to take preventive action. She called out the armed forces
MELO, J.:

On May 1, 2001, President Macapagal-Arroyo, faced by an "angry and violent mob armed with
explosives, firearms, bladed weapons, clubs, stones and other deadly weapons" assaulting and
attempting to break into Malacañang, issued Proclamation No. 38 declaring that there was a state of
rebellion in the National Capital Region. She likewise issued General Order No. 1 directing the Armed
Forces of the Philippines and the Philippine National Police to suppress the rebellion in the National
Capital Region. Warrantless arrests of several alleged leaders and promoters of the "rebellion" were
thereafter effected.
EN BANC
Aggrieved by the warrantless arrests, and the declaration of a "state of rebellion," which allegedly gave
G.R. No. 147780 May 10, 2001 a semblance of legality to the arrests, the following four related petitions were filed before the Court –

PANFILO LACSON, MICHAEL RAY B. AQUINO and CESAR O. MANCAO, petitioners, (1) G. R. No. 147780 for prohibition, injunction, mandamus, and habeas corpus (with an urgent
vs. application for the issuance of temporary restraining order and/or writ of preliminary injunction) filed by
SECRETARY HERNANDO PEREZ, P/DIRECTOR LEANDRO MENDOZA, and P/SR. SUPT. Panfilio M. Lacson, Michael Ray B. Aquino, and Cezar O. Mancao; (2) G. R. No. 147781
REYNALDO BERROYA, respondents. for mandamus and/or review of the factual basis for the suspension of the privilege of the writ of habeas
corpus, with prayer for the suspension of the privilege of the writ of habeas corpus, with prayer for a
---------------------------------------- temporary restraining order filed by Miriam Defensor-Santiago; (3) G. R. No. 147799 for prohibition and
injunction with prayer for a writ of preliminary injunction and/or restraining order filed by Ronaldo A.
G.R. No. 147781 May 10, 2001 Lumbao; and (4) G. R. No. 147810 for certiorari and prohibition filed by the political party Laban ng
Demokratikong Pilipino.
MIRIAM DEFENSOR-SANTIAGO, petitioner,
vs. All the foregoing petitions assail the declaration of a state of rebellion by President Gloria Macapagal-
ANGELO REYES, Secretary of National Defense, ET AL., respondents. Arroyo and the warrantless arrests allegedly effected by virtue thereof, as having no basis both in fact
and in law. Significantly, on May 6, 2001, President Macapagal-Arroyo ordered the lifting of the
---------------------------------------- declaration of a "state of rebellion" in Metro Manila. Accordingly, the instant petitions have been
rendered moot and academic. As to petitioners' claim that the proclamation of a "state of rebellion" is
G.R. No. 147799 May 10, 2001
being used by the authorities to justify warrantless arrests, the Secretary of Justice denies that it has
issued a particular order to arrest specific persons in connection with the "rebellion." He states that what
RONALDO A. LUMBAO, petitioner,
is extant are general instructions to law enforcement officers and military agencies to implement
vs.
Proclamation No. 38. Indeed, as stated in respondents' Joint Comments:
SECRETARY HERNANDO PEREZ, GENERAL DIOMEDIO VILLANUEVA, P/DIRECTOR LEANDRO
MENDOZA, and P/SR. SUPT. REYNALDO BERROYA, respondents.
[I]t is already the declared intention of the Justice Department and police authorities to obtain
regular warrants of arrests from the courts for all acts committed prior to and until May 1, 2001
----------------------------------------
which means that preliminary investigations will henceforth be conducted.
G.R. No. 147810 May 10, 2001
(Comment, G.R. No. 147780, p. 28; G.R. No. 147781, p. 18; G.R. No. 147799, p. 16; G.R. No. 147810,
THE LABAN NG DEMOKRATIKONG PILIPINO, petitioner, p. 24)
vs.
With this declaration, petitioners' apprehensions as to warrantless arrests should be laid to rest.
THE DEPARTMENT OF JUSTICE, SECRETARY HERNANDO PEREZ, THE ARMED FORCES OF
THE PHILIPPINES, GENERAL DIOMEDIO VILLANUEVA, THE PHILIPPINE NATIONAL POLICE,
In quelling or suppressing the rebellion, the authorities may only resort to warrantless arrests of persons
and DIRECTOR GENERAL LEANDRO MENDOZA, respondents.
suspected of rebellion, as provided under Section 5, Rule 113 of the Rules of Court, if the
circumstances so warrant. The warrantless arrest feared by petitioners is, thus, not based on the
RESOLUTION
declaration of a "state of rebellion."
Moreover, petitioners' contention in G. R. No. 147780 (Lacson Petition), 147781 (Defensor-Santiago arrested without a warrant. In point of fact, the authorities have categorically stated that petitioner will
Petition), and 147799 (Lumbao Petition) that they are under imminent danger of being arrested without not be arrested without a warrant.
warrant do not justify their resort to the extraordinary remedies of mandamus and prohibition, since an
individual subjected to warrantless arrest is not without adequate remedies in the ordinary course of G.R. No. 147799
law. Such an individual may ask for a preliminary investigation under Rule 112 of the Rules of Court,
where he may adduce evidence in his defense, or he may submit himself to inquest proceedings to Petitioner Lumbao, leader of the People's Movement against Poverty (PMAP), for his part, argues that
determine whether or not he should remain under custody and correspondingly be charged in court. the declaration of a "state of rebellion" is violative of the doctrine of separation of powers, being an
Further, a person subject of a warrantless arrest must be delivered to the proper judicial authorities encroachment on the domain of the judiciary which has the constitutional prerogative to "determine or
within the periods provided in Article 125 of the Revised Penal Code, otherwise the arresting officer interpret" what took place on May 1, 2001, and that the declaration of a state of rebellion cannot be an
could be held liable for delay in the delivery of detained persons. Should the detention be without legal exception to the general rule on the allocation of the governmental powers.
ground, the person arrested can charge the arresting officer with arbitrary detention. All this is without
We disagree. To be sure, Section 18, Article VII of the Constitution expressly provides that "[t]he
prejudice to his filing an action for damages against the arresting officer under Article 32 of the Civil
President shall be the Commander-in-Chief of all armed forces of the Philippines and whenever it
Code. Verily, petitioners have a surfeit of other remedies which they can avail themselves of, thereby
becomes necessary, he may call out such armed forces to prevent or suppress lawless violence,
making the prayer for prohibition and mandamus improper at this time (Section 2 and 3, Rule 65, Rules
invasion or rebellion…" Thus, we held in Integrated Bar of the Philippines v. Hon. Zamora, (G.R. No.
of Court).1âwphi1.nêt
141284, August 15, 2000):
Aside from the foregoing reasons, several considerations likewise inevitably call for the dismissal of the
x x x The factual necessity of calling out the armed forces is not easily quantifiable and cannot be
petitions at bar.
objectively established since matters considered for satisfying the same is a combination of several
G.R. No. 147780 factors which are not always accessible to the courts. Besides the absence of textual standards that the
court may use to judge necessity, information necessary to arrive at such judgment might also prove
In connection with their alleged impending warrantless arrest, petitioners Lacson, Aquino, and mancao unmanageable for the courts. Certain pertinent information might be difficult to verify, or wholly
pray that the "appropriate court before whom the informations against petitioners are filed be directed to unavailable to the courts. In many instances, the evidence upon which the President might decide that
desist from arraigning and proceeding with the trial of the case, until the instant petition is finally there is a need to call out the armed forces may be of a nature not constituting technical proof.
resolved." This relief is clearly premature considering that as of this date, no complaints or charges
have been filed against any of the petitioners for any crime. And in the event that the same are later On the other hand, the President as Commander-in-Chief has a vast intelligence network to gather
filed, this Court cannot enjoin criminal prosecution conducted in accordance with the Rules of Court, for information, some of which may be classified as highly confidential or affecting the security of the state.
by that time any arrest would have been in pursuant of a duly issued warrant. In the exercise of the power to call, on-the-spot decisions may be imperatively necessary in emergency
situations to avert great loss of human lives and mass destruction of property. x x x
As regards petitioners' prayer that the hold departure orders issued against them be declared null and
void ab initio, it is to be noted that petitioners are not directly assailing the validity of the subject hold (at pp.22-23)
departure orders in their petition. They are not even expressing intention to leave the country in the near
The Court, in a proper case, may look into the sufficiency of the factual basis of the exercise of this
future. The prayer to set aside the same must be made in proper proceedings initiated for that purpose.
power. However, this is no longer feasible at this time, Proclamation No. 38 having been lifted.
Anent petitioners' allegations ex abundante ad cautelam in support of their application for the issuance
G.R. No. 147810
of a writ of habeas corpus, it is manifest that the writ is not called for since its purpose is to relieve
petitioners from unlawful restraint (Ngaya-an v. Balweg, 200 SCRA 149 [1991]), a matter which remains Petitioner Laban ng Demokratikong Pilipino is not a real party-in-interest. The rule requires that a party
speculative up to this very day. must show a personal stake in the outcome of the case or an injury to himself that can be redressed by
a favorable decision so as to warrant an invocation of the court's jurisdiction and to justify the exercise
G.R. No. 147781
of the court's remedial powers in his behalf (KMU Labor Center v. Garcia, Jr., 239 SCRA 386 [1994]).
The petition herein is denominated by petitioner Defensor-Santiago as one for mandamus. It is basic in Here, petitioner has not demonstrated any injury to itself which would justify resort to the Court.
matters relating to petitions for mandamus that the legal right of the petitioner to the performance of a Petitioner is a juridical person not subject to arrest. Thus, it cannot claim to be threatened by a
particular act which is sought to be compelled must be clear and complete. Mandamus will not issue warrantless arrest. Nor is it alleged that its leaders, members, and supporters are being threatened with
unless the right to relief is clear at the time of the award (Palileo v. Ruiz Castro, 85 Phil. 272). Up to the warrantless arrest and detention for the crime of rebellion. Every action must be brought in the name of
present time, petitioner Defensor Santiago has not shown that she is in imminent danger of being
the party whose legal right has been invaded or infringed, or whose legal right is under imminent threat
of invasion or infringement.

At best, the instant petition may be considered as an action for declaratory relief, petitioner claiming that
its right to freedom of expression and freedom of assembly is affected by the declaration of a "state of
rebellion" and that said proclamation is invalid for being contrary to the Constitution.

However, to consider the petition as one for declaratory relief affords little comfort to petitioner, this
Court not having jurisdiction in the first instance over such a petition. Section 5[1], Article VIII of the
Constitution limits the original jurisdiction of the Court to cases affecting ambassadors, other public
ministers and consuls, and over petitions for certiorari, prohibition, mandamus, quo
warranto, and habeas corpus.

WHEREFORE, premises considered, the petitions are hereby DISMISSED. However, in G.R. No.
147780, 147781, and 147799, respondents, consistent and congruent with their undertaking earlier
adverted to, together with their agents, representatives, and all persons acting for and in their behalf,
are hereby enjoined from arresting petitioners therein without the required judicial warrant for all acts
committed in relation to or in connection with the may 1, 2001 siege of Malacañang.

SO ORDERED.
The Joint Implementing Police Visibility Patrols between the PNP NCRPO and the Philippine Marines
partnership in the conduct of visibility patrols in Metro Manila for the suppression of crime prevention
EN BANC and other serious threats to national security.

[G.R. No. 141284. August 15, 2000] 3. SITUATION:

INTEGRATED BAR OF THE PHILIPPINES, petitioner, vs. HON. RONALDO B. ZAMORA, GEN. Criminal incidents in Metro Manila have been perpetrated not only by ordinary criminals but also by
PANFILO M. LACSON, GEN. EDGAR B. AGLIPAY, and GEN. ANGELO REYES, respondents. organized syndicates whose members include active and former police/military personnel whose
training, skill, discipline and firepower prove well-above the present capability of the local police alone to
DECISION handle. The deployment of a joint PNP NCRPO-Philippine Marines in the conduct of police visibility
patrol in urban areas will reduce the incidence of crimes specially those perpetrated by active or former
KAPUNAN, J.:
police/military personnel.
At bar is a special civil action for certiorari and prohibition with prayer for issuance of a temporary
4. MISSION:
restraining order seeking to nullify on constitutional grounds the order of President Joseph Ejercito
Estrada commanding the deployment of the Philippine Marines (the Marines) to join the Philippine The PNP NCRPO will organize a provisional Task Force to conduct joint NCRPO-PM visibility patrols to
National Police (the PNP) in visibility patrols around the metropolis. keep Metro Manila streets crime-free, through a sustained street patrolling to minimize or eradicate all
forms of high-profile crimes especially those perpetrated by organized crime syndicates whose
In view of the alarming increase in violent crimes in Metro Manila, like robberies, kidnappings and
members include those that are well-trained, disciplined and well-armed active or former PNP/Military
carnappings, the President, in a verbal directive, ordered the PNP and the Marines to conduct joint
personnel.
visibility patrols for the purpose of crime prevention and suppression. The Secretary of National
Defense, the Chief of Staff of the Armed Forces of the Philippines (the AFP), the Chief of the PNP and 5. CONCEPT IN JOINT VISIBILITY PATROL OPERATIONS:
the Secretary of the Interior and Local Government were tasked to execute and implement the said
order. In compliance with the presidential mandate, the PNP Chief, through Police Chief Superintendent a. The visibility patrols shall be conducted jointly by the NCRPO [National Capital Regional Police
[1]
Edgar B. Aglipay, formulated Letter of Instruction 02/2000 (the LOI) which detailed the manner by Office] and the Philippine Marines to curb criminality in Metro Manila and to preserve the internal
[2]
which the joint visibility patrols, called Task Force Tulungan, would be conducted. Task security of the state against insurgents and other serious threat to national security, although the
Force Tulungan was placed under the leadership of the Police Chief of Metro Manila. primary responsibility over Internal Security Operations still rests upon the AFP.

Subsequently, the President confirmed his previous directive on the deployment of the Marines in a b. The principle of integration of efforts shall be applied to eradicate all forms of high-profile crimes
Memorandum, dated 24 January 2000, addressed to the Chief of Staff of the AFP and the PNP perpetrated by organized crime syndicates operating in Metro Manila. This concept requires the military
[3]
Chief. In the Memorandum, the President expressed his desire to improve the peace and order and police to work cohesively and unify efforts to ensure a focused, effective and holistic approach in
situation in Metro Manila through a more effective crime prevention program including increased police addressing crime prevention. Along this line, the role of the military and police aside from neutralizing
[4]
patrols. The President further stated that to heighten police visibility in the metropolis, augmentation crime syndicates is to bring a wholesome atmosphere wherein delivery of basic services to the people
[5]
from the AFP is necessary. Invoking his powers as Commander-in-Chief under Section 18, Article VII and development is achieved. Hand-in-hand with this joint NCRPO-Philippine Marines visibility patrols,
of the Constitution, the President directed the AFP Chief of Staff and PNP Chief to coordinate with each local Police Units are responsible for the maintenance of peace and order in their locality.
other for the proper deployment and utilization of the Marines to assist the PNP in preventing or
[6]
suppressing criminal or lawless violence. Finally, the President declared that the services of the c. To ensure the effective implementation of this project, a provisional Task Force TULUNGAN shall be
Marines in the anti-crime campaign are merely temporary in nature and for a reasonable period only, organized to provide the mechanism, structure, and procedures for the integrated planning,
[7]
until such time when the situation shall have improved. coordinating, monitoring and assessing the security situation.
[8]
The LOI explains the concept of the PNP-Philippine Marines joint visibility patrols as follows: xxx.

xxx The selected areas of deployment under the LOI are: Monumento Circle, North Edsa (SM City), Araneta
Shopping Center, Greenhills, SM Megamall, Makati Commercial Center, LRT/MRT Stations and the
2. PURPOSE: [9]
NAIA and Domestic Airport.
On 17 January 2000, the Integrated Bar of the Philippines (the IBP) filed the instant petition to annul LOI The petition has no merit.
02/2000 and to declare the deployment of the Philippine Marines, null and void and unconstitutional,
arguing that: First, petitioner failed to sufficiently show that it is in possession of the requisites of standing to raise the
issues in the petition. Second, the President did not commit grave abuse of discretion amounting to lack
I or excess of jurisdiction nor did he commit a violation of the civilian supremacy clause of the
Constitution.
THE DEPLOYMENT OF THE PHILIPPINE MARINES IN METRO MANILA IS VIOLATIVE OF THE
CONSTITUTION, IN THAT: The power of judicial review is set forth in Section 1, Article VIII of the Constitution, to wit:

A) NO EMERGENCY SITUATION OBTAINS IN METRO MANILA AS WOULD JUSTIFY, EVEN ONLY Section 1. The judicial power shall be vested in one Supreme Court and in such lower courts as may be
REMOTELY, THE DEPLOYMENT OF SOLDIERS FOR LAW ENFORCEMENT WORK; HENCE, SAID established by law.
DEPLOYMENT IS IN DEROGATION OF ARTICLE II, SECTION 3 OF THE CONSTITUTION;
Judicial power includes the duty of the courts of justice to settle actual controversies involving rights
B) SAID DEPLOYMENT CONSTITUTES AN INSIDIOUS INCURSION BY THE MILITARY IN A which are legally demandable and enforceable, and to determine whether or not there has been grave
CIVILIAN FUNCTION OF GOVERNMENT (LAW ENFORCEMENT) IN DEROGATION OF ARTICLE abuse of discretion amounting to lack or excess of jurisdiction on the part of any branch or
XVI, SECTION 5 (4), OF THE CONSTITUTION; instrumentality of the Government.

C) SAID DEPLOYMENT CREATES A DANGEROUS TENDENCY TO RELY ON THE MILITARY TO When questions of constitutional significance are raised, the Court can exercise its power of judicial
PERFORM THE CIVILIAN FUNCTIONS OF THE GOVERNMENT. review only if the following requisites are complied with, namely: (1) the existence of an actual and
appropriate case; (2) a personal and substantial interest of the party raising the constitutional question;
II (3) the exercise of judicial review is pleaded at the earliest opportunity; and (4) the constitutional
[12]
question is the lis mota of the case.
IN MILITARIZING LAW ENFORCEMENT IN METRO MANILA, THE ADMINISTRATION IS
UNWITTINGLY MAKING THE MILITARY MORE POWERFUL THAN WHAT IT SHOULD REALLY BE The IBP has not sufficiently complied with the requisites of standing in this case.
[10]
UNDER THE CONSTITUTION.
Legal standing or locus standi has been defined as a personal and substantial interest in the case such
Asserting itself as the official organization of Filipino lawyers tasked with the bounden duty to uphold the that the party has sustained or will sustain direct injury as a result of the governmental act that is being
rule of law and the Constitution, the IBP questions the validity of the deployment and utilization of the [13]
challenged. The term interest means a material interest, an interest in issue affected by the decree,
Marines to assist the PNP in law enforcement. [14]
as distinguished from mere interest in the question involved, or a mere incidental interest. The gist of
[11]
the question of standing is whether a party alleges such personal stake in the outcome of the
Without granting due course to the petition, the Court in a Resolution, dated 25 January 2000, controversy as to assure that concrete adverseness which sharpens the presentation of issues upon
required the Solicitor General to file his Comment on the petition. On 8 February 2000, the Solicitor which the court depends for illumination of difficult constitutional questions.
[15]

General submitted his Comment.


In the case at bar, the IBP primarily anchors its standing on its alleged responsibility to uphold the rule
The Solicitor General vigorously defends the constitutionality of the act of the President in deploying the of law and the Constitution. Apart from this declaration, however, the IBP asserts no other basis in
Marines, contending, among others, that petitioner has no legal standing; that the question of support of its locus standi. The mere invocation by the IBP of its duty to preserve the rule of law and
deployment of the Marines is not proper for judicial scrutiny since the same involves a political nothing more, while undoubtedly true, is not sufficient to clothe it with standing in this case. This is too
question; that the organization and conduct of police visibility patrols, which feature the team-up of one general an interest which is shared by other groups and the whole citizenry. Based on the standards
police officer and one Philippine Marine soldier, does not violate the civilian supremacy clause in the above-stated, the IBP has failed to present a specific and substantial interest in the resolution of the
Constitution. case. Its fundamental purpose which, under Section 2, Rule 139-A of the Rules of Court, is to elevate
the standards of the law profession and to improve the administration of justice is alien to, and cannot
The issues raised in the present petition are: (1) Whether or not petitioner has legal standing;
be affected by the deployment of the Marines. It should also be noted that the interest of the National
(2) Whether or not the Presidents factual determination of the necessity of calling the armed forces is
President of the IBP who signed the petition, is his alone, absent a formal board resolution authorizing
subject to judicial review; and, (3) Whether or not the calling of the armed forces to assist the PNP in
him to file the present action. To be sure, members of the BAR, those in the judiciary included, have
joint visibility patrols violates the constitutional provisions on civilian supremacy over the military and the
varying opinions on the issue. Moreover, the IBP, assuming that it has duly authorized the National
civilian character of the PNP.
President to file the petition, has not shown any specific injury which it has suffered or may suffer by
virtue of the questioned governmental act. Indeed, none of its members, whom the IBP purportedly confines of the issues raised. Thus, while the parties are in agreement that the power exercised by the
represents, has sustained any form of injury as a result of the operation of the joint visibility President is the power to call out the armed forces, the Court is of the view that the power involved may
[20]
patrols. Neither is it alleged that any of its members has been arrested or that their civil liberties have be no more than the maintenance of peace and order and promotion of the general welfare. For one,
been violated by the deployment of the Marines. What the IBP projects as injurious is the supposed the realities on the ground do not show that there exist a state of warfare, widespread civil unrest or
militarization of law enforcement which might threaten Philippine democratic institutions and may cause anarchy. Secondly, the full brunt of the military is not brought upon the citizenry, a point discussed in the
more harm than good in the long run. Not only is the presumed injury not personal in character, it is latter part of this decision. In the words of the late Justice Irene Cortes in Marcos v. Manglapus:
likewise too vague, highly speculative and uncertain to satisfy the requirement of standing. Since
petitioner has not successfully established a direct and personal injury as a consequence of the More particularly, this case calls for the exercise of the Presidents powers as protector of the
questioned act, it does not possess the personality to assail the validity of the deployment of the peace. [Rossiter, The American Presidency]. The power of the President to keep the peace is not
Marines. This Court, however, does not categorically rule that the IBP has absolutely no standing to limited merely to exercising the commander-in-chief powers in times of emergency or to leading the
raise constitutional issues now or in the future. The IBP must, by way of allegations and proof, satisfy State against external and internal threats to its existence. The President is not only clothed with
this Court that it has sufficient stake to obtain judicial resolution of the controversy. extraordinary powers in times of emergency, but is also tasked with attending to the day-to-day
problems of maintaining peace and order and ensuring domestic tranquility in times when no foreign foe
Having stated the foregoing, it must be emphasized that this Court has the discretion to take appears on the horizon. Wide discretion, within the bounds of law, in fulfilling presidential duties in times
cognizance of a suit which does not satisfy the requirement of legal standing when paramount interest of peace is not in any way diminished by the relative want of an emergency specified in the
[16]
is involved. In not a few cases, the Court has adopted a liberal attitude on the locus standi of a commander-in-chief provision. For in making the President commander-in-chief the enumeration of
petitioner where the petitioner is able to craft an issue of transcendental significance to the powers that follow cannot be said to exclude the Presidents exercising as Commander-in-Chief powers
[17]
people. Thus, when the issues raised are of paramount importance to the public, the Court may short of the calling of the armed forces, or suspending the privilege of the writ of habeas corpus or
[18]
brush aside technicalities of procedure. In this case, a reading of the petition shows that the IBP has declaring martial law, in order to keep the peace, and maintain public order and security.
advanced constitutional issues which deserve the attention of this Court in view of their seriousness,
[21]
novelty and weight as precedents. Moreover, because peace and order are under constant threat and xxx
lawless violence occurs in increasing tempo, undoubtedly aggravated by the Mindanao insurgency
problem, the legal controversy raised in the petition almost certainly will not go away. It will stare us in Nonetheless, even if it is conceded that the power involved is the Presidents power to call out the
the face again. It, therefore, behooves the Court to relax the rules on standing and to resolve the issue armed forces to prevent or suppress lawless violence, invasion or rebellion, the resolution of the
now, rather than later. controversy will reach a similar result.

The President did not commit grave abuse of discretion in calling out the Marines. We now address the Solicitor Generals argument that the issue involved is not susceptible to review by
the judiciary because it involves a political question, and thus, not justiciable.
In the case at bar, the bone of contention concerns the factual determination of the President of the
necessity of calling the armed forces, particularly the Marines, to aid the PNP in visibility patrols. In this As a general proposition, a controversy is justiciable if it refers to a matter which is appropriate for court
[22]
regard, the IBP admits that the deployment of the military personnel falls under the Commander-in- review. It pertains to issues which are inherently susceptible of being decided on grounds recognized
Chief powers of the President as stated in Section 18, Article VII of the Constitution, specifically, the by law. Nevertheless, the Court does not automatically assume jurisdiction over actual constitutional
power to call out the armed forces to prevent or suppress lawless violence, invasion or rebellion. What cases brought before it even in instances that are ripe for resolution. One class of cases wherein the
the IBP questions, however, is the basis for the calling of the Marines under the aforestated provision. Court hesitates to rule on are political questions. The reason is that political questions are concerned
According to the IBP, no emergency exists that would justify the need for the calling of the military to with issues dependent upon the wisdom, not the legality, of a particular act or measure being
assist the police force. It contends that no lawless violence, invasion or rebellion exist to warrant the assailed. Moreover, the political question being a function of the separation of powers, the courts will
calling of the Marines. Thus, the IBP prays that this Court review the sufficiency of the factual basis for not normally interfere with the workings of another co-equal branch unless the case shows a clear need
said troop [Marine] deployment.
[19] for the courts to step in to uphold the law and the Constitution.
[23]
The Solicitor General, on the other hand, contends that the issue pertaining to the necessity of calling As Taada v. Cuenco puts it, political questions refer to those questions which, under the Constitution,
the armed forces is not proper for judicial scrutiny since it involves a political question and the resolution are to be decided by the people in their sovereign capacity, or in regard to which full discretionary
of factual issues which are beyond the review powers of this Court. authority has been delegated to the legislative or executive branch of government. Thus, if an issue is
clearly identified by the text of the Constitution as matters for discretionary action by a particular branch
As framed by the parties, the underlying issues are the scope of presidential powers and limits, and the of government or to the people themselves then it is held to be a political question. In the classic
[24]
extent of judicial review. But, while this Court gives considerable weight to the parties formulation of the formulation of Justice Brennan in Baker v. Carr, [p]rominent on the surface of any case held to
issues, the resolution of the controversy may warrant a creative approach that goes beyond the narrow involve a political question is found a textually demonstrable constitutional commitment of the issue to a
coordinate political department; or a lack of judicially discoverable and manageable standards for unconstitutional, only where such grave abuse of discretion is clearly shown shall the Court interfere
resolving it; or the impossibility of deciding without an initial policy determination of a kind clearly for with the Presidents judgment. To doubt is to sustain.
nonjudicial discretion; or the impossibility of a courts undertaking independent resolution without
expressing lack of the respect due coordinate branches of government; or an unusual need for There is a clear textual commitment under the Constitution to bestow on the President full discretionary
unquestioning adherence to a political decision already made; or the potentiality of embarassment from power to call out the armed forces and to determine the necessity for the exercise of such
multifarious pronouncements by various departments on the one question. power. Section 18, Article VII of the Constitution, which embodies the powers of the President as
Commander-in-Chief, provides in part:
The 1987 Constitution expands the concept of judicial review by providing that (T)he Judicial power
shall be vested in one Supreme Court and in such lower courts as may be established by law.Judicial The President shall be the Commander-in-Chief of all armed forces of the Philippines and whenever it
power includes the duty of the courts of justice to settle actual controversies involving rights which are becomes necessary, he may call out such armed forces to prevent or suppress lawless violence,
legally demandable and enforceable, and to determine whether or not there has been a grave abuse of invasion or rebellion. In case of invasion or rebellion, when the public safety requires it, he may, for a
discretion amounting to lack or excess of jurisdiction on the part of any branch or instrumentality of the period not exceeding sixty days, suspend the privilege of the writ of habeas corpus, or place the
[25] Philippines or any part thereof under martial law.
Government. Under this definition, the Court cannot agree with the Solicitor General that the issue
involved is a political question beyond the jurisdiction of this Court to review. When the grant of power is
qualified, conditional or subject to limitations, the issue of whether the prescribed qualifications or xxx
conditions have been met or the limitations respected, is justiciable - the problem being one of legality
[26] The full discretionary power of the President to determine the factual basis for the exercise of the calling
or validity, not its wisdom. Moreover, the jurisdiction to delimit constitutional boundaries has been
[27] out power is also implied and further reinforced in the rest of Section 18, Article VII which reads, thus:
given to this Court. When political questions are involved, the Constitution limits the determination as
to whether or not there has been a grave abuse of discretion amounting to lack or excess of jurisdiction xxx
[28]
on the part of the official whose action is being questioned.
Within forty-eight hours from the proclamation of martial law or the suspension of the privilege of the writ
By grave abuse of discretion is meant simply capricious or whimsical exercise of judgment that is of habeas corpus, the President shall submit a report in person or in writing to the Congress.The
patent and gross as to amount to an evasion of positive duty or a virtual refusal to perform a duty Congress, voting jointly, by a vote of at least a majority of all its Members in regular or special session,
enjoined by law, or to act at all in contemplation of law, as where the power is exercised in an arbitrary may revoke such proclamation or suspension, which revocation shall not be set aside by the
[29]
and despotic manner by reason of passion or hostility. Under this definition, a court is without power President. Upon the initiative of the President, the Congress may, in the same manner, extend such
to directly decide matters over which full discretionary authority has been delegated. But while this proclamation or suspension for a period to be determined by the Congress, if the invasion or rebellion
Court has no power to substitute its judgment for that of Congress or of the President, it may look into shall persist and public safety requires it.
[30]
the question of whether such exercise has been made in grave abuse of discretion. A showing that
plenary power is granted either department of government, may not be an obstacle to judicial inquiry, The Congress, if not in session, shall within twenty-four hours following such proclamation or
[31]
for the improvident exercise or abuse thereof may give rise to justiciable controversy. suspension, convene in accordance with its rules without need of a call.

When the President calls the armed forces to prevent or suppress lawless violence, invasion or The Supreme Court may review, in an appropriate proceeding filed by any citizen, the sufficiency of the
rebellion, he necessarily exercises a discretionary power solely vested in his wisdom. This is clear from factual basis of the proclamation of martial law or the suspension of the privilege of the writ or the
the intent of the framers and from the text of the Constitution itself. The Court, thus, cannot be called extension thereof, and must promulgate its decision thereon within thirty days from its filing.
upon to overrule the Presidents wisdom or substitute its own. However, this does not prevent an
examination of whether such power was exercised within permissible constitutional limits or whether it A state of martial law does not suspend the operation of the Constitution, nor supplant the functioning of
was exercised in a manner constituting grave abuse of discretion. In view of the constitutional intent to the civil courts or legislative assemblies, nor authorize the conferment of jurisdiction on military courts
give the President full discretionary power to determine the necessity of calling out the armed forces, it and agencies over civilians where civil courts are able to function, nor automatically suspend the
is incumbent upon the petitioner to show that the Presidents decision is totally bereft of factual privilege of the writ.
basis. The present petition fails to discharge such heavy burden as there is no evidence to support the
assertion that there exist no justification for calling out the armed forces. There is, likewise, no evidence The suspension of the privilege of the writ shall apply only to persons judicially charged for rebellion or
to support the proposition that grave abuse was committed because the power to call was exercised in offenses inherent in or directly connected with invasion.
such a manner as to violate the constitutional provision on civilian supremacy over the military. In the
[32]
performance of this Courts duty of purposeful hesitation before declaring an act of another branch as During the suspension of the privilege of the writ, any person thus arrested or detained shall be
judicially charged within three days, otherwise he shall be released.
Under the foregoing provisions, Congress may revoke such proclamation or suspension and the Court Moreover, under Section 18, Article VII of the Constitution, in the exercise of the power to suspend the
may review the sufficiency of the factual basis thereof. However, there is no such equivalent provision privilege of the writ of habeas corpus or to impose martial law, two conditions must concur: (1) there
dealing with the revocation or review of the Presidents action to call out the armed forces. The must be an actual invasion or rebellion and, (2) public safety must require it. These conditions are not
distinction places the calling out power in a different category from the power to declare martial law and required in the case of the power to call out the armed forces. The only criterion is that whenever it
the power to suspend the privilege of the writ of habeas corpus, otherwise, the framers of the becomes necessary, the President may call the armed forces to prevent or suppress lawless violence,
Constitution would have simply lumped together the three powers and provided for their revocation and invasion or rebellion." The implication is that the President is given full discretion and wide latitude in the
review without any qualification. Expressio unius est exclusio alterius. Where the terms are expressly exercise of the power to call as compared to the two other powers.
limited to certain matters, it may not, by interpretation or construction, be extended to other
[33] If the petitioner fails, by way of proof, to support the assertion that the President acted without factual
matters. That the intent of the Constitution is exactly what its letter says, i.e., that the power to call is
fully discretionary to the President, is extant in the deliberation of the Constitutional Commission, to wit: basis, then this Court cannot undertake an independent investigation beyond the pleadings.The factual
necessity of calling out the armed forces is not easily quantifiable and cannot be objectively established
FR. BERNAS. It will not make any difference. I may add that there is a graduated power of the since matters considered for satisfying the same is a combination of several factors which are not
President as Commander-in-Chief. First, he can call out such Armed Forces as may be necessary to always accessible to the courts. Besides the absence of textual standards that the court may use to
suppress lawless violence; then he can suspend the privilege of the writ of habeas corpus, then he can judge necessity, information necessary to arrive at such judgment might also prove unmanageable for
impose martial law. This is a graduated sequence. the courts. Certain pertinent information might be difficult to verify, or wholly unavailable to the courts. In
many instances, the evidence upon which the President might decide that there is a need to call out the
When he judges that it is necessary to impose martial law or suspend the privilege of the writ of habeas armed forces may be of a nature not constituting technical proof.
corpus, his judgment is subject to review. We are making it subject to review by the Supreme Court and
subject to concurrence by the National Assembly. But when he exercises this lesser power of calling on On the other hand, the President as Commander-in-Chief has a vast intelligence network to gather
the Armed Forces, when he says it is necessary, it is my opinion that his judgment cannot be reviewed information, some of which may be classified as highly confidential or affecting the security of the
by anybody. state. In the exercise of the power to call, on-the-spot decisions may be imperatively necessary in
emergency situations to avert great loss of human lives and mass destruction of property.Indeed, the
xxx decision to call out the military to prevent or suppress lawless violence must be done swiftly and
decisively if it were to have any effect at all. Such a scenario is not farfetched when we consider the
FR. BERNAS. Let me just add that when we only have imminent danger, the matter can be handled by present situation in Mindanao, where the insurgency problem could spill over the other parts of the
the first sentence: The President may call out such armed forces to prevent or suppress lawless country. The determination of the necessity for the calling out power if subjected to unfettered judicial
violence, invasion or rebellion. So we feel that that is sufficient for handling imminent danger. scrutiny could be a veritable prescription for disaster, as such power may be unduly straitjacketed by an
injunction or a temporary restraining order every time it is exercised.
MR. DE LOS REYES. So actually, if a President feels that there is imminent danger, the matter can be
handled by the First Sentence: The President....may call out such Armed Forces to prevent or suppress Thus, it is the unclouded intent of the Constitution to vest upon the President, as Commander-in-Chief
lawless violence, invasion or rebellion. So we feel that that is sufficient for handling imminent danger, of of the Armed Forces, full discretion to call forth the military when in his judgment it is necessary to do so
invasion or rebellion, instead of imposing martial law or suspending the writ of habeas corpus, he must in order to prevent or suppress lawless violence, invasion or rebellion. Unless the petitioner can show
necessarily have to call the Armed Forces of the Philippines as their Commander-in-Chief. Is that the that the exercise of such discretion was gravely abused, the Presidents exercise of judgment deserves
idea? to be accorded respect from this Court.

MR. REGALADO. That does not require any concurrence by the legislature nor is it subject to judicial The President has already determined the necessity and factual basis for calling the armed forces. In
[34]
review. his Memorandum, he categorically asserted that, [V]iolent crimes like bank/store robberies, holdups,
[35]
kidnappings and carnappings continue to occur in Metro Manila... We do not doubt the veracity of the
The reason for the difference in the treatment of the aforementioned powers highlights the intent to
Presidents assessment of the situation, especially in the light of present developments. The Court takes
grant the President the widest leeway and broadest discretion in using the power to call out because it
judicial notice of the recent bombings perpetrated by lawless elements in the shopping malls, public
is considered as the lesser and more benign power compared to the power to suspend the privilege of
utilities, and other public places. These are among the areas of deployment described in the LOI
the writ of habeas corpus and the power to impose martial law, both of which involve the curtailment
2000. Considering all these facts, we hold that the President has sufficient factual basis to call for
and suppression of certain basic civil rights and individual freedoms, and thus necessitating safeguards
military aid in law enforcement and in the exercise of this constitutional power.
by Congress and review by this Court.
The deployment of the Marines does not violate the civilian supremacy clause nor does it
infringe the civilian character of the police force.
[45]
Prescinding from its argument that no emergency situation exists to justify the calling of the Marines, the 4. Amateur sports promotion and development;
IBP asserts that by the deployment of the Marines, the civilian task of law enforcement is militarized in
[36] [46]
violation of Section 3, Article II of the Constitution. 5. Development of the culture and the arts;
[47]
We disagree. The deployment of the Marines does not constitute a breach of the civilian supremacy 6. Conservation of natural resources;
clause. The calling of the Marines in this case constitutes permissible use of military assets for civilian [48]
law enforcement. The participation of the Marines in the conduct of joint visibility patrols is appropriately 7. Implementation of the agrarian reform program;
circumscribed. The limited participation of the Marines is evident in the provisions of the LOI itself, [49]
8. Enforcement of customs laws;
which sufficiently provides the metes and bounds of the Marines authority. It is noteworthy that the local
police forces are the ones in charge of the visibility patrols at all times, the real authority belonging to 9. Composite civilian-military law enforcement activities;
[50]

the PNP. In fact, the Metro Manila Police Chief is the overall leader of the PNP-Philippine Marines joint
[37]
visibility patrols. Under the LOI, the police forces are tasked to brief or orient the soldiers on police 10. Conduct of licensure examinations;
[51]
[38] [39]
patrol procedures. It is their responsibility to direct and manage the deployment of the Marines. It
[52]
is, likewise, their duty to provide the necessary equipment to the Marines and render logistical support 11. Conduct of nationwide tests for elementary and high school students;
[40]
to these soldiers. In view of the foregoing, it cannot be properly argued that military authority is
[53]
supreme over civilian authority. Moreover, the deployment of the Marines to assist the PNP does not 12. Anti-drug enforcement activities;
unmake the civilian character of the police force. Neither does it amount to an insidious incursion of the [54]
military in the task of law enforcement in violation of Section 5(4), Article XVI of the Constitution.
[41] 13. Sanitary inspections;
[55]
In this regard, it is not correct to say that General Angelo Reyes, Chief of Staff of the AFP, by his 14. Conduct of census work;
alleged involvement in civilian law enforcement, has been virtually appointed to a civilian post in [56]
15. Administration of the Civil Aeronautics Board;
derogation of the aforecited provision. The real authority in these operations, as stated in the LOI, is
lodged with the head of a civilian institution, the PNP, and not with the military. Such being the case, it [57]
16. Assistance in installation of weather forecasting devices;
does not matter whether the AFP Chief actually participates in the Task Force Tulungan since he does
not exercise any authority or control over the same. Since none of the Marines was incorporated or 17. Peace and order policy formulation in local government units.
[58]

enlisted as members of the PNP, there can be no appointment to civilian position to speak of. Hence,
the deployment of the Marines in the joint visibility patrols does not destroy the civilian character of the This unquestionably constitutes a gloss on executive power resulting from a systematic, unbroken,
PNP. executive practice, long pursued to the knowledge of Congress and, yet, never before
[59]
questioned. What we have here is mutual support and cooperation between the military and civilian
Considering the above circumstances, the Marines render nothing more than assistance required in authorities, not derogation of civilian supremacy.
conducting the patrols. As such, there can be no insidious incursion of the military in civilian affairs nor
can there be a violation of the civilian supremacy clause in the Constitution. In the United States, where a long tradition of suspicion and hostility towards the use of military force for
[60]
domestic purposes has persisted, and whose Constitution, unlike ours, does not expressly provide for
It is worth mentioning that military assistance to civilian authorities in various forms persists in Philippine the power to call, the use of military personnel by civilian law enforcement officers is allowed under
jurisdiction. The Philippine experience reveals that it is not averse to requesting the assistance of the circumstances similar to those surrounding the present deployment of the Philippine Marines. Under
military in the implementation and execution of certain traditionally civil functions. As correctly pointed [61]
the Posse Comitatus Act of the US, the use of the military in civilian law enforcement is generally
out by the Solicitor General, some of the multifarious activities wherein military aid has been rendered, prohibited, except in certain allowable circumstances. A provision of the Act states:
exemplifying the activities that bring both the civilian and the military together in a relationship of
cooperation, are: 1385. Use of Army and Air Force as posse comitatus
[42]
1. Elections; Whoever, except in cases and under circumstances expressly authorized by the Constitution or Act of
Congress, willfully uses any part of the Army or the Air Force as posse comitatus or otherwise to
[43]
2. Administration of the Philippine National Red Cross; execute the laws shall be fined not more than $10,000 or imprisoned not more than two years, or
[62]
[44] both.
3. Relief and rescue operations during calamities and disasters;
To determine whether there is a violation of the Posse Comitatus Act in the use of military personnel, secure in their homes and in the streets, not when the shadows of violence and anarchy constantly lurk
[63]
the US courts apply the following standards, to wit: in their midst.

Were Army or Air Force personnel used by the civilian law enforcement officers at Wounded Knee in WHEREFORE, premises considered, the petition is hereby DISMISSED.
such a manner that the military personnel subjected the citizens to the exercise of military power which
[64] SO ORDERED.
was regulatory, proscriptive, or compulsory George Washington Law Review, pp. 404-433 (1986),
which discusses the four divergent standards for assessing acceptable involvement of military
personnel in civil law enforcement. See likewise HONORED IN THE BREECH: PRESIDENTIAL
AUTHORITY TO EXECUTE THE LAWS WITH MILITARY FORCE, 83 Yale Law Journal, pp. 130-152,
1973. 64 in nature, either presently or prospectively?

xxx

When this concept is transplanted into the present legal context, we take it to mean that military
involvement, even when not expressly authorized by the Constitution or a statute, does not violate the
Posse Comitatus Act unless it actually regulates, forbids or compels some conduct on the part of those
claiming relief. A mere threat of some future injury would be insufficient. (emphasis supplied)

Even if the Court were to apply the above rigid standards to the present case to determine whether
there is permissible use of the military in civilian law enforcement, the conclusion is inevitable that no
violation of the civilian supremacy clause in the Constitution is committed. On this point, the Court
agrees with the observation of the Solicitor General:
[65]
3. The designation of tasks in Annex A does not constitute the exercise of regulatory, proscriptive, or
compulsory military power. First, the soldiers do not control or direct the operation. This is evident from
[66] [67] [68]
Nos. 6, 8(k) and 9(a) of Annex A. These soldiers, second, also have no power to prohibit or
[69]
condemn. In No. 9(d) of Annex A, all arrested persons are brought to the nearest police stations for
proper disposition. And last, these soldiers apply no coercive force. The materials or equipment issued
[70]
to them, as shown in No. 8(c) of Annex A, are all low impact and defensive in character. The
conclusion is that there being no exercise of regulatory, proscriptive or compulsory military power, the
deployment of a handful of Philippine Marines constitutes no impermissible use of military power for
[71]
civilian law enforcement.

It appears that the present petition is anchored on fear that once the armed forces are deployed, the
military will gain ascendancy, and thus place in peril our cherished liberties. Such apprehensions,
however, are unfounded. The power to call the armed forces is just that - calling out the armed
forces. Unless, petitioner IBP can show, which it has not, that in the deployment of the Marines, the
President has violated the fundamental law, exceeded his authority or jeopardized the civil liberties of
the people, this Court is not inclined to overrule the Presidents determination of the factual basis for the
calling of the Marines to prevent or suppress lawless violence.

One last point. Since the institution of the joint visibility patrol in January, 2000, not a single citizen has
complained that his political or civil rights have been violated as a result of the deployment of the
Marines. It was precisely to safeguard peace, tranquility and the civil liberties of the people that the joint
visibility patrol was conceived. Freedom and democracy will be in full bloom only when people feel
G.R. No. 171483 May 3, 2006

KILUSANG MAYO UNO, REPRESENTED BY ITS CHAIRPERSON ELMER C. LABOG AND


SECRETARY GENERAL JOEL MAGLUNSOD, NATIONAL FEDERATION OF LABOR UNIONS –
KILUSANG MAYO UNO (NAFLU-KMU), REPRESENTED BY ITS NATIONAL PRESIDENT,
JOSELITO V. USTAREZ, ANTONIO C. PASCUAL, SALVADOR T. CARRANZA, EMILIA P.
DAPULANG, MARTIN CUSTODIO, JR., AND ROQUE M. TAN, Petitioners,
G.R. No. 171396 May 3, 2006 vs.
HER EXCELLENCY, PRESIDENT GLORIA MACAPAGAL-ARROYO, THE HONORABLE
PROF. RANDOLF S. DAVID, LORENZO TAÑADA III, RONALD LLAMAS, H. HARRY L. ROQUE, EXECUTIVE SECRETARY, EDUARDO ERMITA, THE CHIEF OF STAFF, ARMED FORCES OF THE
JR., JOEL RUIZ BUTUYAN, ROGER R. RAYEL, GARY S. MALLARI, ROMEL REGALADO PHILIPPINES, GENEROSO SENGA, AND THE PNP DIRECTOR GENERAL, ARTURO
BAGARES, CHRISTOPHER F.C. BOLASTIG, Petitioners, LOMIBAO, Respondents.
vs.
GLORIA MACAPAGAL-ARROYO, AS PRESIDENT AND COMMANDER-IN-CHIEF, EXECUTIVE x-------------------------------------x
SECRETARY EDUARDO ERMITA, HON. AVELINO CRUZ II, SECRETARY OF NATIONAL
DEFENSE, GENERAL GENEROSO SENGA, CHIEF OF STAFF, ARMED FORCES OF THE G.R. No. 171400 May 3, 2006
PHILIPPINES, DIRECTOR GENERAL ARTURO LOMIBAO, CHIEF, PHILIPPINE NATIONAL
POLICE, Respondents. ALTERNATIVE LAW GROUPS, INC. (ALG), Petitioner,
vs.
x-------------------------------------x EXECUTIVE SECRETARY EDUARDO R. ERMITA, LT. GEN. GENEROSO SENGA, AND DIRECTOR
GENERAL ARTURO LOMIBAO, Respondents.
G.R. No. 171409 May 3, 2006
G.R. No. 171489 May 3, 2006
NIÑEZ CACHO-OLIVARES AND TRIBUNE PUBLISHING CO., INC., Petitioners,
vs. JOSE ANSELMO I. CADIZ, FELICIANO M. BAUTISTA, ROMULO R. RIVERA, JOSE AMOR M.
HONORABLE SECRETARY EDUARDO ERMITA AND HONORABLE DIRECTOR GENERAL AMORADO, ALICIA A. RISOS-VIDAL, FELIMON C. ABELITA III, MANUEL P. LEGASPI, J.B. JOVY
ARTURO C. LOMIBAO, Respondents. C. BERNABE, BERNARD L. DAGCUTA, ROGELIO V. GARCIA AND INTEGRATED BAR OF THE
PHILIPPINES (IBP), Petitioners,
x-------------------------------------x vs.
HON. EXECUTIVE SECRETARY EDUARDO ERMITA, GENERAL GENEROSO SENGA, IN HIS
G.R. No. 171485 May 3, 2006 CAPACITY AS AFP CHIEF OF STAFF, AND DIRECTOR GENERAL ARTURO LOMIBAO, IN HIS
CAPACITY AS PNP CHIEF, Respondents.
FRANCIS JOSEPH G. ESCUDERO, JOSEPH A. SANTIAGO, TEODORO A. CASINO, AGAPITO A.
AQUINO, MARIO J. AGUJA, SATUR C. OCAMPO, MUJIV S. HATAMAN, JUAN EDGARDO x-------------------------------------x
ANGARA, TEOFISTO DL. GUINGONA III, EMMANUEL JOSEL J. VILLANUEVA, LIZA L. MAZA,
IMEE R. MARCOS, RENATO B. MAGTUBO, JUSTIN MARC SB. CHIPECO, ROILO GOLEZ, G.R. No. 171424 May 3, 2006
DARLENE ANTONINO-CUSTODIO, LORETTA ANN P. ROSALES, JOSEL G. VIRADOR, RAFAEL
V. MARIANO, GILBERT C. REMULLA, FLORENCIO G. NOEL, ANA THERESIA HONTIVEROS- LOREN B. LEGARDA, Petitioner,
BARAQUEL, IMELDA C. NICOLAS, MARVIC M.V.F. LEONEN, NERI JAVIER COLMENARES, vs.
MOVEMENT OF CONCERNED CITIZENS FOR CIVIL LIBERTIES REPRESENTED BY AMADO GAT GLORIA MACAPAGAL-ARROYO, IN HER CAPACITY AS PRESIDENT AND COMMANDER-IN-
INCIONG, Petitioners, CHIEF; ARTURO LOMIBAO, IN HIS CAPACITY AS DIRECTOR-GENERAL OF THE PHILIPPINE
vs. NATIONAL POLICE (PNP); GENEROSO SENGA, IN HIS CAPACITY AS CHIEF OF STAFF OF THE
EDUARDO R. ERMITA, EXECUTIVE SECRETARY, AVELINO J. CRUZ, JR., SECRETARY, DND ARMED FORCES OF THE PHILIPPINES (AFP); AND EDUARDO ERMITA, IN HIS CAPACITY AS
RONALDO V. PUNO, SECRETARY, DILG, GENEROSO SENGA, AFP CHIEF OF STAFF, ARTURO EXECUTIVE SECRETARY, Respondents.
LOMIBAO, CHIEF PNP, Respondents.
DECISION
x-------------------------------------x
SANDOVAL-GUTIERREZ, J.: WHEREAS, the claims of these elements have been recklessly magnified by certain segments of
the national media;
1
All powers need some restraint; practical adjustments rather than rigid formula are necessary. Superior
strength – the use of force – cannot make wrongs into rights. In this regard, the courts should be vigilant WHEREAS, this series of actions is hurting the Philippine State – by obstructing governance
in safeguarding the constitutional rights of the citizens, specifically their liberty. including hindering the growth of the economy and sabotaging the people’s confidence in
government and their faith in the future of this country;
Chief Justice Artemio V. Panganiban’s philosophy of liberty is thus most relevant. He said: "In cases
involving liberty, the scales of justice should weigh heavily against government and in favor of WHEREAS, these actions are adversely affecting the economy;
the poor, the oppressed, the marginalized, the dispossessed and the weak." Laws and actions that
restrict fundamental rights come to the courts "with a heavy presumption against their constitutional WHEREAS, these activities give totalitarian forces of both the extreme Left and extreme Right
2 the opening to intensify their avowed aims to bring down the democratic Philippine State;
validity."

These seven (7) consolidated petitions for certiorari and prohibition allege that in issuing Presidential WHEREAS, Article 2, Section 4 of the our Constitution makes the defense and preservation of the
Proclamation No. 1017 (PP 1017) and General Order No. 5 (G.O. No. 5), President Gloria Macapagal- democratic institutions and the State the primary duty of Government;
Arroyo committed grave abuse of discretion. Petitioners contend that respondent officials of the
Government, in their professed efforts to defend and preserve democratic institutions, are actually WHEREAS, the activities above-described, their consequences, ramifications and collateral effects
trampling upon the very freedom guaranteed and protected by the Constitution. Hence, such issuances constitute a clear and present danger to the safety and the integrity of the Philippine State and of the
are void for being unconstitutional. Filipino people;

Once again, the Court is faced with an age-old but persistently modern problem. How does the On the same day, the President issued G. O. No. 5 implementing PP 1017, thus:
Constitution of a free people combine the degree of liberty, without which, law becomes tyranny, with
3 WHEREAS, over these past months, elements in the political opposition have conspired with
the degree of law, without which, liberty becomes license?
authoritarians of the extreme Left, represented by the NDF-CPP-NPA and the extreme Right,
On February 24, 2006, as the nation celebrated the 20th Anniversary of the Edsa People Power I, represented by military adventurists - the historical enemies of the democratic Philippine State – and
President Arroyo issued PP 1017 declaring a state of national emergency, thus: who are now in a tactical alliance and engaged in a concerted and systematic conspiracy, over a broad
front, to bring down the duly-constituted Government elected in May 2004;
NOW, THEREFORE, I, Gloria Macapagal-Arroyo, President of the Republic of the Philippines and
Commander-in-Chief of the Armed Forces of the Philippines, by virtue of the powers vested upon me by WHEREAS, these conspirators have repeatedly tried to bring down our republican government;
Section 18, Article 7 of the Philippine Constitution which states that: "The President. . . whenever it
WHEREAS, the claims of these elements have been recklessly magnified by certain segments of the
becomes necessary, . . . may call out (the) armed forces to prevent or suppress. . .rebellion. . .," and in
national media;
my capacity as their Commander-in-Chief, do hereby command the Armed Forces of the
Philippines, to maintain law and order throughout the Philippines, prevent or suppress all forms WHEREAS, these series of actions is hurting the Philippine State by obstructing governance, including
of lawless violence as well as any act of insurrection or rebellion and to enforce obedience to all hindering the growth of the economy and sabotaging the people’s confidence in the government and
the laws and to all decrees, orders and regulations promulgated by me personally or upon my
their faith in the future of this country;
direction; and as provided in Section 17, Article 12 of the Constitution do hereby declare a State
of National Emergency. WHEREAS, these actions are adversely affecting the economy;

She cited the following facts as bases: WHEREAS, these activities give totalitarian forces; of both the extreme Left and extreme Right the
opening to intensify their avowed aims to bring down the democratic Philippine State;
WHEREAS, over these past months, elements in the political opposition have conspired with
authoritarians of the extreme Left represented by the NDF-CPP-NPA and the extreme Right, WHEREAS, Article 2, Section 4 of our Constitution makes the defense and preservation of the
represented by military adventurists – the historical enemies of the democratic Philippine democratic institutions and the State the primary duty of Government;
State – who are now in a tactical alliance and engaged in a concerted and systematic conspiracy, over
a broad front, to bring down the duly constituted Government elected in May 2004; WHEREAS, the activities above-described, their consequences, ramifications and collateral effects
constitute a clear and present danger to the safety and the integrity of the Philippine State and of the
WHEREAS, these conspirators have repeatedly tried to bring down the President; Filipino people;
WHEREAS, Proclamation 1017 date February 24, 2006 has been issued declaring a State of National respondents’ task to state the facts behind the questioned Proclamation, however, they are presenting
Emergency; the same, narrated hereunder, for the elucidation of the issues.

NOW, THEREFORE, I GLORIA MACAPAGAL-ARROYO, by virtue of the powers vested in me under On January 17, 2006, Captain Nathaniel Rabonza and First Lieutenants Sonny Sarmiento, Lawrence
the Constitution as President of the Republic of the Philippines, and Commander-in-Chief of the San Juan and Patricio Bumidang, members of the Magdalo Group indicted in the Oakwood mutiny,
Republic of the Philippines, and pursuant to Proclamation No. 1017 dated February 24, 2006, do hereby escaped their detention cell in Fort Bonifacio, Taguig City. In a public statement, they vowed to remain
call upon the Armed Forces of the Philippines (AFP) and the Philippine National Police (PNP), to defiant and to elude arrest at all costs. They called upon the people to "show and proclaim our
prevent and suppress acts of terrorism and lawless violence in the country; displeasure at the sham regime. Let us demonstrate our disgust, not only by going to the streets in
5
protest, but also by wearing red bands on our left arms."
I hereby direct the Chief of Staff of the AFP and the Chief of the PNP, as well as the officers and men of
the AFP and PNP, to immediately carry out the necessary and appropriate actions and measures On February 17, 2006, the authorities got hold of a document entitled "Oplan Hackle I " which detailed
to suppress and prevent acts of terrorism and lawless violence. plans for bombings and attacks during the Philippine Military Academy Alumni Homecoming in Baguio
City. The plot was to assassinate selected targets including some cabinet members and President
On March 3, 2006, exactly one week after the declaration of a state of national emergency and after all 6
Arroyo herself. Upon the advice of her security, President Arroyo decided not to attend the Alumni
these petitions had been filed, the President lifted PP 1017. She issued Proclamation No. 1021 which Homecoming. The next day, at the height of the celebration, a bomb was found and detonated at the
reads: PMA parade ground.

WHEREAS, pursuant to Section 18, Article VII and Section 17, Article XII of the Constitution, On February 21, 2006, Lt. San Juan was recaptured in a communist safehouse in Batangas province.
Proclamation No. 1017 dated February 24, 2006, was issued declaring a state of national emergency; Found in his possession were two (2) flash disks containing minutes of the meetings between members
of the Magdalo Group and the National People’s Army (NPA), a tape recorder, audio cassette
WHEREAS, by virtue of General Order No.5 and No.6 dated February 24, 2006, which were issued on 7
cartridges, diskettes, and copies of subversive documents. Prior to his arrest, Lt. San Juan announced
the basis of Proclamation No. 1017, the Armed Forces of the Philippines (AFP) and the Philippine through DZRH that the "Magdalo’s D-Day would be on February 24, 2006, the 20th Anniversary of Edsa
National Police (PNP), were directed to maintain law and order throughout the Philippines, prevent and I."
suppress all form of lawless violence as well as any act of rebellion and to undertake such action as
may be necessary; On February 23, 2006, PNP Chief Arturo Lomibao intercepted information that members of the PNP-
Special Action Force were planning to defect. Thus, he immediately ordered SAF Commanding General
WHEREAS, the AFP and PNP have effectively prevented, suppressed and quelled the acts lawless Marcelino Franco, Jr. to "disavow" any defection. The latter promptly obeyed and issued a public
violence and rebellion; statement: "All SAF units are under the effective control of responsible and trustworthy officers with
proven integrity and unquestionable loyalty."
NOW, THEREFORE, I, GLORIA MACAPAGAL-ARROYO, President of the Republic of the Philippines,
by virtue of the powers vested in me by law, hereby declare that the state of national emergency has On the same day, at the house of former Congressman Peping Cojuangco, President Cory Aquino’s
ceased to exist. brother, businessmen and mid-level government officials plotted moves to bring down the Arroyo
administration. Nelly Sindayen of TIME Magazine reported that Pastor Saycon, longtime Arroyo critic,
In their presentation of the factual bases of PP 1017 and G.O. No. 5, respondents stated that the
called a U.S. government official about his group’s plans if President Arroyo is ousted. Saycon also
proximate cause behind the executive issuances was the conspiracy among some military officers,
phoned a man code-named Delta. Saycon identified him as B/Gen. Danilo Lim, Commander of the
leftist insurgents of the New People’s Army (NPA), and some members of the political opposition in a 8
4 Army’s elite Scout Ranger. Lim said "it was all systems go for the planned movement against Arroyo."
plot to unseat or assassinate President Arroyo. They considered the aim to oust or assassinate the
President and take-over the reigns of government as a clear and present danger. B/Gen. Danilo Lim and Brigade Commander Col. Ariel Querubin confided to Gen. Generoso Senga,
Chief of Staff of the Armed Forces of the Philippines (AFP), that a huge number of soldiers would join
During the oral arguments held on March 7, 2006, the Solicitor General specified the facts leading to the
the rallies to provide a critical mass and armed component to the Anti-Arroyo protests to be held on
issuance of PP 1017 and G.O. No. 5. Significantly, there was no refutation from petitioners’
February 24, 2005. According to these two (2) officers, there was no way they could possibly stop the
counsels.
soldiers because they too, were breaking the chain of command to join the forces foist to unseat the
The Solicitor General argued that the intent of the Constitution is to give full discretionary powers to President. However, Gen. Senga has remained faithful to his Commander-in-Chief and to the chain of
the President in determining the necessity of calling out the armed forces. He emphasized that none of command. He immediately took custody of B/Gen. Lim and directed Col. Querubin to return to the
the petitioners has shown that PP 1017 was without factual bases. While he explained that it is not Philippine Marines Headquarters in Fort Bonifacio.
Earlier, the CPP-NPA called for intensification of political and revolutionary work within the military and According to petitioner Kilusang Mayo Uno, the police cited PP 1017 as the ground for the dispersal of
the police establishments in order to forge alliances with its members and key officials. NPA spokesman their assemblies.
Gregorio "Ka Roger" Rosal declared: "The Communist Party and revolutionary movement and the entire
people look forward to the possibility in the coming year of accomplishing its immediate task of bringing During the dispersal of the rallyists along EDSA, police arrested (without warrant) petitioner Randolf S.
down the Arroyo regime; of rendering it to weaken and unable to rule that it will not take much longer to David, a professor at the University of the Philippines and newspaper columnist. Also arrested was his
9 companion, Ronald Llamas, president of party-list Akbayan.
end it."

On the other hand, Cesar Renerio, spokesman for the National Democratic Front (NDF) at North At around 12:20 in the early morning of February 25, 2006, operatives of the Criminal Investigation and
Central Mindanao, publicly announced: "Anti-Arroyo groups within the military and police are growing Detection Group (CIDG) of the PNP, on the basis of PP 1017 and G.O. No. 5, raided the Daily
rapidly, hastened by the economic difficulties suffered by the families of AFP officers and enlisted Tribune offices in Manila. The raiding team confiscated news stories by reporters, documents, pictures,
personnel who undertake counter-insurgency operations in the field." He claimed that with the forces of and mock-ups of the Saturday issue. Policemen from Camp Crame in Quezon City were stationed
the national democratic movement, the anti-Arroyo conservative political parties, coalitions, plus the inside the editorial and business offices of the newspaper; while policemen from the Manila Police
13
groups that have been reinforcing since June 2005, it is probable that the President’s ouster is nearing District were stationed outside the building.
its concluding stage in the first half of 2006.
A few minutes after the search and seizure at the Daily Tribune offices, the police surrounded the
Respondents further claimed that the bombing of telecommunication towers and cell sites in Bulacan premises of another pro-opposition paper, Malaya, and its sister publication, the tabloid Abante.
and Bataan was also considered as additional factual basis for the issuance of PP 1017 and G.O. No.
5. So is the raid of an army outpost in Benguet resulting in the death of three (3) soldiers. And also the The raid, according to Presidential Chief of Staff Michael Defensor, is "meant to show a ‘strong
directive of the Communist Party of the Philippines ordering its front organizations to join 5,000 Metro presence,’ to tell media outlets not to connive or do anything that would help the rebels in bringing down
Manila radicals and 25,000 more from the provinces in mass protests.
10 this government." The PNP warned that it would take over any media organization that would not
follow "standards set by the government during the state of national emergency." Director General
By midnight of February 23, 2006, the President convened her security advisers and several cabinet Lomibao stated that "if they do not follow the standards – and the standards are - if they would
members to assess the gravity of the fermenting peace and order situation. She directed both the AFP contribute to instability in the government, or if they do not subscribe to what is in General Order No. 5
and the PNP to account for all their men and ensure that the chain of command remains solid and and Proc. No. 1017 – we will recommend a ‘takeover.’" National Telecommunications’ Commissioner
undivided. To protect the young students from any possible trouble that might break loose on the Ronald Solis urged television and radio networks to "cooperate" with the government for the duration of
streets, the President suspended classes in all levels in the entire National Capital Region. the state of national emergency. He asked for "balanced reporting" from broadcasters when covering
the events surrounding the coup attempt foiled by the government. He warned that his agency will not
For their part, petitioners cited the events that followed after the issuance of PP 1017 and G.O. hesitate to recommend the closure of any broadcast outfit that violates rules set out for media coverage
14
No. 5. when the national security is threatened.

Immediately, the Office of the President announced the cancellation of all programs and activities Also, on February 25, 2006, the police arrested Congressman Crispin Beltran, representing
related to the 20th anniversary celebration of Edsa People Power I; and revoked the permits to hold the Anakpawis Party and Chairman of Kilusang Mayo Uno (KMU), while leaving his farmhouse in
rallies issued earlier by the local governments. Justice Secretary Raul Gonzales stated that political Bulacan. The police showed a warrant for his arrest dated 1985. Beltran’s lawyer explained that the
rallies, which to the President’s mind were organized for purposes of destabilization, are warrant, which stemmed from a case of inciting to rebellion filed during the Marcos regime, had long
cancelled.Presidential Chief of Staff Michael Defensor announced that "warrantless arrests and take- been quashed. Beltran, however, is not a party in any of these petitions.
11
over of facilities, including media, can already be implemented."
When members of petitioner KMU went to Camp Crame to visit Beltran, they were told they could not
Undeterred by the announcements that rallies and public assemblies would not be allowed, groups of be admitted because of PP 1017 and G.O. No. 5. Two members were arrested and detained, while the
protesters (members of Kilusang Mayo Uno [KMU] and National Federation of Labor Unions-Kilusang rest were dispersed by the police.
Mayo Uno [NAFLU-KMU]), marched from various parts of Metro Manila with the intention of converging
at the EDSA shrine. Those who were already near the EDSA site were violently dispersed by huge Bayan Muna Representative Satur Ocampo eluded arrest when the police went after him during a
clusters of anti-riot police. The well-trained policemen used truncheons, big fiber glass shields, water public forum at the Sulo Hotel in Quezon City. But his two drivers, identified as Roel and Art, were taken
cannons, and tear gas to stop and break up the marching groups, and scatter the massed participants. into custody.
The same police action was used against the protesters marching forward to Cubao, Quezon City and
to the corner of Santolan Street and EDSA. That same evening, hundreds of riot policemen broke up an Retired Major General Ramon Montaño, former head of the Philippine Constabulary, was arrested while
EDSA celebration rally held along Ayala Avenue and Paseo de Roxas Street in Makati City.
12 with his wife and golfmates at the Orchard Golf and Country Club in Dasmariñas, Cavite.
Attempts were made to arrest Anakpawis Representative Satur Ocampo, Representative Rafael emergency powers without congressional approval." In addition, petitioners asserted that PP 1017
Mariano, Bayan Muna Representative Teodoro Casiño and Gabriela Representative Liza Maza. Bayan "goes beyond the nature and function of a proclamation as defined under the Revised Administrative
Muna Representative Josel Virador was arrested at the PAL Ticket Office in Davao City. Later, he was Code."
turned over to the custody of the House of Representatives where the "Batasan 5" decided to stay
indefinitely. And lastly, in G.R. No. 171424,petitionerLoren B. Legarda maintained that PP 1017 and G.O. No. 5 are
"unconstitutional for being violative of the freedom of expression, including its cognate rights such as
Let it be stressed at this point that the alleged violations of the rights of Representatives Beltran, Satur freedom of the press and the right to access to information on matters of public concern, all guaranteed
Ocampo, et al., are not being raised in these petitions. under Article III, Section 4 of the 1987 Constitution." In this regard, she stated that these issuances
prevented her from fully prosecuting her election protest pending before the Presidential Electoral
On March 3, 2006, President Arroyo issued PP 1021 declaring that the state of national emergency has Tribunal.
ceased to exist.
In respondents’ Consolidated Comment, the Solicitor General countered that: first, the petitions should
In the interim, these seven (7) petitions challenging the constitutionality of PP 1017 and G.O. No. 5 be dismissed for being moot; second,petitioners in G.R. Nos. 171400 (ALGI), 171424 (Legarda),
were filed with this Court against the above-named respondents. Three (3) of these petitions impleaded 171483 (KMU et al.), 171485 (Escudero et al.) and 171489 (Cadiz et al.) have no legal standing; third, it
President Arroyo as respondent. is not necessary for petitioners to implead President Arroyo as respondent; fourth, PP 1017 has
constitutional and legal basis; and fifth, PP 1017 does not violate the people’s right to free expression
In G.R. No. 171396, petitioners Randolf S. David, et al. assailed PP 1017 on the grounds that (1) it and redress of grievances.
encroaches on the emergency powers of Congress; (2) itis a subterfuge to avoid the constitutional
requirements for the imposition of martial law; and (3) it violates the constitutional guarantees of On March 7, 2006, the Court conducted oral arguments and heard the parties on the above interlocking
freedom of the press, of speech and of assembly. issues which may be summarized as follows:

In G.R. No. 171409, petitioners Ninez Cacho-Olivares and Tribune Publishing Co., Inc. challenged the A. PROCEDURAL:
CIDG’s act of raiding the Daily Tribune offices as a clear case of "censorship" or "prior restraint." They
also claimed that the term "emergency" refers only to tsunami, typhoon, hurricane and similar 1) Whether the issuance of PP 1021 renders the petitions moot and academic.
occurrences, hence, there is "absolutely no emergency" that warrants the issuance of PP 1017.
2) Whether petitioners in 171485 (Escudero et al.), G.R. Nos. 171400 (ALGI), 171483 (KMU et
In G.R. No. 171485, petitioners herein are Representative Francis Joseph G. Escudero, and twenty one al.), 171489(Cadiz et al.), and 171424 (Legarda) have legal standing.
(21) other members of the House of Representatives, including Representatives Satur Ocampo, Rafael
Mariano, Teodoro Casiño, Liza Maza, and Josel Virador. They asserted that PP 1017 and G.O. No. 5 B. SUBSTANTIVE:
constitute "usurpation of legislative powers"; "violation of freedom of expression" and "a declaration of
martial law." They alleged that President Arroyo "gravely abused her discretion in calling out the armed 1) Whetherthe Supreme Court can review the factual bases of PP 1017.
forces without clear and verifiable factual basis of the possibility of lawless violence and a showing that
2) Whether PP 1017 and G.O. No. 5 are unconstitutional.
there is necessity to do so."
a. Facial Challenge
In G.R. No. 171483,petitioners KMU, NAFLU-KMU, and their members averred that PP 1017 and G.O.
No. 5 are unconstitutional because (1) they arrogate unto President Arroyo the power to enact laws and b. Constitutional Basis
decrees; (2) their issuance was without factual basis; and (3) they violate freedom of expression and the
right of the people to peaceably assemble to redress their grievances. c. As Applied Challenge

In G.R. No. 171400, petitioner Alternative Law Groups, Inc. (ALGI) alleged that PP 1017 and G.O. No. 5 A. PROCEDURAL
15 16 17 18
are unconstitutional because they violate (a) Section 4 of Article II, (b) Sections 1, 2, and 4 of
19 20
Article III, (c)Section 23 of Article VI, and (d) Section 17 of Article XII of the Constitution. First, we must resolve the procedural roadblocks.

In G.R. No. 171489, petitioners Jose Anselmo I. Cadiz et al., alleged that PP 1017 is an "arbitrary and I- Moot and Academic Principle
unlawful exercise by the President of her Martial Law powers." And assuming that PP 1017 is not really
a declaration of Martial Law, petitioners argued that "it amounts to an exercise by the President of One of the greatest contributions of the American system to this country is the concept of judicial review
21
enunciated in Marbury v. Madison. This concept rests on the extraordinary simple foundation --
The Constitution is the supreme law. It was ordained by the people, the ultimate source of all political Court has the duty to formulate guiding and controlling constitutional precepts, doctrines or rules. It has
authority. It confers limited powers on the national government. x x x If the government consciously the symbolic function of educating the bench and the bar, and in the present petitions, the military and
35
or unconsciously oversteps these limitations there must be some authority competent to hold it the police, on the extent of the protection given by constitutional guarantees. And lastly, respondents’
in control, to thwart its unconstitutional attempt, and thus to vindicate and preserve inviolate the contested actions are capable of repetition. Certainly, the petitions are subject to judicial review.
will of the people as expressed in the Constitution. This power the courts exercise. This is the
22 In their attempt to prove the alleged mootness of this case, respondents cited Chief Justice Artemio V.
beginning and the end of the theory of judicial review.
36
Panganiban’s Separate Opinion in Sanlakas v. Executive Secretary. However, they failed to take into
23
But the power of judicial review does not repose upon the courts a "self-starting capacity." Courts may account the Chief Justice’s very statement that an otherwise "moot" case may still be decided "provided
exercise such power only when the following requisites are present: first, there must be an actual case the party raising it in a proper case has been and/or continues to be prejudiced or damaged as a direct
or controversy; second, petitioners have to raise a question of constitutionality; third, the constitutional result of its issuance." The present case falls right within this exception to the mootness rule pointed out
question must be raised at the earliest opportunity; and fourth, the decision of the constitutional by the Chief Justice.
24
question must be necessary to the determination of the case itself.
II- Legal Standing
Respondents maintain that the first and second requisites are absent, hence, we shall limit our
discussion thereon. In view of the number of petitioners suing in various personalities, the Court deems it imperative to have
a more than passing discussion on legal standing or locus standi.
An actual case or controversy involves a conflict of legal right, an opposite legal claims susceptible of
37
judicial resolution. It is "definite and concrete, touching the legal relations of parties having adverse legal Locus standi is defined as "a right of appearance in a court of justice on a given question." In private
25 suits, standing is governed by the "real-parties-in interest" rule as contained in Section 2, Rule 3 of the
interest;" a real and substantial controversy admitting of specific relief. The Solicitor General refutes
the existence of such actual case or controversy, contending that the present petitions were rendered 1997 Rules of Civil Procedure, as amended. It provides that "every action must be prosecuted or
"moot and academic" by President Arroyo’s issuance of PP 1021. defended in the name of the real party in interest." Accordingly, the "real-party-in interest" is "the
party who stands to be benefited or injured by the judgment in the suit or the party entitled to
38
Such contention lacks merit. the avails of the suit." Succinctly put, the plaintiff’s standing is based on his own right to the relief
sought.
A moot and academic case is one that ceases to present a justiciable controversy by virtue of
26 27
supervening events, so that a declaration thereon would be of no practical use or value. Generally, The difficulty of determining locus standi arises in public suits. Here, the plaintiff who asserts a "public
28 29
courts decline jurisdiction over such case or dismiss it on ground of mootness. right" in assailing an allegedly illegal official action, does so as a representative of the general public. He
may be a person who is affected no differently from any other person. He could be suing as a
The Court holds that President Arroyo’s issuance of PP 1021 did not render the present petitions moot "stranger," or in the category of a "citizen," or ‘taxpayer." In either case, he has to adequately show that
and academic. During the eight (8) days that PP 1017 was operative, the police officers, according to he is entitled to seek judicial protection. In other words, he has to make out a sufficient interest in the
petitioners, committed illegal acts in implementing it. Are PP 1017 and G.O. No. 5 constitutional or vindication of the public order and the securing of relief as a "citizen" or "taxpayer.
valid? Do they justify these alleged illegal acts? These are the vital issues that must be resolved in
the present petitions. It must be stressed that "an unconstitutional act is not a law, it confers no Case law in most jurisdictions now allows both "citizen" and "taxpayer" standing in public actions. The
30 39
rights, it imposes no duties, it affords no protection; it is in legal contemplation, inoperative." distinction was first laid down in Beauchamp v. Silk, where it was held that the plaintiff in a taxpayer’s
suit is in a different category from the plaintiff in a citizen’s suit. In the former, the plaintiff is affected
The "moot and academic" principle is not a magical formula that can automatically dissuade the courts by the expenditure of public funds, while in the latter, he is but the mere instrument of the public
in resolving a case. Courts will decide cases, otherwise moot and academic, if: first, there is a grave 40
concern. As held by the New York Supreme Court in People ex rel Case v. Collins: "In matter of
31
violation of the Constitution; second, the exceptional character of the situation and the paramount mere public right, however…the people are the real parties…It is at least the right, if not the
32
public interest is involved; third, when constitutional issue raised requires formulation of controlling duty, of every citizen to interfere and see that a public offence be properly pursued and
33
principles to guide the bench, the bar, and the public; and fourth, the case is capable of repetition yet punished, and that a public grievance be remedied." With respect to taxpayer’s suits, Terr v.
34 41
evading review. Jordan held that "the right of a citizen and a taxpayer to maintain an action in courts to restrain
the unlawful use of public funds to his injury cannot be denied."
All the foregoing exceptions are present here and justify this Court’s assumption of jurisdiction over the
instant petitions. Petitioners alleged that the issuance of PP 1017 and G.O. No. 5 violates the However, to prevent just about any person from seeking judicial interference in any official policy or act
Constitution. There is no question that the issues being raised affect the public’s interest, involving as with which he disagreed with, and thus hinders the activities of governmental agencies engaged in
they do the people’s basic rights to freedom of expression, of assembly and of the press. Moreover, the public service, the United State Supreme Court laid down the more stringent "direct injury" test in Ex
42 43
Parte Levitt, later reaffirmed in Tileston v. Ullman. The same Court ruled that for a private individual (2) for taxpayers, there must be a claim of illegal disbursement of public funds or that the tax measure
to invoke the judicial power to determine the validity of an executive or legislative action, he must show is unconstitutional;
that he has sustained a direct injury as a result of that action, and it is not sufficient that he has
a general interest common to all members of the public. (3) for voters, there must be a showing of obvious interest in the validity of the election law in question;

44 (4) for concerned citizens, there must be a showing that the issues raised are of transcendental
This Court adopted the "direct injury" test in our jurisdiction. In People v. Vera, it held that the
person who impugns the validity of a statute must have "a personal and substantial interest in the importance which must be settled early; and
case such that he has sustained, or will sustain direct injury as a result." The Vera doctrine was
45
upheld in a litany of cases, such as, Custodio v. President of the Senate, Manila Race Horse Trainers’ (5) for legislators, there must be a claim that the official action complained of infringes upon their
46 47
Association v. De la Fuente, Pascual v. Secretary of Public Works and Anti-Chinese League of the prerogatives as legislators.
48
Philippines v. Felix.
Significantly, recent decisions show a certain toughening in the Court’s attitude toward legal standing.
However, being a mere procedural technicality, the requirement of locus standi may be waived by the 56
In Kilosbayan, Inc. v. Morato, the Court ruled that the status of Kilosbayan as a people’s organization
Court in the exercise of its discretion. This was done in the 1949 Emergency Powers Cases, Araneta
49 does not give it the requisite personality to question the validity of the on-line lottery contract, more so
v. Dinglasan, where the "transcendental importance" of the cases prompted the Court to act
50 where it does not raise any issue of constitutionality. Moreover, it cannot sue as a taxpayer absent any
liberally. Such liberality was neither a rarity nor accidental. In Aquino v. Comelec, this Court resolved
allegation that public funds are being misused. Nor can it sue as a concerned citizen as it does not
to pass upon the issues raised due to the "far-reaching implications" of the petition notwithstanding its
allege any specific injury it has suffered.
categorical statement that petitioner therein had no personality to file the suit. Indeed, there is a chain of
cases where this liberal policy has been observed, allowing ordinary citizens, members of Congress, 57
In Telecommunications and Broadcast Attorneys of the Philippines, Inc. v. Comelec, the Court
and civic organizations to prosecute actions involving the constitutionality or validity of laws, regulations reiterated the "direct injury" test with respect to concerned citizens’ cases involving constitutional issues.
51
and rulings. It held that "there must be a showing that the citizen personally suffered some actual or threatened
injury arising from the alleged illegal official act."
Thus, the Court has adopted a rule that even where the petitioners have failed to show direct injury,
they have been allowed to sue under the principle of "transcendental importance." Pertinent are the 58
In Lacson v. Perez, the Court ruled that one of the petitioners, Laban ng Demokratikong
following cases: Pilipino (LDP), is not a real party-in-interest as it had not demonstrated any injury to itself or to its
52 leaders, members or supporters.
(1) Chavez v. Public Estates Authority, where the Court ruled that the enforcement of the
constitutional right to information and the equitable diffusion of natural resources are matters of 59
In Sanlakas v. Executive Secretary, the Court ruled that only the petitioners who are members of
transcendental importance which clothe the petitioner with locus standi; Congress have standing to sue, as they claim that the President’s declaration of a state of rebellion is a
53 usurpation of the emergency powers of Congress, thus impairing their legislative powers. As to
(2) Bagong Alyansang Makabayan v. Zamora, wherein the Court held that "given the transcendental
petitioners Sanlakas, Partido Manggagawa, and Social Justice Society, the Court declared them to be
importance of the issues involved, the Court may relax the standing requirements and allow the
devoid of standing, equating them with the LDP in Lacson.
suit to prosper despite the lack of direct injury to the parties seeking judicial review" of the
Visiting Forces Agreement; Now, the application of the above principles to the present petitions.
54
(3) Lim v. Executive Secretary, while the Court noted that the petitioners may not file suit in their The locus standi of petitioners in G.R. No. 171396, particularly David and Llamas, is beyond doubt. The
capacity as taxpayers absent a showing that "Balikatan 02-01" involves the exercise of Congress’ taxing same holds true with petitioners in G.R. No. 171409, Cacho-Olivares and Tribune Publishing Co. Inc.
55
or spending powers, it reiterated its ruling in Bagong Alyansang Makabayan v. Zamora, that in cases They alleged "direct injury" resulting from "illegal arrest" and "unlawful search" committed by police
of transcendental importance, the cases must be settled promptly and definitely and standing
operatives pursuant to PP 1017. Rightly so, the Solicitor General does not question their legal standing.
requirements may be relaxed.
In G.R. No. 171485, the opposition Congressmen alleged there was usurpation of legislative powers.
By way of summary, the following rules may be culled from the cases decided by this Court. Taxpayers, They also raised the issue of whether or not the concurrence of Congress is necessary whenever the
voters, concerned citizens, and legislators may be accorded standing to sue, provided that the following alarming powers incident to Martial Law are used. Moreover, it is in the interest of justice that those
requirements are met: affected by PP 1017 can be represented by their Congressmen in bringing to the attention of the Court
the alleged violations of their basic rights.
(1) the cases involve constitutional issues;
60
In G.R. No. 171400, (ALGI), this Court applied the liberality rule in Philconsa v. Enriquez, Kapatiran distraction to enable him to fully attend to the performance of his official duties and functions. Unlike the
61
Ng Mga Naglilingkod sa Pamahalaan ng Pilipinas, Inc. v. Tan, Association of Small Landowners in the legislative and judicial branch, only one constitutes the executive branch and anything which impairs his
62
Philippines, Inc. v. Secretary of Agrarian Reform, Basco v. Philippine Amusement and Gaming usefulness in the discharge of the many great and important duties imposed upon him by the
63 64
Corporation, and Tañada v. Tuvera, that when the issue concerns a public right, it is sufficient that Constitution necessarily impairs the operation of the Government. However, this does not mean that the
the petitioner is a citizen and has an interest in the execution of the laws. President is not accountable to anyone. Like any other official, he remains accountable to the
68
people but he may be removed from office only in the mode provided by law and that is by
In G.R. No. 171483, KMU’s assertion that PP 1017 and G.O. No. 5 violated its right to peaceful impeachment.
69

assembly may be deemed sufficient to give it legal standing. Organizations may be granted standing
65
to assert the rights of their members. We take judicial notice of the announcement by the Office of B. SUBSTANTIVE
the President banning all rallies and canceling all permits for public assemblies following the issuance of
PP 1017 and G.O. No. 5. I. Review of Factual Bases

In G.R. No. 171489, petitioners, Cadiz et al., who are national officers of the Integrated Bar of the Petitioners maintain that PP 1017 has no factual basis. Hence, it was not "necessary" for President
Philippines (IBP) have no legal standing, having failed to allege any direct or potential injury which the Arroyo to issue such Proclamation.
IBP as an institution or its members may suffer as a consequence of the issuance of PP No. 1017 and
66
G.O. No. 5. In Integrated Bar of the Philippines v. Zamora, the Court held that the mere invocation by The issue of whether the Court may review the factual bases of the President’s exercise of his
the IBP of its duty to preserve the rule of law and nothing more, while undoubtedly true, is not sufficient Commander-in-Chief power has reached its distilled point - from the indulgent days of Barcelon v.
70 71 72
to clothe it with standing in this case. This is too general an interest which is shared by other groups and Baker and Montenegro v. Castaneda to the volatile era of Lansang v. Garcia, Aquino, Jr. v.
73 74
the whole citizenry. However, in view of the transcendental importance of the issue, this Court declares Enrile, and Garcia-Padilla v. Enrile. The tug-of-war always cuts across the line defining "political
that petitioner have locus standi. questions," particularly those questions "in regard to which full discretionary authority has been
75
delegated to the legislative or executive branch of the government." Barcelon and Montenegro were in
In G.R. No. 171424, Loren Legarda has no personality as a taxpayer to file the instant petition as there unison in declaring that the authority to decide whether an exigency has arisen belongs to the
are no allegations of illegal disbursement of public funds. The fact that she is a former Senator is of no President and his decision is final and conclusive on the courts. Lansang took the opposite view.
consequence. She can no longer sue as a legislator on the allegation that her prerogatives as a There, the members of the Court were unanimous in the conviction that the Court has the authority to
lawmaker have been impaired by PP 1017 and G.O. No. 5. Her claim that she is a media personality inquire into the existence of factual bases in order to determine their constitutional sufficiency. From the
will not likewise aid her because there was no showing that the enforcement of these issuances principle of separation of powers, it shifted the focus to the system of checks and balances,
prevented her from pursuing her occupation. Her submission that she has pending electoral protest "under which the President is supreme, x x x only if and when he acts within the sphere allotted
before the Presidential Electoral Tribunal is likewise of no relevance. She has not sufficiently shown that to him by the Basic Law, and the authority to determine whether or not he has so acted is vested
76
PP 1017 will affect the proceedings or result of her case. But considering once more the transcendental in the Judicial Department, which in this respect, is, in turn, constitutionally supreme." In 1973,
77
importance of the issue involved, this Court may relax the standing rules. the unanimous Court of Lansang was divided in Aquino v. Enrile. There, the Court was almost evenly
divided on the issue of whether the validity of the imposition of Martial Law is a political or justiciable
78
It must always be borne in mind that the question of locus standi is but corollary to the bigger question question. Then came Garcia-Padilla v. Enrile which greatly diluted Lansang. It declared that there is a
of proper exercise of judicial power. This is the underlying legal tenet of the "liberality doctrine" on legal need to re-examine the latter case, ratiocinating that "in times of war or national emergency, the
standing. It cannot be doubted that the validity of PP No. 1017 and G.O. No. 5 is a judicial question President must be given absolute control for the very life of the nation and the government is in
which is of paramount importance to the Filipino people. To paraphrase Justice Laurel, the whole of great peril. The President, it intoned, is answerable only to his conscience, the People, and
79
Philippine society now waits with bated breath the ruling of this Court on this very critical matter. The God."
petitions thus call for the application of the "transcendental importance" doctrine, a relaxation of the
80
standing requirements for the petitioners in the "PP 1017 cases."1avvphil.net The Integrated Bar of the Philippines v. Zamora -- a recent case most pertinent to these cases at bar -
- echoed a principle similar to Lansang. While the Court considered the President’s "calling-out" power
This Court holds that all the petitioners herein have locus standi. as a discretionary power solely vested in his wisdom, it stressed that "this does not prevent an
examination of whether such power was exercised within permissible constitutional limits or
Incidentally, it is not proper to implead President Arroyo as respondent. Settled is the doctrine that the whether it was exercised in a manner constituting grave abuse of discretion."This ruling is mainly
67
President, during his tenure of office or actual incumbency, may not be sued in any civil or criminal a result of the Court’s reliance on Section 1, Article VIII of 1987 Constitution which fortifies the authority
case, and there is no need to provide for it in the Constitution or law. It will degrade the dignity of the of the courts to determine in an appropriate action the validity of the acts of the political departments.
high office of the President, the Head of State, if he can be dragged into court litigations while serving Under the new definition of judicial power, the courts are authorized not only "to settle actual
as such. Furthermore, it is important that he be freed from any form of harassment, hindrance or controversies involving rights which are legally demandable and enforceable," but also "to determine
whether or not there has been a grave abuse of discretion amounting to lack or excess of remedy in this, as in all other cases where they have no judge on earth, but to appeal to
85
jurisdiction on the part of any branch or instrumentality of the government." The latter part of the Heaven."
authority represents a broadening of judicial power to enable the courts of justice to review what was
81 Jean-Jacques Rousseau also assumed the need for temporary suspension of democratic processes of
before a forbidden territory, to wit, the discretion of the political departments of the government. It
82 government in time of emergency. According to him:
speaks of judicial prerogative not only in terms of power but also of duty.

As to how the Court may inquire into the President’s exercise of power, Lansang adopted the test that The inflexibility of the laws, which prevents them from adopting themselves to circumstances, may, in
"judicial inquiry can go no further than to satisfy the Court not that the President’s decision is correct," certain cases, render them disastrous and make them bring about, at a time of crisis, the ruin of the
but that "the President did not act arbitrarily." Thus, the standard laid down is not correctness, but State…
83
arbitrariness. In Integrated Bar of the Philippines, this Court further ruled that "it is incumbent upon
the petitioner to show that the President’s decision is totally bereft of factual basis" and that if he It is wrong therefore to wish to make political institutions as strong as to render it impossible to suspend
fails, by way of proof, to support his assertion, then "this Court cannot undertake an independent their operation. Even Sparta allowed its law to lapse...
investigation beyond the pleadings."
If the peril is of such a kind that the paraphernalia of the laws are an obstacle to their preservation, the
Petitioners failed to show that President Arroyo’s exercise of the calling-out power, by issuing PP 1017, method is to nominate a supreme lawyer, who shall silence all the laws and suspend for a moment the
is totally bereft of factual basis. A reading of the Solicitor General’s Consolidated Comment and sovereign authority. In such a case, there is no doubt about the general will, and it clear that the
86
Memorandum shows a detailed narration of the events leading to the issuance of PP 1017, with people’s first intention is that the State shall not perish.
supporting reports forming part of the records. Mentioned are the escape of the Magdalo Group, their
Rosseau did not fear the abuse of the emergency dictatorship or "supreme magistracy" as he termed
audacious threat of the Magdalo D-Day, the defections in the military, particularly in the Philippine
it. For him, it would more likely be cheapened by "indiscreet use." He was unwilling to rely upon an
Marines, and the reproving statements from the communist leaders. There was also the Minutes of the
"appeal to heaven." Instead, he relied upon a tenure of office of prescribed duration to avoid
Intelligence Report and Security Group of the Philippine Army showing the growing alliance between 87
perpetuation of the dictatorship.
the NPA and the military. Petitioners presented nothing to refute such events. Thus, absent any
contrary allegations, the Court is convinced that the President was justified in issuing PP 1017 calling John Stuart Mill concluded his ardent defense of representative government: "I am far from
for military aid. condemning, in cases of extreme necessity, the assumption of absolute power in the form of a
88
temporary dictatorship."
Indeed, judging the seriousness of the incidents, President Arroyo was not expected to simply fold her
arms and do nothing to prevent or suppress what she believed was lawless violence, invasion or Nicollo Machiavelli’s view of emergency powers, as one element in the whole scheme of limited
rebellion. However, the exercise of such power or duty must not stifle liberty. government, furnished an ironic contrast to the Lockean theory of prerogative. He recognized and
attempted to bridge this chasm in democratic political theory, thus:
II. Constitutionality of PP 1017 and G.O. No. 5
Doctrines of Several Political Theorists
Now, in a well-ordered society, it should never be necessary to resort to extra –constitutional measures;
on the Power of the President in Times of Emergency
for although they may for a time be beneficial, yet the precedent is pernicious, for if the practice is once
established for good objects, they will in a little while be disregarded under that pretext but for evil
This case brings to fore a contentious subject -- the power of the President in times of emergency. A
purposes. Thus, no republic will ever be perfect if she has not by law provided for everything, having a
glimpse at the various political theories relating to this subject provides an adequate backdrop for our 89
remedy for every emergency and fixed rules for applying it.
ensuing discussion.
Machiavelli – in contrast to Locke, Rosseau and Mill – sought to incorporate into the constitution a
John Locke, describing the architecture of civil government, called upon the English doctrine of
regularized system of standby emergency powers to be invoked with suitable checks and controls in
prerogative to cope with the problem of emergency. In times of danger to the nation, positive law
time of national danger. He attempted forthrightly to meet the problem of combining a capacious
enacted by the legislature might be inadequate or even a fatal obstacle to the promptness of action
reserve of power and speed and vigor in its application in time of emergency, with effective
necessary to avert catastrophe. In these situations, the Crown retained a prerogative "power to act 90
constitutional restraints.
according to discretion for the public good, without the proscription of the law and sometimes
84
even against it." But Locke recognized that this moral restraint might not suffice to avoid abuse of
Contemporary political theorists, addressing themselves to the problem of response to emergency by
prerogative powers. Who shall judge the need for resorting to the prerogative and how may its 91
constitutional democracies, have employed the doctrine of constitutional dictatorship. Frederick M.
abuse be avoided? Here, Locke readily admitted defeat, suggesting that "the people have no other Watkins saw "no reason why absolutism should not be used as a means for the defense of liberal
institutions," provided it "serves to protect established institutions from the danger of permanent
injury in a period of temporary emergency and is followed by a prompt return to the previous 8) Ultimate responsibility should be maintained for every action taken under a constitutional dictatorship.
92
forms of political life." He recognized the two (2) key elements of the problem of emergency ..
governance, as well as all constitutional governance: increasing administrative powers of the
93 9) The decision to terminate a constitutional dictatorship, like the decision to institute one should never
executive, while at the same time "imposing limitation upon that power." Watkins placed his real
faith in a scheme of constitutional dictatorship. These are the conditions of success of such a be in the hands of the man or men who constitute the dictator. . .
dictatorship: "The period of dictatorship must be relatively short…Dictatorship should always be
strictly legitimate in character…Final authority to determine the need for dictatorship in any 10) No constitutional dictatorship should extend beyond the termination of the crisis for which it was
94
given case must never rest with the dictator himself…" and the objective of such an emergency instituted…
dictatorship should be "strict political conservatism."
11) …the termination of the crisis must be followed by a complete return as possible to the political and
99
95
Carl J. Friedrich cast his analysis in terms similar to those of Watkins. "It is a problem of concentrating governmental conditions existing prior to the initiation of the constitutional dictatorship…
power – in a government where power has consciously been divided – to cope with… situations of
Rossiter accorded to legislature a far greater role in the oversight exercise of emergency powers than
unprecedented magnitude and gravity. There must be a broad grant of powers, subject to equally strong
96 did Watkins. He would secure to Congress final responsibility for declaring the existence or termination
limitations as to who shall exercise such powers, when, for how long, and to what end." Friedrich, too,
of an emergency, and he places great faith in the effectiveness of congressional investigating
offered criteria for judging the adequacy of any of scheme of emergency powers, to wit: "The 100
committees.
emergency executive must be appointed by constitutional means – i.e., he must be legitimate;
he should not enjoy power to determine the existence of an emergency; emergency powers Scott and Cotter, in analyzing the above contemporary theories in light of recent experience, were one
should be exercised under a strict time limitation; and last, the objective of emergency action in saying that, "the suggestion that democracies surrender the control of government to an
97
must be the defense of the constitutional order." authoritarian ruler in time of grave danger to the nation is not based upon sound constitutional
theory." To appraise emergency power in terms of constitutional dictatorship serves merely to distort
Clinton L. Rossiter, after surveying the history of the employment of emergency powers in Great Britain,
the problem and hinder realistic analysis. It matters not whether the term "dictator" is used in its normal
France, Weimar, Germany and the United States, reverted to a description of a scheme of
98 sense (as applied to authoritarian rulers) or is employed to embrace all chief executives administering
"constitutional dictatorship" as solution to the vexing problems presented by emergency. Like Watkins
emergency powers. However used, "constitutional dictatorship" cannot be divorced from the implication
and Friedrich, he stated a priori the conditions of success of the "constitutional dictatorship," thus:
of suspension of the processes of constitutionalism. Thus, they favored instead the "concept of
1) No general regime or particular institution of constitutional dictatorship should be initiated unless it is constitutionalism" articulated by Charles H. McIlwain:
necessary or even indispensable to the preservation of the State and its constitutional order…
A concept of constitutionalism which is less misleading in the analysis of problems of emergency
2) …the decision to institute a constitutional dictatorship should never be in the hands of the man or powers, and which is consistent with the findings of this study, is that formulated by Charles H.
men who will constitute the dictator… McIlwain. While it does not by any means necessarily exclude some indeterminate limitations upon the
substantive powers of government, full emphasis is placed upon procedural limitations, and political
3) No government should initiate a constitutional dictatorship without making specific provisions for its responsibility. McIlwain clearly recognized the need to repose adequate power in government. And in
termination… discussing the meaning of constitutionalism, he insisted that the historical and proper test of
constitutionalism was the existence of adequate processes for keeping government
4) …all uses of emergency powers and all readjustments in the organization of the government should responsible. He refused to equate constitutionalism with the enfeebling of government by an
be effected in pursuit of constitutional or legal requirements… exaggerated emphasis upon separation of powers and substantive limitations on governmental power.
He found that the really effective checks on despotism have consisted not in the weakening of
5) … no dictatorial institution should be adopted, no right invaded, no regular procedure altered any government but, but rather in the limiting of it; between which there is a great and very significant
more than is absolutely necessary for the conquest of the particular crisis . . . difference. In associating constitutionalism with "limited" as distinguished from "weak"
government, McIlwain meant government limited to the orderly procedure of law as opposed to
6) The measures adopted in the prosecution of the a constitutional dictatorship should never be the processes of force. The two fundamental correlative elements of constitutionalism for which
permanent in character or effect… all lovers of liberty must yet fight are the legal limits to arbitrary power and a complete political
101
responsibility of government to the governed.
7) The dictatorship should be carried on by persons representative of every part of the citizenry
interested in the defense of the existing constitutional order. . . In the final analysis, the various approaches to emergency of the above political theorists –- from Lock’s
"theory of prerogative," to Watkins’ doctrine of "constitutional dictatorship" and, eventually, to McIlwain’s
"principle of constitutionalism" --- ultimately aim to solve one real problem in emergency governance, Thus, claims of facial overbreadth are entertained in cases involving statutes which, by their terms,
i.e., that of allotting increasing areas of discretionary power to the Chief Executive, while seek to regulate only "spoken words" and again, that "overbreadth claims, if entertained at all, have
insuring that such powers will be exercised with a sense of political responsibility and under been curtailed when invoked against ordinary criminal laws that are sought to be applied to
106
effective limitations and checks. protected conduct." Here, the incontrovertible fact remains that PP 1017 pertains to a spectrum
of conduct, not free speech, which is manifestly subject to state regulation.
Our Constitution has fairly coped with this problem. Fresh from the fetters of a repressive regime, the
1986 Constitutional Commission, in drafting the 1987 Constitution, endeavored to create a government Second, facial invalidation of laws is considered as "manifestly strong medicine," to be used
102 107
in the concept of Justice Jackson’s "balanced power structure." Executive, legislative, and judicial "sparingly and only as a last resort," and is "generally disfavored;" The reason for this is obvious.
powers are dispersed to the President, the Congress, and the Supreme Court, respectively. Each is Embedded in the traditional rules governing constitutional adjudication is the principle that a person to
supreme within its own sphere. But none has the monopoly of power in times of emergency. Each whom a law may be applied will not be heard to challenge a law on the ground that it may conceivably
108
branch is given a role to serve as limitation or check upon the other. This system does be applied unconstitutionally to others, i.e., in other situations not before the Court. A writer and
not weaken the President, it just limits his power, using the language of McIlwain. In other words, in scholar in Constitutional Law explains further:
times of emergency, our Constitution reasonably demands that we repose a certain amount of faith in
the basic integrity and wisdom of the Chief Executive but, at the same time, it obliges him to operate The most distinctive feature of the overbreadth technique is that it marks an exception to some
within carefully prescribed procedural limitations. of the usual rules of constitutional litigation. Ordinarily, a particular litigant claims that a statute
is unconstitutional as applied to him or her; if the litigant prevails, the courts carve away the
a. "Facial Challenge" unconstitutional aspects of the law by invalidating its improper applications on a case to case
basis. Moreover, challengers to a law are not permitted to raise the rights of third parties and
Petitioners contend that PP 1017 is void on its face because of its "overbreadth." They claim that its can only assert their own interests. In overbreadth analysis, those rules give way; challenges
enforcement encroached on both unprotected and protected rights under Section 4, Article III of the are permitted to raise the rights of third parties; and the court invalidates the entire statute "on its
Constitution and sent a "chilling effect" to the citizens. face," not merely "as applied for" so that the overbroad law becomes unenforceable until a properly
authorized court construes it more narrowly. The factor that motivates courts to depart from the normal
A facial review of PP 1017, using the overbreadth doctrine, is uncalled for. adjudicatory rules is the concern with the "chilling;" deterrent effect of the overbroad statute on third
parties not courageous enough to bring suit. The Court assumes that an overbroad law’s "very
First and foremost, the overbreadth doctrine is an analytical tool developed for testing "on their faces"
103 existence may cause others not before the court to refrain from constitutionally protected speech or
statutes in free speech cases, also known under the American Law as First Amendment cases.
expression." An overbreadth ruling is designed to remove that deterrent effect on the speech of those
A plain reading of PP 1017 shows that it is not primarily directed to speech or even speech-related third parties.
conduct. It is actually a call upon the AFP to prevent or suppress all forms
104 In other words, a facial challenge using the overbreadth doctrine will require the Court to examine PP
of lawless violence. In United States v. Salerno, the US Supreme Court held that "we have not
1017 and pinpoint its flaws and defects, not on the basis of its actual operation to petitioners, but on the
recognized an ‘overbreadth’ doctrine outside the limited context of the First Amendment"
assumption or prediction that its very existence may cause others not before the Court to refrain from
(freedom of speech). 109
constitutionally protected speech or expression. In Younger v. Harris, it was held that:
Moreover, the overbreadth doctrine is not intended for testing the validity of a law that "reflects
[T]he task of analyzing a proposed statute, pinpointing its deficiencies, and requiring correction of these
legitimate state interest in maintaining comprehensive control over harmful, constitutionally unprotected
deficiencies before the statute is put into effect, is rarely if ever an appropriate task for the judiciary. The
conduct." Undoubtedly, lawless violence, insurrection and rebellion are considered "harmful" and
105 combination of the relative remoteness of the controversy, the impact on the legislative process
"constitutionally unprotected conduct." In Broadrick v. Oklahoma, it was held:
of the relief sought, and above all the speculative and amorphous nature of the required line-by-
line analysis of detailed statutes,...ordinarily results in a kind of case that is wholly
It remains a ‘matter of no little difficulty’ to determine when a law may properly be held void on its face
and when ‘such summary action’ is inappropriate. But the plain import of our cases is, at the very unsatisfactory for deciding constitutional questions, whichever way they might be decided.
least, that facial overbreadth adjudication is an exception to our traditional rules of practice and
And third, a facial challenge on the ground of overbreadth is the most difficult challenge to mount
that its function, a limited one at the outset, attenuates as the otherwise unprotected behavior
successfully, since the challenger must establish that there can be no instance when the assailed
that it forbids the State to sanction moves from ‘pure speech’ toward conduct and that conduct –
law may be valid. Here, petitioners did not even attempt to show whether this situation exists.
even if expressive – falls within the scope of otherwise valid criminal laws that reflect legitimate
state interests in maintaining comprehensive controls over harmful, constitutionally Petitioners likewise seek a facial review of PP 1017 on the ground of vagueness. This, too, is
unprotected conduct. unwarranted.
Related to the "overbreadth" doctrine is the "void for vagueness doctrine" which holds that "a law is period to be determined by the Congress, if the invasion or rebellion shall persist and public safety
facially invalid if men of common intelligence must necessarily guess at its meaning and differ requires it.
110
as to its application." It is subject to the same principles governing overbreadth doctrine. For one, it
is also an analytical tool for testing "on their faces" statutes in free speech cases. And like The Congress, if not in session, shall within twenty-four hours following such proclamation or
overbreadth, it is said that a litigant may challenge a statute on its face only if it is vague in all its suspension, convene in accordance with its rules without need of a call.
possible applications. Again, petitioners did not even attempt to show that PP 1017 is vague in
all its application. They also failed to establish that men of common intelligence cannot understand the The Supreme Court may review, in an appropriate proceeding filed by any citizen, the sufficiency of the
meaning and application of PP 1017. factual bases of the proclamation of martial law or the suspension of the privilege of the writ or the
extension thereof, and must promulgate its decision thereon within thirty days from its filing.
b. Constitutional Basis of PP 1017
A state of martial law does not suspend the operation of the Constitution, nor supplant the functioning of
Now on the constitutional foundation of PP 1017. the civil courts or legislative assemblies, nor authorize the conferment of jurisdiction on military courts
and agencies over civilians where civil courts are able to function, nor automatically suspend the
The operative portion of PP 1017 may be divided into three important provisions, thus: privilege of the writ.

First provision: The suspension of the privilege of the writ shall apply only to persons judicially charged for rebellion or
offenses inherent in or directly connected with invasion.
"by virtue of the power vested upon me by Section 18, Artilce VII … do hereby command the Armed
Forces of the Philippines, to maintain law and order throughout the Philippines, prevent or suppress all During the suspension of the privilege of the writ, any person thus arrested or detained shall be
forms of lawless violence as well any act of insurrection or rebellion" judicially charged within three days, otherwise he shall be released.

Second provision: grants the President, as Commander-in-Chief, a "sequence" of graduated powers. From the most to the
least benign, these are: the calling-out power, the power to suspend the privilege of the writ of habeas
"and to enforce obedience to all the laws and to all decrees, orders and regulations promulgated by me 112
corpus, and the power to declare Martial Law. Citing Integrated Bar of the Philippines v. Zamora, the
personally or upon my direction;" Court ruled that the only criterion for the exercise of the calling-out power is that "whenever it becomes
necessary," the President may call the armed forces "to prevent or suppress lawless violence,
Third provision: invasion or rebellion." Are these conditions present in the instant cases? As stated earlier, considering
the circumstances then prevailing, President Arroyo found it necessary to issue PP 1017. Owing to her
"as provided in Section 17, Article XII of the Constitution do hereby declare a State of National
Office’s vast intelligence network, she is in the best position to determine the actual condition of the
Emergency."
country.
First Provision: Calling-out Power
Under the calling-out power, the President may summon the armed forces to aid him in
The first provision pertains to the President’s calling-out power. In Sanlakas v. Executive suppressing lawless violence, invasion and rebellion. This involves ordinary police action. But every
111
Secretary, this Court, through Mr. Justice Dante O. Tinga, held that Section 18, Article VII of the act that goes beyond the President’s calling-out power is considered illegal or ultra vires. For this
Constitution reproduced as follows: reason, a President must be careful in the exercise of his powers. He cannot invoke a greater power
when he wishes to act under a lesser power. There lies the wisdom of our Constitution, the greater the
Sec. 18. The President shall be the Commander-in-Chief of all armed forces of the Philippines power, the greater are the limitations.
and whenever it becomes necessary, he may call out such armed forces to prevent or suppress
lawless violence, invasion or rebellion. In case of invasion or rebellion, when the public safety It is pertinent to state, however, that there is a distinction between the President’s authority to declare a
requires it, he may, for a period not exceeding sixty days, suspend the privilege of the writ of habeas "state of rebellion" (in Sanlakas) and the authority to proclaim a state of national emergency. While
corpus or place the Philippines or any part thereof under martial law. Within forty-eight hours from the President Arroyo’s authority to declare a "state of rebellion" emanates from her powers as Chief
proclamation of martial law or the suspension of the privilege of the writ of habeas corpus, the President Executive, the statutory authority cited in Sanlakas was Section 4, Chapter 2, Book II of the Revised
shall submit a report in person or in writing to the Congress. The Congress, voting jointly, by a vote of at Administrative Code of 1987, which provides:
least a majority of all its Members in regular or special session, may revoke such proclamation or
suspension, which revocation shall not be set aside by the President. Upon the initiative of the
President, the Congress may, in the same manner, extend such proclamation or suspension for a
SEC. 4. – Proclamations. – Acts of the President fixing a date or declaring a status or condition of public Based on the above disquisition, it is clear that PP 1017 is not a declaration of Martial Law. It is merely
moment or interest, upon the existence of which the operation of a specific law or regulation is made to an exercise of President Arroyo’s calling-out power for the armed forces to assist her in preventing
depend, shall be promulgated in proclamations which shall have the force of an executive order. or suppressing lawless violence.

President Arroyo’s declaration of a "state of rebellion" was merely an act declaring a status or condition Second Provision: "Take Care" Power
of public moment or interest, a declaration allowed under Section 4 cited above. Such declaration, in the
words of Sanlakas, is harmless, without legal significance, and deemed not written. In these cases, PP The second provision pertains to the power of the President to ensure that the laws be faithfully
1017 is more than that. In declaring a state of national emergency, President Arroyo did not only rely on executed. This is based on Section 17, Article VII which reads:
Section 18, Article VII of the Constitution, a provision calling on the AFP to prevent or suppress lawless
violence, invasion or rebellion. She also relied on Section 17, Article XII, a provision on the State’s SEC. 17. The President shall have control of all the executive departments, bureaus, and offices. He
extraordinary power to take over privately-owned public utility and business affected with public interest. shall ensure that the laws be faithfully executed.
Indeed, PP 1017 calls for the exercise of an awesome power. Obviously, such Proclamation cannot be 115
As the Executive in whom the executive power is vested, the primary function of the President is to
deemed harmless, without legal significance, or not written, as in the case of Sanlakas.
enforce the laws as well as to formulate policies to be embodied in existing laws. He sees to it that all
Some of the petitioners vehemently maintain that PP 1017 is actually a declaration of Martial Law. It is laws are enforced by the officials and employees of his department. Before assuming office, he is
no so. What defines the character of PP 1017 are its wordings. It is plain therein that what the President required to take an oath or affirmation to the effect that as President of the Philippines, he will, among
116
invoked was her calling-out power. others, "execute its laws." In the exercise of such function, the President, if needed, may employ the
powers attached to his office as the Commander-in-Chief of all the armed forces of the
117 118
The declaration of Martial Law is a "warn[ing] to citizens that the military power has been called upon by country, including the Philippine National Police under the Department of Interior and Local
119
the executive to assist in the maintenance of law and order, and that, while the emergency lasts, they Government.
must, upon pain of arrest and punishment, not commit any acts which will in any way render more
difficult the restoration of order and the enforcement of law."
113 Petitioners, especially Representatives Francis Joseph G. Escudero, Satur Ocampo, Rafael Mariano,
Teodoro Casiño, Liza Maza, and Josel Virador argue that PP 1017 is unconstitutional as it arrogated
In his "Statement before the Senate Committee on Justice" on March 13, 2006, Mr. Justice Vicente V. upon President Arroyo the power to enact laws and decrees in violation of Section 1, Article VI of the
114 Constitution, which vests the power to enact laws in Congress. They assail the clause "to enforce
Mendoza, an authority in constitutional law, said that of the three powers of the President as
Commander-in-Chief, the power to declare Martial Law poses the most severe threat to civil liberties. It obedience to all the laws and to all decrees, orders and regulations promulgated by me
is a strong medicine which should not be resorted to lightly. It cannot be used to stifle or persecute personally or upon my direction."
critics of the government. It is placed in the keeping of the President for the purpose of enabling him to
secure the people from harm and to restore order so that they can enjoy their individual freedoms. In Petitioners’ contention is understandable. A reading of PP 1017 operative clause shows that it was
120
fact, Section 18, Art. VII, provides: lifted from Former President Marcos’ Proclamation No. 1081, which partly reads:

A state of martial law does not suspend the operation of the Constitution, nor supplant the functioning of NOW, THEREFORE, I, FERDINAND E. MARCOS, President of the Philippines by virtue of the powers
the civil courts or legislative assemblies, nor authorize the conferment of jurisdiction on military courts vested upon me by Article VII, Section 10, Paragraph (2) of the Constitution, do hereby place the entire
and agencies over civilians where civil courts are able to function, nor automatically suspend the Philippines as defined in Article 1, Section 1 of the Constitution under martial law and, in my capacity as
privilege of the writ. their Commander-in-Chief, do hereby command the Armed Forces of the Philippines, to maintain
law and order throughout the Philippines, prevent or suppress all forms of lawless violence as
Justice Mendoza also stated that PP 1017 is not a declaration of Martial Law. It is no more than a call well as any act of insurrection or rebellion and to enforce obedience to all the laws and decrees,
by the President to the armed forces to prevent or suppress lawless violence. As such, it cannot be orders and regulations promulgated by me personally or upon my direction.
used to justify acts that only under a valid declaration of Martial Law can be done. Its use for any other
purpose is a perversion of its nature and scope, and any act done contrary to its command is ultra vires. We all know that it was PP 1081 which granted President Marcos legislative power. Its enabling clause
states: "to enforce obedience to all the laws and decrees, orders and regulations promulgated by
Justice Mendoza further stated that specifically, (a) arrests and seizures without judicial warrants; (b) me personally or upon my direction." Upon the other hand, the enabling clause of PP 1017 issued by
ban on public assemblies; (c) take-over of news media and agencies and press censorship; and (d) President Arroyo is: to enforce obedience to all the laws and to all decrees, orders and
issuance of Presidential Decrees, are powers which can be exercised by the President as Commander- regulations promulgated by me personally or upon my direction."
in-Chief only where there is a valid declaration of Martial Law or suspension of the writ of habeas
corpus. Is it within the domain of President Arroyo to promulgate "decrees"?
PP 1017 states in part: "to enforce obedience to all the laws and decrees x x x promulgated by me As this Court stated earlier, President Arroyo has no authority to enact decrees. It follows that these
personally or upon my direction." decrees are void and, therefore, cannot be enforced. With respect to "laws," she cannot call the military
to enforce or implement certain laws, such as customs laws, laws governing family and property
The President is granted an Ordinance Power under Chapter 2, Book III of Executive Order No. 292 relations, laws on obligations and contracts and the like. She can only order the military, under PP
(Administrative Code of 1987). She may issue any of the following: 1017, to enforce laws pertinent to its duty to suppress lawless violence.

Sec. 2. Executive Orders. — Acts of the President providing for rules of a general or permanent Third Provision: Power to Take Over
character in implementation or execution of constitutional or statutory powers shall be promulgated in
executive orders. The pertinent provision of PP 1017 states:

Sec. 3. Administrative Orders. — Acts of the President which relate to particular aspect of governmental x x x and to enforce obedience to all the laws and to all decrees, orders, and regulations promulgated
operations in pursuance of his duties as administrative head shall be promulgated in administrative by me personally or upon my direction; and as provided in Section 17, Article XII of the Constitution
orders. do hereby declare a state of national emergency.

Sec. 4. Proclamations. — Acts of the President fixing a date or declaring a status or condition of public The import of this provision is that President Arroyo, during the state of national emergency under PP
moment or interest, upon the existence of which the operation of a specific law or regulation is made to 1017, can call the military not only to enforce obedience "to all the laws and to all decrees x x x" but
depend, shall be promulgated in proclamations which shall have the force of an executive order. also to act pursuant to the provision of Section 17, Article XII which reads:

Sec. 5. Memorandum Orders. — Acts of the President on matters of administrative detail or of Sec. 17. In times of national emergency, when the public interest so requires, the State may, during the
subordinate or temporary interest which only concern a particular officer or office of the Government emergency and under reasonable terms prescribed by it, temporarily take over or direct the operation of
shall be embodied in memorandum orders. any privately-owned public utility or business affected with public interest.

Sec. 6. Memorandum Circulars. — Acts of the President on matters relating to internal administration, What could be the reason of President Arroyo in invoking the above provision when she issued PP
which the President desires to bring to the attention of all or some of the departments, agencies, 1017?
bureaus or offices of the Government, for information or compliance, shall be embodied in
memorandum circulars. The answer is simple. During the existence of the state of national emergency, PP 1017 purports to
grant the President, without any authority or delegation from Congress, to take over or direct the
Sec. 7. General or Special Orders. — Acts and commands of the President in his capacity as operation of any privately-owned public utility or business affected with public interest.
Commander-in-Chief of the Armed Forces of the Philippines shall be issued as general or special
orders. This provision was first introduced in the 1973 Constitution, as a product of the "martial law" thinking of
122
the 1971 Constitutional Convention. In effect at the time of its approval was President Marcos’ Letter
President Arroyo’s ordinance power is limited to the foregoing issuances. She cannot of Instruction No. 2 dated September 22, 1972 instructing the Secretary of National Defense to take
issue decrees similar to those issued by Former President Marcos under PP 1081. Presidential over "the management, control and operation of the Manila Electric Company, the Philippine Long
Decrees are laws which are of the same category and binding force as statutes because they were Distance Telephone Company, the National Waterworks and Sewerage Authority, the Philippine
issued by the President in the exercise of his legislative power during the period of Martial Law under National Railways, the Philippine Air Lines, Air Manila (and) Filipinas Orient Airways . . . for the
121
the 1973 Constitution. successful prosecution by the Government of its effort to contain, solve and end the present national
emergency."
This Court rules that the assailed PP 1017 is unconstitutional insofar as it grants President
Arroyo the authority to promulgate "decrees." Legislative power is peculiarly within the province of Petitioners, particularly the members of the House of Representatives, claim that President Arroyo’s
the Legislature. Section 1, Article VI categorically states that "[t]he legislative power shall be vested inclusion of Section 17, Article XII in PP 1017 is an encroachment on the legislature’s emergency
in the Congress of the Philippines which shall consist of a Senate and a House of powers.
Representatives." To be sure, neither Martial Law nor a state of rebellion nor a state of emergency can
justify President Arroyo’s exercise of legislative power by issuing decrees. This is an area that needs delineation.

Can President Arroyo enforce obedience to all decrees and laws through the military? A distinction must be drawn between the President’s authority to declare "a state of national
emergency" and to exercise emergency powers. To the first, as elucidated by the Court, Section 18,
Article VII grants the President such power, hence, no legitimate constitutional objection can be raised. Section 17, Article XII must be understood as an aspect of the emergency powers clause. The taking
But to the second, manifold constitutional issues arise. over of private business affected with public interest is just another facet of the emergency powers
generally reposed upon Congress. Thus, when Section 17 states that the "the State may, during the
Section 23, Article VI of the Constitution reads: emergency and under reasonable terms prescribed by it, temporarily take over or direct the
operation of any privately owned public utility or business affected with public interest," it refers
SEC. 23. (1) The Congress, by a vote of two-thirds of both Houses in joint session assembled, voting to Congress, not the President. Now, whether or not the President may exercise such power is
separately, shall have the sole power to declare the existence of a state of war. dependent on whether Congress may delegate it to him pursuant to a law prescribing the reasonable
125
terms thereof. Youngstown Sheet & Tube Co. et al. v. Sawyer, held:
(2) In times of war or other national emergency, the Congress may, by law, authorize the President,
for a limited period and subject to such restrictions as it may prescribe, to exercise powers necessary It is clear that if the President had authority to issue the order he did, it must be found in some provision
and proper to carry out a declared national policy. Unless sooner withdrawn by resolution of the of the Constitution. And it is not claimed that express constitutional language grants this power to the
Congress, such powers shall cease upon the next adjournment thereof. President. The contention is that presidential power should be implied from the aggregate of his powers
under the Constitution. Particular reliance is placed on provisions in Article II which say that "The
It may be pointed out that the second paragraph of the above provision refers not only to war but also to
executive Power shall be vested in a President . . . .;" that "he shall take Care that the Laws be faithfully
"other national emergency." If the intention of the Framers of our Constitution was to withhold from the
executed;" and that he "shall be Commander-in-Chief of the Army and Navy of the United States.
President the authority to declare a "state of national emergency" pursuant to Section 18, Article VII
(calling-out power) and grant it to Congress (like the declaration of the existence of a state of war), then The order cannot properly be sustained as an exercise of the President’s military power as
the Framers could have provided so. Clearly, they did not intend that Congress should first authorize Commander-in-Chief of the Armed Forces. The Government attempts to do so by citing a number of
the President before he can declare a "state of national emergency." The logical conclusion then is that cases upholding broad powers in military commanders engaged in day-to-day fighting in a theater of
President Arroyo could validly declare the existence of a state of national emergency even in the war. Such cases need not concern us here. Even though "theater of war" be an expanding concept,
absence of a Congressional enactment. we cannot with faithfulness to our constitutional system hold that the Commander-in-Chief of
the Armed Forces has the ultimate power as such to take possession of private property in
But the exercise of emergency powers, such as the taking over of privately owned public utility or
order to keep labor disputes from stopping production. This is a job for the nation’s lawmakers,
business affected with public interest, is a different matter. This requires a delegation from Congress.
not for its military authorities.
Courts have often said that constitutional provisions in pari materia are to be construed together.
Nor can the seizure order be sustained because of the several constitutional provisions that
Otherwise stated, different clauses, sections, and provisions of a constitution which relate to the same
123 grant executive power to the President. In the framework of our Constitution, the President’s
subject matter will be construed together and considered in the light of each other. Considering
power to see that the laws are faithfully executed refutes the idea that he is to be a
that Section 17 of Article XII and Section 23 of Article VI, previously quoted, relate to national
lawmaker. The Constitution limits his functions in the lawmaking process to the recommending
emergencies, they must be read together to determine the limitation of the exercise of emergency
of laws he thinks wise and the vetoing of laws he thinks bad. And the Constitution is neither
powers.
silent nor equivocal about who shall make laws which the President is to execute. The first
Generally, Congress is the repository of emergency powers. This is evident in the tenor of Section section of the first article says that "All legislative Powers herein granted shall be vested in a
126
23 (2), Article VI authorizing it to delegate such powers to the President. Certainly, a body cannot Congress of the United States. . ."
delegate a power not reposed upon it. However, knowing that during grave emergencies, it may not
Petitioner Cacho-Olivares, et al. contends that the term "emergency" under Section 17, Article XII refers
be possible or practicable for Congress to meet and exercise its powers, the Framers of our
to "tsunami," "typhoon," "hurricane"and"similar occurrences." This is a limited view of "emergency."
Constitution deemed it wise to allow Congress to grant emergency powers to the President, subject to
certain conditions, thus: Emergency, as a generic term, connotes the existence of conditions suddenly intensifying the degree of
existing danger to life or well-being beyond that which is accepted as normal. Implicit in this definitions
(1) There must be a war or other emergency. 127
are the elements of intensity, variety, and perception. Emergencies, as perceived by legislature or
(2) The delegation must be for a limited period only. executive in the United Sates since 1933, have been occasioned by a wide range of situations,
128 129
classifiable under three (3) principal heads: a)economic, b) natural disaster, and c) national
130
(3) The delegation must be subject to such restrictions as the Congress may prescribe. security.

124
(4) The emergency powers must be exercised to carry out a national policy declared by Congress.
"Emergency," as contemplated in our Constitution, is of the same breadth. It may include rebellion, legislative policy according to prescribed standards; no, not even when that Republic was fighting a total
economic crisis, pestilence or epidemic, typhoon, flood, or other similar catastrophe of nationwide war, or when it was engaged in a life-and-death struggle to preserve the Union. The truth is that under
131
proportions or effect. This is evident in the Records of the Constitutional Commission, thus: our concept of constitutional government, in times of extreme perils more than in normal circumstances
‘the various branches, executive, legislative, and judicial,’ given the ability to act, are called upon ‘to
MR. GASCON. Yes. What is the Committee’s definition of "national emergency" which appears in perform the duties and discharge the responsibilities committed to them respectively."
Section 13, page 5? It reads:
Following our interpretation of Section 17, Article XII, invoked by President Arroyo in issuing PP 1017,
When the common good so requires, the State may temporarily take over or direct the operation of any this Court rules that such Proclamation does not authorize her during the emergency to temporarily take
privately owned public utility or business affected with public interest. over or direct the operation of any privately owned public utility or business affected with public interest
without authority from Congress.
MR. VILLEGAS. What I mean is threat from external aggression, for example, calamities or natural
disasters. Let it be emphasized that while the President alone can declare a state of national emergency,
however, without legislation, he has no power to take over privately-owned public utility or business
MR. GASCON. There is a question by Commissioner de los Reyes. What about strikes and riots? affected with public interest. The President cannot decide whether exceptional circumstances exist
warranting the take over of privately-owned public utility or business affected with public interest. Nor
MR. VILLEGAS. Strikes, no; those would not be covered by the term "national emergency."
can he determine when such exceptional circumstances have ceased. Likewise, without
legislation, the President has no power to point out the types of businesses affected with public interest
MR. BENGZON. Unless they are of such proportions such that they would paralyze government
service.
132 that should be taken over. In short, the President has no absolute authority to exercise all the powers of
the State under Section 17, Article VII in the absence of an emergency powers act passed by Congress.
xxxxxx
c. "AS APPLIED CHALLENGE"
MR. TINGSON. May I ask the committee if "national emergency" refers to military national
emergency or could this be economic emergency?" One of the misfortunes of an emergency, particularly, that which pertains to security, is that military
necessity and the guaranteed rights of the individual are often not compatible. Our history reveals that
MR. VILLEGAS. Yes, it could refer to both military or economic dislocations. in the crucible of conflict, many rights are curtailed and trampled upon. Here, the right against
unreasonable search and seizure; the right against warrantless arrest; and the freedom of
133
MR. TINGSON. Thank you very much. speech, of expression, of the press, and of assembly under the Bill of Rights suffered the greatest
blow.
It may be argued that when there is national emergency, Congress may not be able to convene and,
therefore, unable to delegate to the President the power to take over privately-owned public utility or Of the seven (7) petitions, three (3) indicate "direct injury."
business affected with public interest.
In G.R. No. 171396, petitioners David and Llamas alleged that, on February 24, 2006, they were
134
In Araneta v. Dinglasan, this Court emphasized that legislative power, through which extraordinary arrested without warrants on their way to EDSA to celebrate the 20th Anniversary of People Power
measures are exercised, remains in Congress even in times of crisis. I. The arresting officers cited PP 1017 as basis of the arrest.

"x x x In G.R. No. 171409, petitioners Cacho-Olivares and Tribune Publishing Co., Inc. claimed that on
February 25, 2006, the CIDG operatives "raided and ransacked without warrant" their office. Three
After all the criticisms that have been made against the efficiency of the system of the separation of policemen were assigned to guard their office as a possible "source of destabilization." Again, the basis
powers, the fact remains that the Constitution has set up this form of government, with all its defects was PP 1017.
and shortcomings, in preference to the commingling of powers in one man or group of men. The Filipino
people by adopting parliamentary government have given notice that they share the faith of other And in G.R. No. 171483, petitioners KMU and NAFLU-KMU et al. alleged that their members were
democracy-loving peoples in this system, with all its faults, as the ideal. The point is, under this "turned away and dispersed" when they went to EDSA and later, to Ayala Avenue, to celebrate the 20th
framework of government, legislation is preserved for Congress all the time, not excepting periods of Anniversary of People Power I.
crisis no matter how serious. Never in the history of the United States, the basic features of whose
Constitution have been copied in ours, have specific functions of the legislative branch of enacting laws A perusal of the "direct injuries" allegedly suffered by the said petitioners shows that they resulted from
been surrendered to another department – unless we regard as legislating the carrying out of a the implementation, pursuant to G.O. No. 5, of PP 1017.
Can this Court adjudge as unconstitutional PP 1017 and G.O. No 5 on the basis of these illegal acts? In The basic problem underlying all these military actions – or threats of the use of force as the most
general, does the illegal implementation of a law render it unconstitutional? recent by the United States against Iraq – consists in the absence of an agreed definition of terrorism.

Settled is the rule that courts are not at liberty to declare statutes invalid although they may be Remarkable confusion persists in regard to the legal categorization of acts of violence either by states,
135
abused and misabused and may afford an opportunity for abuse in the manner of by armed groups such as liberation movements, or by individuals.
136
application. The validity of a statute or ordinance is to be determined from its general purpose and
137
its efficiency to accomplish the end desired, not from its effects in a particular case. PP 1017 is The dilemma can by summarized in the saying "One country’s terrorist is another country’s freedom
merely an invocation of the President’s calling-out power. Its general purpose is to command the AFP to fighter." The apparent contradiction or lack of consistency in the use of the term "terrorism" may further
suppress all forms of lawless violence, invasion or rebellion. It had accomplished the end desired which be demonstrated by the historical fact that leaders of national liberation movements such as Nelson
prompted President Arroyo to issue PP 1021. But there is nothing in PP 1017 allowing the police, Mandela in South Africa, Habib Bourgouiba in Tunisia, or Ahmed Ben Bella in Algeria, to mention only a
expressly or impliedly, to conduct illegal arrest, search or violate the citizens’ constitutional rights. few, were originally labeled as terrorists by those who controlled the territory at the time, but later
became internationally respected statesmen.
Now, may this Court adjudge a law or ordinance unconstitutional on the ground that its implementor
committed illegal acts? The answer is no. The criterion by which the validity of the statute or ordinance What, then, is the defining criterion for terrorist acts – the differentia specifica distinguishing those acts
is to be measured is the essential basis for the exercise of power, and not a mere incidental result from eventually legitimate acts of national resistance or self-defense?
138
arising from its exertion. This is logical. Just imagine the absurdity of situations when laws maybe
declared unconstitutional just because the officers implementing them have acted arbitrarily. If this were Since the times of the Cold War the United Nations Organization has been trying in vain to reach a
so, judging from the blunders committed by policemen in the cases passed upon by the Court, majority consensus on the basic issue of definition. The organization has intensified its efforts recently, but has
of the provisions of the Revised Penal Code would have been declared unconstitutional a long time been unable to bridge the gap between those who associate "terrorism" with any violent act by non-
ago. state groups against civilians, state functionaries or infrastructure or military installations, and those who
believe in the concept of the legitimate use of force when resistance against foreign occupation or
President Arroyo issued G.O. No. 5 to carry into effect the provisions of PP 1017. General orders are against systematic oppression of ethnic and/or religious groups within a state is concerned.
"acts and commands of the President in his capacity as Commander-in-Chief of the Armed Forces of
the Philippines." They are internal rules issued by the executive officer to his subordinates precisely for The dilemma facing the international community can best be illustrated by reference to the contradicting
the proper and efficientadministration of law. Such rules and regulations create no relation except categorization of organizations and movements such as Palestine Liberation Organization (PLO) –
139
between the official who issues them and the official who receives them. They are based on and are which is a terrorist group for Israel and a liberation movement for Arabs and Muslims – the Kashmiri
140
the product of, a relationship in which power is their source, and obedience, their object. For these resistance groups – who are terrorists in the perception of India, liberation fighters in that of Pakistan –
reasons, one requirement for these rules to be valid is that they must be reasonable, not arbitrary or the earlier Contras in Nicaragua – freedom fighters for the United States, terrorists for the Socialist
capricious. camp – or, most drastically, the Afghani Mujahedeen (later to become the Taliban movement): during
the Cold War period they were a group of freedom fighters for the West, nurtured by the United States,
G.O. No. 5 mandates the AFP and the PNP to immediately carry out the "necessary and appropriate and a terrorist gang for the Soviet Union. One could go on and on in enumerating examples of
actions and measures to suppress and prevent acts of terrorism and lawless violence." conflicting categorizations that cannot be reconciled in any way – because of opposing political interests
that are at the roots of those perceptions.
Unlike the term "lawless violence" which is unarguably extant in our statutes and the Constitution, and
which is invariably associated with "invasion, insurrection or rebellion," the phrase "acts of terrorism" is How, then, can those contradicting definitions and conflicting perceptions and evaluations of one and
still an amorphous and vague concept. Congress has yet to enact a law defining and punishing acts of the same group and its actions be explained? In our analysis, the basic reason for these striking
terrorism. inconsistencies lies in the divergent interest of states. Depending on whether a state is in the position of
an occupying power or in that of a rival, or adversary, of an occupying power in a given territory, the
In fact, this "definitional predicament" or the "absence of an agreed definition of terrorism" confronts not definition of terrorism will "fluctuate" accordingly. A state may eventually see itself as protector of the
only our country, but the international community as well. The following observations are quite apropos: rights of a certain ethnic group outside its territory and will therefore speak of a "liberation struggle," not
of "terrorism" when acts of violence by this group are concerned, and vice-versa.
In the actual unipolar context of international relations, the "fight against terrorism" has become one of
the basic slogans when it comes to the justification of the use of force against certain states and against The United Nations Organization has been unable to reach a decision on the definition of terrorism
groups operating internationally. Lists of states "sponsoring terrorism" and of terrorist organizations are exactly because of these conflicting interests of sovereign states that determine in each and every
set up and constantly being updated according to criteria that are not always known to the public, but instance how a particular armed movement (i.e. a non-state actor) is labeled in regard to the terrorists-
are clearly determined by strategic interests.
142
freedom fighter dichotomy. A "policy of double standards" on this vital issue of international affairs has persons or things to be seized." The plain import of the language of the Constitution is that searches,
been the unavoidable consequence. seizures and arrests are normally unreasonable unless authorized by a validly issued search warrant
or warrant of arrest. Thus, the fundamental protection given by this provision is that between person
This "definitional predicament" of an organization consisting of sovereign states – and not of peoples, in and police must stand the protective authority of a magistrate clothed with power to issue or refuse to
spite of the emphasis in the Preamble to the United Nations Charter! – has become even more serious issue search warrants or warrants of arrest.
143

in the present global power constellation: one superpower exercises the decisive role in the Security
144
Council, former great powers of the Cold War era as well as medium powers are increasingly being In the Brief Account submitted by petitioner David, certain facts are established: first, he was arrested
marginalized; and the problem has become even more acute since the terrorist attacks of 11 September without warrant; second, the PNP operatives arrested him on the basis of PP 1017; third, he was
141
2001 I the United States. brought at Camp Karingal, Quezon City where he was fingerprinted, photographed and booked like a
criminal suspect; fourth,he was treated brusquely by policemen who "held his head and tried to push
The absence of a law defining "acts of terrorism" may result in abuse and oppression on the part of the him" inside an unmarked car; fifth, he was charged with Violation of Batas Pambansa Bilang No.
police or military. An illustration is when a group of persons are merely engaged in a drinking spree. Yet 145
880 and Inciting to Sedition; sixth, he was detained for seven (7) hours; and seventh,he was
the military or the police may consider the act as an act of terrorism and immediately arrest them eventually released for insufficiency of evidence.
pursuant to G.O. No. 5. Obviously, this is abuse and oppression on their part. It must be remembered
that an act can only be considered a crime if there is a law defining the same as such and imposing the Section 5, Rule 113 of the Revised Rules on Criminal Procedure provides:
corresponding penalty thereon.
Sec. 5. Arrest without warrant; when lawful. - A peace officer or a private person may, without a
So far, the word "terrorism" appears only once in our criminal laws, i.e., in P.D. No. 1835 dated January warrant, arrest a person:
16, 1981 enacted by President Marcos during the Martial Law regime. This decree is entitled "Codifying
The Various Laws on Anti-Subversion and Increasing The Penalties for Membership in Subversive (a) When, in his presence, the person to be arrested has committed, is actually committing, or is
Organizations." The word "terrorism" is mentioned in the following provision: "That one who conspires attempting to commit an offense.
with any other person for the purpose of overthrowing the Government of the Philippines x x x by force,
violence, terrorism, x x x shall be punished by reclusion temporal x x x." (b) When an offense has just been committed and he has probable cause to believe based on personal
knowledge of facts or circumstances that the person to be arrested has committed it; and
P.D. No. 1835 was repealed by E.O. No. 167 (which outlaws the Communist Party of the Philippines)
enacted by President Corazon Aquino on May 5, 1985. These two (2) laws, however, do not define x x x.
"acts of terrorism." Since there is no law defining "acts of terrorism," it is President Arroyo alone, under
Neither of the two (2) exceptions mentioned above justifies petitioner David’s warrantless arrest. During
G.O. No. 5, who has the discretion to determine what acts constitute terrorism. Her judgment on this
the inquest for the charges of inciting to sedition and violation of BP 880, all that the arresting
aspect is absolute, without restrictions. Consequently, there can be indiscriminate arrest without
officers could invoke was their observation that some rallyists were wearing t-shirts with the
warrants, breaking into offices and residences, taking over the media enterprises, prohibition and
invective "Oust Gloria Now" and their erroneous assumption that petitioner David was the leader of the
dispersal of all assemblies and gatherings unfriendly to the administration. All these can be effected in 146
rally. Consequently, the Inquest Prosecutor ordered his immediate release on the ground of
the name of G.O. No. 5. These acts go far beyond the calling-out power of the President. Certainly, they
insufficiency of evidence. He noted that petitioner David was not wearing the subject t-shirt and even if
violate the due process clause of the Constitution. Thus, this Court declares that the "acts of terrorism"
he was wearing it, such fact is insufficient to charge him with inciting to sedition. Further, he also
portion of G.O. No. 5 is unconstitutional.
stated that there is insufficient evidence for the charge of violation of BP 880 as it was not even known
147
Significantly, there is nothing in G.O. No. 5 authorizing the military or police to commit acts beyond what whether petitioner David was the leader of the rally.
are necessary and appropriate to suppress and prevent lawless violence, the limitation of their
But what made it doubly worse for petitioners David et al. is that not only was their right against
authority in pursuing the Order. Otherwise, such acts are considered illegal.
warrantless arrest violated, but also their right to peaceably assemble.
We first examine G.R. No. 171396 (David et al.)
Section 4 of Article III guarantees:
The Constitution provides that "the right of the people to be secured in their persons, houses, papers
No law shall be passed abridging the freedom of speech, of expression, or of the press, or the right of
and effects against unreasonable search and seizure of whatever nature and for any purpose shall
the people peaceably to assemble and petition the government for redress of grievances.
be inviolable, and no search warrant or warrant of arrest shall issue except upon probable cause to
be determined personally by the judge after examination under oath or affirmation of the complainant "Assembly" means a right on the part of the citizens to meet peaceably for consultation in respect to
and the witnesses he may produce, and particularly describing the place to be searched and the public affairs. It is a necessary consequence of our republican institution and complements the right of
speech. As in the case of freedom of expression, this right is not to be limited, much less denied, except G.R. No. 171409, (Cacho-Olivares, et al.) presents another facet of freedom of speech i.e., the freedom
on a showing of a clear and present danger of a substantive evil that Congress has a right to prevent. of the press. Petitioners’ narration of facts, which the Solicitor General failed to refute, established the
In other words, like other rights embraced in the freedom of expression, the right to assemble is not following: first, the Daily Tribune’s offices were searched without warrant;second, the police operatives
subject to previous restraint or censorship. It may not be conditioned upon the prior issuance of a permit seized several materials for publication; third, the search was conducted at about 1:00 o’ clock in the
or authorization from the government authorities except, of course, if the assembly is intended to be morning of February 25, 2006; fourth, the search was conducted in the absence of any official of
held in a public place, a permit for the use of such place, and not for the assembly itself, may be validly the Daily Tribune except the security guard of the building; and fifth, policemen stationed themselves at
required. the vicinity of the Daily Tribune offices.

The ringing truth here is that petitioner David, et al. were arrested while they were exercising their right Thereafter, a wave of warning came from government officials. Presidential Chief of Staff Michael
to peaceful assembly. They were not committing any crime, neither was there a showing of a clear and Defensor was quoted as saying that such raid was "meant to show a ‘strong presence,’ to tell media
present danger that warranted the limitation of that right. As can be gleaned from circumstances, the outlets not to connive or do anything that would help the rebels in bringing down this
charges of inciting to sedition and violation of BP 880 were mere afterthought. Even the Solicitor government." Director General Lomibao further stated that "if they do not follow the standards –and
General, during the oral argument, failed to justify the arresting officers’ conduct. In De Jonge v. the standards are if they would contribute to instability in the government, or if they do not
148
Oregon, it was held that peaceable assembly cannot be made a crime, thus: subscribe to what is in General Order No. 5 and Proc. No. 1017 – we will recommend
a ‘takeover.’" National Telecommunications Commissioner Ronald Solis urged television and radio
Peaceable assembly for lawful discussion cannot be made a crime. The holding of meetings for networks to "cooperate" with the government for the duration of the state of national emergency. He
peaceable political action cannot be proscribed. Those who assist in the conduct of such meetings warned that his agency will not hesitate to recommend the closure of any broadcast outfit that
cannot be branded as criminals on that score. The question, if the rights of free speech and peaceful violates rules set out for media coverage during times when the national security is
151
assembly are not to be preserved, is not as to the auspices under which the meeting was held but as to threatened.
its purpose; not as to the relations of the speakers, but whether their utterances transcend the bounds
of the freedom of speech which the Constitution protects. If the persons assembling have committed The search is illegal. Rule 126 of The Revised Rules on Criminal Procedure lays down the steps in the
crimes elsewhere, if they have formed or are engaged in a conspiracy against the public peace and conduct of search and seizure. Section 4 requires that a search warrant be issued upon probable
order, they may be prosecuted for their conspiracy or other violations of valid laws. But it is a different cause in connection with one specific offence to be determined personally by the judge after
matter when the State, instead of prosecuting them for such offenses, seizes upon mere examination under oath or affirmation of the complainant and the witnesses he may produce. Section
participation in a peaceable assembly and a lawful public discussion as the basis for a criminal 8 mandates that the search of a house, room, or any other premise be made in the presence of
charge. the lawful occupant thereof or any member of his family or in the absence of the latter, in the presence
of two (2) witnesses of sufficient age and discretion residing in the same locality. And Section 9 states
On the basis of the above principles, the Court likewise considers the dispersal and arrest of the that the warrant must direct that it be served in the daytime, unless the property is on the person or in
members of KMU et al. (G.R. No. 171483) unwarranted. Apparently, their dispersal was done merely on the place ordered to be searched, in which case a direction may be inserted that it be served at any
the basis of Malacañang’s directive canceling all permits previously issued by local government units. time of the day or night. All these rules were violated by the CIDG operatives.
This is arbitrary. The wholesale cancellation of all permits to rally is a blatant disregard of the principle
that "freedom of assembly is not to be limited, much less denied, except on a showing of a clear Not only that, the search violated petitioners’ freedom of the press. The best gauge of a free and
149
and present danger of a substantive evil that the State has a right to prevent." Tolerance is the democratic society rests in the degree of freedom enjoyed by its media. In the Burgos v. Chief of
152
rule and limitation is the exception. Only upon a showing that an assembly presents a clear and present Staff this Court held that --
danger that the State may deny the citizens’ right to exercise it. Indeed, respondents failed to show or
convince the Court that the rallyists committed acts amounting to lawless violence, invasion or rebellion. As heretofore stated, the premises searched were the business and printing offices of the "Metropolitan
With the blanket revocation of permits, the distinction between protected and unprotected assemblies Mail" and the "We Forum" newspapers. As a consequence of the search and seizure, these premises
was eliminated. were padlocked and sealed, with the further result that the printing and publication of said
newspapers were discontinued.
Moreover, under BP 880, the authority to regulate assemblies and rallies is lodged with the local
government units. They have the power to issue permits and to revoke such permits after due notice Such closure is in the nature of previous restraint or censorship abhorrent to the freedom of the
and hearing on the determination of the presence of clear and present danger. Here, petitioners were press guaranteed under the fundamental law, and constitutes a virtual denial of petitioners'
150 freedom to express themselves in print. This state of being is patently anathematic to a
not even notified and heard on the revocation of their permits. The first time they learned of it was at
the time of the dispersal. Such absence of notice is a fatal defect. When a person’s right is restricted by democratic framework where a free, alert and even militant press is essential for the political
government action, it behooves a democratic government to see to it that the restriction is fair, enlightenment and growth of the citizenry.
reasonable, and according to procedure.
While admittedly, the Daily Tribune was not padlocked and sealed like the "Metropolitan Mail" and "We It is not based on Proclamation 1017, Your Honor, because there is nothing in 1017 which says that the
Forum" newspapers in the above case, yet it cannot be denied that the CIDG operatives exceeded their police could go and inspect and gather clippings from Daily Tribune or any other newspaper.
enforcement duties. The search and seizure of materials for publication, the stationing of policemen in
the vicinity of the The Daily Tribune offices, and the arrogant warning of government officials to media, SR. ASSO. JUSTICE PUNO:
are plain censorship. It is that officious functionary of the repressive government who tells the citizen
that he may speak only if allowed to do so, and no more and no less than what he is permitted to say on Is it based on any law?
153
pain of punishment should he be so rash as to disobey. Undoubtedly, the The Daily Tribune was
SOLGEN BENIPAYO:
subjected to these arbitrary intrusions because of its anti-government sentiments. This Court cannot
tolerate the blatant disregard of a constitutional right even if it involves the most defiant of our citizens. As far as I know, no, Your Honor, from the facts, no.
Freedom to comment on public affairs is essential to the vitality of a representative democracy. It is the
duty of the courts to be watchful for the constitutional rights of the citizen, and against any stealthy SR. ASSO. JUSTICE PUNO:
154
encroachments thereon. The motto should always be obsta principiis.
So, it has no basis, no legal basis whatsoever?
Incidentally, during the oral arguments, the Solicitor General admitted that the search of
the Tribune’s offices and the seizure of its materials for publication and other papers are illegal; and that SOLGEN BENIPAYO:
the same are inadmissible "for any purpose," thus:
Maybe so, Your Honor. Maybe so, that is why I said, I don’t know if it is premature to say this, we do
JUSTICE CALLEJO: not condone this. If the people who have been injured by this would want to sue them, they can
156
sue and there are remedies for this.
You made quite a mouthful of admission when you said that the policemen, when inspected the Tribune
for the purpose of gathering evidence and you admitted that the policemen were able to get the Likewise, the warrantless arrests and seizures executed by the police were, according to the Solicitor
clippings. Is that not in admission of the admissibility of these clippings that were taken from the General, illegal and cannot be condoned, thus:
Tribune?
CHIEF JUSTICE PANGANIBAN:
SOLICITOR GENERAL BENIPAYO:
There seems to be some confusions if not contradiction in your theory.
Under the law they would seem to be, if they were illegally seized, I think and I know, Your Honor, and
155
these are inadmissible for any purpose. SOLICITOR GENERAL BENIPAYO:

xxxxxxxxx I don’t know whether this will clarify. The acts, the supposed illegal or unlawful acts committed on the
occasion of 1017, as I said, it cannot be condoned. You cannot blame the President for, as you said,
157
SR. ASSO. JUSTICE PUNO: a misapplication of the law. These are acts of the police officers, that is their responsibility.

These have been published in the past issues of the Daily Tribune; all you have to do is to get those The Dissenting Opinion states that PP 1017 and G.O. No. 5 are constitutional in every aspect and
past issues. So why do you have to go there at 1 o’clock in the morning and without any search "should result in no constitutional or statutory breaches if applied according to their letter."
warrant? Did they become suddenly part of the evidence of rebellion or inciting to sedition or what?
The Court has passed upon the constitutionality of these issuances. Its ratiocination has been
SOLGEN BENIPAYO: exhaustively presented. At this point, suffice it to reiterate that PP 1017 is limited to the calling out by
the President of the military to prevent or suppress lawless violence, invasion or rebellion. When in
Well, it was the police that did that, Your Honor. Not upon my instructions. implementing its provisions, pursuant to G.O. No. 5, the military and the police committed acts which
violate the citizens’ rights under the Constitution, this Court has to declare such acts unconstitutional
SR. ASSO. JUSTICE PUNO: and illegal.
Are you saying that the act of the policeman is illegal, it is not based on any law, and it is not based on In this connection, Chief Justice Artemio V. Panganiban’s concurring opinion, attached hereto, is
Proclamation 1017. considered an integral part of this ponencia.
SOLGEN BENIPAYO: SUMMATION
In sum, the lifting of PP 1017 through the issuance of PP 1021 – a supervening event – would have Perhaps, the vital lesson that we must learn from the theorists who studied the various competing
normally rendered this case moot and academic. However, while PP 1017 was still operative, illegal political philosophies is that, it is possible to grant government the authority to cope with crises without
acts were committed allegedly in pursuance thereof. Besides, there is no guarantee that PP 1017, or surrendering the two vital principles of constitutionalism: the maintenance of legal limits to arbitrary
158
one similar to it, may not again be issued. Already, there have been media reports on April 30, 2006 power, and political responsibility of the government to the governed.
that allegedly PP 1017 would be reimposed "if the May 1 rallies" become "unruly and violent."
Consequently, the transcendental issues raised by the parties should not be "evaded;" they must now WHEREFORE, the Petitions are partly granted. The Court rules that PP 1017
be resolved to prevent future constitutional aberration. is CONSTITUTIONAL insofar as it constitutes a call by President Gloria Macapagal-Arroyo on the
AFP to prevent or suppress lawless violence. However, the provisions of PP 1017 commanding the
The Court finds and so holds that PP 1017 is constitutional insofar as it constitutes a call by the AFP to enforce laws not related to lawless violence, as well as decrees promulgated by the President,
President for the AFP to prevent or suppress lawless violence. The proclamation is sustained by are declared UNCONSTITUTIONAL. In addition, the provision in PP 1017 declaring national
Section 18, Article VII of the Constitution and the relevant jurisprudence discussed earlier. However, PP emergency under Section 17, Article VII of the Constitution is CONSTITUTIONAL, but such declaration
1017’s extraneous provisions giving the President express or implied power (1) to issue decrees; (2) to does not authorize the President to take over privately-owned public utility or business affected with
direct the AFP to enforce obedience to all laws even those not related to lawless violence as well as public interest without prior legislation.
decrees promulgated by the President; and (3) to impose standards on media or any form of prior
restraint on the press, are ultra vires and unconstitutional. The Court also rules that under Section 17, G.O. No. 5 is CONSTITUTIONAL since it provides a standard by which the AFP and the PNP should
Article XII of the Constitution, the President, in the absence of a legislation, cannot take over privately- implement PP 1017, i.e. whatever is "necessary and appropriate actions and measures to
owned public utility and private business affected with public interest. suppress and prevent acts of lawless violence." Considering that "acts of terrorism" have not yet
been defined and made punishable by the Legislature, such portion of G.O. No. 5 is
In the same vein, the Court finds G.O. No. 5 valid. It is an Order issued by the President – acting as declared UNCONSTITUTIONAL.
Commander-in-Chief – addressed to subalterns in the AFP to carry out the provisions of PP 1017.
Significantly, it also provides a valid standard – that the military and the police should take only the The warrantless arrest of Randolf S. David and Ronald Llamas; the dispersal and warrantless arrest of
"necessary and appropriate actions and measures to suppress and prevent acts of lawless the KMU and NAFLU-KMU members during their rallies, in the absence of proof that these petitioners
violence."But the words "acts of terrorism" found in G.O. No. 5 have not been legally defined and were committing acts constituting lawless violence, invasion or rebellion and violating BP 880; the
made punishable by Congress and should thus be deemed deleted from the said G.O. While "terrorism" imposition of standards on media or any form of prior restraint on the press, as well as the warrantless
has been denounced generally in media, no law has been enacted to guide the military, and eventually search of the Tribune offices and whimsical seizure of its articles for publication and other materials, are
the courts, to determine the limits of the AFP’s authority in carrying out this portion of G.O. No. 5. declared UNCONSTITUTIONAL.

On the basis of the relevant and uncontested facts narrated earlier, it is also pristine clear that (1) the No costs.
warrantless arrest of petitioners Randolf S. David and Ronald Llamas; (2) the dispersal of the rallies and
warrantless arrest of the KMU and NAFLU-KMU members; (3) the imposition of standards on media or SO ORDERED.
any prior restraint on the press; and (4) the warrantless search of the Tribune offices and the whimsical
seizures of some articles for publication and other materials, are not authorized by the Constitution, the
law and jurisprudence. Not even by the valid provisions of PP 1017 and G.O. No. 5.

Other than this declaration of invalidity, this Court cannot impose any civil, criminal or administrative
sanctions on the individual police officers concerned. They have not been individually identified and
given their day in court. The civil complaints or causes of action and/or relevant criminal Informations
have not been presented before this Court. Elementary due process bars this Court from making any
specific pronouncement of civil, criminal or administrative liabilities.

It is well to remember that military power is a means to an end and substantive civil rights are
ends in themselves. How to give the military the power it needs to protect the Republic without
unnecessarily trampling individual rights is one of the eternal balancing tasks of a democratic
state.During emergency, governmental action may vary in breadth and intensity from normal times, yet
they should not be arbitrary as to unduly restrain our people’s liberty.
Republic of the Philippines This Memorandum, which was labeled ‘secret’ on its all pages, also outlined the responsibilities of each
SUPREME COURT of the party signatories, as follows:
Manila
Responsibilities of the Provincial Government:
EN BANC
1) The Provincial Government shall source the funds and logistics needed for the activation of the CEF;
G.R. No. 187298 July 03, 2012
2) The Provincial Government shall identify the Local Government Units which shall participate in the
JAMAR M. KULAYAN, TEMOGEN S. TULAWIE, HJI. MOH. YUSOP ISMI, JULHAJAN AWADI, and operations and to propose them for the approval of the parties to this agreement;
SPO1 SATTAL H. JADJULI, Petitioners,
vs. 3) The Provincial Government shall ensure that there will be no unilateral action(s) by the CEF without
GOV. ABDUSAKUR M. TAN, in his capacity as Governor of Sulu; GEN. JUANCHO SABAN, COL. the knowledge and approval by both parties.
EUGENIO CLEMEN PN, P/SUPT. JULASIRIM KASIM and P/SUPT. BIENVENIDO G. LATAG, in
their capacity as officers of the Phil. Marines and Phil. National Police, Responsibilities of AFP/PNP/ TF ICRC (Task Force ICRC):
respectively, Respondents.
1) The AFP/PNP shall remain the authority as prescribed by law in military operations and law
DECISION enforcement;

SERENO, J.: 2) The AFP/PNP shall ensure the orderly deployment of the CEF in the performance of their assigned
task(s);
On 15 January 2009, three members from the International Committee of the Red Cross (ICRC) were
1
kidnapped in the vicinity of the Provincial Capitol in Patikul, Sulu. Andres Notter, a Swiss national and 3) The AFP/PNP shall ensure the safe movements of the CEF in identified areas of operation(s);
head of the ICRC in Zamboanga City, Eugenio Vagni, an Italian national and ICRC delegate, and Marie
4) The AFP/PNP shall provide the necessary support and/or assistance as called for in the course of
Jean Lacaba, a Filipino engineer, were purportedly inspecting a water and sanitation project for the Sulu 8
operation(s)/movements of the CEF.
Provincial Jail when inspecting a water and sanitation project for the Sulu Provincial Jail when they were
seized by three armed men who were later confirmed to be members of the Abu Sayyaf Group Meanwhile, Ronaldo Puno, then Secretary of the Department of Interior and Local Government,
2
(ASG). The leader of the alleged kidnappers was identified as Raden Abu, a former guard at the Sulu announced to the media that government troops had cornered some one hundred and twenty (120) Abu
Provincial Jail. News reports linked Abu to Albader Parad, one of the known leaders of the Abu Sayyaf. 9
Sayyaf members along with the three (3) hostages. However, the ASG made
On 21 January 2009, a task force was created by the ICRC and the Philippine National Police (PNP), contact with the authorities and demanded that the military pull its troops back from the jungle
3
which then organized a parallel local group known as the Local Crisis Committee. The local group, later 10
area. The government troops yielded and went back to their barracks; the Philippine Marines withdrew
renamed Sulu Crisis Management Committee, convened under the leadership of respondent Abdusakur to their camp, while police and civilian forces pulled back from the terrorists’ stronghold by ten (10) to
Mahail Tan, the Provincial Governor of Sulu. Its armed forces component was headed by respondents fifteen (15) kilometers. Threatening that one of the hostages will be beheaded, the ASG further
General Juancho Saban, and his deputy, Colonel Eugenio Clemen. The PNP component was headed demanded the evacuation of the military camps and bases in the different barangays in Jolo. The
11

by respondent Police Superintendent Bienvenido G. Latag, the Police Deputy Director for Operations of authorities were given no later than 2:00 o’clock in the afternoon of 31 March 2009 to comply.
12
4
the Autonomous Region of Muslim Mindanao (ARMM).
On 31 March 2009, Governor Tan issued Proclamation No. 1, Series of 2009 (Proclamation 1-09),
Governor Tan organized the Civilian Emergency Force (CEF), a group of armed male civilians coming 13
declaring a state of emergency in the province of Sulu. It cited the kidnapping incident as a ground for
5
from different municipalities, who were redeployed to surrounding areas of Patikul. The organization of the said declaration, describing it as a terrorist act pursuant to the Human Security
6
the CEF was embodied in a "Memorandum of Understanding" entered into
Act (R.A. 9372). It also invoked Section 465 of the Local Government Code of 1991 (R.A. 7160), which
between three parties: the provincial government of Sulu, represented by Governor Tan; the Armed bestows on the Provincial Governor the power to carry out emergency measures during man-made and
Forces of the Philippines, represented by Gen. Saban; and the Philippine National Police, represented natural disasters and calamities, and to call upon the appropriate national law enforcement agencies to
by P/SUPT. Latag. The Whereas clauses of the Memorandum alluded to the extraordinary situation in suppress disorder and lawless violence.
Sulu, and the willingness of civilian supporters of the municipal mayors to offer their services in order
7
that "the early and safe rescue of the hostages may be achieved."
In the same Proclamation, respondent Tan called upon the PNP and the CEF to set up checkpoints and lack or excess of jurisdiction, as it threatened fundamental freedoms guaranteed under Article III of the
chokepoints, conduct general search and seizures including arrests, and other actions necessary to 1987 Constitution.
ensure public safety. The pertinent portion of the proclamation states:
Petitioners contend that Proclamation No. 1 and its Implementing Guidelines were issued ultra vires,
NOW, THEREFORE, BY VIRTUE OF THE POWERS VESTED IN ME BY LAW, I, ABDUSAKUR and thus null and void, for violating Sections 1 and 18, Article VII of the Constitution, which grants the
MAHAIL TAN, GOVERNOR OF THE PROVINCE OF SULU, DO HEREBY DECLARE A STATE OF President sole authority to exercise emergency powers and calling-out powers as the chief executive of
20
EMERGENCY IN THE PROVINCE OF SULU, AND CALL ON THE PHILIPPINE NATIONAL POLICE the Republic and commander-in-chief of the armed forces. Additionally, petitioners claim that the
WITH THE ASSISTANCE OF THE ARMED FORCES OF THE PHILIPPINES AND THE CIVILIAN Provincial Governor is not authorized by any law to create civilian armed forces under his command,
EMERGENCY FORCE TO IMPLEMENT THE FOLLOWING: nor regulate and limit the issuances of PTCFORs to his own private army.

1. The setting-up of checkpoints and chokepoints in the province; In his Comment, Governor Tan contended that petitioners violated the doctrine on hierarchy of courts
when they filed the instant petition directly in the court of last resort, even if both the Court of Appeals
2. The imposition of curfew for the entire province subject to such Guidelines as may be issued by (CA) and the Regional Trial Courts (RTC) possessed concurrent jurisdiction with the
proper authorities;
21
Supreme Court under Rule 65. This is the only procedural defense raised by respondent Tan.
3. The conduct of General Search and Seizure including arrests in the pursuit of the kidnappers and Respondents Gen. Juancho Saban, Col. Eugenio Clemen, P/SUPT. Julasirim Kasim, and P/SUPT.
their supporters; and Bienvenido Latag did not file their respective Comments.1âwphi1

4. To conduct such other actions or police operations as may be necessary to ensure public safety. On the substantive issues, respondents deny that Proclamation 1-09 was issued ultra vires, as
Governor Tan allegedly acted pursuant to Sections 16 and 465 of the Local Government Code, which
DONE AT THE PROVINCIAL CAPITOL, PROVINCE OF SULU THIS empowers the Provincial Governor to carry out emergency measures during calamities and disasters,
14 and to call upon the appropriate national law enforcement agencies to suppress disorder, riot, lawless
31STDAY OF MARCH 2009. Sgd. Abdusakur M. Tan Governor. 22
violence, rebellion or sedition. Furthermore, the Sangguniang Panlalawigan of Sulu authorized the
On 1 April 2009, SPO1 Sattal Jadjuli was instructed by his superior to report to respondent P/SUPT. declaration of a state of emergency as evidenced by Resolution No. 4, Series of 2009 issued on 31
23
15
Julasirim Kasim. Upon arriving at the police station, he was booked, and interviewed about his March 2009 during its regular session.
relationship to Musin, Jaiton, and Julamin, who were all his deceased relatives. Upon admitting that he
The threshold issue in the present case is whether or not Section 465, in relation to Section 16, of the
was indeed related to the three, he was detained. After a few hours, former Punong Barangay Juljahan
Local Government Code authorizes the respondent governor to declare a state of emergency, and
Awadi, Hadji Hadjirul Bambra, Abdugajir Hadjirul, as well as PO2 Marcial Hajan, SPO3 Muhilmi Ismula,
16 exercise the powers enumerated under Proclamation 1-09, specifically the conduct of general searches
Punong Barangay Alano Mohammad and jeepney driver Abduhadi Sabdani, were also arrested. The
17 and seizures. Subsumed herein is the secondary question of whether or not the provincial governor is
affidavit of the apprehending officer alleged that they were suspected ASG supporters and were being
similarly clothed with authority to convene the CEF under the said provisions.
arrested under Proclamation 1-09. The following day, 2 April 2009, the hostage Mary Jane Lacaba was
released by the ASG. We grant the petition.

On 4 April 2009, the office of Governor Tan distributed to civic organizations, copies of the "Guidelines I. Transcendental public Importance warrants a relaxation of the Doctrine of Hierarchy of Courts
for the Implementation of Proclamation No. 1, Series of 2009 Declaring a State of Emergency in the
18
Province of Sulu." These Guidelines suspended all Permits to Carry We first dispose of respondents’ invocation of the doctrine of hierarchy of courts which allegedly
prevents judicial review by this Court in the present case, citing for this specific purpose, Montes v.
Firearms Outside of Residence (PTCFORs) issued by the Chief of the PNP, and allowed civilians to 24
Court of Appeals and Purok Bagong Silang Association, Inc. v. Yuipco. Simply put, the
seek exemption from the gun ban only by applying to the Office of the Governor and obtaining the
appropriate identification cards. The said guidelines also allowed general searches and seizures in doctrine provides that where the issuance of an extraordinary writ is also within the competence of the
designated checkpoints and chokepoints. CA or the RTC, it is in either of these courts and not in the Supreme Court, that the specific action for
the issuance of such writ must be sought unless special and important laws are clearly and specifically
On 16 April 2009, Jamar M. Kulayan, Temogen S. Tulawie, Hadji Mohammad Yusop Ismi, Ahajan set forth in the petition. The reason for this is that this Court is a court of last resort and must so remain
Awadi, and SPO1 Sattal H. Jadjuli, residents of Patikul, Sulu, filed the present Petition for Certiorari and if it is to perform the functions assigned to it by the Constitution and immemorial tradition. It cannot be
19
Prohibition, claiming that Proclamation 1-09 was issued with grave abuse of discretion amounting to burdened with deciding cases in the first instance.
25
26
The said rule, however, is not without exception. In Chavez v. PEA-Amari, the Court stated: Evidently, the triple reasons We advanced at the start of Our ruling are justified under the foregoing
exceptions. Every bad, unusual incident where police officers figure in generates public interest and
PEA and AMARI claim petitioner ignored the judicial hierarchy by seeking relief directly from the Court. people watch what will be done or not done to them. Lack of disciplinary steps taken against them
The principle of hierarchy of courts applies generally to cases involving factual questions. As it is not a erode public confidence in the police institution. As petitioners themselves assert, the restrictive custody
trier of facts, the Court cannot entertain cases involving factual issues. The instant case, however, of policemen under investigation is an existing practice, hence, the issue is bound to crop up every now
raises constitutional questions of transcendental importance to the public. The Court can resolve this and then. The matter is capable of repetition or susceptible of recurrence. It better be resolved now for
case without determining any factual issue related to the case. Also, the instant case is a petition for 31
the education and guidance of all concerned. (Emphasis supplied)
mandamus which falls under the original jurisdiction of the Court under Section 5, Article VIII of the
27
Constitution. We resolve to exercise primary jurisdiction over the instant case. Hence, the instant petition is given due course, impressed as it is with transcendental public
importance.
The instant case stems from a petition for certiorari and prohibition, over which the Supreme Court
28
possesses original jurisdiction. More crucially, this case involves acts of a public official which pertain II. Only the President is vested with calling-out powers, as the commander-in-chief of the Republic
to restrictive custody, and is thus impressed with transcendental public importance that would warrant
the relaxation of the general rule. The Court would be remiss in its constitutional duties were it to i. One executive, one commander-in-chief
dismiss the present petition solely due to claims of judicial hierarchy. 32
As early as Villena v. Secretary of Interior, it has already been established that there is one repository
29 of executive powers, and that is the President of the Republic. This means that when Section 1, Article
In David v. Macapagal-Arroyo, the Court highlighted the transcendental public importance involved in
33
cases that concern restrictive custody, because judicial review in these cases serves as "a VII of the Constitution speaks of executive power, it is granted to the President and no one else. As
manifestation of the crucial defense of civilians ‘in police power’ cases due to the diminution of their emphasized by Justice Jose P. Laurel, in his ponencia in Villena:
30
basic liberties under the guise of a state of emergency." Otherwise, the importance of the high tribunal
as the court of last resort would be put to naught, considering the nature of "emergency" cases, wherein With reference to the Executive Department of the government, there is one purpose which is crystal-
the proclamations and issuances are inherently short-lived. In finally disposing of the claim that the clear and is readily visible without the projection of judicial searchlight, and that is the establishment of a
issue had become moot and academic, the Court also cited transcendental public importance as an single, not plural, Executive. The first section of Article VII of the Constitution, dealing with the Executive
exception, stating: Department, begins with the enunciation of the principle that "The executive power shall be vested in a
President of the Philippines." This means that the President of the Philippines is the Executive of the
34
Sa kabila ng pagiging akademiko na lamang ng mga isyu tungkol sa mahigpit na pangangalaga Government of the Philippines, and no other.
(restrictive custody) at pagmonitor ng galaw (monitoring of movements) ng nagpepetisyon,
dedesisyunan namin ito (a) dahil sa nangingibabaw na interes ng madla na nakapaloob dito, Corollarily, it is only the President, as Executive, who is authorized to exercise emergency powers as
provided under Section 23, Article VI, of the Constitution, as well as what became known as the calling-
(b) dahil sa posibilidad na maaaring maulit ang pangyayari at (c) dahil kailangang maturuan ang out powers under Section 7, Article VII thereof.
kapulisan tungkol dito.
ii. The exceptional character of Commander-in-Chief powers dictate that they are exercised by one
The moot and academic principle is not a magical formula that can automatically dissuade the courts in president
resolving a case. Courts will decide cases, otherwise moot and academic, if: first, there is a grave
violation of the Constitution; second, the exceptional character of the situation and the paramount public Springing from the well-entrenched constitutional precept of One President is the notion that there are
interest is involved; third, when [the] constitutional issue raised requires formulation of controlling certain acts which, by their very nature, may only be performed by the president as the Head of the
principles to guide the bench, the bar, and the public; and fourth, the case is capable of repetition yet State. One of these acts or prerogatives is the bundle of Commander-in-Chief powers to which the
evading review. "calling-out" powers constitutes a portion. The President’s Emergency Powers, on the other hand, is
balanced only by the legislative act of Congress, as embodied in the second paragraph of Section 23,
…There is no question that the issues being raised affect the public interest, involving as they do the Article 6 of the Constitution:
people’s basic rights to freedom of expression, of assembly and of the press. Moreover, the
Article 6, Sec 23(2). In times of war or other national emergency, the Congress may, by law, authorize
Court has the duty to formulate guiding and controlling constitutional precepts, doctrines or rules. It has the President, for a limited period and subject to such restrictions as it may prescribe, to exercise
the symbolic function of educating the bench and the bar, and in the present petitions, the military and powers necessary and proper to carry out a declared national policy. Unless sooner withdrawn by
35
the police, on the extent of the protection given by constitutional guarantees. And lastly, respondents resolution of the Congress, such powers shall cease upon the next adjournment thereof.
contested actions are capable of repetition. Certainly, the petitions are subject to judicial review.
Article 7, Sec 18. The President shall be the Commander-in-Chief of all armed forces of the Philippines There is a clear textual commitment under the Constitution to bestow on the President full discretionary
and whenever it becomes necessary, he may call out such armed forces to prevent or suppress lawless power to call out the armed forces and to determine the necessity for the exercise of such
43
violence, invasion or rebellion. In case of invasion or rebellion, when the public safety requires it, he power. (Emphasis supplied)
may, for a period not exceeding sixty days, suspend the privilege of the writ of habeas corpus or place
the Philippines or any part thereof under martial law. Within forty-eight hours from the proclamation of Under the foregoing provisions, Congress may revoke such proclamation or suspension and the Court
martial law or the suspension of the privilege of the writ of habeas corpus, the President shall submit a may review the sufficiency of the factual basis thereof. However, there is no such equivalent provision
report in person or in writing to the Congress. The Congress, voting jointly, by a vote of at least a dealing with the revocation or review of the President’s action to call out the armed forces. The
majority of all its Members in regular or special session, may revoke such proclamation or suspension, distinction places the calling out power in a different category from the power to declare martial law and
which revocation shall not be set aside by the President. Upon the initiative of the President, the the power to suspend the privilege of the writ of habeas corpus, otherwise, the framers of the
Congress may, in the same manner, extend such proclamation or suspension for a period to be Constitution would have simply lumped together the three powers and provided for their revocation and
44
determined by the Congress, if the invasion or rebellion shall persist and public safety requires it. review without any qualification.

The Congress, if not in session, shall, within twenty-four hours following such proclamation or That the power to call upon the armed forces is discretionary on the president is clear from the
36 deliberation of the Constitutional Commission:
suspension, convene in accordance with its rules without need of a call.

The power to declare a state of martial law is subject to the Supreme Court’s authority to review the FR. BERNAS. It will not make any difference. I may add that there is a graduated power of the
37 President as Commander-in-Chief. First, he can call out such Armed Forces as may be necessary to
factual basis thereof. By constitutional fiat, the calling-out powers, which is of lesser gravity than the
power to declare martial law, is bestowed upon the President alone. As noted in Villena, "(t)here are suppress lawless violence; then he can suspend the privilege of the writ of habeas corpus, then he can
certain constitutional powers and prerogatives of the Chief Executive of the Nation which must be impose martial law. This is a graduated sequence.
exercised by him in person and no amount of approval or ratification will validate the exercise of any of
those powers by any other person. Such, for instance, is his power to suspend the writ of habeas When he judges that it is necessary to impose martial law or suspend the privilege of the writ of habeas
corpus and proclaim martial law x x x.
38 corpus, his judgment is subject to review. We are making it subject to review by the Supreme Court and
subject to concurrence by the National Assembly. But when he exercises this lesser power of calling on
39 the Armed Forces, when he says it is necessary, it is my opinion that his judgment cannot be reviewed
Indeed, while the President is still a civilian, Article II, Section 3 of the Constitution mandates that
civilian authority is, at all times, supreme over the military, making the civilian president the nation’s by anybody.
supreme military leader. The net effect of Article II, Section 3, when read with Article VII,
xxx xxx xxx
Section 18, is that a civilian President is the ceremonial, legal and administrative head of the armed
forces. The Constitution does not require that the President must be possessed of military training and MR. REGALADO. That does not require any concurrence by the legislature nor is it subject to judicial
talents, but as Commander-in-Chief, he has the power to direct military operations and to determine review.
military strategy. Normally, he would be expected to delegate the actual command of the armed forces
40 The reason for the difference in the treatment of the aforementioned powers highlights the intent to
to military experts; but the ultimate power is his. As Commander-in-Chief, he is authorized to direct the
grant the President the widest leeway and broadest discretion in using the power to call out because it
movements of the naval and military forces placed by law at his command, and to employ them in the
41 is considered as the lesser and more benign power compared to the power to suspend the privilege of
manner he may deem most effectual.
the writ of habeas corpus and the power to impose martial law, both of which involve the curtailment
42
In the case of Integrated Bar of the Philippines v. Zamora, the Court had occasion to rule that the and suppression of certain basic civil rights and individual freedoms, and thus necessitating safeguards
calling-out powers belong solely to the President as commander-in-chief: by Congress and review by this Court.

When the President calls the armed forces to prevent or suppress lawless violence, invasion or x x x Thus, it is the unclouded intent of the Constitution to vest upon the President, as Commander-in-
rebellion, he necessarily exercises a discretionary power solely vested in his wisdom. This is clear from Chief of the Armed Forces, full discretion to call forth the military when in his judgment it is necessary to
45
the intent of the framers and from the text of the Constitution itself. The Court, thus, cannot be called do so in order to prevent or suppress lawless violence, invasion or rebellion. (Emphasis Supplied)
upon to overrule the President’s wisdom or substitute its own. However, this does not prevent an 46
In the more recent case of Constantino, Jr. v. Cuisia, the Court characterized these powers as
examination of whether such power was exercised within permissible constitutional limits or whether it
exclusive to the President, precisely because they are of exceptional import:
was exercised in a manner constituting grave abuse of discretion. In view of the constitutional intent to
give the President full discretionary power to determine the necessity of calling out the armed forces, it These distinctions hold true to this day as they remain embodied in our fundamental law. There are
is incumbent upon the petitioner to show that the President’s decision is totally bereft of factual basis. certain presidential powers which arise out of exceptional circumstances, and if exercised, would
involve the suspension of fundamental freedoms, or at least call for the supersedence of executive xxx xxx xxx
prerogatives over those exercised by co-equal branches of government. The declaration of martial law,
the suspension of the writ of habeas corpus, and the exercise of the pardoning power, notwithstanding Mr. de los Reyes: But in exceptional cases, even the operational control can be taken over by the
the judicial determination of guilt of the accused, all fall within this special class that demands the National Police Commission?
exclusive exercise by the President of the constitutionally vested power. The list is by no means 51
exclusive, but there must be a showing that the executive power in question is of similar gravitas and Mr. Natividad: If the situation is beyond the capacity of the local governments. (Emphases supplied)
47
exceptional import.
Furthermore according to the framers, it is still the President who is authorized to exercise supervision
In addition to being the commander-in-chief of the armed forces, the President also acts as the leader of and control over the police, through the National Police Commission:
the country’s police forces, under the mandate of Section 17, Article VII of the Constitution, which
Mr. Rodrigo: Just a few questions. The President of the Philippines is the Commander-in-Chief of all the
provides that, "The President shall have control of all the executive departments, bureaus, and offices.
armed forces.
He shall ensure that the laws be faithfully executed." During the deliberations of the Constitutional
Commission on the framing of this provision, Fr. Bernas defended the retention of the word "control," Mr. Natividad: Yes, Madam President.
employing the same rationale of singularity of the office of the president, as the only Executive under
48
the presidential form of government. Mr. Rodrigo: Since the national police is not integrated with the armed forces, I do not suppose they
come under the Commander-in-Chief powers of the President of the Philippines.
Regarding the country’s police force, Section 6, Article XVI of the Constitution states that: "The State
shall establish and maintain one police force, which shall be national in scope and civilian in character, Mr. Natividad: They do, Madam President. By law, they are under the supervision and control of the
to be administered and controlled by a national police commission. The authority of local executives President of the Philippines.
49
over the police units in their jurisdiction shall be provided by law."
Mr. Rodrigo: Yes, but the President is not the Commander-in-Chief of the national police.
A local chief executive, such as the provincial governor, exercises operational supervision over the
50
police, and may exercise control only in day-to-day operations, viz: Mr. Natividad: He is the President.

Mr. Natividad: By experience, it is not advisable to provide either in our Constitution or by law full control Mr. Rodrigo: Yes, the Executive. But they do not come under that specific provision that the President is
of the police by the local chief executive and local executives, the mayors. By our experience, this has the Commander-in-Chief of all the armed forces.
spawned warlordism, bossism and sanctuaries for vices and abuses. If the national government does
not have a mechanism to supervise these 1,500 legally, technically separate police forces, plus 61 city Mr. Natividad: No, not under the Commander-in-Chief provision.
police forces, fragmented police system, we will have a lot of difficulty in presenting a modern
professional police force. So that a certain amount of supervision and control will have to be exercised Mr. Rodrigo: There are two other powers of the President. The
by the national government.
President has control over ministries, bureaus and offices, and supervision over local governments.
For example, if a local government, a town cannot handle its peace and order problems or police Under which does the police fall, under control or under supervision?
problems, such as riots, conflagrations or organized crime, the national government may come in,
Mr. Natividad: Both, Madam President.
especially if requested by the local executives. Under that situation, if they come in under such an
extraordinary situation, they will be in control. But if the day-to-day business of police investigation of
Mr. Rodrigo: Control and supervision.
crime, crime prevention, activities, traffic control, is all lodged in the mayors, and if they are in complete
operational control of the day-to-day business of police service, what the national government would Mr. Natividad: Yes, in fact, the National Police Commission is under the Office of the President.
52

control would be the administrative aspect.


In the discussions of the Constitutional Commission regarding the above provision it is clear that the
xxx xxx xxx framers never intended for local chief executives to exercise unbridled control over the police in
emergency situations. This is without prejudice to their authority over police units in their jurisdiction as
Mr. de los Reyes: so the operational control on a day-to-day basis, meaning, the usual duties being
provided by law, and their prerogative to seek assistance from the police in day to day situations, as
performed by the ordinary policemen, will be under the supervision of the local executives?
contemplated by the Constitutional Commission. But as a civilian agency of the government, the police,
through the NAPOLCOM, properly comes within, and is subject to, the exercise by the President of the
Mr. Natividad: Yes, Madam President. 53
power of executive control.
iii. The provincial governor does not possess the same calling-out powers as the President the Local Government Code nor in any law on which the broad and unwarranted powers granted to the
Governor may be based.
Given the foregoing, respondent provincial governor is not endowed with the power to call upon the
armed forces at his own bidding. In issuing the assailed proclamation, Governor Tan exceeded his Petitioners cite the implementation of "General Search and Seizure including arrests in the pursuit of the
57
authority when he declared a state of emergency and called upon the Armed Forces, the police, and his kidnappers and their supporters," as being violative of the constitutional proscription on general search
own Civilian Emergency Force. The calling-out powers contemplated under the Constitution is exclusive warrants and general seizures. Petitioners rightly assert that this alone would be sufficient to render the
to the President. An exercise by another official, even if he is the local chief executive, is ultra vires, and proclamation void, as general searches and seizures are proscribed, for being violative of the rights
may not be justified by the invocation of Section 465 of the Local Government Code, as will be enshrined in the Bill of Rights, particularly:
discussed subsequently.
The right of the people to be secure in their persons, houses, papers, and effects against unreasonable
Respondents, however, justify this stance by stating that nowhere in the seminal case of David v. searches and seizures of whatever nature and for any purpose shall be inviolable, and no search
Arroyo, which dealt squarely with the issue of the declaration of a state of emergency, does it limit the warrant or warrant of arrest shall issue except upon probable cause to be determined personally by the
said authority to the President alone. Respondents contend that the ruling in David expressly limits the judge after examination under oath or affirmation of the complainant and the witnesses he may
58
authority to declare a national emergency, a condition which covers the entire country, and does not produce, and particularly describing the place to be searched and the persons or things to be seized.
54
include emergency situations in local government units. This claim is belied by the clear intent of the
framers that in all situations involving threats to security, such as lawless violence, invasion or rebellion, In fact, respondent governor has arrogated unto himself powers exceeding even the martial law powers
even in localized areas, it is still the President who possesses the sole authority to exercise calling-out of the President, because as the Constitution itself declares, "A state of martial law does not suspend
powers. As reflected in the Journal of the Constitutional Commission: the operation of the Constitution, nor supplant the functioning of the civil courts or legislative
assemblies, nor authorize the conferment of the jurisdiction on military courts and agencies over
59
Thereafter, Mr. Padilla proposed on line 29 to insert the phrase OR PUBLIC DISORDER in lieu of civilians where civil courts are able to function, nor automatically suspend the privilege of the writ."
"invasion or rebellion." Mr. Sumulong stated that the committee could not accept the amendment
because under the first section of Section 15, the President may call out and make use of the armed We find, and so hold, that there is nothing in the Local Government Code which justifies the acts
forces to prevent or suppress not only lawless violence but even invasion or rebellion without declaring sanctioned under the said Proclamation. Not even Section 465 of the said Code, in relation to Section
martial law. He observed that by deleting "invasion or rebellion" and substituting PUBLIC DISORDER, 16, which states:
the President would have to declare martial law before he can make use of the armed forces to prevent
or suppress lawless invasion or rebellion. Section 465. The Chief Executive: Powers, Duties, Functions, and Compensation.

Mr. Padilla, in reply thereto, stated that the first sentence contemplates a lighter situation where there is xxx xxx xxx
some lawless violence in a small portion of the country or public disorder in another at which times, the
(b) For efficient, effective and economical governance the purpose of which is the general welfare of the
armed forces can be called to prevent or suppress these incidents. He noted that the Commander-in-
province and its inhabitants pursuant to Section 16 of this Code, the provincial governor shall:
Chief can do so in a minor degree but he can also exercise such powers should the situation worsen.
The words "invasion or rebellion" to be eliminated on line 14 are covered by the following sentence (1) Exercise general supervision and control over all programs, projects, services, and activities of the
which provides for "invasion or rebellion." He maintained that the proposed amendment does not mean provincial government, and in this connection, shall:
that under such circumstances, the President cannot call on the armed forces to prevent or suppress
55
the same. (Emphasis supplied) xxx xxx xxx

III. Section 465 of the Local (vii) Carry out such emergency measures as may be necessary during and in the aftermath of man-
made and natural disasters and calamities;
Government Code cannot be invoked to justify the powers enumerated under Proclamation 1-09
(2) Enforce all laws and ordinances relative to the governance of the province and the exercise of the
Respondent governor characterized the kidnapping of the three ICRC workers as a terroristic act, and appropriate corporate powers provided for under Section 22 of this Code, implement all approved
56
used this incident to justify the exercise of the powers enumerated under Proclamation 1-09. He policies, programs, projects, services and activities of the province and, in addition to the foregoing,
invokes Section 465, in relation to Section 16, of the Local Government Code, which purportedly allows shall:
the governor to carry out emergency measures and call upon the appropriate national law enforcement
agencies for assistance. But a closer look at the said proclamation shows that there is no provision in xxx xxx xxx
(vi) Call upon the appropriate national law enforcement agencies to suppress disorder, riot, lawless Section 5, Article II of the 1973 Constitution, which allowed LGUs to create their own sources of
62
violence, rebellion or sedition or to apprehend violators of the law when public interest so requires and revenue. During the interpellation made by Mr. Tirol addressed to Mr. de Pedro, the latter emphasized
the police forces of the component city or municipality where the disorder or violation is happening are that "Decentralization is an administrative concept and the process of shifting and delegating power
inadequate to cope with the situation or the violators. from a central point to subordinate levels to promote independence, responsibility, and quicker decision-
making. … (I)t does not involve any transfer of final authority from the national to field levels, nor
Section 16. General Welfare. - Every local government unit shall exercise the powers expressly diminution of central office powers and responsibilities. Certain government agencies, including the
granted, those necessarily implied therefrom, as well as powers necessary, appropriate, or incidental for police force, are exempted from the decentralization process because their functions are not inherent in
its efficient and effective governance, and those which are essential to the promotion of the general 63
local government units."
welfare. Within their respective territorial jurisdictions, local government units shall ensure and support,
among other things, the preservation and enrichment of culture, promote health and safety, enhance IV. Provincial governor is not authorized to convene CEF
the right of the people to a balanced ecology, encourage and support the development of appropriate
and self-reliant scientific and technological capabilities, improve public morals, enhance economic Pursuant to the national policy to establish one police force, the organization of private citizen armies is
prosperity and social justice, promote full employment among their residents, maintain peace and order, proscribed. Section 24 of Article XVIII of the Constitution mandates that:
and preserve the comfort and convenience of their inhabitants. (Emphases supplied)
Private armies and other armed groups not recognized by duly constituted authority shall be dismantled.
Respondents cannot rely on paragraph 1, subparagraph (vii) of Article 465 above, as the said provision All paramilitary forces including Civilian Home Defense Forces (CHDF) not consistent with the citizen
expressly refers to calamities and disasters, whether man-made or natural. The governor, as local chief armed force established in this Constitution, shall be dissolved or, where appropriate, converted into the
executive of the province, is certainly empowered to enact and implement emergency measures during regular force.
these occurrences. But the kidnapping incident in the case at bar cannot be considered as a calamity or
a disaster. Respondents cannot find any legal mooring under this provision to justify their actions. Additionally, Section 21of Article XI states that, "The preservation of peace and order within the regions
shall be the responsibility of the local police agencies which shall be organized, maintained, supervised,
Paragraph 2, subparagraph (vi) of the same provision is equally inapplicable for two reasons. First, the and utilized in accordance with applicable laws. The defense and security of the regions shall be the
Armed Forces of the Philippines does not fall under the category of a "national law enforcement responsibility of the National Government."
agency," to which the National Police Commission (NAPOLCOM) and its departments belong.
Taken in conjunction with each other, it becomes clear that the Constitution does not authorize the
Its mandate is to uphold the sovereignty of the Philippines, support the Constitution, and defend the organization of private armed groups similar to the CEF convened by the respondent Governor. The
Republic against all enemies, foreign and domestic. Its aim is also to secure the integrity of the national framers of the Constitution were themselves wary of armed citizens’ groups, as shown in the following
60 proceedings:
territory.

Second, there was no evidence or even an allegation on record that the local police forces were MR. GARCIA: I think it is very clear that the problem we have here is a paramilitary force operating
inadequate to cope with the situation or apprehend the violators. If they were inadequate, the recourse under the cloak, under the mantle of legality is creating a lot of problems precisely by being able to
of the provincial governor was to ask the assistance of the Secretary of Interior and Local Government, operate as an independent private army for many regional warlords. And at the same time, this I think
or such other authorized officials, for the assistance of national law enforcement agencies. has been the thrust, the intent of many of the discussions and objections to the paramilitary units and
the armed groups.
The Local Government Code does not involve the diminution of central powers inherently vested in the
National Government, especially not the prerogatives solely granted by the Constitution to the President MR. PADILLA: My proposal covers two parts: the private armies of political warlords and other armed
in matters of security and defense. torces not recognized by constituted authority which shall be dismantled and dissolved. In my trips to
the provinces, I heard of many abuses committed by the CHDF (Civilian Home Defense Forces),
The intent behind the powers granted to local government units is fiscal, economic, and administrative specially in Escalante, Negros Occidental. But I do not know whether a particular CHDF is approved or
in nature.1âwphi1The Code is concerned only with powers that would make the delivery of basic authorized by competent authority. If it is not authorized, then the CHDF will have to be dismantled. If
61
services more effective to the constituents, and should not be unduly stretched to confer calling-out some CHDFs, say in other provinces, are authorized by constituted authority, by the Armed Forces of
powers on local executives. the Philippines, through the Chief of Staff or the Minister of National Defense, if they are recognized and
authorized, then they will not be dismantled. But I cannot give a categorical answer to any specific
In the sponsorship remarks for Republic Act 7160, it was stated that the devolution of powers is a step CHDF unit, only the principle that if they are armed forces which are not authorized, then they should be
towards the autonomy of local government units (LGUs), and is actually an experiment whose success 64
dismantled. (Emphasis supplied)
heavily relies on the power of taxation of the LGUs. The underpinnings of the Code can be found in
Thus, with the discussions in the Constitutional Commission as guide, the creation of the Civilian
Emergency Force (CEF) in the present case, is also invalid.

WHEREFORE, the instant petition is GRANTED. Judgment is rendered commanding respondents to


desist from further proceedings m implementing Proclamation No. 1, Series of 2009, and its
Implementing Guidelines. The said proclamation and guidelines are hereby declared NULL and VOID
for having been issued in grave abuse of discretion, amounting to lack or excess of jurisdiction.

SO ORDERED.

MARIA LOURDES P. A. SERENO


Associate Justice
addressed to the satisfaction of petitioners, the Court recognizes these values as of paramount
importance to our civil society, even if not determinative of the resolution of this petition. Had the
relevant issue before us been the right of the Senate to compel the testimony of petitioners, the
constitutional questions raised by them would have come to fore. Such a scenario could have very well
been presented to the Court in such manner, without the petitioners having had to violate a direct order
from their commanding officer. Instead, the Court has to resolve whether petitioners may be subjected
to military discipline on account of their defiance of a direct order of the AFP Chief of Staff.

The solicited writs of certiorari and prohibition do not avail; the petition must be denied.

I.
EN BANC
The petitioners are high-ranking officers of the Armed Forces of the Philippines (AFP). Both petitioners,
G.R. No. 170165 August 15, 2006 Brigadier General Francisco Gudani (Gen. Gudani) and Lieutenant Colonel Alexander Balutan (Col.
Balutan), belonged to the Philippine Marines. At the time of the subject incidents, both Gen. Gudani and
B/GEN. (RET.) FRANCISCO V. GUDANI AND LT. COL. ALEXANDER F. BALUTAN Petitioners, Col. Balutan were assigned to the Philippine Military Academy (PMA) in Baguio City, the former as the
2
vs. PMA Assistant Superintendent, and the latter as the Assistant Commandant of Cadets.
LT./GEN. GENEROSO S. SENGA CORONA, AS CHIEF OF STAFF OF THE CARPIO-MORALES,
ARMED FORCES OF THE CALLEJO, SR., PHILIPPINES, COL. GILBERTO AZCUNA, JOSE C. ROA On 22 September 2005, Senator Rodolfo Biazon (Sen. Biazon) invited several senior officers of the
AS THE PRE-TRIAL TINGA, INVESTIGATING OFFICER, THE CHICO-NAZARIO, PROVOST AFP to appear at a public hearing before the Senate Committee on National Defense and Security
MARSHALL GENERAL GARCIA, and OF THE ARMED FORCES OF THE PHILIPPINES AND THE (Senate Committee) scheduled on 28 September 2005. The hearing was scheduled after topics
GENERAL COURT-MARTIAL, Respondents. concerning the conduct of the 2004 elections emerged in the public eye, particularly allegations of
massive cheating and the surfacing of copies of an audio excerpt purportedly of a phone conversation
DECISION between President Gloria Macapagal Arroyo and an official of the Commission on Elections
(COMELEC) widely reputed as then COMELEC Commissioner Virgilio Garcillano. At the time of the
TINGA, J.: 2004 elections, Gen. Gudani had been designated as commander, and Col. Balutan a member, of
"Joint Task Force Ranao" by the AFP Southern Command. "Joint Task Force Ranao" was tasked with
A most dangerous general proposition is foisted on the Court — that soldiers who defy orders of their the maintenance of peace and order during the 2004 elections in the provinces of Lanao del Norte and
superior officers are exempt 3
Lanao del Sur. `

from the strictures of military law and discipline if such defiance is predicated on an act otherwise valid Gen. Gudani, Col. Balutan, and AFP Chief of Staff Lieutenant General Generoso Senga (Gen. Senga)
under civilian law. Obedience and deference to the military chain of command and the President as were among the several AFP officers who received a letter invitation from Sen. Biazon to attend the 28
commander-in-chief are the cornerstones of a professional military in the firm cusp of civilian control. September 2005 hearing. On 23 September 2005, Gen. Senga replied through a letter to Sen. Biazon
These values of obedience and deference expected of military officers are content-neutral, beyond the that he would be unable to attend the hearing due to a previous commitment in Brunei, but he
sway of the officer’s own sense of what is prudent or rash, or more elementally, of right or wrong. A self- nonetheless "directed other officers from the AFP who were invited to attend the hearing."
4

righteous military invites itself as the scoundrel’s activist solution to the "ills" of participatory democracy.
On 26 September 2005, the Office of the Chief of Staff of the AFP issued a Memorandum addressed to
1
Petitioners seek the annulment of a directive from President Gloria Macapagal-Arroyo enjoining them the Superintendent of the PMA Gen. Cristolito P. Baloing (Gen. Baloing). It was signed by Lt. Col.
and other military officers from testifying before Congress without the President’s consent. Petitioners 5
Hernando DCA Iriberri in behalf of Gen. Senga. Noting that Gen. Gudani and Col. Balutan had been
also pray for injunctive relief against a pending preliminary investigation against them, in preparation for invited to attend the Senate Committee hearing on 28 September 2005, the Memorandum directed the
possible court-martial proceedings, initiated within the military justice system in connection with 6
two officers to attend the hearing. Conformably, Gen. Gudani and Col. Balutan filed their respective
petitioners’ violation of the aforementioned directive. requests for travel authority addressed to the PMA Superintendent.

The Court is cognizant that petitioners, in their defense, invoke weighty constitutional principles that On 27 September 2005, Gen. Senga wrote a letter to Sen. Biazon, requesting the postponement of the
center on fundamental freedoms enshrined in the Bill of Rights. Although these concerns will not be hearing scheduled for the following day, since the AFP Chief of Staff was himself unable to attend said
hearing, and that some of the invited officers also could not attend as they were "attending to other
12
urgent operational matters." By this time, both Gen. Gudani and Col. Balutan had already departed Galarpe, both petitioners invoked their right to remain silent. The following day, Gen. Gudani was
13
Baguio for Manila to attend the hearing. compulsorily retired from military service, having reached the age of 56.

Then on the evening of 27 September 2005, at around 10:10 p.m., a message was transmitted to the In an Investigation Report dated 6 October 2005, the OPMG recommended that petitioners be charged
PMA Superintendent from the office of Gen. Senga, stating as follows: with violation of Article of War 65, on willfully disobeying a superior officer, in relation to Article of War
14
97, on conduct prejudicial to the good order and military discipline. As recommended, the case was
PER INSTRUCTION OF HER EXCELLENCY PGMA, NO AFP PERSONNEL SHALL APPEAR referred to a Pre-Trial Investigation Officer (PTIO) preparatory to trial by the General Court Martial
BEFORE ANY CONGRESSIONAL OR SENATE HEARING WITHOUT HER APPROVAL. INFORM 15
(GCM). Consequently, on 24 October 2005, petitioners were separately served with Orders
7
BGEN FRANCISCO F GUDANI AFP AND LTC ALEXANDER BALUTAN PA (GSC) ACCORDINGLY. respectively addressed to them and signed by respondent Col. Gilbert Jose C. Roa, the Pre-Trial
Investigating Officer of the PTIO. The Orders directed petitioners to appear in person before Col. Roa at
The following day, Gen. Senga sent another letter to Sen. Biazon, this time informing the senator that 16 17
the Pre-Trial Investigation of the Charges for violation of Articles 65 and 97 of Commonwealth Act
"no approval has been granted by the President to any AFP officer to appear" before the hearing 18
No. 408, and to submit their counter-affidavits and affidavits of witnesses at the Office of the Judge
scheduled on that day. Nonetheless, both Gen. Gudani and Col. Balutan were present as the hearing 19
Advocate General. The Orders were accompanied by respective charge sheets against petitioners,
started, and they both testified as to the conduct of the 2004 elections. accusing them of violating Articles of War 65 and 97.

The Office of the Solicitor General (OSG), representing the respondents before this Court, has offered It was from these premises that the present petition for certiorari and prohibition was filed, particularly
additional information surrounding the testimony of Gen. Gudani and Col. Balutan. The OSG manifests seeking that (1) the order of President Arroyo coursed through Gen. Senga preventing petitioners from
that the couriers of the AFP Command Center had attempted to deliver the radio message to Gen. testifying before Congress without her prior approval be declared unconstitutional; (2) the charges
Gudani’s residence in a subdivision in Parañaque City late in the night of 27 September 2005, but they stated in the charge sheets against petitioners be quashed; and (3) Gen. Senga, Col. Galarpe, Col.
were not permitted entry by the subdivision guards. The next day, 28 September 2005, shortly before Roa, and their successors-in-interest or persons acting for and on their behalf or orders, be permanently
the start of the hearing, a copy of Gen. Senga’s letter to Sen. Biazon sent earlier that day was handed enjoined from proceeding against petitioners, as a consequence of their having testified before the
at the Senate by Commodore Amable B. Tolentino of the AFP Office for Legislative Affairs to Gen. Senate on 28 September 2005.
20

Gudani, who replied that he already had a copy. Further, Gen. Senga called Commodore Tolentino on
the latter’s cell phone and asked to talk to Gen. Gudani, but Gen. Gudani refused. In response, Gen. Petitioners characterize the directive from President Arroyo requiring her prior approval before any AFP
Senga instructed Commodore Tolentino to inform Gen. Gudani that "it was an order," yet Gen. Gudani personnel appear before Congress as a "gag order," which violates the principle of separation of
8
still refused to take Gen. Senga’s call. powers in government as it interferes with the investigation of the Senate Committee conducted in aid of
legislation. They also equate the "gag order" with culpable violation of the Constitution, particularly in
A few hours after Gen. Gudani and Col. Balutan had concluded their testimony, the office of Gen. relation to the public’s constitutional right to information and transparency in matters of public concern.
Senga issued a statement which noted that the two had appeared before the Senate Committee "in Plaintively, petitioners claim that "the Filipino people have every right to hear the [petitioners’]
spite of the fact that a guidance has been given that a Presidential approval should be sought prior to testimonies," and even if the "gag order" were unconstitutional, it still was tantamount to "the crime of
such an appearance;" that such directive was "in keeping with the time[-]honored principle of the Chain obstruction of justice." Petitioners further argue that there was no law prohibiting them from testifying
of Command;" and that the two officers "disobeyed a legal order, in violation of A[rticles of] W[ar] 65 before the Senate, and in fact, they were appearing in obeisance to the authority of Congress to
(Willfully Disobeying Superior Officer), hence they will be subjected to General Court Martial conduct inquiries in aid of legislation.
proceedings x x x" Both Gen. Gudani and Col. Balutan were likewise relieved of their assignments
9
then. Finally, it is stressed in the petition that Gen. Gudani was no longer subject to military jurisdiction on
account of his compulsory retirement on 4 October 2005. It is pointed out that Article 2, Title I of the
On the very day of the hearing, 28 September 2005, President Gloria-Macapagal-Arroyo issued Articles of War defines persons subject to military law as "all officers and soldiers in the active service"
Executive Order No. 464 (E.O. 464). The OSG notes that the E.O. "enjoined officials of the executive of the AFP.
department including the military establishment from appearing in any legislative inquiry without her
10
approval." This Court subsequently ruled on the constitutionality of the said executive order in Senate II.
11
v. Ermita. The relevance of E.O. 464 and Senate to the present petition shall be discussed forthwith.
We first proceed to define the proper litigable issues. Notably, the guilt or innocence of petitioners in
In the meantime, on 30 September 2005, petitioners were directed by General Senga, through Col. violating Articles 65 and 97 of the Articles of War is not an issue before this Court, especially
Henry A. Galarpe of the AFP Provost Marshal General, to appear before the Office of the Provost considering that per records, petitioners have not yet been subjected to court martial proceedings.
Marshal General (OPMG) on 3 October 2005 for investigation. During their appearance before Col. Owing to the absence of such proceedings, the correct inquiry should be limited to whether respondents
could properly initiate such proceedings preparatory to a formal court-martial, such as the
aforementioned preliminary investigation, on the basis of petitioners’ acts surrounding their testimony During the deliberations in Senate, the Court was very well aware of the pendency of this petition as
before the Senate on 28 September 2005. Yet this Court, consistent with the principle that it is not a well as the issues raised herein. The decision in Senate was rendered with the comfort that the
21
trier of facts at first instance, is averse to making any authoritative findings of fact, for that function is nullification of portions of E.O. 464 would bear no impact on the present petition since petitioners herein
first for the court-martial court to fulfill. were not called to task for violating the executive order. Moreover, the Court was then cognizant
that Senate and this case would ultimately hinge on disparate legal issues.
Thus, we limit ourselves to those facts that are not controverted before the Court, having been Relevantly, Senate purposely did not touch upon or rule on the faculty of the President, under the aegis
commonly alleged by petitioners and the OSG (for respondents). Petitioners were called by the Senate 26
of the commander-in-chief powers to require military officials from securing prior consent before
Committee to testify in its 28 September 2005 hearing. Petitioners attended such hearing and testified appearing before Congress. The pertinent factors in considering that question are markedly outside of
before the Committee, despite the fact that the day before, there was an order from Gen. Senga (which those which did become relevant in adjudicating the issues raised in Senate. It is in this petition that
in turn was sourced "per instruction" from President Arroyo) prohibiting them from testifying without the those factors come into play.
prior approval of the President. Petitioners do not precisely admit before this Court that they had
learned of such order prior to their testimony, although the OSG asserts that at the very least, Gen. At this point, we wish to dispose of another peripheral issue before we strike at the heart of the matter.
22
Gudani already knew of such order before he testified. Yet while this fact may be ultimately material in General Gudani argues that he can no longer fall within the jurisdiction of the court-martial, considering
the court-martial proceedings, it is not determinative of this petition, which as stated earlier, does not his retirement last 4 October 2005. He cites Article 2, Title I of Commonwealth Act No. 408, which
proffer as an issue whether petitioners are guilty of violating the Articles of War. defines persons subject to military law as, among others, "all officers and soldiers in the active service
of the [AFP]," and points out that he is no longer in the active service.
What the Court has to consider though is whether the violation of the aforementioned order of Gen.
27
Senga, which emanated from the President, could lead to any investigation for court-martial of This point was settled against Gen. Gudani’s position in Abadilla v. Ramos, where the Court declared
23
petitioners. It has to be acknowledged as a general principle that AFP personnel of whatever rank are that an officer whose name was dropped from the roll of officers cannot be considered to be outside the
liable under military law for violating a direct order of an officer superior in rank. Whether petitioners did jurisdiction of military authorities when military justice proceedings were initiated against him before the
violate such an order is not for the Court to decide, but it will be necessary to assume, for the purposes termination of his service. Once jurisdiction has been acquired over the officer, it continues until his
of this petition, that petitioners did so. case is terminated. Thus, the Court held:

III. The military authorities had jurisdiction over the person of Colonel Abadilla at the time of the alleged
offenses. This jurisdiction having been vested in the military authorities, it is retained up to the end of
Preliminarily, we must discuss the effect of E.O. 464 and the Court’s ruling in Senate on the present the proceedings against Colonel Abadilla. Well-settled is the rule that jurisdiction once acquired is not
petition. Notably, it is not alleged that petitioners were in any way called to task for violating E.O. 28
lost upon the instance of the parties but continues until the case is terminated.
464, but instead, they were charged for violating the direct order of Gen. Senga not to appear
before the Senate Committee, an order that stands independent of the executive Citing Colonel Winthrop’s treatise on Military Law, the Court further stated:
order. Distinctions are called for, since Section 2(b) of E.O. 464 listed "generals and flag officers of the
Armed Forces of the Philippines and such other officers who in the judgment of the Chief of Staff are We have gone through the treatise of Colonel Winthrop and We find the following passage which goes
covered by the executive privilege," as among those public officials required in Section 3 of E.O. 464 "to against the contention of the petitioners, viz —
secure prior consent of the President prior to appearing before either House of Congress." The Court
24
in Senate declared both Section 2(b) and Section 3 void, and the impression may have been left 3. Offenders in general — Attaching of jurisdiction. It has further been held, and is now settled law, in
following Senate that it settled as doctrine, that the President is prohibited from requiring military regard to military offenders in general, that if the military jurisdiction has once duly attached to them
personnel from attending congressional hearings without having first secured prior presidential consent. previous to the date of the termination of their legal period of service, they may be brought to trial by
That impression is wrong. court-martial after that date, their discharge being meanwhile withheld. This principle has mostly been
applied to cases where the offense was committed just prior to the end of the term. In such cases the
Senate turned on the nature of executive privilege, a presidential prerogative which is encumbered by interests of discipline clearly forbid that the offender should go unpunished. It is held therefore that if
significant limitations. Insofar as E.O. 464 compelled officials of the executive branch to seek prior before the day on which his service legally terminates and his right to a discharge is complete,
presidential approval before appearing before Congress, the notion of executive control also comes into proceedings with a view to trial are commenced against him — as by arrest or the service of
25
consideration. However, the ability of the President to require a military official to secure prior consent charges, — the military jurisdiction will fully attach and once attached may be continued by a
before appearing before Congress pertains to a wholly different and independent specie of presidential trial by court-martial ordered and held after the end of the term of the enlistment of the accused
29
authority—the commander-in-chief powers of the President. By tradition and jurisprudence, the xxx
commander-in-chief powers of the President are not encumbered by the same degree of restriction as
that which may attach to executive privilege or executive control.
Thus, military jurisdiction has fully attached to Gen. Gudani inasmuch as both the acts complained of includes the ability of the President to restrict the travel, movement and speech of military officers,
and the initiation of the proceedings against him occurred before he compulsorily retired on 4 October activities which may otherwise be sanctioned under civilian law.
2005. We see no reason to unsettle the Abadilla doctrine. The OSG also points out that under Section
38
28 of Presidential Decree No. 1638, as amended, "[a]n officer or enlisted man carried in the retired list Reference to Kapunan, Jr. v. De Villa is useful in this regard. Lt. Col. Kapunan was ordered confined
30 under "house arrest" by then Chief of Staff (later President) Gen. Fidel Ramos. Kapunan was also
[of the Armed Forces of the Philippines] shall be subject to the Articles of War x x x" To this citation,
petitioners do not offer any response, and in fact have excluded the matter of Gen. Gudani’s retirement ordered, as a condition for his house arrest, that he may not issue any press statements or give any
as an issue in their subsequent memorandum. press conference during his period of detention. The Court unanimously upheld such restrictions,
noting:
IV.
[T]he Court is of the view that such is justified by the requirements of military discipline. It cannot be
We now turn to the central issues. gainsaid that certain liberties of persons in the military service, including the freedom of speech,
may be circumscribed by rules of military discipline. Thus, to a certain degree, individual rights
Petitioners wish to see annulled the "gag order" that required them to secure presidential consent prior may be curtailed, because the effectiveness of the military in fulfilling its duties under the law
to their appearance before the Senate, claiming that it violates the constitutional right to information and depends to a large extent on the maintenance of discipline within its ranks. Hence, lawful orders
transparency in matters of public concern; or if not, is tantamount at least to the criminal acts of must be followed without question and rules must be faithfully complied with, irrespective of a
obstruction of justice and grave coercion. However, the proper perspective from which to consider this soldier's personal views on the matter. It is from this viewpoint that the restrictions imposed on
issue entails the examination of the basis and authority of the President to issue such an order in the 39
petitioner Kapunan, an officer in the AFP, have to be considered.
first place to members of the AFP and the determination of whether such an order is subject to any
limitations. Any good soldier, or indeed any ROTC cadet, can attest to the fact that the military way of life
circumscribes several of the cherished freedoms of civilian life. It is part and parcel of the military
The vitality of the tenet that the President is the commander-in-chief of the Armed Forces is most crucial package. Those who cannot abide by these limitations normally do not pursue a military career and
to the democratic way of life, to civilian supremacy over the military, and to the general stability of our instead find satisfaction in other fields; and in fact many of those discharged from the service are
representative system of government. The Constitution reposes final authority, control and supervision inspired in their later careers precisely by their rebellion against the regimentation of military life.
of the AFP to the President, a civilian who is not a member of the armed forces, and whose duties as Inability or unwillingness to cope with military discipline is not a stain on character, for the military mode
commander-in-chief represent only a part of the organic duties imposed upon the office, the other is a highly idiosyncratic path which persons are not generally conscripted into, but volunteer themselves
31
functions being clearly civil in nature. Civilian supremacy over the military also countermands the to be part of. But for those who do make the choice to be a soldier, significant concessions to personal
notion that the military may bypass civilian authorities, such as civil courts, on matters such as freedoms are expected. After all, if need be, the men and women of the armed forces may be
32
conducting warrantless searches and seizures. commanded upon to die for country, even against their personal inclinations.

Pursuant to the maintenance of civilian supremacy over the military, the Constitution has allocated It may be so that military culture is a remnant of a less democratic era, yet it has been fully integrated
specific roles to the legislative and executive branches of government in relation to military affairs. into the democratic system of governance. The constitutional role of the armed forces is as protector of
Military appropriations, as with all other appropriations, are determined by Congress, as is the power to 40
the people and of the State. Towards this end, the military must insist upon a respect for duty and a
33 41
declare the existence of a state of war. Congress is also empowered to revoke a proclamation of discipline without counterpart in civilian life. The laws and traditions governing that discipline have a
34 42
martial law or the suspension of the writ of habeas corpus. The approval of the Commission on long history; but they are founded on unique military exigencies as powerful now as in the past. In the
Appointments is also required before the President can promote military officers from the rank of colonel end, it must be borne in mind that the armed forces has a distinct subculture with unique needs, a
35 43
or naval captain. Otherwise, on the particulars of civilian dominance and administration over the specialized society separate from civilian society. In the elegant prose of the eminent British military
military, the Constitution is silent, except for the commander-in-chief clause which is fertile in meaning historian, John Keegan:
and
[Warriors who fight wars have] values and skills [which] are not those of politicians and diplomats. They
36
implication as to whatever inherent martial authority the President may possess. are those of a world apart, a very ancient world, which exists in parallel with the everyday world but
does not belong to it. Both worlds change over time, and the warrior world adopts in step to the civilian.
The commander-in-chief provision in the Constitution is denominated as Section 18, Article VII, which It follows it, however, at a distance. The distance can never be closed, for the culture of the warrior can
begins with the simple declaration that "[t]he President shall be the Commander-in-Chief of all armed never be that of civilization itself….
44
37
forces of the Philippines x x x" Outside explicit constitutional limitations, such as those found in
Section 5, Article XVI, the commander-in-chief clause vests on the President, as commander-in-chief, Critical to military discipline is obeisance to the military chain of command. Willful disobedience of a
absolute authority over the persons and actions of the members of the armed forces. Such authority 45
superior officer is punishable by court-martial under Article 65 of the Articles of War. "An individual
soldier is not free to ignore the lawful orders or duties assigned by his immediate superiors. For there witness the birth of his first-born, or to attend the funeral of a parent. Yet again, military life calls for
would be an end of all discipline if the seaman and marines on board a ship of war [or soldiers deployed considerable personal sacrifices during the period of conscription, wherein the higher duty is not to self
in the field], on a distant service, were permitted to act upon their own opinion of their rights [or their but to country.
opinion of the
Indeed, the military practice is to require a soldier to obtain permission from the commanding officer
President’s intent], and to throw off the authority of the commander whenever they supposed it to be before he/she may leave his destination. A soldier who goes from the properly appointed place of duty
46
unlawfully exercised." or absents from his/her command, guard, quarters, station, or camp without proper leave is subject to
48
punishment by court-martial. It is even clear from the record that petitioners had actually requested for
Further traditional restrictions on members of the armed forces are those imposed on free speech and 49
travel authority from the PMA in Baguio City to Manila, to attend the Senate Hearing. Even petitioners
mobility.1âwphi1Kapunan is ample precedent in justifying that a soldier may be restrained by a superior are well aware that it was necessary for them to obtain permission from their superiors before they
officer from speaking out on certain matters. As a general rule, the discretion of a military officer to could travel to Manila to attend the Senate Hearing.
restrain the speech of a soldier under his/her command will be accorded deference, with minimal regard
if at all to the reason for such restraint. It is integral to military discipline that the soldier’s speech be with It is clear that the basic position of petitioners impinges on these fundamental principles we have
the consent and approval of the military commander. discussed. They seek to be exempted from military justice for having traveled to the Senate to testify
before the Senate Committee against the express orders of Gen. Senga, the AFP Chief of Staff. If
The necessity of upholding the ability to restrain speech becomes even more imperative if the soldier petitioners’ position is affirmed, a considerable exception would be carved from the unimpeachable right
desires to speak freely on political matters. The Constitution requires that "[t]he armed forces shall be of military officers to restrict the speech and movement of their juniors. The ruinous consequences to
insulated from partisan politics," and that ‘[n]o member of the military shall engage directly or indirectly the chain of command and military discipline simply cannot warrant the Court’s imprimatur on
47
in any partisan political activity, except to vote." Certainly, no constitutional provision or military petitioner’s position.
indoctrination will eliminate a soldier’s ability to form a personal political opinion, yet it is vital that such
opinions be kept out of the public eye. For one, political belief is a potential source of discord among V.
people, and a military torn by political strife is incapable of fulfilling its constitutional function as
protectors of the people and of the State. For another, it is ruinous to military discipline to foment an Still, it would be highly myopic on our part to resolve the issue solely on generalities surrounding military
atmosphere that promotes an active dislike of or dissent against the President, the commander-in-chief discipline. After all, petitioners seek to impress on us that their acts are justified as they were
of the armed forces. Soldiers are constitutionally obliged to obey a President they may dislike or responding to an invitation from the Philippine Senate, a component of the legislative branch of
distrust. This fundamental principle averts the country from going the way of banana republics. government. At the same time, the order for them not to testify ultimately came from the President, the
head of the executive branch of government and the commander-in-chief of the armed forces.
Parenthetically, it must be said that the Court is well aware that our country’s recent past is marked by
regime changes wherein active military dissent from the chain of command formed a key, though not Thus, we have to consider the question: may the President prevent a member of the armed forces from
exclusive, element. The Court is not blind to history, yet it is a judge not of history but of the testifying before a legislative inquiry? We hold that the President has constitutional authority to do so, by
Constitution. The Constitution, and indeed our modern democratic order, frown in no uncertain terms on virtue of her power as commander-in-chief, and that as a consequence a military officer who defies
a politicized military, informed as they are on the trauma of absolute martial rule. Our history might imply such injunction is liable under military justice. At the same time, we also hold that any chamber of
that a political military is part of the natural order, but this view cannot be affirmed by the legal order. Congress which seeks the appearance before it of a military officer against the consent of the President
The evolutionary path of our young democracy necessitates a reorientation from this view, reliant as our has adequate remedies under law to compel such attendance. Any military official whom Congress
socio-political culture has become on it. At the same time, evolution mandates a similar demand that summons to testify before it may be compelled to do so by the President. If the President is not so
our system of governance be more responsive to the needs and aspirations of the citizenry, so as to inclined, the President may be commanded by judicial order to compel the attendance of the military
avoid an environment vulnerable to a military apparatus able at will to exert an undue influence in our officer. Final judicial orders have the force of the law of the land which the President has the duty to
50
polity. faithfully execute.

Of possibly less gravitas, but of equal importance, is the principle that mobility of travel is another Explication of these principles is in order.
necessary restriction on members of the military. A soldier cannot leave his/her post without the consent
of the commanding officer. The reasons are self-evident. The commanding officer has to be aware at all As earlier noted, we ruled in Senate that the President may not issue a blanket requirement of prior
times of the location of the troops under command, so as to be able to appropriately respond to any consent on executive officials summoned by the legislature to attend a congressional hearing. In doing
exigencies. For the same reason, commanding officers have to be able to restrict the movement or so, the Court recognized the considerable limitations on executive privilege, and affirmed that the
travel of their soldiers, if in their judgment, their presence at place of call of duty is necessary. At times, privilege must be formally invoked on specified grounds. However, the ability of the President to
this may lead to unsentimental, painful consequences, such as a soldier being denied permission to prevent military officers from testifying before Congress does not turn on executive privilege,
but on the Chief Executive’s power as commander-in-chief to control the actions and speech of legislation is not "absolute or unlimited", and its exercise is circumscribed by Section 21, Article VI of the
58
members of the armed forces. The President’s prerogatives as commander-in-chief are not Constitution. From these premises, the Court enjoined the Senate Blue Ribbon Committee from
hampered by the same limitations as in executive privilege. requiring the petitioners in Bengzon from testifying and producing evidence before the committee,
holding that the inquiry in question did not involve any intended legislation.
Our ruling that the President could, as a general rule, require military officers to seek presidential
approval before appearing before Congress is based foremost on the notion that a contrary rule unduly Senate affirmed both the Arnault and Bengzon rulings. It elucidated on the constitutional scope and
diminishes the prerogatives of the President as commander-in-chief. Congress holds significant control limitations on the constitutional power of congressional inquiry. Thus:
over the armed forces in matters such as budget appropriations and the approval of higher-rank
51 As discussed in Arnault, the power of inquiry, "with process to enforce it," is grounded on the necessity
promotions, yet it is on the President that the Constitution vests the title as commander-in-chief and all
the prerogatives and functions appertaining to the position. Again, the exigencies of military discipline of information in the legislative process. If the information possessed by executive officials on the
and the chain of command mandate that the President’s ability to control the individual members of the operation of their offices is necessary for wise legislation on that subject, by parity of reasoning,
armed forces be accorded the utmost respect. Where a military officer is torn between obeying the Congress has the right to that information and the power to compel the disclosure thereof.
President and obeying the Senate, the Court will without hesitation affirm that the officer has to choose
the President. After all, the Constitution prescribes that it is the President, and not the Senate, who is As evidenced by the American experience during the so-called "McCarthy era", however, the right of
the commander-in-chief of the armed forces.
52 Congress to conduct inquirites in aid of legislation is, in theory, no less susceptible to abuse than
executive or judicial power. It may thus be subjected to judicial review pursuant to the Court’s certiorari
At the same time, the refusal of the President to allow members of the military to appear before powers under Section 1, Article VIII of the Constitution.
Congress is still subject to judicial relief. The Constitution itself recognizes as one of the legislature’s
53
functions is the conduct of inquiries in aid of legislation. Inasmuch as it is ill-advised for Congress to For one, as noted in Bengzon v. Senate Blue Ribbon Committee, the inquiry itself might not properly be
interfere with the President’s power as commander-in-chief, it is similarly detrimental for the President to in aid of legislation, and thus beyond the constitutional power of Congress. Such inquiry could not usurp
unduly interfere with Congress’s right to conduct legislative inquiries. The impasse did not come to pass judicial functions. Parenthetically, one possible way for Congress to avoid such result as occurred
in this petition, since petitioners testified anyway despite the presidential prohibition. Yet the Court is in Bengzon is to indicate in its invitations to the public officials concerned, or to any person for that
aware that with its pronouncement today that the President has the right to require prior consent from matter, the possible needed statute which prompted the need for the inquiry. Given such statement in
members of the armed forces, the clash may soon loom or actualize. its invitations, along with the usual indication of the subject of inquiry and the questions relative to and in
furtherance thereof, there would be less room for speculation on the part of the person invited on
We believe and hold that our constitutional and legal order sanctions a modality by which members of whether the inquiry is in aid of legislation.
the military may be compelled to attend legislative inquiries even if the President desires otherwise, a
modality which does not offend the Chief Executive’s prerogatives as commander-in-chief. The remedy Section 21, Article VI likewise establishes critical safeguards that proscribe the legislative power of
lies with the courts. inquiry. The provision requires that the inquiry be done in accordance with the Senate or House’s duly
published rules of procedure, necessarily implying the constitutional infirmity of an inquiry conducted
The fact that the executive branch is an equal, coordinate branch of government to the legislative without duly published rules of procedure. Section 21 also mandates that the rights of persons
creates a wrinkle to any basic rule that persons summoned to testify before Congress must do so. appearing in or affected by such inquiries be respected, an imposition that obligates Congress to
There is considerable interplay between the legislative and executive branches, informed by due adhere to the guarantees in the Bill of Rights.
deference and respect as to their various constitutional functions. Reciprocal courtesy idealizes this
relationship; hence, it is only as a last resort that one branch seeks to compel the other to a particular These abuses are, of course, remediable before the courts, upon the proper suit filed by the persons
mode of behavior. The judiciary, the third coordinate branch of government, does not enjoy a similar affected, even if they belong to the executive branch. Nonetheless, there may be exceptional
dynamic with either the legislative or executive branches. Whatever weakness inheres on judicial power circumstances… wherein a clear pattern of abuse of the legislative power of inquiry might be
due to its inability to originate national policies and legislation, such is balanced by the fact that it is the established, resulting in palpable violations of the rights guaranteed to members of the executive
branch empowered by the Constitution to compel obeisance to its rulings by the other branches of department under the Bill of Rights. In such instances, depending on the particulars of each case,
59
government. attempts by the Executive Branch to forestall these abuses may be accorded judicial sanction .

54
As evidenced by Arnault v. Nazareno and Bengzon v. Senate Blue Ribbon Committee, among
55 In Senate, the Court ruled that the President could not impose a blanket prohibition barring executive
others, the Court has not shirked from reviewing the exercise by Congress of its power of legislative officials from testifying before Congress without the President’s consent notwithstanding the invocation
56
inquiry. Arnault recognized that the legislative power of inquiry and the process to enforce it, "is an of executive privilege to justify such prohibition. The Court did not rule that the power to conduct
57
essential and appropriate auxiliary to the legislative function." On the other legislative inquiry ipso facto superseded the claim of executive privilege, acknowledging instead that the
hand, Bengzon acknowledged that the power of both houses of Congress to conduct inquiries in aid of viability of executive privilege stood on a case to case basis. Should neither branch yield to the other
branch’s assertion, the constitutional recourse is to the courts, as the final arbiter if the dispute. It is only principle that is the Constitution, the embodiment of the national conscience. The Constitution simply
the courts that can compel, with conclusiveness, attendance or non-attendance in legislative inquiries. does not permit the infraction which petitioners have allegedly committed, and moreover, provides for
an orderly manner by which the same result could have been achieved without offending constitutional
Following these principles, it is clear that if the President or the Chief of Staff refuses to allow a member principles.
of the AFP to appear before Congress, the legislative body seeking such testimony may seek judicial
relief to compel the attendance. Such judicial action should be directed at the heads of the executive WHEREFORE, the petition is DENIED. No pronouncement as to costs.
branch or the armed forces, the persons who wield authority and control over the actions of the officers
concerned. The legislative purpose of such testimony, as well as any defenses against the same — SO ORDERED.
whether grounded on executive privilege, national security or similar concerns — would be accorded
due judicial evaluation. All the constitutional considerations pertinent to either branch of government DANTE O. TINGA
may be raised, assessed, and ultimately weighed against each other. And once the courts speak with Associate Justice
finality, both branches of government have no option but to comply with the decision of the courts,
whether the effect of the decision is to their liking or disfavor.

Courts are empowered, under the constitutional principle of judicial review, to arbitrate disputes
between the legislative and executive branches of government on the proper constitutional parameters
60
of power. This is the fair and workable solution implicit in the constitutional allocation of powers among
the three branches of government. The judicial filter helps assure that the particularities of each case
would ultimately govern, rather than any overarching principle unduly inclined towards one branch of
government at the expense of the other. The procedure may not move as expeditiously as some may
desire, yet it ensures thorough deliberation of all relevant and cognizable issues before one branch is
compelled to yield to the other. Moreover, judicial review does not preclude the legislative and executive
branches from negotiating a mutually acceptable solution to the impasse. After all, the two branches,
exercising as they do functions and responsibilities that are political in nature, are free to smooth over
the thorns in their relationship with a salve of their own choosing.

And if emphasis be needed, if the courts so rule, the duty falls on the shoulders of the President,
as commander-in-chief, to authorize the appearance of the military officers before Congress.
Even if the President has earlier disagreed with the notion of officers appearing before the
legislature to testify, the Chief Executive is nonetheless obliged to comply with the final orders
of the courts.

Petitioners have presented several issues relating to the tenability or wisdom of the President’s order on
them and other military officers not to testify before Congress without the President’s consent. Yet these
issues ultimately detract from the main point — that they testified before the Senate despite an order
61
from their commanding officer and their commander-in-chief for them not to do so, in contravention of
the traditions of military discipline which we affirm today.1âwphi1 The issues raised by petitioners could
have very well been raised and properly adjudicated if the proper procedure was observed. Petitioners
could have been appropriately allowed to testify before the Senate without having to countermand their
Commander-in-chief and superior officer under the setup we have prescribed.

We consider the other issues raised by petitioners unnecessary to the resolution of this petition.

Petitioners may have been of the honest belief that they were defying a direct order of their
Commander-in-Chief and Commanding General in obeisance to a paramount idea formed within their
consciences, which could not be lightly ignored. Still, the Court, in turn, is guided by the superlative
Republic of the Philippines Petitioner maintains that sometime in August, 1988, respondent governor, in his official capacity as
SUPREME COURT Provincial Governor Tarlac, entered into and executed a Loan Agreement with Lingkod Tarlac
Manila Foundation, Inc., a non-stock and non-profit organization headed by the governor himself as chairman
and controlled by his brother-in-law as executive director, trustee, and secretary; that the said Loan
EN BANC Agreement was never authorized and approved by the Provincial Board, in direct contravention of the
provisions of the Local Government Code; that the said Agreement is wholly one-sided in favor of the
G.R. No. 99031 October 15, 1991 Foundation and grossly inimical to the interest of the Provincial Government (because it did not provide
for interest or for any type security and it did not provide for suretyship and comptrollership or audit to
RODOLFO D. LLAMAS, petitioner,
control the safe disbursement of said loan); that a total amount of P20,000,000.00 was disbursed to the
vs.
aforesaid Foundation; that the transactions constitute a fraudulent scheme to defraud the Provincial
EXECUTIVE SECRETARY OSCAR ORBOS and MARIANO UN OCAMPO III, respondents.
Government; and that the said Agreement is wholly unconstitutional, illegal, a immoral. (Annex "A",
Mauricio Law Office for petitioner. Petition)

Ongkiko, Bucoy, Dizon & Associates for private respondent. On the other hand, it is the contention of respondent governor that "the funds were intended to generate
livelihood project among the residents of Tarlac and the use of the Lingkod Tarlac Foundation, Inc. was
authorized by law and considered the best alternative as a matter of judgment." (pp. 12-13, Appeal
Memorandom); that he resigned from the said Foundation in order to forestall any suspicion that he
PARAS, J.: would influence it; that it is not true that the Loan Agreement did not provide for continuing audit by the
Provincial Government because the Memorandum of Agreement provides otherwise; and that the
The case before Us calls for a determination of whether or not the President of the Philippines has the Agreement is not manifestly and grossly disadvantageous to the Provincial Government and respondent
power to grant executive clemency in administrative cases. In connection therewith, two important governor did not and would not profit thereby because it provided sufficient safeguards for repayment.
questions are also put in issue, namely, whether or not the grant of executive clemency and the reason (Annex "A", Petition)
therefore, are political questions beyond judicial review, and whether or not the questioned act was
characterized by grave abuse of discretion amounting to lack of jurisdiction. After trial, the Secretary of the then Department of Local Government rendered a decision dated
September 21, 1990, dispositive portion of which reads:
Petitioner Rodolfo D. Llamas is the incumbent Vice-Governor of the Province of Tarlac and, on March 1,
1991 he assumed, by virtue of a decision of the Office of the President, the governorship (p. 1, Petition). WHEREFORE, Governor Mariano Un Ocampo III is, as he hereby found guilty of having violated
Private respondent Mariano Un Ocampo III is the incumbent Governor of the Province of Tarlac and Section 3(g) of Republic Act No.3019, otherwise known as the Anti-Graft and Corrupt Practices Act,
was suspended from office for a period of 90 days. Public respondent Oscar Orbos was the Executive which act amounts to serious neglect of duty and/or abuse of authority, for which tilp penalty of
Secretary at the time of the filing of this petition and is being impleaded herein in that official capacity for suspension from office for a period of ninety (90) days, effective upon the finality of this Decision, is
having issued, by authority of the President, the assailed Resolution granting executive clemency to hereby imposed upon him. (p. 3, Petition)
respondent governor.
Parenthetically, be it noted that the Resolution imposed not a preventive suspension but a penalty of
Sometime in 1989, petiotioner, together with Tarlac Board Members Marcelino Aganon, Jr. and Arnaldo suspension.
P. Dizon, filed on June 13, 1989 a verified complaint dated June 7, 1989 against respondent governor
before the then Department of Local Government (DLG, for short), charging him with alleged violation of Respondent govemor moved for a reconsideration of the abovequoted decision but the same was
Section 203(2) (f) 203(2) (p), and 208(w), of Batas Pambansa (B.P.) Blg. 337, otherwise known as the denied on October 19, 1990. Aggrieved, he appealed the DLG decision dated September 21, 1990 and
Local Government Code, and other appropriate laws, among them, the Anti-Graft and Corrupt Practices the order of denial dated October 19, 1990 to the Office of the President (O.P. Case No. 4480).
ACt. Prior to that, petitoner filed with the Office of the Omdusman a verified complainant dated
November 10, 1988 against respondent governor for the latter's alleged viloation of Section 3-G of On February 26, 1991, herein public respondent Executive Secretary issued a Resolution dismissing
Republic Act. (R.A.) No. 3019, otherwise known as the Anti-Graft and Corrupt Practices Act. respondent governor's appeal and affirming the September 21, 1990 DLG decision.

The complaint before the DLG, docketed as Administrative Case 10459, was subsequently tried, where Subsequently, and pursuant to Sec. 66, Chapter 4 of B.P. Blg. 337, to the effect that the decision of the
both petitioner and respondent govemor presented their respective evidence. Office of the President in administrative suspension of local officials shall be immediately executory
without prejudice to appeal to appropriate courts, petitioner, on March 1, 1991, took his oath of office as
acting governor. Under the administrative suspension order, petitioner had up to May 31, 1991 as acting
governor. On the same date (March 1, 1991), respondent govemor moved for a reconsideration of the always justify the means. Be that as it may, but without belaboring the propriety of the loan agreement
Executive Secretary's Resolution, to which petitioner filed an opposition. From the allegations of the aforementioned, some measure of leniency may be accorded petitioner as the purpose of his suspen
petitioner in his petition, respondent govemor accepted his suspension and turned over his office to sion may have made its mark.
petitioner.
WHEREFORE, Governor Mariano Un Ocampo III is hereby granted executive clemency in the sense
To the surprise of petitioner, however, respondent govemor on March 19, 1991, issued an that his ninety-day suspension is hereby reduced to the period already served.
"administrative order" dated March 8, 1991, in which the latter signified his intention to "(continue, as I
am bound to exercise my fimctions as govemor and shall hold office at my residence," in the belief that SO ORDERED.
"the pendency of my Motion for Reconsideration precludes the coming into finality as executory the
DLG decision." (Annex "E", Petition; p. 10, Comment). And, as categorically stated in the petition, the (Annex "F", Petition; pp. 25-26, Rollo)
reassumption ceremony by respondent governor was held on May 21, 1991 (p. 8, Petition).
By virtue of the aforequoted Resolution, respondent governor reassumed the governorship of the
Without ruling on respondent governor's Motion for Reconsideration, public respondent issued a province, allegedly withou any notification made to the petitioner.
Resolution dated May 15, 1991, in O.P. Case No. 4480, which reads:
Petitioner posits that the issuance by public respondent of the May 15, 1991 Resolution was "whimsical,
This refers to the petition of Gov. Mariano Un Ocampo III of Tarlac for executive clemency, interposed capricious and despotic, and constituted grave abuse of discretion amounting lack of jurisdiction," (p. 6,
in connection with the decision of the Secretary of then Department of Local Governmen (DLG) dated petition) basically on the ground th executive clemency could be granted by the President only in
21 September 1990, as affirmed in a Resolution of this Office dated 26 February 1991, suspending criminal cases as there is nothing in the statute books or even in the Constitution which allows the grant
petitioner from office for period of ninety (90) days upon the finality of said decision. thereof in administrative cases. Petitioner also contends that since respondent governor refused to
recognize his suspension (having reassumed the governorship in gross defiance of the suspension
As will be recalled, the DLG Secretary imposed the penalty of suspension upon his finding that order), executive clemency cannot apply to him; that his rights to due process were violated because
petitioner was guilty of serious neglect of duty and/or abuse of authority for entering into a loan contract the grant of executive clemency was so sudden that he was not even notified thereof; and that despite a
— with the Lingkod Tarlac Foundation, Inc. (LTFI) — grossly/manifestly disadvantageous to Tarlac finding by public respondent of impropriety in the loan transaction entered into by respondent governor,
Province. In his letter-petition of 10 May 1991, thereby pleading for a thirty (30)-day reduction of his the former failed to justify the reduction of the penalty of suspension on the latter. Petitioner further
suspension, petitioner invited attention to the DLG Secretary's decision clearing him of having alleges that the exftutive clemency granted by public respondent was "the product of a hocus-pocus
personally benefitted from the questioned transaction. In the same letter, petitioner manifests serving strategy" (p. 1, Manifestation with Motion, etc.) because there was allegedly no real petition for the grant
more than sixty (60) days of the ninety-day suspension. Previously, petitioner submitted documents and of executive clemency filed by respondent govemor.
letters from his constituents tending to show the relative success of his livelihood loan program pursue
under the aegis of the LTFI and/or the Foundation's credible loan repayment record. To cite some: Batas Pambansa Blg. 337 provides:

1. Certification of the Chairman,Tarlac Integrated Livelihood Cooperative, Inc., attesting to the full Sec. 63. Preventive Suspension. — (1) Preventive suspension may be imposed by the Minister of Local
payment of its loan (P15.05 M) plus interest with LTFI; Government if the respondent is a provincial or city official, ...

2. Certification of the Manager, Rural Bank of Geron (Tarlac), Inc., attesting to the gradual liquidation of (2) Preventive suspension may be imposed at any time after the issues are joined, when there is
the loan granted to family-borrowers out of funds provided by LTFI; reasonable ground to believe that the respondent has committed the act or acts complained of, when
the evidence of culpability is strong, when the gravity of the offense s warrants, or when the
3. Letter of Jover's Phil., expressing gratitude for the loan assistance extended for its export activities by continuance in office of the respondent coul influence the witnesses or pose a threat to the safety and
LTFI; integrity the records and other evidence. In all cases, preventive suspension shall not extend beyond
sixty days after the start of said suspension.
4. Letter of the Tarlac Provincial Agricultural Officer i forming that the proceeds of the loan from LTFI
have bee utilized in hybrid com production; and (3) At the expiration of sixty days, the suspended official shall be deemed reinstated in office without
prejudice to the continuation the proceedings against him until its termination. (Emphasis supplied)
5. Letter of the President of the Federation of Tobacco Leaf Producers of Tarlac, Inc., informing of the
payment of 76 of the amount (P203,966.00) loaned to the Federation for tobacco production. It is admitted by petitioner that since March 1, 1991, he has assumed the governorship. A portion of the
petition is hereon der quoted as follows:
Petitioner's act, vis-a-vis the loan to LTFI, may have been promp by an over eagerness to accelerate
the delivery of livelihood services to his provincemates. As the truism goes, however, the end does not
7. [On February 28, 1991], and in accordance with the provisions of the Local Government Code (Sec. b. In the meantime that this action is pending, and irnmediately upon the filing hereof, a temporary
66, Chapter 4, Batas Pambansa Blg. 337), to the effect that the decision of the Office of the President in restraining order be issued stopping the Respondents from enforcing, in any manner, the aforesaid
an administrative suspension of local officials shall be immediately executory without prejudice to contested resolution, and Respondent Ocampo, firom continuing with his reassumption of the
appeal to appropriate courts, Petitioner Llamas took his oath of office as acting govemor. Under the governorship. IN THE ALTERNATIVE, that a cease and desist order be issued against Respondent
administrative suspension order, Llamas had up to May 31 [sic 29] 1991 as acting governor; Ocampo stopping him from continuing with hiii reassumption of the governorship.

8. A copy of this oath of office is attached and made a part hereof as Annex B; Let us first deal with the issue on jurisdiction. Respondent govemor avers that since under the
Constitution fiffl discretionary authority is granted to the President on the exercise of executive
9. Significantly, this oath of office was sworn to by Petitioner Llamas before Secretary Santos of the clemency, the same constitutes a political question which is beyond judicial review.
newly created Department Interior and Local Government, as shown by the lower portion Annex B, and
by a picture of the oathtaking itself, attached and mad a part hereof as Annex B-1; Such a rule does not hold true in the case at bar. While it is true that courts cannot inquire into the
manner in which the President's discretionary powers are exercised or into the wisdom for its exercise,
10. Subsequently, Petitioner Llamas and Respondent Ocampo met, where Ocampo was shown Llamas' it is also a settled rule that when the issue involved concerns the validity of such discretionary powers or
oath of office. During meeting, held in the presence of all department heads at the provi cial capitol and whether said powers are within the limits prescribed by the Constitution, We will not decline to exercise
in the presence of various local government offici and representatives of the media, Ocampo agreed to our power of judicial review. And such review does not constitute a modification or correction of the act
turn over reigns of the provincial government to Petitioner; of the President, nor does it constitute interference with the functions of the President. In this
connection, the case of Tanada and Macapagal vs. Cuenco, et al., 103 Phil. 1051, is very enlightening,
11. In fact, Ocampo had asked the department heads and all other officials of the provincial government and We quote:
of Tarlac to extend their cooperation to Llamas, during the ninety days that the latter would assume the
governorship; Elsewhere in this treatise the well-known and well-established principle is considered that it is not within
the province of the courts to pass judgment upon the policy of legislative or executive action. Where,
12. And, as if this was not enough, Ocampo even made announcements in the media that he was therefore, discretionary powers are granted by the Consfitution or by statute, the manner in which those
allowing Petitioner Llamas to perform his functions as acting governor at the Office of the Govern at the powers are exercised is not subject to judicial review. The courts, therefore, concern themselves only
Capitol where he (Ocampo) used to hold office (true enough Ocampo has subsequently allowed Llamas with the question as to the existence and extent of these discretionary powers.
to hold office at the of the Governor, with Ocampo even escorting the acting therein last March 4, 1991);
As distinguished from the judicial, the legislative and executive departments are spoken of as the
l 3. An account of Ocampo's acceptance of his suspension and of his having turned over his office to political departments of government because in very many cases their action is necessarily dictated by
Petitioner Llamas was published, front page, in the March 5, 1991 issue of the Manila Bulletin. A copy of considerations of public or political policy. These considerations of public or political policy of course will
this news account is attached and made a part hereof as Annex C); not permit the legislature to violate constitutional provisions, or the executive to exercise authority not
granted him by the Constitution or by statute, but, within these limits, they do permit the departments,
14. Furthermore, various other officials, President Aquino Rep. Jose Cojuangco included, have
separately or together, to recognize that a certain set of facts exists or that a given status exists, and
extended recognition to Petitions Llamas' assumption of the governorship. Llamas met with President
these determinations, together with the consequences that flow therefrom, may not be traversed in the
Aquino and Rep. Cojuangco and, during this meeting, the two highest officials of the land have asked
courts. (Willoughby on the Constitution of the United States, Vol. 3, p. 1326).
Llamas to discharge his duties acting governor;
xxx xxx xxx
15. Secretary Santos, for that matter, has issued a designation to Tarlac Senior Board Member Aganon,
dated March 18, 1991, a pointing bim as acting vice governor of the province, "in view of the suspension What is generally meant, when it is said that a question is political, and not judicial, is that it is a matter
of Gov. Mariano Un Ocampo III, and the assumption Vice Governor Rodolfo Llamas as acting which is to be exercised by the people in their primary political capacity, or that it has been specifically
governor." A copy of this designation is attached and made a part hereof as Annex D; delegated to some other department or particular officer of the goverrunent, with discretionary power to
act. See State vs. Cunningham, 81 Wis. 497, 51 L.R.A. 561; In Re Gunn, 50 Fan. 155; 32 Pac. 470,
xxx xxx xxx
948, 19 L. RA. 519; Green vs. Mills, 69 Fed. 852, 16, C. CA 516, 30 L.R.A- 90; Fletcher vs. Tuttle, 151
30. ... [T]he reassumption ceremony by [Governor] Ocampo was held [in the] morning of May 21, 1991 111, 41, 37 N.E. 683, 25 L.R.A. 143, 42 Am. St. Rep. 220. Thus the Legislature may in its discretion
... (pp- 2-4 & 7, Petition; pp. 3-5 & 8, Rollo) determine whether it will pass a law or submit a proposed constitutional amendment to the people. The
courts have no judicial control over such matters, not merely because they involve political question, but
It is prayed in the instant petition dated May 21, 1991 that: because they are matters which the people have by the Constitute delegated to the Legislature. The
Governor may exercise the powers delegated to him, free from judicial control, so long as he observes
the laws and acts within the limits of the power conferred. His discretionary acts cannot be controllable, For misfeasance or malfeasance ... any [elective official] could ... be proceeded against administratively
not primarily because they are of a political nature, but because the Constitution and laws have placed or ... criminally. In either case, his culpability must be established ...
the particular matter under his control. But every officer under a constitutional government must act
according to law and subject him to the restraining and controlling power of the people, acting through It is also important to note that respondent govemor's Motion for Reconsideration filed on March 1, 1991
the courts, as well as through the executive or the Legislature. One department is just as representative was withdrawn in his petition for the grant of executive clemency, which fact rendered the Resolution
as the other, and the judiciary the department which is charged with the special duty of determinining dated February 26, 1991 affirming the DLG Decision (which found respondent governor guilty of neglect
the limitations which the law places upon all official action. The recognition of this principle, unknown of duty and/or abuse of authority and which suspended him for ninety (90) days) final.
except in Great Britain and America, is necessary, to "the end that the government may be one of laws
and not men" — words which Webster said were the greatest contained in any written constitutional Moreover, applying the doctrine "Ubi lex non distinguit, nec nos distinguire debemos," We cannot
document. sustain petitioner's view. In other words, if the law does not distinguish, so We must no distinguish. The
Constitution does not distinguish between which cases executive clemency may be exercised by the
Besides, under the 1987 Constitution, the Supreme Court has been conferred an "expanded President, with the sole exclusion of impeachment cases. By the same token, if executive clemency
jurisdiction" to review the decisions of the other branches and agencies of the government to determine may be exercised only in criminal cases, it would indeed be unnecessary to provide for the exclusion of
whether or not they have acted within the bounds of the Constitution (See Art. VIII, Sec. 1, Constitution). impeachment cases from the coverage of Article VII, Section 19 of the Constitution. Following
"Yet, in the exercise thereof, the Court is to merely check whether or not the govermental branch or petitioner's proposed interpretation, cases of impeachment are automatically excluded inasmuch as the
agency has gone beyond the constitutional limits of its jurisdiction, not that it erred or has a different same do not necessarily involve criminal offenses.
view" (Co vs. Electoral Tribunal of the House of Representatives & Ong, G.R. Nos. 92191-92 and
Balanquit vs. Electoral Tribunal of the House of Representatives & Ong, G.R Nos. 92202-03, July 30, In the same vein, We do not clearly see any valid and convincing reason why the President cannot
1991). grant executive clemency in administrative cases. It is Our considered view that if the President can
grant reprieves, commutations and pardons, and remit fines and forfeitures in criminal cases, with much
In the case at bar, the nature of the question for determination is not purely political. Here, we are called more reason can she grant executive clemency in administrative cases, which are clearly less serious
upon to decide whether under the Constitution the President may grant executive clemency in than criminal offenses.
administrative cases. We must not overlook the fact that the exercise by the President of her power of
executive clemency is subject to constitutional l'um'tations. We will merely check whether the particular A number of laws impliedly or expressly recognize or support the exercise of the executive clemency in
measure in question has been in accordance with law. In so doing, We will not concern ourselves with administrative cases.
the reasons or motives which actuate the President as such is clearly beyond our power of judicial
Under Sec. 43 of P.D. 807, "In meritorious cases, ..., the President may commute or remove
review.
administrative penalties or disabilities issued upon officers and employees, in disciplinary cases, subject
Petitioner's main argument is that the President may grant executive clemency only in criminal cases, to such terms and conditions as he may impose in the interest of the service."
based on Article VII, Section 19 of the Constitution which reads:
During the deliberations of the Constitutional Commission, a subject of deliberations was the proposed
Sec. 19. Except in cases of impeachment, or as otherwise pro vided in this Constitution, the President amendment to Art. VII, Sec. 19 which reads as follows: "However, the power to grant executive
may grant reprieves, commu tations, and pardons, and remit fines and forfeitures, after conviction by clemency for violation of corrupt practices laws may be limited by legislation."The Constitutional
final judgment. Commission, however, voted to remove the amendment, since it was in derogation of the powers of the
President. As Mr. Natividad stated:
He shall also have the power to grant amnesty with the concurrence of a majority of all the members of
the Congress. (Emphasis supplied) I am also against this provision which will again chip more powers from the President. In case of other
criminals convicted in our society we extend probation to them while in this case, they have already
According to the petitioner, the qualifying phrase "after conviction by final judgment" applies solely to been convicted and we offer mercy. The only way we can offer mercy to them is through this executive
criminal cases, and no other law allows the grant of executive clemency or pardon to anyone who has clemency extended to them by the President. If we still close this avenue to them, they would be
been "convicted in an administrative case," allegedly because the word "conviction" refers only to prejudiced even worse than the murderers and the more vicious killers in our society ....
criminal cases (par. 22-b, c, d, Petition). Petitioner, however, describes in his very own words,
respondent governor as one who has been "convicted in an administrative case" (par. 22-a, petition). The proposal was primarily intended to prevent the President from protecting his cronies. Manifestly,
Thus, petitioner concedes that the word "conviction" may be used either in a criminal case or in an however, the Commission preferred to trust in the discretion of Presidents and refrained from putting
administrative case. In Layno, Sr. vs. Sandiganbayan, 136 SCRA 536, We ruled: additional limitations on his clemency powers. (II RECORD of the Constitutional Commission, 392, 418-
419, 524-525)
It is evident from the intent of the Constitutional Commission, therefore, that the President's executive over all executive departments, may substitute her decision for that of her subordinate, most especially
clemency powers may not be limited in terms of coverage, except as already provided in the where the basis therefor would be to serve the greater public interest. It is clearly within the power of the
Constitution, that is, "no pardon, amnesty, parole, or suspension of sentence for violation of election President not only to grant "executive clemency" but also to reverse or modify a ruling issued by a
laws, rules and regulations shall be granted by the President without the favorable recommendation of subordinate against an erring public official, where a reconsideration of the facts alleged would support
the COMELEC" (Article IX, C, Section 5, Constitution). If those already adjudged guilty criminally in the same. It is in this sense that the alleged executive clemency was granted, after adducing reasons
court may be pardoned, those adjudged guilty administratively should likewise be extended the same that subserve the public interest. — "the relative success of . . . livelihood loan program." (pp. 39-40,
benefit. Comment of public respondent)

In criminal cases, the quantum of evidence required to convict an individual is proof beyond reasonable We wish to stress however that when we say the President can grant executive clemency in
doubt, but the Constitution grants to the President the power to pardon the act done by the proved administrative cases, We refer only to all administrative cases in the Executive branch, not in the
criminal and in the process exempts him from punishment therefor. On the other hand, in administrative Judicial or Legislative branches of the government.
cases, the quantum of evidence required is mere substantial evidence to support a decision, not to
mention that as to the admissibility of evidence, administrative bodies are not bound by the technical Noteworthy is the fact that on March 1, 1991, respondent governor filed a motion for reconsideration
and rigid rules of admissibility prescribed in criminal cases. It will therefore be unjust and unfair for those and the same may be regarded as implicitly resolved, not only because of its withdrawal but also
found guilty administratively of some charge if the same effects of pardon or executive clemency cannot because of the executive clemency which in effect reduced the penalty, conformably with the power of
be extended to them, even in the sense of modifying a decision to subserve the interest of the public. "control."
(p. 34, Comment of public respondent)
On petitioner's argument that private respondent's motion for reconsideration has abated the running of
Of equal importance are the following provisions of Executive Order No. 292, otherwise known as the the reglementary period for finality of judgment in O.P. Case No. 4480 (that is, there being no final
Administrative Code of 1987, Section I, Book III of which provides: judgment to speak of, the pardon granted was premature and of no effect, We reiterate the doctrine that
upon acceptance of a presidential pardon, the grantee is deemed to have waived any appeal which he
SECTION 1. Power of Control. — The President shall have control of all the executive departments, may have filed. Thus, it was held that:
bureaus, and offices. He shall ensure that the laws be faithfully executed.
The commutation of the penalty is impressed with legal significance. That is an exercise of executive
SECTION 38. Definition of Administrative Relationships. — Unless otherwise expressly stated in the clemency embraced in the pardoning power. According to the Constitution: "The President may except
Code or in other laws defining the special relationships of particular agencies, administrative in cases of impeachment, grant reprieves, commutations and pardons, remit fines and forfeitures and,
relationships shall be categorized and defined as follows: with the concurrence of the Batasang Pambansa, grant amnesty. "Once granted, it is binding and
effective. It serves to put an end to this appeal." (Mansanto v. Factoran, Jr., G.R. No. 78239, 170 SCRA
(1) Supervision and Control. — Supervision and control shall include authority to act directly whenever a 190. 196). (See also Peo v. Crisola, 129 SCRA 13)
specific function is entrusted by law or regulation to a subordinate; direct the performance of duty;
restrain the commission of acts; review, approve, reverse or modify acts and decisions of subordinate Consequently, respondent governor's acceptance of the presidential pardon "serves to put an end" to
officials or units; determine priorities in the execution of plans and programs. Unless a different meaning the motion for reconsideration and renders the subject decision final, that of the period already served.
is explicitly provided in the specific law governing the relationship of particular agencies the word
"control" shall encompass supervision and control as defined in this paragraph. ... (emphasis supplied) Finally, petitioner's argument that his constitutional rights to due process were violated is uruneritorious.
Pardon has been defined as "the private, though official, act of the executive magistrate, delivered to
The disciplinary authority to investigate, suspend, and remove provincial or city officials devolves at the the individual for whose benefit it is intended and not communicated officially to the court. ..." (Bernas,
first instance on the Department of Interior and Local Government (Secs. 61 and 65, B.P. Blg. 337) and The Constitution of the Philippines, Vol. II, First Ed. 1988, pp. 239-240, citing U.S. v. Wilson, 7 Pet. 150
ultimately on the President (Sec. 66). Implicit in this authority, however, is the "supervision and control" [U.S. 1833]). Thus, assuming that petitioner was not notified of the subject pardon, it is only because
power of the President to reduce, if circumstances so warrant, the imposable penalty or to modify the said notice is unnecessary. Besides, petitioner's claim that respondent governor has not begun serve
suspension or removal order, even "in the sense" of granting executive clemency. "Control," within the sentence is belied by his very own factual allegations in his petition, more particularly that he served as
meaning of the Constitution, is the power to substitute one's own judgment for that of a subordinate. Acting Governor of Tarlac effective from the date he took his Oath of Office on February 28, 1991 up to
Under the doctrine of Qualified Political Agency, the different executive departments are mere adjuncts the time respondent govemor reassumed the governorship of Tarlac on May 21, 1991 (par. 30 petition).
of the President. Their acts are presumptively the acts of the President until countermanded or It is, therefore, error to say that private respondent did not serve any portion of the 90-day suspension
reprobated by her (Vinena v. Secretary, 67 Phil. 451; Free Telephone Workers Union vs. Minister of meted upon him.
Labor and Employment, 108 SCRA 767 [1981]). Replying upon this view, it is urged by the Solicitor
General that in the present case, the President, in the exercise of her power of supervision and control
We fail to see any grave abuse of discretion amounting to lack or in excess of jurisdiction committed by
public respondent.

WHEREFORE, judgment is hereby rendered: (1) DECLARING that the President did not act arbitrarily
or with abuse, much less grave abuse of discretion in issuing the May 15, 1991 Resolution granting on
the grounds mentioned therein, executive clemency to respondent governor and that, accordingly, the
same is not unconstitutional (without prejudice to criminal proceedings which have been filed or may be
filed against respondent governor), and (2) DENYING the rest of the prayers in the petition for being
unmeritorious, moot and academic. No costs.

SO ORDERED.
G.R. No. 138680 October 10, 2000

INTEGRATED BAR OF THE PHILIPPINES, Represented by its National President, Jose Aguila
Grapilon, petitioners,
vs.
JOSEPH EJERCITO ESTRADA, in his capacity as President, Republic of the Philippines, and
HON. DOMINGO SIAZON, in his capacity as Secretary of Foreign Affairs, respondents.

x-----------------------x
EN BANC
G.R. No. 138698 October 10, 2000
G.R. No. 138570 October 10, 2000
JOVITO R. SALONGA, WIGBERTO TAÑADA, ZENAIDA QUEZON-AVENCEÑA, ROLANDO
SIMBULAN, PABLITO V. SANIDAD, MA. SOCORRO I. DIOKNO, AGAPITO A. AQUINO, JOKER P.
BAYAN (Bagong Alyansang Makabayan), a JUNK VFA MOVEMENT, BISHOP TOMAS
ARROYO, FRANCISCO C. RIVERA JR., RENE A.V. SAGUISAG, KILOSBAYAN, MOVEMENT OF
MILLAMENA (Iglesia Filipina Independiente), BISHOP ELMER BOLOCAN (United Church of
ATTORNEYS FOR BROTHERHOOD, INTEGRITY AND NATIONALISM, INC. (MABINI), petitioners,
Christ of the Phil.), DR. REYNALDO LEGASCA, MD, KILUSANG MAMBUBUKID NG PILIPINAS,
vs.
KILUSANG MAYO UNO, GABRIELA, PROLABOR, and the PUBLIC INTEREST LAW
THE EXECUTIVE SECRETARY, THE SECRETARY OF FOREIGN AFFAIRS, THE SECRETARY OF
CENTER, petitioners,
NATIONAL DEFENSE, SENATE PRESIDENT MARCELO B. FERNAN, SENATOR BLAS F. OPLE,
vs.
SENATOR RODOLFO G. BIAZON, AND ALL OTHER PERSONS ACTING THEIR CONTROL,
EXECUTIVE SECRETARY RONALDO ZAMORA, FOREIGN AFFAIRS SECRETARY DOMINGO
SUPERVISION, DIRECTION, AND INSTRUCTION IN RELATION TO THE VISITING FORCES
SIAZON, DEFENSE SECRETARY ORLANDO MERCADO, BRIG. GEN. ALEXANDER AGUIRRE,
AGREEMENT (VFA), respondents.
SENATE PRESIDENT MARCELO FERNAN, SENATOR FRANKLIN DRILON, SENATOR BLAS
OPLE, SENATOR RODOLFO BIAZON, and SENATOR FRANCISCO TATAD, respondents.
DECISION
x-----------------------x
BUENA, J.:
G.R. No. 138572 October 10, 2000
Confronting the Court for resolution in the instant consolidated petitions for certiorari and prohibition are
issues relating to, and borne by, an agreement forged in the turn of the last century between the
PHILIPPINE CONSTITUTION ASSOCIATION, INC.(PHILCONSA), EXEQUIEL B. GARCIA,
Republic of the Philippines and the United States of America -the Visiting Forces Agreement.
AMADOGAT INCIONG, CAMILO L. SABIO, AND RAMON A. GONZALES, petitioners,
vs.
The antecedents unfold.
HON. RONALDO B. ZAMORA, as Executive Secretary, HON. ORLANDO MERCADO, as Secretary
of National Defense, and HON. DOMINGO L. SIAZON, JR., as Secretary of Foreign On March 14, 1947, the Philippines and the United States of America forged a Military Bases
Affairs, respondents. Agreement which formalized, among others, the use of installations in the Philippine territory by United
States military personnel. To further strengthen their defense and security relationship, the Philippines
x-----------------------x
and the United States entered into a Mutual Defense Treaty on August 30, 1951. Under the treaty, the
parties agreed to respond to any external armed attack on their territory, armed forces, public vessels,
G.R. No. 138587 October 10, 2000 1
and aircraft.
TEOFISTO T. GUINGONA, JR., RAUL S. ROCO, and SERGIO R. OSMEÑA III, petitioners,
In view of the impending expiration of the RP-US Military Bases Agreement in 1991, the Philippines and
vs.
JOSEPH E. ESTRADA, RONALDO B. ZAMORA, DOMINGO L. SIAZON, JR., ORLANDO B. the United States negotiated for a possible extension of the military bases agreement. On September
16, 1991, the Philippine Senate rejected the proposed RP-US Treaty of Friendship, Cooperation and
MERCADO, MARCELO B. FERNAN, FRANKLIN M. DRILON, BLAS F. OPLE and RODOLFO G.
Security which, in effect, would have extended the presence of US military bases in the
BIAZON, respondents. 2
Philippines. With the expiration of the RP-US Military Bases Agreement, the periodic military exercises
x-----------------------x conducted between the two countries were held in abeyance. Notwithstanding, the defense and security
relationship between the Philippines and the United States of America continued pursuant to the Mutual "1. The term ‘military personnel’ refers to military members of the United States Army, Navy, Marine
Defense Treaty. Corps, Air Force, and Coast Guard.

On July 18, 1997, the United States panel, headed by US Defense Deputy Assistant Secretary for Asia "2. The term ‘civilian personnel’ refers to individuals who are neither nationals of, nor ordinary residents
Pacific Kurt Campbell, met with the Philippine panel, headed by Foreign Affairs Undersecretary Rodolfo in the Philippines and who are employed by the United States armed forces or who are accompanying
Severino Jr., to exchange notes on "the complementing strategic interests of the United States and the the United States armed forces, such as employees of the American Red Cross and the United
Philippines in the Asia-Pacific region." Both sides discussed, among other things, the possible elements Services Organization.
of the Visiting Forces Agreement (VFA for brevity). Negotiations by both panels on the VFA led to a
3 "Article II
consolidated draft text, which in turn resulted to a final series of conferences and negotiations that
culminated in Manila on January 12 and 13, 1998. Thereafter, then President Fidel V. Ramos approved Respect for Law
the VFA, which was respectively signed by public respondent Secretary Siazon and Unites States
Ambassador Thomas Hubbard on February 10, 1998. "It is the duty of the United States personnel to respect the laws of the Republic of the Philippines and to
abstain from any activity inconsistent with the spirit of this agreement, and, in particular, from any
On October 5, 1998, President Joseph E. Estrada, through respondent Secretary of Foreign Affairs, political activity in the Philippines. The Government of the United States shall take all measures within
4 its authority to ensure that this is done.
ratified the VFA.

On October 6, 1998, the President, acting through respondent Executive Secretary Ronaldo Zamora, "Article III
5 Entry and Departure
officially transmitted to the Senate of the Philippines, the Instrument of Ratification, the letter of the
6
President and the VFA, for concurrence pursuant to Section 21, Article VII of the 1987 Constitution.
The Senate, in turn, referred the VFA to its Committee on Foreign Relations, chaired by Senator Blas F. "1. The Government of the Philippines shall facilitate the admission of United States personnel and their
Ople, and its Committee on National Defense and Security, chaired by Senator Rodolfo G. Biazon, for departure from the Philippines in connection with activities covered by this agreement.
their joint consideration and recommendation. Thereafter, joint public hearings were held by the two
7 "2. United States military personnel shall be exempt from passport and visa regulations upon entering
Committees.
and departing the Philippines.
8
On May 3, 1999, the Committees submitted Proposed Senate Resolution No. 443 recommending the
"3. The following documents only, which shall be presented on demand, shall be required in respect of
concurrence of the Senate to the VFA and the creation of a Legislative Oversight Committee to oversee
United States military personnel who enter the Philippines:
its implementation. Debates then ensued.
"(a) personal identity card issued by the appropriate United States authority showing full name, date of
On May 27, 1999, Proposed Senate Resolution No. 443 was approved by the Senate, by a two-thirds
9 birth, rank or grade and service number (if any), branch of service and photograph;
(2/3) vote of its members. Senate Resolution No. 443 was then re-numbered as Senate Resolution No.
10
18. "(b) individual or collective document issued by the appropriate United States authority, authorizing the
travel or visit and identifying the individual or group as United States military personnel; and
On June 1, 1999, the VFA officially entered into force after an Exchange of Notes between respondent
Secretary Siazon and United States Ambassador Hubbard. "(c) the commanding officer of a military aircraft or vessel shall present a declaration of health, and
when required by the cognizant representative of the Government of the Philippines, shall conduct a
The VFA, which consists of a Preamble and nine (9) Articles, provides for the mechanism for regulating
quarantine inspection and will certify that the aircraft or vessel is free from quarantinable diseases. Any
the circumstances and conditions under which US Armed Forces and defense personnel may be
quarantine inspection of United States aircraft or United States vessels or cargoes thereon shall be
present in the Philippines, and is quoted in its full text, hereunder:
conducted by the United States commanding officer in accordance with the international health
"Article I regulations as promulgated by the World Health Organization, and mutually agreed procedures.
Definitions
"4. United States civilian personnel shall be exempt from visa requirements but shall present, upon
"As used in this Agreement, ‘United States personnel’ means United States military and civilian demand, valid passports upon entry and departure of the Philippines.
personnel temporarily in the Philippines in connection with activities approved by the Philippine
"5. If the Government of the Philippines has requested the removal of any United States personnel from
Government.
its territory, the United States authorities shall be responsible for receiving the person concerned within
"Within this definition: its own territory or otherwise disposing of said person outside of the Philippines.
"Article IV (1) offenses solely against the property or security of the United States or offenses solely against the
property or person of United States personnel; and
Driving and Vehicle Registration
(2) offenses arising out of any act or omission done in performance of official duty.
"1. Philippine authorities shall accept as valid, without test or fee, a driving permit or license issued by
the appropriate United States authority to United States personnel for the operation of military or official (c) The authorities of either government may request the authorities of the other government to waive
vehicles. their primary right to exercise jurisdiction in a particular case.

"2. Vehicles owned by the Government of the United States need not be registered, but shall have (d) Recognizing the responsibility of the United States military authorities to maintain good order and
appropriate markings. discipline among their forces, Philippine authorities will, upon request by the United States, waive their
primary right to exercise jurisdiction except in cases of particular importance to the Philippines. If the
"Article V Government of the Philippines determines that the case is of particular importance, it shall communicate
Criminal Jurisdiction such determination to the United States authorities within twenty (20) days after the Philippine
authorities receive the United States request.
"1. Subject to the provisions of this article:
(e) When the United States military commander determines that an offense charged by authorities of
(a) Philippine authorities shall have jurisdiction over United States personnel with respect to offenses the Philippines against United states personnel arises out of an act or omission done in the
committed within the Philippines and punishable under the law of the Philippines. performance of official duty, the commander will issue a certificate setting forth such determination. This
certificate will be transmitted to the appropriate authorities of the Philippines and will constitute sufficient
(b) United States military authorities shall have the right to exercise within the Philippines all criminal
proof of performance of official duty for the purposes of paragraph 3(b)(2) of this Article. In those cases
and disciplinary jurisdiction conferred on them by the military law of the United States over United
where the Government of the Philippines believes the circumstances of the case require a review of the
States personnel in the Philippines.
duty certificate, United States military authorities and Philippine authorities shall consult immediately.
"2. (a) Philippine authorities exercise exclusive jurisdiction over United States personnel with respect to Philippine authorities at the highest levels may also present any information bearing on its validity.
offenses, including offenses relating to the security of the Philippines, punishable under the laws of the United States military authorities shall take full account of the Philippine position. Where appropriate,
Philippines, but not under the laws of the United States. United States military authorities will take disciplinary or other action against offenders in official duty
cases, and notify the Government of the Philippines of the actions taken.
(b) United States authorities exercise exclusive jurisdiction over United States personnel with respect to
offenses, including offenses relating to the security of the United States, punishable under the laws of (f) If the government having the primary right does not exercise jurisdiction, it shall notify the authorities
the United States, but not under the laws of the Philippines. of the other government as soon as possible.

(c) For the purposes of this paragraph and paragraph 3 of this article, an offense relating to security (g) The authorities of the Philippines and the United States shall notify each other of the disposition of
means: all cases in which both the authorities of the Philippines and the United States have the right to exercise
jurisdiction.
(1) treason;
"4. Within the scope of their legal competence, the authorities of the Philippines and United States shall
(2) sabotage, espionage or violation of any law relating to national defense. assist each other in the arrest of United States personnel in the Philippines and in handling them over to
authorities who are to exercise jurisdiction in accordance with the provisions of this article.
"3. In cases where the right to exercise jurisdiction is concurrent, the following rules shall apply:
"5. United States military authorities shall promptly notify Philippine authorities of the arrest or detention
(a) Philippine authorities shall have the primary right to exercise jurisdiction over all offenses committed of United States personnel who are subject of Philippine primary or exclusive jurisdiction. Philippine
by United States personnel, except in cases provided for in paragraphs 1(b), 2 (b), and 3 (b) of this authorities shall promptly notify United States military authorities of the arrest or detention of any United
Article. States personnel.

(b) United States military authorities shall have the primary right to exercise jurisdiction over United "6. The custody of any United States personnel over whom the Philippines is to exercise jurisdiction
States personnel subject to the military law of the United States in relation to. shall immediately reside with United States military authorities, if they so request, from the commission
of the offense until completion of all judicial proceedings. United States military authorities shall, upon
formal notification by the Philippine authorities and without delay, make such personnel available to "11. United States personnel shall be subject to trial only in Philippine courts of ordinary jurisdiction, and
those authorities in time for any investigative or judicial proceedings relating to the offense with which shall not be subject to the jurisdiction of Philippine military or religious courts.
the person has been charged in extraordinary cases, the Philippine Government shall present its
position to the United States Government regarding custody, which the United States Government shall "Article VI
take into full account. In the event Philippine judicial proceedings are not completed within one year, the Claims
United States shall be relieved of any obligations under this paragraph. The one-year period will not
include the time necessary to appeal. Also, the one-year period will not include any time during which "1. Except for contractual arrangements, including United States foreign military sales letters of offer
scheduled trial procedures are delayed because United States authorities, after timely notification by and acceptance and leases of military equipment, both governments waive any and all claims against
Philippine authorities to arrange for the presence of the accused, fail to do so. each other for damage, loss or destruction to property of each other’s armed forces or for death or injury
to their military and civilian personnel arising from activities to which this agreement applies.
"7. Within the scope of their legal authority, United States and Philippine authorities shall assist each
other in the carrying out of all necessary investigation into offenses and shall cooperate in providing for "2. For claims against the United States, other than contractual claims and those to which paragraph 1
the attendance of witnesses and in the collection and production of evidence, including seizure and, in applies, the United States Government, in accordance with United States law regarding foreign claims,
proper cases, the delivery of objects connected with an offense. will pay just and reasonable compensation in settlement of meritorious claims for damage, loss,
personal injury or death, caused by acts or omissions of United States personnel, or otherwise incident
"8. When United States personnel have been tried in accordance with the provisions of this Article and to the non-combat activities of the United States forces.
have been acquitted or have been convicted and are serving, or have served their sentence, or have
had their sentence remitted or suspended, or have been pardoned, they may not be tried again for the "Article VII
same offense in the Philippines. Nothing in this paragraph, however, shall prevent United States military Importation and Exportation
authorities from trying United States personnel for any violation of rules of discipline arising from the act
"1. United States Government equipment, materials, supplies, and other property imported into or
or omission which constituted an offense for which they were tried by Philippine authorities.
acquired in the Philippines by or on behalf of the United States armed forces in connection with
"9. When United States personnel are detained, taken into custody, or prosecuted by Philippine activities to which this agreement applies, shall be free of all Philippine duties, taxes and other similar
authorities, they shall be accorded all procedural safeguards established by the law of the Philippines. charges. Title to such property shall remain with the United States, which may remove such property
At the minimum, United States personnel shall be entitled: from the Philippines at any time, free from export duties, taxes, and other similar charges. The
exemptions provided in this paragraph shall also extend to any duty, tax, or other similar charges which
(a) To a prompt and speedy trial; would otherwise be assessed upon such property after importation into, or acquisition within, the
Philippines. Such property may be removed from the Philippines, or disposed of therein, provided that
(b) To be informed in advance of trial of the specific charge or charges made against them and to have disposition of such property in the Philippines to persons or entities not entitled to exemption from
reasonable time to prepare a defense; applicable taxes and duties shall be subject to payment of such taxes, and duties and prior approval of
the Philippine Government.
(c) To be confronted with witnesses against them and to cross examine such witnesses;
"2. Reasonable quantities of personal baggage, personal effects, and other property for the personal
(d) To present evidence in their defense and to have compulsory process for obtaining witnesses; use of United States personnel may be imported into and used in the Philippines free of all duties, taxes
and other similar charges during the period of their temporary stay in the Philippines. Transfers to
(e) To have free and assisted legal representation of their own choice on the same basis as nationals of persons or entities in the Philippines not entitled to import privileges may only be made upon prior
the Philippines; approval of the appropriate Philippine authorities including payment by the recipient of applicable duties
and taxes imposed in accordance with the laws of the Philippines. The exportation of such property and
(f) To have the service of a competent interpreter; and
of property acquired in the Philippines by United States personnel shall be free of all Philippine duties,
(g) To communicate promptly with and to be visited regularly by United States authorities, and to have taxes, and other similar charges.
such authorities present at all judicial proceedings. These proceedings shall be public unless the court,
"Article VIII
in accordance with Philippine laws, excludes persons who have no role in the proceedings.
Movement of Vessels and Aircraft
"10. The confinement or detention by Philippine authorities of United States personnel shall be carried
out in facilities agreed on by appropriate Philippine and United States authorities. United States
Personnel serving sentences in the Philippines shall have the right to visits and material assistance.
"1. Aircraft operated by or for the United States armed forces may enter the Philippines upon approval b. Is the Supreme Court deprived of its jurisdiction over offenses punishable by reclusion perpetua or
of the Government of the Philippines in accordance with procedures stipulated in implementing higher?
arrangements.
IV
"2. Vessels operated by or for the United States armed forces may enter the Philippines upon approval
of the Government of the Philippines. The movement of vessels shall be in accordance with Does the VFA violate:
international custom and practice governing such vessels, and such agreed implementing
arrangements as necessary. a. the equal protection clause under Section 1, Article III of the Constitution?

"3. Vehicles, vessels, and aircraft operated by or for the United States armed forces shall not be subject b. the Prohibition against nuclear weapons under Article II, Section 8?
to the payment of landing or port fees, navigation or over flight charges, or tolls or other use charges,
c. Section 28 (4), Article VI of the Constitution granting the exemption from taxes and duties for the
including light and harbor dues, while in the Philippines. Aircraft operated by or for the United States
equipment, materials supplies and other properties imported into or acquired in the Philippines by, or on
armed forces shall observe local air traffic control regulations while in the Philippines. Vessels owned or
behalf, of the US Armed Forces?
operated by the United States solely on United States Government non-commercial service shall not be
subject to compulsory pilotage at Philippine ports. LOCUS STANDI
"Article IX At the outset, respondents challenge petitioner’s standing to sue, on the ground that the latter have not
Duration and Termination shown any interest in the case, and that petitioners failed to substantiate that they have sustained, or
12
will sustain direct injury as a result of the operation of the VFA. Petitioners, on the other hand, counter
"This agreement shall enter into force on the date on which the parties have notified each other in
that the validity or invalidity of the VFA is a matter of transcendental importance which justifies their
writing through the diplomatic channel that they have completed their constitutional requirements for 13
standing.
entry into force. This agreement shall remain in force until the expiration of 180 days from the date on
which either party gives the other party notice in writing that it desires to terminate the agreement." A party bringing a suit challenging the constitutionality of a law, act, or statute must show "not only that
11 the law is invalid, but also that he has sustained or in is in immediate, or imminent danger of sustaining
Via these consolidated petitions for certiorari and prohibition, petitioners - as legislators, non-
some direct injury as a result of its enforcement, and not merely that he suffers thereby in some
governmental organizations, citizens and taxpayers - assail the constitutionality of the VFA and impute
indefinite way." He must show that he has been, or is about to be, denied some right or privilege to
to herein respondents grave abuse of discretion in ratifying the agreement.
which he is lawfully entitled, or that he is about to be subjected to some burdens or penalties by reason
14
We have simplified the issues raised by the petitioners into the following: of the statute complained of.

I In the case before us, petitioners failed to show, to the satisfaction of this Court, that they have
sustained, or are in danger of sustaining any direct injury as a result of the enforcement of the VFA. As
Do petitioners have legal standing as concerned citizens, taxpayers, or legislators to question the taxpayers, petitioners have not established that the VFA involves the exercise by Congress of its taxing
15
constitutionality of the VFA? or spending powers. On this point, it bears stressing that a taxpayer’s suit refers to a case where the
act complained of directly involves the illegal disbursement of public funds derived from
16 17
II taxation. Thus, in Bugnay Const. & Development Corp. vs. Laron , we held:

Is the VFA governed by the provisions of Section 21, Article VII or of Section 25, Article XVIII of the "x x x it is exigent that the taxpayer-plaintiff sufficiently show that he would be benefited or injured by the
Constitution? judgment or entitled to the avails of the suit as a real party in interest. Before he can invoke the power of
judicial review, he must specifically prove that he has sufficient interest in preventing the illegal
III expenditure of money raised by taxation and that he will sustain a direct injury as a result of the
enforcement of the questioned statute or contract. It is not sufficient that he has merely a general
Does the VFA constitute an abdication of Philippine sovereignty? interest common to all members of the public."

a. Are Philippine courts deprived of their jurisdiction to hear and try offenses committed by US military Clearly, inasmuch as no public funds raised by taxation are involved in this case, and in the absence of
personnel? any allegation by petitioners that public funds are being misspent or illegally expended, petitioners, as
taxpayers, have no legal standing to assail the legality of the VFA.
Similarly, Representatives Wigberto Tañada, Agapito Aquino and Joker Arroyo, as petitioners- Although courts generally avoid having to decide a constitutional question based on the doctrine of
legislators, do not possess the requisite locus standi to maintain the present suit. While this Court, separation of powers, which enjoins upon the departments of the government a becoming respect for
18 25
in Phil. Constitution Association vs. Hon. Salvador Enriquez, sustained the legal standing of a each others’ acts, this Court nevertheless resolves to take cognizance of the instant petitions.
member of the Senate and the House of Representatives to question the validity of a presidential veto
or a condition imposed on an item in an appropriation bull, we cannot, at this instance, similarly uphold APPLICABLE CONSTITUTIONAL PROVISION
petitioners’ standing as members of Congress, in the absence of a clear showing of any direct injury to
their person or to the institution to which they belong. One focal point of inquiry in this controversy is the determination of which provision of the Constitution
applies, with regard to the exercise by the senate of its constitutional power to concur with the VFA.
Beyond this, the allegations of impairment of legislative power, such as the delegation of the power of Petitioners argue that Section 25, Article XVIII is applicable considering that the VFA has for its subject
Congress to grant tax exemptions, are more apparent than real. While it may be true that petitioners the presence of foreign military troops in the Philippines. Respondents, on the contrary, maintain that
pointed to provisions of the VFA which allegedly impair their legislative powers, petitioners failed Section 21, Article VII should apply inasmuch as the VFA is not a basing arrangement but an
however to sufficiently show that they have in fact suffered direct injury. agreement which involves merely the temporary visits of United States personnel engaged in joint
military exercises.
In the same vein, petitioner Integrated Bar of the Philippines (IBP) is stripped of standing in these
cases. As aptly observed by the Solicitor General, the IBP lacks the legal capacity to bring this suit in The 1987 Philippine Constitution contains two provisions requiring the concurrence of the Senate on
the absence of a board resolution from its Board of Governors authorizing its National President to treaties or international agreements. Section 21, Article VII, which herein respondents invoke, reads:
19
commence the present action.
"No treaty or international agreement shall be valid and effective unless concurred in by at least two-
Notwithstanding, in view of the paramount importance and the constitutional significance of the issues thirds of all the Members of the Senate."
raised in the petitions, this Court, in the exercise of its sound discretion, brushes aside the procedural
barrier and takes cognizance of the petitions, as we have done in the early Emergency Powers Section 25, Article XVIII, provides:
20
Cases, where we had occasion to rule:
"After the expiration in 1991 of the Agreement between the Republic of the Philippines and the United
"x x x ordinary citizens and taxpayers were allowed to question the constitutionality of several executive States of America concerning Military Bases, foreign military bases, troops, or facilities shall not be
orders issued by President Quirino although they were involving only an indirect and general interest allowed in the Philippines except under a treaty duly concurred in by the senate and, when the
shared in common with the public. The Court dismissed the objection that they were not proper parties Congress so requires, ratified by a majority of the votes cast by the people in a national referendum
and ruled that ‘transcendental importance to the public of these cases demands that they be held for that purpose, and recognized as a treaty by the other contracting State."
settled promptly and definitely, brushing aside, if we must, technicalities of procedure.’ We have
Section 21, Article VII deals with treatise or international agreements in general, in which case, the
since then applied the exception in many other cases. (Association of Small Landowners in the
concurrence of at least two-thirds (2/3) of all the Members of the Senate is required to make the subject
Philippines, Inc. v. Sec. of Agrarian Reform, 175 SCRA 343)." (Underscoring Supplied)
treaty, or international agreement, valid and binding on the part of the Philippines. This provision lays
21
This principle was reiterated in the subsequent cases of Gonzales vs. COMELEC, Daza vs. down the general rule on treatise or international agreements and applies to any form of treaty with a
22 23
Singson, and Basco vs. Phil. Amusement and Gaming Corporation, where we emphatically held: wide variety of subject matter, such as, but not limited to, extradition or tax treatise or those economic in
nature. All treaties or international agreements entered into by the Philippines, regardless of subject
"Considering however the importance to the public of the case at bar, and in keeping with the Court’s matter, coverage, or particular designation or appellation, requires the concurrence of the Senate to be
duty, under the 1987 Constitution, to determine whether or not the other branches of the government valid and effective.
have kept themselves within the limits of the Constitution and the laws and that they have not abused
the discretion given to them, the Court has brushed aside technicalities of procedure and has taken In contrast, Section 25, Article XVIII is a special provision that applies to treaties which involve the
cognizance of this petition. x x x" presence of foreign military bases, troops or facilities in the Philippines. Under this provision, the
concurrence of the Senate is only one of the requisites to render compliance with the constitutional
24 requirements and to consider the agreement binding on the Philippines. Section 25, Article XVIII further
Again, in the more recent case of Kilosbayan vs. Guingona, Jr., thisCourt ruled that in cases of
transcendental importance, the Court may relax the standing requirements and allow a suit to requires that "foreign military bases, troops, or facilities" may be allowed in the Philippines only by virtue
prosper even where there is no direct injury to the party claiming the right of judicial review. of a treaty duly concurred in by the Senate, ratified by a majority of the votes cast in a national
referendum held for that purpose if so required by Congress, and recognized as such by the other
contracting state.
It is our considered view that both constitutional provisions, far from contradicting each other, actually It is a rudiment in legal hermenuetics that when no distinction is made by law, the Court should not
share some common ground. These constitutional provisions both embody phrases in the negative and distinguish- Ubi lex non distinguit nec nos distinguire debemos.
thus, are deemed prohibitory in mandate and character. In particular, Section 21 opens with the clause
"No treaty x x x," and Section 25 contains the phrase "shall not be allowed." Additionally, in both In like manner, we do not subscribe to the argument that Section 25, Article XVIII is not controlling since
instances, the concurrence of the Senate is indispensable to render the treaty or international no foreign military bases, but merely foreign troops and facilities, are involved in the VFA. Notably, a
agreement valid and effective. perusal of said constitutional provision reveals that the proscription covers "foreign military bases,
troops, or facilities." Stated differently, this prohibition is not limited to the entry of troops and facilities
To our mind, the fact that the President referred the VFA to the Senate under Section 21, Article VII, without any foreign bases being established. The clause does not refer to "foreign military bases,
and that the Senate extended its concurrence under the same provision, is immaterial. For in either troops, or facilities" collectively but treats them as separate and independent subjects. The use of
case, whether under Section 21, Article VII or Section 25, Article XVIII, the fundamental law is comma and the disjunctive word "or" clearly signifies disassociation and independence of one thing
28
crystalline that the concurrence of the Senate is mandatory to comply with the strict constitutional from the others included in the enumeration, such that, the provision contemplates three different
requirements. situations - a military treaty the subject of which could be either (a) foreign bases, (b) foreign troops, or
(c) foreign facilities - any of the three standing alone places it under the coverage of Section 25, Article
On the whole, the VFA is an agreement which defines the treatment of United States troops and XVIII.
personnel visiting the Philippines. It provides for the guidelines to govern such visits of military
personnel, and further defines the rights of the United States and the Philippine government in the To this end, the intention of the framers of the Charter, as manifested during the deliberations of the
matter of criminal jurisdiction, movement of vessel and aircraft, importation and exportation of 1986 Constitutional Commission, is consistent with this interpretation:
equipment, materials and supplies.
"MR. MAAMBONG. I just want to address a question or two to Commissioner Bernas.
Undoubtedly, Section 25, Article XVIII, which specifically deals with treaties involving foreign military
bases, troops, or facilities, should apply in the instant case. To a certain extent and in a limited sense, This formulation speaks of three things: foreign military bases, troops or facilities. My first question is: If
however, the provisions of section 21, Article VII will find applicability with regard to the issue and for the the country does enter into such kind of a treaty, must it cover the three-bases, troops or
sole purpose of determining the number of votes required to obtain the valid concurrence of the Senate, facilities-or could the treaty entered into cover only one or two?
as will be further discussed hereunder.
FR. BERNAS. Definitely, it can cover only one. Whether it covers only one or it covers three, the
It is a finely-imbedded principle in statutory construction that a special provision or law prevails over a requirement will be the same.
general one. Lex specialis derogat generali. Thus, where there is in the same statute a particular
enactment and also a general one which, in its most comprehensive sense, would include what is MR. MAAMBONG. In other words, the Philippine government can enter into a treaty covering not
embraced in the former, the particular enactment must be operative, and the general enactment must bases but merely troops?
be taken to affect only such cases within its general language which are not within the provision of the
26 FR. BERNAS. Yes.
particular enactment.
27 MR. MAAMBONG. I cannot find any reason why the government can enter into a treaty covering only
In Leveriza vs. Intermediate Appellate Court, we enunciated:
troops.
"x x x that another basic principle of statutory construction mandates that general legislation must give
FR. BERNAS. Why not? Probably if we stretch our imagination a little bit more, we will find some. We
way to a special legislation on the same subject, and generally be so interpreted as to embrace only 29
just want to cover everything." (Underscoring Supplied)
cases in which the special provisions are not applicable (Sto. Domingo vs. de los Angeles, 96 SCRA
139), that a specific statute prevails over a general statute (De Jesus vs. People, 120 SCRA 760) and Moreover, military bases established within the territory of another state is no longer viable because of
that where two statutes are of equal theoretical application to a particular case, the one designed the alternatives offered by new means and weapons of warfare such as nuclear weapons, guided
therefor specially should prevail (Wil Wilhensen Inc. vs. Baluyot, 83 SCRA 38)." missiles as well as huge sea vessels that can stay afloat in the sea even for months and years without
returning to their home country. These military warships are actually used as substitutes for a land-
Moreover, it is specious to argue that Section 25, Article XVIII is inapplicable to mere transient
home base not only of military aircraft but also of military personnel and facilities. Besides, vessels are
agreements for the reason that there is no permanent placing of structure for the establishment of a
mobile as compared to a land-based military headquarters.
military base. On this score, the Constitution makes no distinction between "transient’ and "permanent".
Certainly, we find nothing in Section 25, Article XVIII that requires foreign troops or facilities to be At this juncture, we shall then resolve the issue of whether or not the requirements of Section 25 were
stationed or placed permanently in the Philippines. complied with when the Senate gave its concurrence to the VFA.
Section 25, Article XVIII disallows foreign military bases, troops, or facilities in the country, unless the Petitioners content that the phrase "recognized as a treaty," embodied in section 25, Article XVIII,
following conditions are sufficiently met, viz: (a) it must be under a treaty; (b) the treaty must be duly means that the VFA should have the advice and consent of the United States Senate pursuant to its
concurred in by the Senate and, when so required by congress, ratified by a majority of the votes cast own constitutional process, and that it should not be considered merely an executive agreement by the
by the people in a national referendum; and (c) recognized as a treaty by the other contracting state. United States.

There is no dispute as to the presence of the first two requisites in the case of the VFA. The In opposition, respondents argue that the letter of United States Ambassador Hubbard stating that the
concurrence handed by the Senate through Resolution No. 18 is in accordance with the provisions of VFA is binding on the United States Government is conclusive, on the point that the VFA is recognized
the Constitution, whether under the general requirement in Section 21, Article VII, or the specific as a treaty by the United States of America. According to respondents, the VFA, to be binding, must
mandate mentioned in Section 25, Article XVIII, the provision in the latter article requiring ratification by only be accepted as a treaty by the United States.
a majority of the votes cast in a national referendum being unnecessary since Congress has not
required it. This Court is of the firm view that the phrase "recognized as a treaty" means that the other contracting
32
party accepts or acknowledges the agreement as a treaty. To require the other contracting state, the
As to the matter of voting, Section 21, Article VII particularly requires that a treaty or international United States of America in this case, to submit the VFA to the United States Senate for concurrence
33
agreement, to be valid and effective, must be concurred in by at least two-thirds of all the members pursuant to its Constitution, is to accord strict meaning to the phrase.
of the Senate. On the other hand, Section 25, Article XVIII simply provides that the treaty be "duly
concurred in by the Senate." Well-entrenched is the principle that the words used in the Constitution are to be given their ordinary
meaning except where technical terms are employed, in which case the significance thus attached to
34
Applying the foregoing constitutional provisions, a two-thirds vote of all the members of the Senate is them prevails. Its language should be understood in the sense they have in common use.
clearly required so that the concurrence contemplated by law may be validly obtained and deemed
present. While it is true that Section 25, Article XVIII requires, among other things, that the treaty-the Moreover, it is inconsequential whether the United States treats the VFA only as an executive
35
VFA, in the instant case-be "duly concurred in by the Senate," it is very true however that said provision agreement because, under international law, an executive agreement is as binding as a treaty. To be
must be related and viewed in light of the clear mandate embodied in Section 21, Article VII, which in sure, as long as the VFA possesses the elements of an agreement under international law, the said
more specific terms, requires that the concurrence of a treaty, or international agreement, be made by a agreement is to be taken equally as a treaty.
two -thirds vote of all the members of the Senate. Indeed, Section 25, Article XVIII must not be treated
in isolation to section 21, Article, VII. A treaty, as defined by the Vienna Convention on the Law of Treaties, is "an international instrument
concluded between States in written form and governed by international law, whether embodied in a
36
As noted, the "concurrence requirement" under Section 25, Article XVIII must be construed in relation to single instrument or in two or more related instruments, and whatever its particular designation." There
the provisions of Section 21, Article VII. In a more particular language, the concurrence of the Senate are many other terms used for a treaty or international agreement, some of which are: act, protocol,
contemplated under Section 25, Article XVIII means that at least two-thirds of all the members of the agreement, compromis d’ arbitrage, concordat, convention, declaration, exchange of notes, pact,
Senate favorably vote to concur with the treaty-the VFA in the instant case. statute, charter and modus vivendi. All writers, from Hugo Grotius onward, have pointed out that the
names or titles of international agreements included under the general term treaty have little or no legal
37
Under these circumstances, the charter provides that the Senate shall be composed of twenty-four (24) significance. Certain terms are useful, but they furnish little more than mere description.
30
Senators. Without a tinge of doubt, two-thirds (2/3) of this figure, or not less than sixteen (16)
members, favorably acting on the proposal is an unquestionable compliance with the requisite number Article 2(2) of the Vienna Convention provides that "the provisions of paragraph 1 regarding the use of
of votes mentioned in Section 21 of Article VII. The fact that there were actually twenty-three (23) terms in the present Convention are without prejudice to the use of those terms, or to the meanings
31 which may be given to them in the internal law of the State."
incumbent Senators at the time the voting was made, will not alter in any significant way the
circumstance that more than two-thirds of the members of the Senate concurred with the proposed
VFA, even if the two-thirds vote requirement is based on this figure of actual members (23). In this Thus, in international law, there is no difference between treaties and executive agreements in their
regard, the fundamental law is clear that two-thirds of the 24 Senators, or at least 16 favorable votes, binding effect upon states concerned, as long as the negotiating functionaries have remained within
38
suffice so as to render compliance with the strict constitutional mandate of giving concurrence to the their powers. International law continues to make no distinction between treaties and executive
39
subject treaty. agreements: they are equally binding obligations upon nations.

Having resolved that the first two requisites prescribed in Section 25, Article XVIII are present, we shall In our jurisdiction, we have recognized the binding effect of executive agreements even without the
40
now pass upon and delve on the requirement that the VFA should be recognized as a treaty by the concurrence of the Senate or Congress. In Commissioner of Customs vs. Eastern Sea Trading, we
United States of America. had occasion to pronounce:
"x x x the right of the Executive to enter into binding agreements without the necessity of subsequent intention of the State to sign the treaty subject to ratification appears from the full powers of its
44
congressional approval has been confirmed by long usage. From the earliest days of our history we representative, or was expressed during the negotiation.
have entered into executive agreements covering such subjects as commercial and consular relations,
most-favored-nation rights, patent rights, trademark and copyright protection, postal and navigation In our jurisdiction, the power to ratify is vested in the President and not, as commonly believed, in the
arrangements and the settlement of claims. The validity of these has never been seriously questioned legislature. The role of the Senate is limited only to giving or withholding its consent, or concurrence, to
45
by our courts. the ratification.

"x x x x x x x x x With the ratification of the VFA, which is equivalent to final acceptance, and with the exchange of notes
between the Philippines and the United States of America, it now becomes obligatory and incumbent on
"Furthermore, the United States Supreme Court has expressly recognized the validity and our part, under the principles of international law, to be bound by the terms of the agreement. Thus, no
46
constitutionality of executive agreements entered into without Senate approval. (39 Columbia Law less than Section 2, Article II of the Constitution, declares that the Philippines adopts the generally
Review, pp. 753-754) (See, also, U.S. vs. Curtis Wright Export Corporation, 299 U.S. 304, 81 L. ed. accepted principles of international law as part of the law of the land and adheres to the policy of peace,
255; U.S. vs. Belmont, 301 U.S. 324, 81 L. ed. 1134; U.S. vs. Pink, 315 U.S. 203, 86 L. ed. 796; equality, justice, freedom, cooperation and amity with all nations.
Ozanic vs. U.S. 188 F. 2d. 288; Yale Law Journal, Vol. 15 pp. 1905-1906; California Law Review,
Vol. 25, pp. 670-675; Hyde on International Law [revised Edition], Vol. 2, pp. 1405, 1416-1418; As a member of the family of nations, the Philippines agrees to be bound by generally accepted rules
willoughby on the U.S. Constitution Law, Vol. I [2d ed.], pp. 537-540; Moore, International Law for the conduct of its international relations. While the international obligation devolves upon the state
Digest, Vol. V, pp. 210-218; Hackworth, International Law Digest, Vol. V, pp. 390-407). (Italics and not upon any particular branch, institution, or individual member of its government, the Philippines
Supplied)" (Emphasis Ours) is nonetheless responsible for violations committed by any branch or subdivision of its government or
any official thereof. As an integral part of the community of nations, we are responsible to assure that
47
The deliberations of the Constitutional Commission which drafted the 1987 Constitution is enlightening our government, Constitution and laws will carry out our international obligation. Hence, we cannot
and highly-instructive: readily plead the Constitution as a convenient excuse for non-compliance with our obligations, duties
and responsibilities under international law.
"MR. MAAMBONG. Of course it goes without saying that as far as ratification of the other state is
concerned, that is entirely their concern under their own laws. Beyond this, Article 13 of the Declaration of Rights and Duties of States adopted by the International
Law Commission in 1949 provides: "Every State has the duty to carry out in good faith its obligations
FR. BERNAS. Yes, but we will accept whatever they say. If they say that we have done everything to arising from treaties and other sources of international law, and it may not invoke provisions in its
41 48
make it a treaty, then as far as we are concerned, we will accept it as a treaty." constitution or its laws as an excuse for failure to perform this duty."

The records reveal that the United States Government, through Ambassador Thomas C. Hubbard, has Equally important is Article 26 of the convention which provides that "Every treaty in force is binding
42
stated that the United States government has fully committed to living up to the terms of the VFA. For upon the parties to it and must be performed by them in good faith." This is known as the principle
as long as the united States of America accepts or acknowledges the VFA as a treaty, and binds itself of pacta sunt servanda which preserves the sanctity of treaties and have been one of the most
further to comply with its obligations under the treaty, there is indeed marked compliance with the fundamental principles of positive international law, supported by the jurisprudence of international
mandate of the Constitution. 49
tribunals.

Worth stressing too, is that the ratification, by the President, of the VFA and the concurrence of the NO GRAVE ABUSE OF DISCRETION
Senate should be taken as a clear an unequivocal expression of our nation’s consent to be bound by
said treaty, with the concomitant duty to uphold the obligations and responsibilities embodied In the instant controversy, the President, in effect, is heavily faulted for exercising a power and
thereunder. performing a task conferred upon him by the Constitution-the power to enter into and ratify treaties.
Through the expediency of Rule 65 of the Rules of Court, petitioners in these consolidated cases
Ratification is generally held to be an executive act, undertaken by the head of the state or of the impute grave abuse of discretion on the part of the chief Executive in ratifying the VFA, and referring
43
government, as the case may be, through which the formal acceptance of the treaty is proclaimed. A the same to the Senate pursuant to the provisions of Section 21, Article VII of the Constitution.
State may provide in its domestic legislation the process of ratification of a treaty. The consent of the
State to be bound by a treaty is expressed by ratification when: (a) the treaty provides for such On this particular matter, grave abuse of discretion implies such capricious and whimsical exercise of
ratification, (b) it is otherwise established that the negotiating States agreed that ratification should be judgment as is equivalent to lack of jurisdiction, or, when the power is exercised in an arbitrary or
required, (c) the representative of the State has signed the treaty subject to ratification, or (d) the despotic manner by reason of passion or personal hostility, and it must be so patent and gross as to
50
amount to an evasion of positive duty enjoined or to act at all in contemplation of law.
57
By constitutional fiat and by the intrinsic nature of his office, the President, as head of State, is the sole For the role of the Senate in relation to treaties is essentially legislative in character; the Senate, as an
organ and authority in the external affairs of the country. In many ways, the President is the chief independent body possessed of its own erudite mind, has the prerogative to either accept or reject the
architect of the nation’s foreign policy; his "dominance in the field of foreign relations is (then) proposed agreement, and whatever action it takes in the exercise of its wide latitude of discretion,
51
conceded." Wielding vast powers an influence, his conduct in the external affairs of the nation, as pertains to the wisdom rather than the legality of the act. In this sense, the Senate partakes a principal,
52
Jefferson describes, is "executive altogether." yet delicate, role in keeping the principles of separation of powers and of checks and balances alive and
vigilantly ensures that these cherished rudiments remain true to their form in a democratic government
As regards the power to enter into treaties or international agreements, the Constitution vests the same such as ours. The Constitution thus animates, through this treaty-concurring power of the Senate, a
in the President, subject only to the concurrence of at least two-thirds vote of all the members of the healthy system of checks and balances indispensable toward our nation’s pursuit of political maturity
Senate. In this light, the negotiation of the VFA and the subsequent ratification of the agreement are and growth. True enough, rudimentary is the principle that matters pertaining to the wisdom of a
exclusive acts which pertain solely to the President, in the lawful exercise of his vast executive and legislative act are beyond the ambit and province of the courts to inquire.
diplomatic powers granted him no less than by the fundamental law itself. Into the field of negotiation
53
the Senate cannot intrude, and Congress itself is powerless to invade it. Consequently, the acts or In fine, absent any clear showing of grave abuse of discretion on the part of respondents, this Court- as
judgment calls of the President involving the VFA-specifically the acts of ratification and entering into a the final arbiter of legal controversies and staunch sentinel of the rights of the people - is then without
treaty and those necessary or incidental to the exercise of such principal acts - squarely fall within the power to conduct an incursion and meddle with such affairs purely executive and legislative in character
sphere of his constitutional powers and thus, may not be validly struck down, much less calibrated by and nature. For the Constitution no less, maps out the distinct boundaries and limits the metes and
this Court, in the absence of clear showing of grave abuse of power or discretion. bounds within which each of the three political branches of government may exercise the powers
exclusively and essentially conferred to it by law.
It is the Court’s considered view that the President, in ratifying the VFA and in submitting the same to
the Senate for concurrence, acted within the confines and limits of the powers vested in him by the WHEREFORE, in light of the foregoing disquisitions, the instant petitions are hereby DISMISSED.
Constitution. It is of no moment that the President, in the exercise of his wide latitude of discretion and
in the honest belief that the VFA falls within the ambit of Section 21, Article VII of the Constitution, SO ORDERED.
referred the VFA to the Senate for concurrence under the aforementioned provision. Certainly, no
abuse of discretion, much less a grave, patent and whimsical abuse of judgment, may be imputed to the
President in his act of ratifying the VFA and referring the same to the Senate for the purpose of
complying with the concurrence requirement embodied in the fundamental law. In doing so, the
President merely performed a constitutional task and exercised a prerogative that chiefly pertains to the
functions of his office. Even if he erred in submitting the VFA to the Senate for concurrence under the
provisions of Section 21 of Article VII, instead of Section 25 of Article XVIII of the Constitution, still, the
President may not be faulted or scarred, much less be adjudged guilty of committing an abuse of
discretion in some patent, gross, and capricious manner.

For while it is conceded that Article VIII, Section 1, of the Constitution has broadened the scope of
judicial inquiry into areas normally left to the political departments to decide, such as those relating to
national security, it has not altogether done away with political questions such as those which arise in
54
the field of foreign relations. The High Tribunal’s function, as sanctioned by Article VIII, Section 1, "is
merely (to) check whether or not the governmental branch or agency has gone beyond the
constitutional limits of its jurisdiction, not that it erred or has a different view. In the absence of a
showing… (of) grave abuse of discretion amounting to lack of jurisdiction, there is no occasion for the
55
Court to exercise its corrective power…It has no power to look into what it thinks is apparent error."

As to the power to concur with treaties, the constitution lodges the same with the Senate
56
alone.1âwphi1 Thus, once the Senate performs that power, or exercises its prerogative within the
boundaries prescribed by the Constitution, the concurrence cannot, in like manner, be viewed to
constitute an abuse of power, much less grave abuse thereof. Corollarily, the Senate, in the exercise of
its discretion and acting within the limits of such power, may not be similarly faulted for having simply
performed a task conferred and sanctioned by no less than the fundamental law.
Republic of the Philippines On May 9, 2003, then Ambassador Francis J. Ricciardone sent US Embassy Note No. 0470 to the
SUPREME COURT Department of Foreign Affairs (DFA) proposing the terms of the non-surrender bilateral agreement
Manila (Agreement, hereinafter) between the USA and the RP.
7
EN BANC Via Exchange of Notes No. BFO-028-03 dated May 13, 2003 (E/N BFO-028-03, hereinafter), the RP,
represented by then DFA Secretary Ople, agreed with and accepted the US proposals embodied under
G.R. No. 159618 February 1, 2011 the US Embassy Note adverted to and put in effect the Agreement with the US government. In esse,
the Agreement aims to protect what it refers to and defines as "persons" of the RP and US from
BAYAN MUNA, as represented by Rep. SATUR OCAMPO, Rep. CRISPIN BELTRAN, and Rep. 8
frivolous and harassment suits that might be brought against them in international tribunals. It is
LIZA L. MAZA,Petitioner, reflective of the increasing pace of the strategic security and defense partnership between the two
vs. countries. As of May 2, 2003, similar bilateral agreements have been effected by and between the US
ALBERTO ROMULO, in his capacity as Executive Secretary, and BLAS F. OPLE, in his capacity and 33 other countries.
9

as Secretary of Foreign Affairs, Respondents.


The Agreement pertinently provides as follows:
DECISION
1. For purposes of this Agreement, "persons" are current or former Government officials, employees
VELASCO, JR., J.: (including contractors), or military personnel or nationals of one Party.
The Case 2. Persons of one Party present in the territory of the other shall not, absent the express consent of the
1 first Party,
This petition for certiorari, mandamus and prohibition under Rule 65 assails and seeks to nullify the
Non-Surrender Agreement concluded by and between the Republic of the Philippines (RP) and the (a) be surrendered or transferred by any means to any international tribunal for any purpose, unless
United States of America (USA). such tribunal has been established by the UN Security Council, or
The Facts (b) be surrendered or transferred by any means to any other entity or third country, or expelled to a third
country, for the purpose of surrender to or transfer to any international tribunal, unless such tribunal has
Petitioner Bayan Muna is a duly registered party-list group established to represent the marginalized
been established by the UN Security Council.
sectors of society. Respondent Blas F. Ople, now deceased, was the Secretary of Foreign Affairs during
the period material to this case. Respondent Alberto Romulo was impleaded in his capacity as then 3. When the [US] extradites, surrenders, or otherwise transfers a person of the Philippines to a third
2
Executive Secretary. country, the [US] will not agree to the surrender or transfer of that person by the third country to any
international tribunal, unless such tribunal has been established by the UN Security Council, absent the
Rome Statute of the International Criminal Court
express consent of the Government of the Republic of the Philippines [GRP].
3
Having a key determinative bearing on this case is the Rome Statute establishing the International
4. When the [GRP] extradites, surrenders, or otherwise transfers a person of the [USA] to a third
Criminal Court (ICC) with "the power to exercise its jurisdiction over persons for the most serious crimes
4 country, the [GRP] will not agree to the surrender or transfer of that person by the third country to any
of international concern x x x and shall be complementary to the national criminal jurisdictions." The
international tribunal, unless such tribunal has been established by the UN Security Council, absent the
serious crimes adverted to cover those considered grave under international law, such as genocide,
5 express consent of the Government of the [US].
crimes against humanity, war crimes, and crimes of aggression.
5. This Agreement shall remain in force until one year after the date on which one party notifies the
On December 28, 2000, the RP, through Charge d’Affaires Enrique A. Manalo, signed the Rome
other of its intent to terminate the Agreement. The provisions of this Agreement shall continue to apply
Statute which, by its terms, is "subject to ratification, acceptance or approval" by the signatory
6 with respect to any act occurring, or any allegation arising, before the effective date of termination.
states. As of the filing of the instant petition, only 92 out of the 139 signatory countries appear to have
completed the ratification, approval and concurrence process. The Philippines is not among the 92. In response to a query of then Solicitor General Alfredo L. Benipayo on the status of the non-surrender
agreement, Ambassador Ricciardone replied in his letter of October 28, 2003 that the exchange of
RP-US Non-Surrender Agreement
diplomatic notes constituted a legally binding agreement under international law; and that, under US
10
law, the said agreement did not require the advice and consent of the US Senate.
In this proceeding, petitioner imputes grave abuse of discretion to respondents in concluding and the Senate for concurrence, contravenes and undermines the Rome Statute and other treaties. But
ratifying the Agreement and prays that it be struck down as unconstitutional, or at least declared as because respondents expectedly raised it, we shall first tackle the issue of petitioner’s legal standing.
without force and effect.
The Court’s Ruling
For their part, respondents question petitioner’s standing to maintain a suit and counter that
the Agreement, being in the nature of an executive agreement, does not require Senate concurrence for This petition is bereft of merit.
its efficacy. And for reasons detailed in their comment, respondents assert the constitutionality of
the Agreement. Procedural Issue: Locus Standi of Petitioner

The Issues Petitioner, through its three party-list representatives, contends that the issue of the validity or invalidity
of the Agreement carries with it constitutional significance and is of paramount importance that justifies
12
I. WHETHER THE [RP] PRESIDENT AND THE [DFA] SECRETARY x x x GRAVELY ABUSED THEIR its standing. Cited in this regard is what is usually referred to as the emergency powers cases, in
DISCRETION AMOUNTING TO LACK OR EXCESS OF JURISDICTION FOR CONCLUDING THE RP- which ordinary citizens and taxpayers were accorded the personality to question the constitutionality of
US NON SURRENDER AGREEMENT BY MEANS OF [E/N] BFO-028-03 DATED 13 MAY 2003, executive issuances.
WHEN THE PHILIPPINE GOVERNMENT HAS ALREADY SIGNED THE ROME STATUTE OF THE 13
[ICC] ALTHOUGH THIS IS PENDING RATIFICATION BY THE PHILIPPINE SENATE. Locus standi is "a right of appearance in a court of justice on a given question." Specifically, it is "a
party’s personal and substantial interest in a case where he has sustained or will sustain direct injury as
14 15
A. Whether by entering into the x x x Agreement Respondents gravely abused their discretion when a result" of the act being challenged, and "calls for more than just a generalized grievance." The
16
they capriciously abandoned, waived and relinquished our only legitimate recourse through the Rome term "interest" refers to material interest, as distinguished from one that is merely incidental. The
Statute of the [ICC] to prosecute and try "persons" as defined in the x x x Agreement, x x x or literally rationale for requiring a party who challenges the validity of a law or international agreement to allege
any conduit of American interests, who have committed crimes of genocide, crimes against humanity, such a personal stake in the outcome of the controversy is "to assure the concrete adverseness which
war crimes and the crime of aggression, thereby abdicating Philippine Sovereignty. sharpens the presentation of issues upon which the court so largely depends for illumination of difficult
17
constitutional questions."
B. Whether after the signing and pending ratification of the Rome Statute of the [ICC] the [RP] President
and the [DFA] Secretary x x x are obliged by the principle of good faith to refrain from doing all acts Locus standi, however, is merely a matter of procedure and it has been recognized that, in some cases,
which would substantially impair the value of the undertaking as signed. suits are not brought by parties who have been personally injured by the operation of a law or any other
government act, but by concerned citizens, taxpayers, or voters who actually sue in the public
18 19
C. Whether the x x x Agreement constitutes an act which defeats the object and purpose of the Rome interest. Consequently, in a catena of cases, this Court has invariably adopted a liberal stance on
Statute of the International Criminal Court and contravenes the obligation of good faith inherent in the locus standi.
signature of the President affixed on the Rome Statute of the International Criminal Court, and if so
whether the x x x Agreement is void and unenforceable on this ground. Going by the petition, petitioner’s representatives pursue the instant suit primarily as concerned citizens
raising issues of transcendental importance, both for the Republic and the citizenry as a whole.
D. Whether the RP-US Non-Surrender Agreement is void and unenforceable for grave abuse of
discretion amounting to lack or excess of jurisdiction in connection with its execution. When suing as a citizen to question the validity of a law or other government action, a petitioner needs
to meet certain specific requirements before he can be clothed with standing. Francisco, Jr. v.
20
II. WHETHER THE RP-US NON SURRENDER AGREEMENT IS VOID AB INITIO FOR Nagmamalasakit na mga Manananggol ng mga Manggagawang Pilipino, Inc. expounded on this
CONTRACTING OBLIGATIONS THAT ARE EITHER IMMORAL OR OTHERWISE AT VARIANCE requirement, thus:
WITH UNIVERSALLY RECOGNIZED PRINCIPLES OF INTERNATIONAL LAW.
In a long line of cases, however, concerned citizens, taxpayers and legislators when specific
III. WHETHER THE x x x AGREEMENT IS VALID, BINDING AND EFFECTIVE WITHOUT THE requirements have been met have been given standing by this Court.
CONCURRENCE BY AT LEAST TWO-THIRDS (2/3) OF ALL THE MEMBERS OF THE SENATE x x
11 When suing as a citizen, the interest of the petitioner assailing the constitutionality of a statute must be
x.
direct and personal. He must be able to show, not only that the law or any government act is invalid, but
The foregoing issues may be summarized into two: first, whether or not the Agreement was contracted also that he sustained or is in imminent danger of sustaining some direct injury as a result of its
validly, which resolves itself into the question of whether or not respondents gravely abused their enforcement, and not merely that he suffers thereby in some indefinite way. It must appear that the
discretion in concluding it; and second, whether or not the Agreement, which has not been submitted to person complaining has been or is about to be denied some right or privilege to which he is lawfully
entitled or that he is about to be subjected to some burdens or penalties by reason of the statute or act
complained of. In fine, when the proceeding involves the assertion of a public right, the mere fact that In another perspective, the terms "exchange of notes" and "executive agreements" have been used
21
he is a citizen satisfies the requirement of personal interest. interchangeably, exchange of notes being considered a form of executive agreement that becomes
29
binding through executive action. On the other hand, executive agreements concluded by the
In the case at bar, petitioner’s representatives have complied with the qualifying conditions or specific President "sometimes take the form of exchange of notes and at other times that of more formal
requirements exacted under the locus standi rule. As citizens, their interest in the subject matter of the 30
documents denominated ‘agreements’ or ‘protocols.’" As former US High Commissioner to the
petition is direct and personal. At the very least, their assertions questioning the Agreement are made of Philippines Francis B. Sayre observed in his work, The Constitutionality of Trade Agreement Acts:
a public right, i.e., to ascertain that the Agreement did not go against established national policies,
practices, and obligations bearing on the State’s obligation to the community of nations. The point where ordinary correspondence between this and other governments ends and agreements –
whether denominated executive agreements or exchange of notes or otherwise – begin, may
At any event, the primordial importance to Filipino citizens in general of the issue at hand impels the 31
sometimes be difficult of ready ascertainment. x x x
Court to brush aside the procedural barrier posed by the traditional requirement of locus standi, as we
have done in a long line of earlier cases, notably in the old but oft-cited emergency powers It is fairly clear from the foregoing disquisition that E/N BFO-028-03––be it viewed as the Non-Surrender
22 23
cases and Kilosbayan v. Guingona, Jr. In cases of transcendental importance, we wrote again Agreement itself, or as an integral instrument of acceptance thereof or as consent to be bound––is a
24
in Bayan v. Zamora, "The Court may relax the standing requirements and allow a suit to prosper even recognized mode of concluding a legally binding international written contract among nations.
where there is no direct injury to the party claiming the right of judicial review."
Senate Concurrence Not Required
Moreover, bearing in mind what the Court said in Tañada v. Angara, "that it will not shirk, digress from
or abandon its sacred duty and authority to uphold the Constitution in matters that involve grave abuse Article 2 of the Vienna Convention on the Law of Treaties defines a treaty as "an international
of discretion brought before it in appropriate cases, committed by any officer, agency, instrumentality or agreement concluded between states in written form and governed by international law, whether
25 embodied in a single instrument or in two or more related instruments and whatever its particular
department of the government," we cannot but resolve head on the issues raised before us. Indeed,
32
where an action of any branch of government is seriously alleged to have infringed the Constitution or is designation." International agreements may be in the form of (1) treaties that require legislative
done with grave abuse of discretion, it becomes not only the right but in fact the duty of the judiciary to concurrence after executive ratification; or (2) executive agreements that are similar to treaties, except
settle it. As in this petition, issues are precisely raised putting to the fore the propriety of that they do not require legislative concurrence and are usually less formal and deal with a narrower
33
the Agreement pending the ratification of the Rome Statute. range of subject matters than treaties.

Validity of the RP-US Non-Surrender Agreement Under international law, there is no difference between treaties and executive agreements in terms of
34
their binding effects on the contracting states concerned, as long as the negotiating functionaries have
35
Petitioner’s initial challenge against the Agreement relates to form, its threshold posture being that E/N remained within their powers. Neither, on the domestic sphere, can one be held valid if it violates the
36
BFO-028-03 cannot be a valid medium for concluding the Agreement. Constitution. Authorities are, however, agreed that one is distinct from another for accepted reasons
37
apart from the concurrence-requirement aspect. As has been observed by US constitutional scholars,
Petitioners’ contention––perhaps taken unaware of certain well-recognized international doctrines, a treaty has greater "dignity" than an executive agreement, because its constitutional efficacy is beyond
practices, and jargons––is untenable. One of these is the doctrine of incorporation, as expressed in 38
doubt, a treaty having behind it the authority of the President, the Senate, and the people; a ratified
Section 2, Article II of the Constitution, wherein the Philippines adopts the generally accepted principles 39
treaty, unlike an executive agreement, takes precedence over any prior statutory enactment.
of international law and international jurisprudence as part of the law of the land and adheres to the
26
policy of peace, cooperation, and amity with all nations. An exchange of notes falls "into the category Petitioner parlays the notion that the Agreement is of dubious validity, partaking as it does of the nature
27
of inter-governmental agreements," which is an internationally accepted form of international of a treaty; hence, it must be duly concurred in by the Senate. Petitioner takes a cue
agreement. The United Nations Treaty Collections (Treaty Reference Guide) defines the term as from Commissioner of Customs v. Eastern Sea Trading, in which the Court reproduced the following
follows: observations made by US legal scholars: "[I]nternational agreements involving political issues or
changes of national policy and those involving international arrangements of a permanent character
An "exchange of notes" is a record of a routine agreement, that has many similarities with the private usually take the form of treaties [while] those embodying adjustments of detail carrying out well
law contract. The agreement consists of the exchange of two documents, each of the parties being in established national policies and traditions and those involving arrangements of a more or less
the possession of the one signed by the representative of the other. Under the usual procedure, the 40
temporary nature take the form of executive agreements."
accepting State repeats the text of the offering State to record its assent. The signatories of the letters
may be government Ministers, diplomats or departmental heads. The technique of exchange of notes is Pressing its point, petitioner submits that the subject of the Agreement does not fall under any of the
frequently resorted to, either because of its speedy procedure, or, sometimes, to avoid the process of subject-categories that are enumerated in the Eastern Sea Trading case, and that may be covered by
28
legislative approval. an executive agreement, such as commercial/consular relations, most-favored nation rights, patent
47
rights, trademark and copyright protection, postal and navigation arrangements and settlement of Petitioner’s reliance on Adolfo is misplaced, said case being inapplicable owing to different factual
claims. milieus. There, the Court held that an executive agreement cannot be used to amend a duly ratified and
existing treaty, i.e., the Bases Treaty. Indeed, an executive agreement that does not require the
In addition, petitioner foists the applicability to the instant case of Adolfo v. CFI of Zambales and concurrence of the Senate for its ratification may not be used to amend a treaty that, under the
41
Merchant, holding that an executive agreement through an exchange of notes cannot be used to Constitution, is the product of the ratifying acts of the Executive and the Senate. The presence of a
amend a treaty. treaty, purportedly being subject to amendment by an executive agreement, does not obtain under the
premises.
We are not persuaded.
Considering the above discussion, the Court need not belabor at length the third main issue raised,
The categorization of subject matters that may be covered by international agreements mentioned referring to the validity and effectivity of the Agreement without the concurrence by at least two-thirds of
in Eastern Sea Trading is not cast in stone. There are no hard and fast rules on the propriety of 48
all the members of the Senate. The Court has, in Eastern Sea Trading, as reiterated in Bayan, given
49

entering, on a given subject, into a treaty or an executive agreement as an instrument of international recognition to the obligatory effect of executive agreements without the concurrence of the Senate:
relations. The primary consideration in the choice of the form of agreement is the parties’ intent and
desire to craft an international agreement in the form they so wish to further their respective interests. x x x [T]he right of the Executive to enter into binding agreements without the necessity of subsequent
Verily, the matter of form takes a back seat when it comes to effectiveness and binding effect of the Congressional approval has been confirmed by long usage. From the earliest days of our history, we
enforcement of a treaty or an executive agreement, as the parties in either international agreement have entered executive agreements covering such subjects as commercial and consular relations, most
42
each labor under the pacta sunt servanda principle. favored-nation rights, patent rights, trademark and copyright protection, postal and navigation
arrangements and the settlement of claims. The validity of these has never been seriously questioned
As may be noted, almost half a century has elapsed since the Court rendered its decision in Eastern by our courts.
Sea Trading. Since then, the conduct of foreign affairs has become more complex and the domain of
international law wider, as to include such subjects as human rights, the environment, and the sea. In The Agreement Not in Contravention of the Rome Statute
fact, in the US alone, the executive agreements executed by its President from 1980 to 2000 covered
subjects such as defense, trade, scientific cooperation, aviation, atomic energy, environmental It is the petitioner’s next contention that the Agreement undermines the establishment of the ICC and is
43
cooperation, peace corps, arms limitation, and nuclear safety, among others. Surely, the enumeration null and void insofar as it unduly restricts the ICC’s jurisdiction and infringes upon the effectivity of the
in Eastern Sea Trading cannot circumscribe the option of each state on the matter of which the Rome Statute. Petitioner posits that the Agreement was constituted solely for the purpose of providing
international agreement format would be convenient to serve its best interest. As Francis Sayre said in individuals or groups of individuals with immunity from the jurisdiction of the ICC; and such grant of
his work referred to earlier: immunity through non-surrender agreements allegedly does not legitimately fall within the scope of Art.
98 of the Rome Statute. It concludes that state parties with non-surrender agreements are prevented
x x x It would be useless to undertake to discuss here the large variety of executive agreements as such from meeting their obligations under the Rome Statute, thereby constituting a breach of Arts.
concluded from time to time. Hundreds of executive agreements, other than those entered into under 50 51 52 53
27, 86, 89 and 90 thereof.
the trade-agreement act, have been negotiated with foreign governments. x x x They cover such
subjects as the inspection of vessels, navigation dues, income tax on shipping profits, the admission of Petitioner stresses that the overall object and purpose of the Rome Statute is to ensure that those
civil air craft, custom matters and commercial relations generally, international claims, postal matters, responsible for the worst possible crimes are brought to justice in all cases, primarily by states, but as a
the registration of trademarks and copyrights, etc. x x x last resort, by the ICC; thus, any agreement—like the non-surrender agreement—that precludes the
ICC from exercising its complementary function of acting when a state is unable to or unwilling to do so,
44
And lest it be overlooked, one type of executive agreement is a treaty-authorized or a treaty- defeats the object and purpose of the Rome Statute.
45
implementing executive agreement, which necessarily would cover the same matters subject of the
underlying treaty. Petitioner would add that the President and the DFA Secretary, as representatives of a signatory of the
Rome Statute, are obliged by the imperatives of good faith to refrain from performing acts that
But over and above the foregoing considerations is the fact that––save for the situation and matters substantially devalue the purpose and object of the Statute, as signed. Adding a nullifying ingredient to
46
contemplated in Sec. 25, Art. XVIII of the Constitution ––when a treaty is required, the Constitution the Agreement, according to petitioner, is the fact that it has an immoral purpose or is otherwise at
does not classify any subject, like that involving political issues, to be in the form of, and ratified as, a variance with a priorly executed treaty.
treaty. What the Constitution merely prescribes is that treaties need the concurrence of the Senate by a
vote defined therein to complete the ratification process. Contrary to petitioner’s pretense, the Agreement does not contravene or undermine, nor does it differ
from, the Rome Statute. Far from going against each other, one complements the other. As a matter of
fact, the principle of complementarity underpins the creation of the ICC. As aptly pointed out by
respondents and admitted by petitioners, the jurisdiction of the ICC is to "be complementary to national 2. The Court may not proceed with a request for surrender which would require the requested State to
54
criminal jurisdictions [of the signatory states]." Art. 1 of the Rome Statute pertinently provides: act inconsistently with its obligations under international agreements pursuant to which the consent of a
sending State is required to surrender a person of that State to the Court, unless the Court can first
Article 1 obtain the cooperation of the sending State for the giving of consent for the surrender.
The Court
Moreover, under international law, there is a considerable difference between a State-Party and a
An International Crimininal Court ("the Court") is hereby established. It x x x shall have the power to signatory to a treaty. Under the Vienna Convention on the Law of Treaties, a signatory state is only
exercise its jurisdiction over persons for the most serious crimes of international concern, as referred 58
obliged to refrain from acts which would defeat the object and purpose of a treaty; whereas a State-
to in this Statute, and shall be complementary to national criminal jurisdictions. The jurisdiction and Party, on the other hand, is legally obliged to follow all the provisions of a treaty in good faith.
functioning of the Court shall be governed by the provisions of this Statute. (Emphasis ours.)
In the instant case, it bears stressing that the Philippines is only a signatory to the Rome Statute and
Significantly, the sixth preambular paragraph of the Rome Statute declares that "it is the duty of every not a State-Party for lack of ratification by the Senate. Thus, it is only obliged to refrain from acts which
State to exercise its criminal jurisdiction over those responsible for international crimes." This provision would defeat the object and purpose of the Rome Statute. Any argument obliging the Philippines to
indicates that primary jurisdiction over the so-called international crimes rests, at the first instance, with follow any provision in the treaty would be premature.
the state where the crime was committed; secondarily, with the ICC in appropriate situations
55
contemplated under Art. 17, par. 1 of the Rome Statute. As a result, petitioner’s argument that State-Parties with non-surrender agreements are prevented from
meeting their obligations under the Rome Statute, specifically Arts. 27, 86, 89 and 90, must fail. These
56
Of particular note is the application of the principle of ne bis in idem under par. 3 of Art. 20, Rome articles are only legally binding upon State-Parties, not signatories.
Statute, which again underscores the primacy of the jurisdiction of a state vis-a-vis that of the ICC. As
far as relevant, the provision states that "no person who has been tried by another court for conduct x x Furthermore, a careful reading of said Art. 90 would show that the Agreement is not incompatible with
x [constituting crimes within its jurisdiction] shall be tried by the [International Criminal] Court with the Rome Statute. Specifically, Art. 90(4) provides that "[i]f the requesting State is a State not Party to
respect to the same conduct x x x." this Statute the requested State, if it is not under an international obligation to extradite the person to
the requesting State, shall give priority to the request for surrender from the Court. x x x" In applying the
The foregoing provisions of the Rome Statute, taken collectively, argue against the idea of jurisdictional provision, certain undisputed facts should be pointed out: first, the US is neither a State-Party nor a
conflict between the Philippines, as party to the non-surrender agreement, and the ICC; or the idea of signatory to the Rome Statute; and second, there is an international agreement between the US and the
the Agreement substantially impairing the value of the RP’s undertaking under the Rome Statute. Philippines regarding extradition or surrender of persons, i.e., the Agreement. Clearly, even assuming
Ignoring for a while the fact that the RP signed the Rome Statute ahead of the Agreement, it is that the Philippines is a State-Party, the Rome Statute still recognizes the primacy of international
abundantly clear to us that the Rome Statute expressly recognizes the primary jurisdiction of states, like agreements entered into between States, even when one of the States is not a State-Party to the Rome
the RP, over serious crimes committed within their respective borders, the complementary jurisdiction of Statute.
the ICC coming into play only when the signatory states are unwilling or unable to prosecute.
Sovereignty Limited by International Agreements
Given the above consideration, petitioner’s suggestion––that the RP, by entering into the Agreement,
violated its duty required by the imperatives of good faith and breached its commitment under the Petitioner next argues that the RP has, through the Agreement, abdicated its sovereignty by bargaining
57
Vienna Convention to refrain from performing any act tending to impair the value of a treaty, e.g., the away the jurisdiction of the ICC to prosecute US nationals, government officials/employees or military
Rome Statute––has to be rejected outright. For nothing in the provisions of the Agreement, in relation to personnel who commit serious crimes of international concerns in the Philippines. Formulating
the Rome Statute, tends to diminish the efficacy of the Statute, let alone defeats the purpose of the ICC. petitioner’s argument a bit differently, the RP, by entering into the Agreement, does thereby abdicate its
Lest it be overlooked, the Rome Statute contains a proviso that enjoins the ICC from seeking the sovereignty, abdication being done by its waiving or abandoning its right to seek recourse through the
surrender of an erring person, should the process require the requested state to perform an act that Rome Statute of the ICC for erring Americans committing international crimes in the country.
would violate some international agreement it has entered into. We refer to Art. 98(2) of the Rome
Statute, which reads: We are not persuaded. As it were, the Agreement is but a form of affirmance and confirmance of the
Philippines’ national criminal jurisdiction. National criminal jurisdiction being primary, as explained
Article 98 above, it is always the responsibility and within the prerogative of the RP either to prosecute criminal
Cooperation with respect to waiver of immunity offenses equally covered by the Rome Statute or to accede to the jurisdiction of the ICC. Thus, the
and consent to surrender Philippines may decide to try "persons" of the US, as the term is understood in the Agreement, under
our national criminal justice system. Or it may opt not to exercise its criminal jurisdiction over its erring
xxxx citizens or over US "persons" committing high crimes in the country and defer to the secondary criminal
jurisdiction of the ICC over them. As to "persons" of the US whom the Philippines refuses to prosecute, process undermined its treaty obligations under the Rome Statute, contrary to international law
64
the country would, in effect, accord discretion to the US to exercise either its national criminal principles.
jurisdiction over the "person" concerned or to give its consent to the referral of the matter to the ICC for
trial. In the same breath, the US must extend the same privilege to the Philippines with respect to The Court is not persuaded. Suffice it to state in this regard that the non-surrender agreement, as aptly
"persons" of the RP committing high crimes within US territorial jurisdiction. described by the Solicitor General, "is an assertion by the Philippines of its desire to try and punish
crimes under its national law. x x x The agreement is a recognition of the primacy and competence of
In the context of the Constitution, there can be no serious objection to the Philippines agreeing to the country’s judiciary to try offenses under its national criminal laws and dispense justice fairly and
undertake the things set forth in the Agreement. Surely, one State can agree to waive jurisdiction—to judiciously."
the extent agreed upon—to subjects of another State due to the recognition of the principle of
59 Petitioner, we believe, labors under the erroneous impression that the Agreement would allow Filipinos
extraterritorial immunity. What the Court wrote in Nicolas v. Romulo —a case involving the
implementation of the criminal jurisdiction provisions of the RP-US Visiting Forces Agreement—is and Americans committing high crimes of international concern to escape criminal trial and punishment.
apropos: This is manifestly incorrect. Persons who may have committed acts penalized under the Rome Statute
can be prosecuted and punished in the Philippines or in the US; or with the consent of the RP or the
Nothing in the Constitution prohibits such agreements recognizing immunity from jurisdiction or some US, before the ICC, assuming, for the nonce, that all the formalities necessary to bind both countries to
aspects of jurisdiction (such as custody), in relation to long-recognized subjects of such immunity like the Rome Statute have been met. For perspective, what the Agreement contextually prohibits is the
Heads of State, diplomats and members of the armed forces contingents of a foreign State allowed to surrender by either party of individuals to international tribunals, like the ICC, without the consent of the
enter another State’s territory. x x x other party, which may desire to prosecute the crime under its existing laws. With the view we take of
things, there is nothing immoral or violative of international law concepts in the act of the Philippines of
To be sure, the nullity of the subject non-surrender agreement cannot be predicated on the postulate assuming criminal jurisdiction pursuant to the non-surrender agreement over an offense considered
that some of its provisions constitute a virtual abdication of its sovereignty. Almost every time a state criminal by both Philippine laws and the Rome Statute.
enters into an international agreement, it voluntarily sheds off part of its sovereignty. The Constitution,
as drafted, did not envision a reclusive Philippines isolated from the rest of the world. It even adheres, No Grave Abuse of Discretion
60
as earlier stated, to the policy of cooperation and amity with all nations.
Petitioner’s final point revolves around the necessity of the Senate’s concurrence in the Agreement. And
By their nature, treaties and international agreements actually have a limiting effect on the otherwise without specifically saying so, petitioner would argue that the non-surrender agreement was executed
encompassing and absolute nature of sovereignty. By their voluntary act, nations may decide to by the President, thru the DFA Secretary, in grave abuse of discretion.
surrender or waive some aspects of their state power or agree to limit the exercise of their otherwise
exclusive and absolute jurisdiction. The usual underlying consideration in this partial surrender may be The Court need not delve on and belabor the first portion of the above posture of petitioner, the same
the greater benefits derived from a pact or a reciprocal undertaking of one contracting party to grant the having been discussed at length earlier on. As to the second portion, We wish to state that petitioner
same privileges or immunities to the other. On the rationale that the Philippines has adopted the virtually faults the President for performing, through respondents, a task conferred the President by the
generally accepted principles of international law as part of the law of the land, a portion of sovereignty Constitution—the power to enter into international agreements.
61
may be waived without violating the Constitution. Such waiver does not amount to an unconstitutional
diminution or deprivation of jurisdiction of Philippine courts.
62 By constitutional fiat and by the nature of his or her office, the President, as head of state and
65
government, is the sole organ and authority in the external affairs of the country. The Constitution
Agreement Not Immoral/Not at Variance vests in the President the power to enter into international agreements, subject, in appropriate cases, to
with Principles of International Law the required concurrence votes of the Senate. But as earlier indicated, executive agreements may be
validly entered into without such concurrence. As the President wields vast powers and influence, her
Petitioner urges that the Agreement be struck down as void ab initio for imposing immoral obligations conduct in the external affairs of the nation is, as Bayan would put it, "executive altogether." The right of
66
and/or being at variance with allegedly universally recognized principles of international law. The the President to enter into or ratify binding executive agreements has been confirmed by long practice.
immoral aspect proceeds from the fact that the Agreement, as petitioner would put it, "leaves criminals
immune from responsibility for unimaginable atrocities that deeply shock the conscience of humanity; x In thus agreeing to conclude the Agreement thru E/N BFO-028-03, then President Gloria Macapagal-
63 Arroyo, represented by the Secretary of Foreign Affairs, acted within the scope of the authority and
x x it precludes our country from delivering an American criminal to the [ICC] x x x."
discretion vested in her by the Constitution. At the end of the day, the President––by ratifying, thru her
The above argument is a kind of recycling of petitioner’s earlier position, which, as already discussed, deputies, the non-surrender agreement––did nothing more than discharge a constitutional duty and
contends that the RP, by entering into the Agreement, virtually abdicated its sovereignty and in the exercise a prerogative that pertains to her office.
While the issue of ratification of the Rome Statute is not determinative of the other issues raised herein, 9851, such as requiring the consent of the US before the Philippines can exercise such option, requires
it may perhaps be pertinent to remind all and sundry that about the time this petition was interposed, an amendatory law. In line with this scenario, the view strongly argues that the Agreement prevents the
67
such issue of ratification was laid to rest in Pimentel, Jr. v. Office of the Executive Secretary. As the Philippines—without the consent of the US—from surrendering to any international tribunal US
Court emphasized in said case, the power to ratify a treaty, the Statute in that instance, rests with the nationals accused of crimes covered by RA 9851, and, thus, in effect amends Sec. 17 of RA 9851.
President, subject to the concurrence of the Senate, whose role relative to the ratification of a treaty is Consequently, the view is strongly impressed that the Agreement cannot be embodied in a simple
limited merely to concurring in or withholding the ratification. And concomitant with this treaty-making executive agreement in the form of an exchange of notes but must be implemented through an
power of the President is his or her prerogative to refuse to submit a treaty to the Senate; or having extradition law or a treaty with the corresponding formalities.
68
secured the latter’s consent to the ratification of the treaty, refuse to ratify it. This prerogative, the
Court hastened to add, is the President’s alone and cannot be encroached upon via a writ of Moreover, consonant with the foregoing view, citing Sec. 2, Art. II of the Constitution, where the
mandamus. Barring intervening events, then, the Philippines remains to be just a signatory to the Rome Philippines adopts, as a national policy, the "generally accepted principles of international law as part of
69 the law of the land," the Court is further impressed to perceive the Rome Statute as declaratory of
Statute. Under Art. 125 thereof, the final acts required to complete the treaty process and, thus, bring
it into force, insofar as the Philippines is concerned, have yet to be done. customary international law. In other words, the Statute embodies principles of law which constitute
customary international law or custom and for which reason it assumes the status of an enforceable
Agreement Need Not Be in the Form of a Treaty domestic law in the context of the aforecited constitutional provision. As a corollary, it is argued that any
derogation from the Rome Statute principles cannot be undertaken via a mere executive agreement,
On December 11, 2009, then President Arroyo signed into law Republic Act No. (RA) 9851, otherwise which, as an exclusive act of the executive branch, can only implement, but cannot amend or repeal, an
known as the "Philippine Act on Crimes Against International Humanitarian Law, Genocide, and Other existing law. The Agreement, so the argument goes, seeks to frustrate the objects of the principles of
Crimes Against Humanity." Sec. 17 of RA 9851, particularly the second paragraph thereof, provides: law or alters customary rules embodied in the Rome Statute.

Section 17. Jurisdiction. – x x x x Prescinding from the foregoing premises, the view thus advanced considers
the Agreement inefficacious, unless it is embodied in a treaty duly ratified with the concurrence of the
In the interest of justice, the relevant Philippine authorities may dispense with the investigation or Senate, the theory being that a Senate- ratified treaty partakes of the nature of a municipal law that can
prosecution of a crime punishable under this Act if another court or international tribunal is already amend or supersede another law, in this instance Sec. 17 of RA 9851 and the status of the Rome
conducting the investigation or undertaking the prosecution of such crime. Instead, the Statute as constitutive of enforceable domestic law under Sec. 2, Art. II of the Constitution.
authorities may surrender or extradite suspected or accused persons in the Philippines to the
appropriate international court, if any, or to another State pursuant to the applicable extradition laws and We are unable to lend cogency to the view thus taken. For one, we find that the Agreement does not
treaties. (Emphasis supplied.) amend or is repugnant to RA 9851. For another, the view does not clearly state what precise principles
of law, if any, the Agreement alters. And for a third, it does not demonstrate in the concrete how
A view is advanced that the Agreement amends existing municipal laws on the State’s obligation in the Agreement seeks to frustrate the objectives of the principles of law subsumed in the Rome Statute.
relation to grave crimes against the law of nations, i.e., genocide, crimes against humanity and war
crimes. Relying on the above-quoted statutory proviso, the view posits that the Philippine is required to Far from it, as earlier explained, the Agreement does not undermine the Rome Statute as the former
surrender to the proper international tribunal those persons accused of the grave crimes defined under merely reinforces the primacy of the national jurisdiction of the US and the Philippines in prosecuting
RA 9851, if it does not exercise its primary jurisdiction to prosecute them. criminal offenses committed by their respective citizens and military personnel, among others. The
jurisdiction of the ICC pursuant to the Rome Statute over high crimes indicated thereat is clearly and
The basic premise rests on the interpretation that if it does not decide to prosecute a foreign national for unmistakably complementary to the national criminal jurisdiction of the signatory states.
violations of RA 9851, the Philippines has only two options, to wit: (1) surrender the accused to the
proper international tribunal; or (2) surrender the accused to another State if such surrender is "pursuant Moreover, RA 9851 clearly: (1) defines and establishes the crimes against international humanitarian
to the applicable extradition laws and treaties." But the Philippines may exercise these options only in 70
law, genocide and other crimes against humanity; (2) provides penal sanctions and criminal liability for
cases where "another court or international tribunal is already conducting the investigation or 71
their commission; and (3) establishes special courts for the prosecution of these crimes and for the
undertaking the prosecution of such crime;" otherwise, the Philippines must prosecute the crime before 72
State to exercise primary criminal jurisdiction. Nowhere in RA 9851 is there a proviso that goes
its own courts pursuant to RA 9851. against the tenor of the Agreement.

Posing the situation of a US national under prosecution by an international tribunal for any crime under The view makes much of the above quoted second par. of Sec. 17, RA 9851 as requiring the
RA 9851, the Philippines has the option to surrender such US national to the international tribunal if it Philippine State to surrender to the proper international tribunal those persons accused of crimes
decides not to prosecute such US national here. The view asserts that this option of the Philippines sanctioned under said law if it does not exercise its primary jurisdiction to prosecute such persons. This
under Sec. 17 of RA 9851 is not subject to the consent of the US, and any derogation of Sec. 17 of RA view is not entirely correct, for the above quoted proviso clearly provides discretion to the Philippine
State on whether to surrender or not a person accused of the crimes under RA 9851. The statutory 118, Part I, Title 18 of the United States Code Annotated (USCA) provides for the criminal offense of
proviso uses the word "may." It is settled doctrine in statutory construction that the word "may" denotes "war crimes" which is similar to the war crimes found in both the Rome Statute and RA 9851, thus:
73
discretion, and cannot be construed as having mandatory effect. Thus, the pertinent second
pararagraph of Sec. 17, RA 9851 is simply permissive on the part of the Philippine State.1avvphi1 (a) Offense – Whoever, whether inside or outside the United States, commits a war crime, in any of the
circumstances described in subsection (b), shall be fined under this title or imprisoned for life or any
Besides, even granting that the surrender of a person is mandatorily required when the Philippines does term of years, or both, and if death results to the victim, shall also be subject to the penalty of death.
not exercise its primary jurisdiction in cases where "another court or international tribunal is already
conducting the investigation or undertaking the prosecution of such crime," still, the tenor of (b) Circumstances – The circumstances referred to in subsection (a) are that the person committing
the Agreement is not repugnant to Sec. 17 of RA 9851. Said legal proviso aptly provides that the such war crime or the victim of such war crime is a member of the Armed Forces of the United States or
surrender may be made "to another State pursuant to the applicable extradition laws and treaties." The a national of the United States (as defined in Section 101 of the Immigration and Nationality Act).
Agreement can already be considered a treaty following this Court’s decision in Nicolas v.
74 75
Romulo which cited Weinberger v. Rossi. In Nicolas, We held that "an executive agreement is a (c) Definition – As used in this Section the term "war crime" means any conduct –
‘treaty’ within the meaning of that word in international law and constitutes enforceable domestic law
76 (1) Defined as a grave breach in any of the international conventions signed at Geneva 12 August 1949,
vis-à-vis the United States."
or any protocol to such convention to which the United States is a party;
Likewise, the Philippines and the US already have an existing extradition treaty, i.e., RP-US Extradition
(2) Prohibited by Article 23, 25, 27 or 28 of the Annex to the Hague Convention IV, Respecting the Laws
Treaty, which was executed on November 13, 1994. The pertinent Philippine law, on the other hand, is
and Customs of War on Land, signed 18 October 1907;
Presidential Decree No. 1069, issued on January 13, 1977. Thus, the Agreement, in conjunction with
the RP-US Extradition Treaty, would neither violate nor run counter to Sec. 17 of RA 9851. (3) Which constitutes a grave breach of common Article 3 (as defined in subsection [d]) when
77 committed in the context of and in association with an armed conflict not of an international character; or
The view’s reliance on Suplico v. Neda is similarly improper. In that case, several petitions were filed
questioning the power of the President to enter into foreign loan agreements. However, before the (4) Of a person who, in relation to an armed conflict and contrary to the provisions of the Protocol on
petitions could be resolved by the Court, the Office of the Solicitor General filed a Manifestation and Prohibitions or Restrictions on the Use of Mines, Booby-Traps and Other Devices as amended at
Motion averring that the Philippine Government decided not to continue with the ZTE National Geneva on 3 May 1996 (Protocol II as amended on 3 May 1996), when the United States is a party to
Broadband Network Project, thus rendering the petition moot. In resolving the case, the Court took 80
such Protocol, willfully kills or causes serious injury to civilians. 1avvphi1
judicial notice of the act of the executive department of the Philippines (the President) and found the
petition to be indeed moot. Accordingly, it dismissed the petitions. Similarly, in December 2009, the US adopted a law that criminalized genocide, to wit:

In his dissent in the abovementioned case, Justice Carpio discussed the legal implications of an §1091. Genocide
executive agreement. He stated that "an executive agreement has the force and effect of law x x x [it]
78
cannot amend or repeal prior laws." Hence, this argument finds no application in this case seeing as (a) Basic Offense – Whoever, whether in the time of peace or in time of war and with specific intent to
RA 9851 is a subsequent law, not a prior one. Notably, this argument cannot be found in the ratio destroy, in whole or in substantial part, a national, ethnic, racial or religious group as such–
decidendi of the case, but only in the dissenting opinion.
(1) kills members of that group;
The view further contends that the RP-US Extradition Treaty is inapplicable to RA 9851 for the reason
that under par. 1, Art. 2 of the RP-US Extradition Treaty, "[a]n offense shall be an extraditable offense if (2) causes serious bodily injury to members of that group;
79
it is punishable under the laws in both Contracting Parties x x x," and thereby concluding that while the
Philippines has criminalized under RA 9851 the acts defined in the Rome Statute as war crimes, (3) causes the permanent impairment of the mental faculties of members of the group through drugs,
genocide and other crimes against humanity, there is no similar legislation in the US. It is further argued torture, or similar techniques;
that, citing U.S. v. Coolidge, in the US, a person cannot be tried in the federal courts for an international
(4) subjects the group to conditions of life that are intended to cause the physical destruction of the
crime unless Congress adopts a law defining and punishing the offense.
group in whole or in part;
This view must fail.
(5) imposes measures intended to prevent births within the group; or
On the contrary, the US has already enacted legislation punishing the high crimes mentioned earlier. In
(6) transfers by force children of the group to another group;
fact, as early as October 2006, the US enacted a law criminalizing war crimes. Section 2441, Chapter
81
shall be punished as provided in subsection (b). within the group; the group in whole or in part;

Arguing further, another view has been advanced that the current US laws do not cover every crime (e) Forcibly transferring children of the group to (5) imposes measures intended to prevent births
listed within the jurisdiction of the ICC and that there is a gap between the definitions of the different another group. within the group; or
crimes under the US laws versus the Rome Statute. The view used a report written by Victoria K. Holt
and Elisabeth W. Dallas, entitled "On Trial: The US Military and the International Criminal Court," as its (6) transfers by force children of the group to
basis. another group;

At the outset, it should be pointed out that the report used may not have any weight or value under shall be punished as provided in subsection (b).
international law. Article 38 of the Statute of the International Court of Justice (ICJ) lists the sources of
international law, as follows: (1) international conventions, whether general or particular, establishing
rules expressly recognized by the contesting states; (2) international custom, as evidence of a general Article 8 (d) Definition – As used in this Section the term
practice accepted as law; (3) the general principles of law recognized by civilized nations; and (4) War Crimes "war crime" means any conduct –
subject to the provisions of Article 59, judicial decisions and the teachings of the most highly qualified
2. For the purpose of this Statute, "war crimes" (1) Defined as a grave breach in any of the
publicists of the various nations, as subsidiary means for the determination of rules of law. The report
means: international conventions signed at Geneva 12
does not fall under any of the foregoing enumerated sources. It cannot even be considered as the
August 1949, or any protocol to such convention
"teachings of highly qualified publicists." A highly qualified publicist is a scholar of public international (a) Grave breaches of the Geneva Conventions of
82 to which the United States is a party;
law and the term usually refers to legal scholars or "academic writers." It has not been shown that the 12 August 1949, namely, any of the following acts
83
authors of this report are highly qualified publicists. against persons or property protected under the (2) Prohibited by Article 23, 25, 27 or 28 of the
provisions of the relevant Geneva Convention: x x Annex to the Hague Convention IV, Respecting
Assuming arguendo that the report has weight, still, the perceived gaps in the definitions of the crimes 84
x the Laws and Customs of War on Land, signed
are nonexistent. To highlight, the table below shows the definitions of genocide and war crimes under
18 October 1907;
the Rome Statute vis-à-vis the definitions under US laws: (b) Other serious violations of the laws and customs
applicable in international armed conflict, within the (3) Which constitutes a grave breach of common
Rome Statute US Law 85
established framework of international law, namely, Article 3 (as defined in subsection [d] ) when
any of the following acts: committed in the context of and in association
Article 6 §1091. Genocide with an armed conflict not of an international
xxxx character; or
Genocide
(a) Basic Offense – Whoever, whether in the
(c) In the case of an armed conflict not of an (4) Of a person who, in relation to an armed
For the purpose of this Statute, "genocide" means time of peace or in time of war and with specific
international character, serious violations of article 3 conflict and contrary to the provisions of the
any of the following acts committed with intent to intent to destroy, in whole or in substantial part, a
common to the four Geneva Conventions of 12 Protocol on Prohibitions or Restrictions on the
destroy, in whole or in part, a national, ethnical, national, ethnic, racial or religious group as
August 1949, namely, any of the following acts Use of Mines, Booby-Traps and Other Devices
racial or religious group, as such: such–
committed against persons taking no active part in as amended at Geneva on 3 May 1996 (Protocol
(a) Killing members of the group; (1) kills members of that group; the hostilities, including members of armed forces II as amended on 3 May 1996), when the United
who have laid down their arms and those placed States is a party to such Protocol, willfully kills or
hors de combat by sickness, wounds, detention or 86
(b) Causing serious bodily or mental harm to (2) causes serious bodily injury to members of causes serious injury to civilians.
members of the group; that group; any other cause:

(c) Deliberately inflicting on the group conditions of (3) causes the permanent impairment of the xxxx
life calculated to bring about its physical destruction mental faculties of members of the group
in whole or in part; through drugs, torture, or similar techniques; (d) Paragraph 2 (c) applies to armed conflicts not of
an international character and thus does not apply to
(d) Imposing measures intended to prevent births (4) subjects the group to conditions of life that situations of internal disturbances and tensions,
are intended to cause the physical destruction of such as riots, isolated and sporadic acts of violence
or other acts of a similar nature. its history this Court has recognized and applied the law of war as including that part of the law of
nations which prescribes, for the conduct of war, the status, rights and duties of enemy nations as well
97
(e) Other serious violations of the laws and customs as of enemy individuals." It went on further to explain that Congress had not undertaken the task of
applicable in armed conflicts not of an international codifying the specific offenses covered in the law of war, thus:
character, within the established framework of
international law, namely, any of the following acts: x It is no objection that Congress in providing for the trial of such offenses has not itself undertaken to
x x. codify that branch of international law or to mark its precise boundaries, or to enumerate or define by
statute all the acts which that law condemns. An Act of Congress punishing ‘the crime of piracy as
defined by the law of nations is an appropriate exercise of its constitutional authority, Art. I, s 8, cl. 10,
Evidently, the gaps pointed out as to the definition of the crimes are not present. In fact, the report itself
‘to define and punish’ the offense since it has adopted by reference the sufficiently precise definition of
stated as much, to wit:
international law. x x x Similarly by the reference in the 15th Article of War to ‘offenders or offenses that
Few believed there were wide differences between the crimes under the jurisdiction of the Court and x x x by the law of war may be triable by such military commissions. Congress has incorporated by
crimes within the Uniform Code of Military Justice that would expose US personnel to the Court. Since reference, as within the jurisdiction of military commissions, all offenses which are defined as such by
98
US military lawyers were instrumental in drafting the elements of crimes outlined in the Rome Statute, the law of war x x x, and which may constitutionally be included within that jurisdiction. x x x
they ensured that most of the crimes were consistent with those outlined in the UCMJ and gave (Emphasis supplied.)
strength to complementarity for the US. Small areas of potential gaps between the UCMJ and the Rome
87 This rule finds an even stronger hold in the case of crimes against humanity. It has been held that
Statute, military experts argued, could be addressed through existing military laws. x x x
genocide, war crimes and crimes against humanity have attained the status of customary international
99
The report went on further to say that "[a]ccording to those involved, the elements of crimes laid out in law. Some even go so far as to state that these crimes have attained the status of jus cogens.
88
the Rome Statute have been part of US military doctrine for decades." Thus, the argument proffered
Customary international law or international custom is a source of international law as stated in the
cannot stand. 100
Statute of the ICJ. It is defined as the "general and consistent practice of states recognized and
101
Nonetheless, despite the lack of actual domestic legislation, the US notably follows the doctrine of followed by them from a sense of legal obligation." In order to establish the customary status of a
89
incorporation. As early as 1900, the esteemed Justice Gray in The Paquete Habana case already held particular norm, two elements must concur: State practice, the objective element; and opinio juris sive
102
international law as part of the law of the US, to wit: necessitates, the subjective element.

International law is part of our law, and must be ascertained and administered by the courts of justice of State practice refers to the continuous repetition of the same or similar kind of acts or norms by
103
appropriate jurisdiction as often as questions of right depending upon it are duly presented for their States. It is demonstrated upon the existence of the following elements: (1) generality; (2) uniformity
104
determination. For this purpose, where there is no treaty and no controlling executive or legislative act and consistency; and (3) duration. While, opinio juris, the psychological element, requires that the
or judicial decision, resort must be had to the customs and usages of civilized nations, and, as evidence state practice or norm "be carried out in such a way, as to be evidence of a belief that this practice is
105
of these, to the works of jurists and commentators who by years of labor, research, and experience rendered obligatory by the existence of a rule of law requiring it."
have made themselves peculiarly well acquainted with the subjects of which they treat. Such works are 106
"The term ‘jus cogens’ means the ‘compelling law.’" Corollary, "a jus cogens norm holds the highest
resorted to by judicial tribunals, not for the speculations of their authors concerning what the law ought 107
90 hierarchical position among all other customary norms and principles." As a result, jus cogens norms
to be, but for the trustworthy evidence of what the law really is. (Emphasis supplied.) 108
are deemed "peremptory and non-derogable." When applied to international crimes, "jus cogens
Thus, a person can be tried in the US for an international crime despite the lack of domestic legislation. crimes have been deemed so fundamental to the existence of a just international legal order that states
109
91 92
The cited ruling in U.S. v. Coolidge, which in turn is based on the holding in U.S. v. Hudson, only cannot derogate from them, even by agreement."
93
applies to common law and not to the law of nations or international law. Indeed, the Court in U.S. v.
These jus cogens crimes relate to the principle of universal jurisdiction, i.e., "any state may exercise
Hudson only considered the question, "whether the Circuit Courts of the United States can exercise a
94 jurisdiction over an individual who commits certain heinous and widely condemned offenses, even when
common law jurisdiction in criminal cases." Stated otherwise, there is no common law crime in the US 110
no other recognized basis for jurisdiction exists." "The rationale behind this principle is that the crime
but this is considerably different from international law.
committed is so egregious that it is considered to be committed against all members of the international
111 112
The US doubtless recognizes international law as part of the law of the land, necessarily including community" and thus granting every State jurisdiction over the crime.
95
international crimes, even without any local statute. In fact, years later, US courts would apply
Therefore, even with the current lack of domestic legislation on the part of the US, it still has both the
international law as a source of criminal liability despite the lack of a local statute criminalizing it as
96 doctrine of incorporation and universal jurisdiction to try these crimes.
such. So it was that in Ex Parte Quirin the US Supreme Court noted that "[f]rom the very beginning of
Consequently, no matter how hard one insists, the ICC, as an international tribunal, found in the Rome Absent the widespread/consistent-practice-of-states factor, the second or the psychological element
Statute is not declaratory of customary international law. must be deemed non-existent, for an inquiry on why states behave the way they do presupposes, in the
first place, that they are actually behaving, as a matter of settled and consistent practice, in a certain
The first element of customary international law, i.e., "established, widespread, and consistent practice manner. This implicitly requires belief that the practice in question is rendered obligatory by the
113 117
on the part of States," does not, under the premises, appear to be obtaining as reflected in this simple existence of a rule of law requiring it. Like the first element, the second element has likewise not been
114
reality: As of October 12, 2010, only 114 States have ratified the Rome Statute, subsequent to its shown to be present.
coming into force eight (8) years earlier, or on July 1, 2002. The fact that 114 States out of a total of
115
194 countries in the world, or roughly 58.76%, have ratified the Rome Statute casts doubt on whether Further, the Rome Statute itself rejects the concept of universal jurisdiction over the crimes enumerated
118
or not the perceived principles contained in the Statute have attained the status of customary law and therein as evidenced by it requiring State consent. Even further, the Rome Statute specifically and
should be deemed as obligatory international law. The numbers even tend to argue against the urgency unequivocally requires that: "This Statute is subject to ratification, acceptance or approval by signatory
119
of establishing international criminal courts envisioned in the Rome Statute. Lest it be overlooked, the States." These clearly negate the argument that such has already attained customary status.
Philippines, judging by the action or inaction of its top officials, does not even feel bound by the Rome
Statute. Res ipsa loquitur. More than eight (8) years have elapsed since the Philippine representative More importantly, an act of the executive branch with a foreign government must be afforded great
signed the Statute, but the treaty has not been transmitted to the Senate for the ratification process. respect. The power to enter into executive agreements has long been recognized to be lodged with the
President. As We held in Neri v. Senate Committee on Accountability of Public Officers and
And this brings us to what Fr. Bernas, S.J. aptly said respecting the application of the concurring Investigations, "[t]he power to enter into an executive agreement is in essence an executive power. This
elements, thus: authority of the President to enter into executive agreements without the concurrence of the Legislature
120
has traditionally been recognized in Philippine jurisprudence." The rationale behind this principle is
Custom or customary international law means "a general and consistent practice of states followed by the inviolable doctrine of separation of powers among the legislative, executive and judicial branches of
them from a sense of legal obligation [opinio juris] x x x." This statement contains the two basic the government. Thus, absent any clear contravention of the law, courts should exercise utmost caution
elements of custom: the material factor, that is how the states behave, and the psychological factor or in declaring any executive agreement invalid.
subjective factor, that is, why they behave the way they do.
In light of the above consideration, the position or view that the challenged RP-US Non-Surrender
xxxx Agreement ought to be in the form of a treaty, to be effective, has to be rejected.

The initial factor for determining the existence of custom is the actual behavior of states. This includes WHEREFORE, the petition for certiorari, mandamus and prohibition is hereby DISMISSED for lack of
several elements: duration, consistency, and generality of the practice of states. merit. No costs.

The required duration can be either short or long. x x x SO ORDERED.

xxxx PRESBITERO J. VELASCO, JR.


Associate Justice
Duration therefore is not the most important element. More important is the consistency and the
generality of the practice. x x x

xxxx

Once the existence of state practice has been established, it becomes necessary to determine
why states behave the way they do. Do states behave the way they do because they consider it
obligatory to behave thus or do they do it only as a matter of courtesy? Opinio juris, or the belief that a
certain form of behavior is obligatory, is what makes practice an international rule. Without it, practice is
116
not law. (Emphasis added.)

Evidently, there is, as yet, no overwhelming consensus, let alone prevalent practice, among the different
countries in the world that the prosecution of internationally recognized crimes of genocide, etc. should
be handled by a particular international criminal court.
lack of consensus was eventually cured when the two nations concluded the Visiting Forces Agreement
(V FA) in 1999.

The entry of American troops into Philippine soil is proximately rooted in the international anti-terrorism
campaign declared by President George W. Bush in reaction to the tragic events that occurred on
September 11, 2001. On that day, three (3) commercial aircrafts were hijacked, flown and smashed into
the twin towers of the World Trade Center in New York City and the Pentagon building in Washington,
D.C. by terrorists with alleged links to the al-Qaeda ("the Base"), a Muslim extremist organization
headed by the infamous Osama bin Laden. Of no comparable historical parallels, these acts caused
EN BANC billions of dollars worth of destruction of property and incalculable loss of hundreds of lives.

G.R. No. 151445 April 11, 2002 On February 1, 2002, petitioners Arthur D. Lim and Paulino P. Ersando filed this petition for certiorari
2
and prohibition, attacking the constitutionality of the joint exercise. They were joined subsequently by
ARTHUR D. LIM and PAULINO R. ERSANDO, petitioners, SANLAKAS and PARTIDO NG MANGGAGAWA, both party-Iist organizations, who filed a petition-in-
vs. intervention on February 11, 2002.
HONORABLE EXECUTIVE SECRETARY as alter ego of HER EXCELLENCEY GLORIA
MACAPAGAL-ARROYO, and HONORABLE ANGELO REYES in his capacity as Secretary of Lim and Ersando filed suit in their capacities as citizens, lawyers and taxpayers. SANLAKAS and
National Defense, respondents. PARTIDO, on the other hand, aver that certain members of their organization are residents of
Zamboanga and Sulu, and hence will be directly affected by the operations being conducted in
---------------------------------------- Mindanao. They likewise pray for a relaxation on the rules relative to locus standi citing the
unprecedented importance of the issue involved.
SANLAKAS and PARTIDO NG MANGGAGAWA, petitioners-intervenors,
vs. On February 71 2002 the Senate conducted a hearing on the "Balikatan" exercise wherein Vice-
GLORIA MACAPAGA-ARROYO, ALBERTO ROMULO, ANGELO REYES, respondents. President Teofisto T. Guingona, Jr., who is concurrently Secretary of Foreign. Affairs, presented the
3
Draft Terms of Reference (TOR). Five days later, he approved the TOR, which we quote hereunder:
DE LEON, JR., J.:
I. POLICY LEVEL
This case involves a petition for certiorari and prohibition as well as a petition-in-intervention, praying
that respondents be restrained from proceeding with the so-called "Balikatan 02-1" and that after due 1. The Exercise shall be consistent with the Philippine Constitution and all its activities shall be in
notice and hearing, that judgment be rendered issuing a permanent writ of injunction and/or prohibition consonance with the laws of the land and the provisions of the RP-US Visiting Forces Agreement
against the deployment of U.S. troops in Basilan and Mindanao for being illegal and in violation of the (VFA).
Constitution.
2. The conduct of this training Exercise is in accordance with pertinent United Nations resolutions
The facts are as follows: against global terrorism as understood by the respective parties.

Beginning January of this year 2002, personnel from the armed forces of the United States of America 3. No permanent US basing and support facilities shall be established. Temporary structures such as
started arriving in Mindanao to take part, in conjunction with the Philippine military, in "Balikatan 02-1." those for troop billeting, classroom instruction and messing may be set up for use by RP and US Forces
These so-called "Balikatan" exercises are the largest combined training operations involving Filipino during the Exercise.
and American troops. In theory, they are a simulation of joint military maneuvers pursuant to the Mutual
1
Defense Treaty, a bilateral defense agreement entered into by the Philippines and the United States in 4. The Exercise shall be implemented jointly by RP and US Exercise Co-Directors under the authority of
1951. the Chief of Staff, AFP. In no instance will US Forces operate independently during field training
exercises (FTX). AFP and US Unit Commanders will retain command over their respective forces under
Prior to the year 2002, the last "Balikatan" was held in 1995. This was due to the paucity of any formal the overall authority of the Exercise Co-Directors. RP and US participants shall comply with operational
agreement relative to the treatment of United States personnel visiting the Philippines. In the meantime, instructions of the AFP during the FTX.
the respective governments of the two countries agreed to hold joint exercises on a reduced scale. The
5. The exercise shall be conducted and completed within a period of not more than six months, with the
projected participation of 660 US personnel and 3,800 RP Forces. The Chief of Staff, AFP shall direct
the Exercise Co-Directors to wind up and terminate the Exercise and other activities within the six b. Local media relations will be the concern of the AFP and all public affairs guidelines shall be jointly
month Exercise period. developed by RP and US Forces.

6. The Exercise is a mutual counter-terrorism advising, assisting and training Exercise relative to c. Socio-Economic Assistance Projects shall be planned and executed jointly by RP and US Forces in
Philippine efforts against the ASG, and will be conducted on the Island of Basilan. Further advising, accordance with their respective laws and regulations, and in consultation with community and local
assisting and training exercises shall be conducted in Malagutay and the Zamboanga area. Related government officials.
activities in Cebu will be for support of the Exercise.
Contemporaneously, Assistant Secretary for American Affairs Minerva Jean A. Falcon and United
7. Only 160 US Forces organized in 12-man Special Forces Teams shall be deployed with AFP field, States Charge d' Affaires Robert Fitts signed the Agreed Minutes of the discussion between the Vice-
4
commanders. The US teams shall remain at the Battalion Headquarters and, when approved, Company President and Assistant Secretary Kelly.
Tactical headquarters where they can observe and assess the performance of the AFP Forces.
Petitioners Lim and Ersando present the following arguments:
8. US exercise participants shall not engage in combat, without prejudice to their right of self-defense.
I
9. These terms of Reference are for purposes of this Exercise only and do not create additional legal
obligations between the US Government and the Republic of the Philippines. THE PHILIPPINES AND THE UNITED STATES SIGNED THE MUTUAL DEFENSE TREATY (MDT) in
1951 TO PROVIDE MUTUAL MILITARY ASSIST ANCE IN ACCORDANCE WITH THE
II. EXERCISE LEVEL 'CONSTITUTIONAL PROCESSE-S' OF EACH COUNTRY ONLY IN THE CASE OF AN ARMED
ATTACK BY AN EXTERNAL AGGRESSOR, MEANING A THIRD COUNTRY AGAINST ONE OF
1. TRAINING THEM.

a. The Exercise shall involve the conduct of mutual military assisting, advising and training of RP and BY NO STRETCH OF THE IMAGINA TION CAN IT BE SAID THAT THE ABU SAYYAF BANDITS IN
US Forces with the primary objective of enhancing the operational capabilities of both forces to combat BASILAN CONSTITUTE AN EXTERNAL ARMED FORCE THAT HAS SUBJECT THE PHILIPPINES
terrorism. TO AN ARMED EXTERNAL ATTACK TO WARRANT U.S. MILITARY ASSISTANCE UNDER THE
MDT OF 1951.
b. At no time shall US Forces operate independently within RP territory.
II
c. Flight plans of all aircraft involved in the exercise will comply with the local air traffic regulations.
NEITHER DOES THE VFA OF 1999 AUTHORIZE AMERICAN SOLDIERS TO ENGAGE IN COMBAT
2. ADMINISTRATION & LOGISTICS OPERATIONS IN PHILIPPINE TERRITORY, NOT EVEN TO FIRE BACK "IF FIRED UPON".

a. RP and US participants shall be given a country and area briefing at the start of the Exercise. This Substantially the same points are advanced by petitioners SANLAKAS and PARTIDO.
briefing shall acquaint US Forces on the culture and sensitivities of the Filipinos and the provisions of
the VF A. The briefing shall also promote the full cooperation on the part of the RP and US participants In his Comment, the Solicitor General points to infirmities in the petitions regarding, inter alia, Lim and
for the successful conduct of the Exercise. Ersando's standing to file suit, the prematurity of the action, as well as the impropriety of availing
of certiorari to ascertain a question of fact. Anent their locus standi, the Solicitor General argues
b. RP and US participating forces may share, in accordance with their respective laws and regulations, that first, they may not file suit in their capacities as, taxpayers inasmuch as it has not been shown that
in the use of their resources, equipment and other assets. They will use their respective logistics "Balikatan 02-1 " involves the exercise of Congress' taxing or spending powers. Second, their being
channels. lawyers does not invest them with sufficient personality to initiate the case, citing our ruling
5
in Integrated Bar of the Philippines v. Zamora. Third, Lim and Ersando have failed to demonstrate
c. Medical evaluation shall be jointly planned and executed utilizing RP and US assets and resources.
the requisite showing of direct personal injury. We agree.
d. Legal liaison officers from each respective party shall be appointed by the Exercise Directors.
It is also contended that the petitioners are indulging in speculation. The Solicitor General is of the view
3. PUBLIC AFFAIRS that since the Terms of Reference are clear as to the extent and duration of "Balikatan 02-1," the issues
raised by petitioners are premature, as they are based only on a fear of future violation of the Terms of
a. Combined RP-US Information Bureaus shall be established at the Exercise Directorate in Reference. Even petitioners' resort to a special civil action for certiorari is assailed on the ground that
Zamboanga City and at GHQ, AFP in Camp Aguinaldo, Quezon City. the writ may only issue on the basis of established facts.
Apart from these threshold issues, the Solicitor General claims that there is actually no question of through joint training with its American counterparts; the "Balikatan" is the largest such training exercise
constitutionality involved. The true object of the instant suit, it is said, is to obtain an interpretation of the directly supporting the MDT's objectives. It is this treaty to which the V FA adverts and the obligations
V FA. The Solicitor General asks that we accord due deference to the executive determination that thereunder which it seeks to reaffirm.
"Balikatan 02-1" is covered by the VFA, considering the President's monopoly in the field of foreign
relations and her role as commander-in-chief of the Philippine armed forces. The lapse of the US-Philippine Bases Agreement in 1992 and the decision not to renew it created a
vacuum in US-Philippine defense relations, that is, until it was replaced by the Visiting Forces
Given the primordial importance of the issue involved, it will suffice to reiterate our view on this point in Agreement. It should be recalled that on October 10, 2000, by a vote of eleven to three, this Court
7
a related case: upheld the validity of the VFA. The V FA provides the "regulatory mechanism" by which "United States
military and civilian personnel [may visit] temporarily in the Philippines in connection with activities
Notwithstanding, in view of the paramount importance and the constitutional significance of the issues approved by the Philippine Government." It contains provisions relative to entry and departure of
raised in the petitions, this Court, in the exercise of its sound discretion, brushes aside the procedural American personnel, driving and vehicle registration, criminal jurisdiction, claims, importation and
barrier and takes cognizance of the petitions, as we have done in the early Emergency Powers exportation, movement of vessels and aircraft, as well as the duration of the agreement and its
Cases, where we had occasion to rule: termination. It is the VFA which gives continued relevance to the MDT despite the passage of years. Its
primary goal is to facilitate the promotion of optimal cooperation between American and Philippine
'x x x ordinary citizens and taxpayers were allowed to question the constitutionality of several executive military forces in the event of an attack by a common foe.
orders issued by President Quirino although they were involving only an indirect and general interest
shared in common with the public. The Court dismissed the objection that they were not proper parties The first question that should be addressed is whether "Balikatan 02-1" is covered by the Visiting
and ruled that 'transcendental importance to the public of these cases demands that they be Forces Agreement. To resolve this, it is necessary to refer to the V FA itself: Not much help can be had
settled promptly and definitely, brushing aside, if we must, technicalities of procedure.' We have therefrom, unfortunately, since the terminology employed is itself the source of the problem. The VFA
since then applied the exception in many other cases. [citation omitted] permits United States personnel to engage, on an impermanent basis, in "activities," the exact meaning
of which was left undefined. The expression is ambiguous, permitting a wide scope of undertakings
This principle was reiterated in the subsequent cases of Gonzales vs. COMELEC, Daza vs. Singson, 8
subject only to the approval of the Philippine government. The sole encumbrance placed on its
and Basco vs. Phil, Amusement and Gaming Corporation, where we emphatically held: definition is couched in the negative, in that United States personnel must "abstain from any
9
activity inconsistent with the spirit of this agreement, and in particular, from any political activity." All
Considering however the importance to the public of the case at bar, and in keeping with the Court's
other activities, in other words, are fair game.
duty, under the 1987 Constitution, to determine whether or not the other branches of the government
have kept themselves within the limits of the Constitution and the laws that they have not abused the We are not left completely unaided, however. The Vienna Convention on the Law of Treaties, which
discretion given to them, the Court has brushed aside technicalities of procedure and has taken contains provisos governing interpretations of international agreements, state:
cognizance of this petition. xxx'
SECTION 3. INTERPRETATION OF TREATIES
Again, in the more recent case of Kilosbayan vs. Guingona, Jr., this Court ruled that in cases of
transcendental importance, the Court may relax the standing requirements and allow a suit to Article 31
prosper even where there is no direct injury to the party claiming the right of judicial review.
General rule of interpretation
Although courts generally avoid having to decide a constitutional question based on the doctrine of
separation of powers, which enjoins upon the department of the government a becoming respect for 1. A treaty shall be interpreted in good faith ill accordance with the ordinary meaning to be given to the
6
each other's act, this Court nevertheless resolves to take cognizance of the instant petition. tenus of the treaty in their context and in the light of its object and purpose.

Hence, we treat with similar dispatch the general objection to the supposed prematurity of the action. At 2. The context for the purpose of the interpretation of a treaty shall comprise, in addition to the text,
any rate, petitioners' concerns on the lack of any specific regulation on the latitude of activity US including its preamble and annexes:
personnel may undertake and the duration of their stay has been addressed in the Terms of Reference.
(a) any agreement relating to the treaty which was made between all the parties in connexion with the
The holding of "Balikatan 02-1" must be studied in the framework of the treaty antecedents to which the conclusion of the treaty;
Philippines bound itself. The first of these is the Mutual Defense Treaty (MDT, for brevity). The MDT has
been described as the "core" of the defense relationship between the Philippines and its traditional ally, (b) any instrument which was made by one or more parties in connexion with the conclusion of the
the United States. Its aim is to enhance the strategic and technological capabilities of our armed forces treaty and accepted by the other parties as an instrument related to the party .
3. There shall be taken into account, together with the context: certain leeway in negotiation. In this manner, visiting US forces may sojourn in Philippine territory for
purposes other than military. As conceived, the joint exercises may include training on new techniques
(a) any subsequent agreement between the parties regarding the interpretation of the treaty or the of patrol and surveillance to protect the nation's marine resources, sea search-and-rescue operations to
application of its provisions; assist vessels in distress, disaster relief operations, civic action projects such as the building of school
houses, medical and humanitarian missions, and the like.
(b) any subsequent practice in the application of the treaty which establishes the agreement of the
parties regarding its interpretation; Under these auspices, the VFA gives legitimacy to the current Balikatan exercises. It is only logical to
assume that .'Balikatan 02-1," a "mutual anti- terrorism advising, assisting and training exercise," falls
(c) any relevant rules of international law applicable in the relations between the parties. under the umbrella of sanctioned or allowable activities in the context of the agreement. Both the history
and intent of the Mutual Defense Treaty and the V FA support the conclusion that combat-
4. A special meaning shall be given to a term if it is established that the parties so intended.
related activities -as opposed to combat itself -such as the one subject of the instant petition, are indeed
Article 32 authorized.

Supplementary means of interpretation That is not the end of the matter, though. Granted that "Balikatan 02-1" is permitted under the terms of
the VFA, what may US forces legitimately do in furtherance of their aim to provide advice, assistance
Recourse may be had to supplementary means of interpretation, including the preparatory work of the and training in the global effort against terrorism? Differently phrased, may American troops actually
treaty and the circumstances of its conclusion, in order to confirm the meaning resulting from the engage in combat in Philippine territory? The Terms of Reference are explicit enough. Paragraph 8 of
application of article 31, or to determine the meaning when the interpretation according to article 31 : section I stipulates that US exercise participants may not engage in combat "except in self-
defense." We wryly note that this sentiment is admirable in the abstract but difficult in implementation.
(a) leaves the meaning ambiguous or obscure; or The target of "Balikatan 02-1 I" the Abu Sayyaf, cannot reasonably be expected to sit idly while the
battle is brought to their very doorstep. They cannot be expected to pick and choose their targets for
(b) leads to a result which is manifestly absurd unreasonable. they will not have the luxury of doing so. We state this point if only to signify our awareness that the
parties straddle a fine line, observing the honored legal maxim "Nemo potest facere per alium quod non
It is clear from the foregoing that the cardinal rule of interpretation must involve an examination of the 11
potest facere per directum." The indirect violation is actually petitioners' worry, that in reality,
text, which is presumed to verbalize the parties' intentions. The Convention likewise dictates what may "Balikatan 02-1 " is actually a war principally conducted by the United States government, and that the
be used as aids to deduce the meaning of terms, which it refers to as the context of the treaty, as well provision on self-defense serves only as camouflage to conceal the true nature of the exercise. A clear
as other elements may be taken into account alongside the aforesaid context. As explained by a writer pronouncement on this matter thereby becomes crucial.
on the Convention ,
In our considered opinion, neither the MDT nor the V FA allow foreign troops to engage in an offensive
[t]he Commission's proposals (which were adopted virtually without change by the conference and are war on Philippine territory. We bear in mind the salutary proscription stated in the Charter of the United
now reflected in Articles 31 and 32 of the Convention) were clearly based on the view that the text of a Nations, to wit:
treaty must be presumed to be the authentic expression of the intentions of the parties; the Commission
accordingly came down firmly in favour of the view that 'the starting point of interpretation is the Article 2
elucidation of the meaning of the text, not an investigation ab initio into the intentions of the parties'.
This is not to say that the travauxpreparatoires of a treaty , or the circumstances of its conclusion, are The Organization and its Members, in pursuit of the Purposes stated in Article 1, shall act in accordance
relegated to a subordinate, and wholly ineffective, role. As Professor Briggs points out, no rigid temporal with the following Principles.
prohibition on resort to travaux preparatoires of a treaty was intended by the use of the phrase
'supplementary means of interpretation' in what is now Article 32 of the Vienna Convention. The xxx xxx xxx xxx
distinction between the general rule of interpretation and the supplementary means of interpretation is
intended rather to ensure that the supplementary means do not constitute an alternative, autonomous 4. All Members shall refrain in their international relations from the threat or use of force against the
method of interpretation divorced from the general rule.
10 territorial integrity or political independence of any state, or in any other manner inconsistent with the
Purposes of the United Nations.
The Terms of Reference rightly fall within the context of the VFA.
xxx xxx xxx xxx
After studied reflection, it appeared farfetched that the ambiguity surrounding the meaning of the word
.'activities" arose from accident. In our view, it was deliberately made that way to give both parties a
In the same manner, both the Mutual Defense Treaty and the Visiting Forces Agreement, as in all other This is not exactly helpful in solving the problem at hand since in trying to find a middle ground, it favors
treaties and international agreements to which the Philippines is a party, must be read in the context of neither one law nor the other, which only leaves the hapless seeker with an unsolved dilemma. Other
the 1987 Constitution. In particular, the Mutual Defense Treaty was concluded way before the present more traditional approaches may offer valuable insights.
Charter, though it nevertheless remains in effect as a valid source of international obligation. The
present Constitution contains key provisions useful in determining the extent to which foreign military From the perspective of public international law, a treaty is favored over municipal law pursuant to the
troops are allowed in Philippine territory. Thus, in the Declaration of Principles and State Policies, it is principle of pacta sunt servanda. Hence, "[e]very treaty in force is binding upon the parties to it and
14
provided that: must be performed by them in good faith." Further, a party to a treaty is not allowed to "invoke the
15
provisions of its internal law as justification for its failure to perform a treaty."
xxx xxx xxx xxx
Our Constitution espouses the opposing view. Witness our jurisdiction as I stated in section 5 of Article
SEC. 2. The Philippines renounces war as an instrument of national policy, adopts the generally VIII:
accepted principles of international law as part of the law of the land and adheres to the policy of peace,
equality, justice, freedom, cooperation, and amity with all nations. The Supreme Court shall have the following powers:

xxx xxx xxx xxx xxx xxx xxx xxx

SEC. 7. The State shall pursue an independent foreign policy. In its relations with other states the (2) Review, revise, reverse, modify, or affirm on appeal or certiorari, as the law or the Rules of Court
paramount consideration shall be national sovereignty, territorial integrity, national interest, and the right may provide, final judgments and order of lower courts in:
to self- determination.
(A) All cases in which the constitutionality or validity of any treaty, international or executive
SEC. 8. The Philippines, consistent with the national interest, adopts and pursues a policy of freedom agreement, law, presidential decree, proclamation, order, instruction, ordinance, or regulation is in
from nuclear weapons in the country. question.

xxx xxx xxx xxx xxx xxx xxx xxx


16
The Constitution also regulates the foreign relations powers of the Chief Executive when it provides that In Ichong v. Hernandez, we ruled that the provisions of a treaty are always subject to qualification or
"[n]o treaty or international agreement shall be valid and effective unless concurred in by at least two- amendment by a subsequent law, or that it is subject to the police power of the State. In Gonzales v.
12 17
thirds of all the members of the Senate." Even more pointedly, the Transitory Provisions state: Hechanova,

Sec. 25. After the expiration in 1991 of the Agreement between the Republic of the Philippines and the xxx As regards the question whether an international agreement may be invalidated by our courts,
United States of America concerning Military Bases, foreign military bases, troops or facilities shall not suffice it to say that the Constitution of the Philippines has clearly settled it in the affirmative, by
be allowed in the Philippines except under a treaty duly concurred in by the Senate and, when the providing, in Section 2 of Article VIII thereof, that the Supreme Court may not be deprived "of its
Congress so requires, ratified by a majority of the votes cast by the people in a national referendum jurisdiction to review, revise, reverse, modify, or affirm on appeal, certiorari, or writ of error as the law or
held for that purpose, and recognized as a treaty by the other contracting state. the rules of court may provide, final judgments and decrees of inferior courts in -( I) All cases in which
the constitutionality or validity of any treaty, law, ordinance, or executive order or regulation is in
The aforequoted provisions betray a marked antipathy towards foreign military presence in the country, question." In other words, our Constitution authorizes the nullification of a treaty, not only when it
or of foreign influence in general. Hence, foreign troops are allowed entry into the Philippines only by conflicts with the fundamental law, but, also, when it runs counter to an act of Congress.
way of direct exception. Conflict arises then between the fundamental law and our obligations arising
from international agreements. The foregoing premises leave us no doubt that US forces are prohibited / from engaging in an offensive
war on Philippine territory.
A rather recent formulation of the relation of international law vis-a-vis municipal law was expressed
13 Yet a nagging question remains: are American troops actively engaged in combat alongside Filipino
in Philip Morris, Inc. v. Court of Appeals, to wit:
soldiers under the guise of an alleged training and assistance exercise? Contrary to what petitioners
xxx Withal, the fact that international law has been made part of the law of the land does not by any would have us do, we cannot take judicial notice of the events transpiring down south,18 as reported
means imply the primacy of international law over national law in the municipal sphere. Under the from the saturation coverage of the media. As a rule, we do not take cognizance of newspaper or
doctrine of incorporation as applied in most countries, rules of international law are given a standing electronic reports per se, not because of any issue as to their truth, accuracy, or impartiality, but for the
equal, not superior, to national legislation. simple reason that facts must be established in accordance with the rules of evidence. As a result, we
cannot accept, in the absence of concrete proof, petitioners' allegation that the Arroyo government is
engaged in "doublespeak" in trying to pass off as a mere training exercise an offensive effort by foreign
troops on native soil. The petitions invite us to speculate on what is really happening in Mindanao, to
issue I make factual findings on matters well beyond our immediate perception, and this we are
understandably loath to do.

It is all too apparent that the determination thereof involves basically a question of fact. On this point, we
must concur with the Solicitor General that the present subject matter is not a fit topic for a special civil
action for certiorari. We have held in too many instances that questions of fact are not entertained in
such a remedy. The sole object of the writ is to correct errors of jurisdiction or grave abuse of discretion:
The phrase "grave abuse of discretion" has a precise meaning in law, denoting abuse of discretion "too
patent and gross as to amount to an evasion of a positive duty, or a virtual refusal to perform the duty
enjoined or act in contemplation of law, or where the power is exercised in an arbitrary and despotic
19
manner by reason of passion and personal hostility."
20
In this connection, it will not be amiss to add that the Supreme Court is not a trier of facts.

Under the expanded concept of judicial power under the Constitution, courts are charged with the duty
"to determine whether or not there has been a grave abuse of discretion amounting to lack or excess of
21
jurisdiction on the part of any branch or instrumentality of the government." From the facts obtaining,
we find that the holding of "Balikatan 02-1" joint military exercise has not intruded into that penumbra of
error that would otherwise call for correction on our part. In other words, respondents in the case at bar
have not committed grave abuse of discretion amounting to lack or excess of jurisdiction.

WHEREFORE, the petition and the petition-in-intervention are hereby DISMISSED without prejudice to
the filing of a new petition sufficient in form and substance in the proper Regional Trial Court.

SO ORDERED.
Republic of the Philippines These are petitions for certiorari, etc. as special civil actions and/or for review of the Decision of the
SUPREME COURT Court of Appeals in Lance Corporal Daniel J. Smith v. Hon. Benjamin T. Pozon, et al., in CA-G.R. SP
Manila No. 97212, dated January 2, 2007.

EN BANC The facts are not disputed.

G.R. No. 175888 February 11, 2009 Respondent Lance Corporal (L/CPL) Daniel Smith is a member of the United States Armed Forces. He
was charged with the crime of rape committed against a Filipina, petitioner herein, sometime on
SUZETTE NICOLAS y SOMBILON, Petitioner, November 1, 2005, as follows:
vs.
ALBERTO ROMULO, in his capacity as Secretary of Foreign Affairs; RAUL GONZALEZ, in his The undersigned accused LCpl. Daniel Smith, Ssgt. Chad Brian Carpentier, Dominic Duplantis, Keith
capacity as Secretary of Justice; EDUARDO ERMITA, in his capacity as Executive Secretary; Silkwood and Timoteo L. Soriano, Jr. of the crime of Rape under Article 266-A of the Revised Penal
RONALDO PUNO, in his capacity as Secretary of the Interior and Local Government; SERGIO Code, as amended by Republic Act 8353, upon a complaint under oath filed by Suzette S. Nicolas,
APOSTOL, in his capacity as Presidential Legal Counsel; and L/CPL. DANIEL which is attached hereto and made an integral part hereof as Annex "A," committed as follows:
SMITH, Respondents.
"That on or about the First (1st) day of November 2005, inside the Subic Bay Freeport Zone, Olongapo
x - - - - - - - - - - - - - - - - - - - - - - -x City and within the jurisdiction of this Honorable Court, the above-named accused’s (sic), being then
members of the United States Marine Corps, except Timoteo L. Soriano, Jr., conspiring, confederating
G.R. No. 176051 February 11, 2009 together and mutually helping one another, with lewd design and by means of force, threat and
intimidation, with abuse of superior strength and taking advantage of the intoxication of the victim, did
JOVITO R. SALONGA, WIGBERTO E. TAÑADA, JOSE DE LA RAMA, EMILIO C. CAPULONG, H. then and there willfully, unlawfully and feloniously sexually abuse and have sexual intercourse with or
HARRY L. ROQUE, JR., FLORIN HILBAY, and BENJAMIN POZON, Petitioners, carnal knowledge of one Suzette S. Nicolas, a 22-year old unmarried woman inside a Starex Van with
vs. Plate No. WKF-162, owned by Starways Travel and Tours, with Office address at 8900 P. Victor St.,
DANIEL SMITH, SECRETARY RAUL GONZALEZ, PRESIDENTIAL LEGAL COUNSEL SERGIO Guadalupe, Makati City, and driven by accused Timoteo L. Soriano, Jr., against the will and consent of
APOSTOL, SECRETARY RONALDO PUNO, SECRETARY ALBERTO ROMULO, The Special 16th the said Suzette S. Nicolas, to her damage and prejudice.
Division of the COURT OF APPEALS, and all persons acting in their capacity, Respondents.
1
CONTRARY TO LAW."
x - - - - - - - - - - - - - - - - - - - - - - -x
Pursuant to the Visiting Forces Agreement (VFA) between the Republic of the Philippines and the
G.R. No. 176222 February 11, 2009 United States, entered into on February 10, 1998, the United States, at its request, was granted custody
of defendant Smith pending the proceedings.
BAGONG ALYANSANG MAKABAYAN (BAYAN), represented by Dr. Carol Araullo; GABRIELA,
represented by Emerenciana de Jesus; BAYAN MUNA, represented by Rep. Satur Ocampo; During the trial, which was transferred from the Regional Trial Court (RTC) of Zambales to the RTC of
GABRIELA WOMEN'S PARTY, represented by Rep. Liza Maza; KILUSANG MAYO UNO (KMU), Makati for security reasons, the United States Government faithfully complied with its undertaking to
represented by Elmer Labog; KILUSANG MAGBUBUKID NG PILIPINAS (KMP), represented by bring defendant Smith to the trial court every time his presence was required.
Willy Marbella; LEAGUE OF FILIPINO STUDENTS (LFS), represented by Vencer Crisostomo; and
THE PUBLIC INTEREST LAW CENTER, represented by Atty. Rachel Pastores, Petitioners, On December 4, 2006, the RTC of Makati, following the end of the trial, rendered its Decision, finding
vs. defendant Smith guilty, thus:
PRESIDENT GLORIA MACAPAGAL-ARROYO, in her capacity as concurrent Defense Secretary,
EXECUTIVE SECRETARY EDUARDO ERMITA, FOREIGN AFFAIRS SECRETARY ALBERTO WHEREFORE, premises considered, for failure of the prosecution to adduce sufficient evidence against
ROMULO, JUSTICE SECRETARY RAUL GONZALEZ, AND INTERIOR AND LOCAL GOVERNMENT accused S/SGT. CHAD BRIAN CARPENTER, L/CPL. KEITH SILKWOOD AND L/CPL. DOMINIC
SECRETARY RONALDO PUNO,Respondents. DUPLANTIS, all of the US Marine Corps assigned at the USS Essex, are hereby ACQUITTED to the
crime charged.
DECISION
The prosecution having presented sufficient evidence against accused L/CPL. DANIEL J. SMITH, also
AZCUNA, J.: of the US Marine Corps at the USS Essex, this Court hereby finds him GUILTY BEYOND
REASONABLE DOUBT of the crime of RAPE defined under Article 266-A, paragraph 1 (a) of the
Revised Penal Code, as amended by R.A. 8353, and, in accordance with Article 266-B, first paragraph WHEREFORE, all the foregoing considered, we resolved to DISMISS the petition for having become
3
thereof, hereby sentences him to suffer the penalty of reclusion perpetua together with the accessory moot.
penalties provided for under Article 41 of the same Code.
Hence, the present actions.
Pursuant to Article V, paragraph No. 10, of the Visiting Forces Agreement entered into by the
Philippines and the United States, accused L/CPL. DANIEL J. SMITH shall serve his sentence in the The petitions were heard on oral arguments on September 19, 2008, after which the parties submitted
facilities that shall, thereafter, be agreed upon by appropriate Philippine and United States authorities. their memoranda.
Pending agreement on such facilities, accused L/CPL. DANIEL J. SMITH is hereby temporarily
committed to the Makati City Jail. Petitioners contend that the Philippines should have custody of defendant L/CPL Smith because, first of
all, the VFA is void and unconstitutional.
Accused L/CPL. DANIEL J. SMITH is further sentenced to indemnify complainant SUZETTE S.
NICOLAS in the amount of ₱50,000.00 as compensatory damages plus ₱50,000.00 as moral damages. This issue had been raised before, and this Court resolved in favor of the constitutionality of the VFA.
4
This was in Bayan v. Zamora, brought by Bayan, one of petitioners in the present cases.
2
SO ORDERED.
Against the barriers of res judicata vis-à-vis Bayan, and stare decisis vis-à-vis all the parties, the
As a result, the Makati court ordered Smith detained at the Makati jail until further orders. reversal of the previous ruling is sought on the ground that the issue is of primordial importance,
involving the sovereignty of the Republic, as well as a specific mandate of the Constitution.
On December 29, 2006, however, defendant Smith was taken out of the Makati jail by a contingent of
Philippine law enforcement agents, purportedly acting under orders of the Department of the Interior The provision of the Constitution is Art. XVIII, Sec. 25 which states:
and Local Government, and brought to a facility for detention under the control of the United States
government, provided for under new agreements between the Philippines and the United States, Sec. 25. After the expiration in 1991 of the Agreement between the Philippines and the United States of
referred to as the Romulo-Kenney Agreement of December 19, 2006 which states: America concerning Military Bases, foreign military bases, troops, or facilities shall not be allowed in the
Philippines except under a treaty duly concurred in by the Senate and, when the Congress so requires,
The Government of the Republic of the Philippines and the Government of the United States of America ratified by a majority of the votes cast by the people in a national referendum held for that purpose, and
agree that, in accordance with the Visiting Forces Agreement signed between our two nations, Lance recognized as a treaty by the other contracting State.
Corporal Daniel J. Smith, United States Marine Corps, be returned to U.S. military custody at the U.S.
Embassy in Manila. The reason for this provision lies in history and the Philippine experience in regard to the United States
military bases in the country.

(Sgd.) Kristie A. Kenney (Sgd.) Alberto G. Romulo It will be recalled that under the Philippine Bill of 1902, which laid the basis for the Philippine
Representative of the United States Representative of the Republic Commonwealth and, eventually, for the recognition of independence, the United States agreed to cede
of America of the Philippines to the Philippines all the territory it acquired from Spain under the Treaty of Paris, plus a few islands
later added to its realm, except certain naval ports and/or military bases and facilities, which the United
DATE: 12-19-06 DATE: December 19, 2006 States retained for itself.

and the Romulo-Kenney Agreement of December 22, 2006 which states: This is noteworthy, because what this means is that Clark and Subic and the other places in the
Philippines covered by the RP-US Military Bases Agreement of 1947 were not Philippine territory, as
The Department of Foreign Affairs of the Republic of the Philippines and the Embassy of the United they were excluded from the cession and retained by the US.
States of America agree that, in accordance with the Visiting Forces Agreement signed between the two
nations, upon transfer of Lance Corporal Daniel J. Smith, United States Marine Corps, from the Makati Accordingly, the Philippines had no jurisdiction over these bases except to the extent allowed by the
City Jail, he will be detained at the first floor, Rowe (JUSMAG) Building, U.S. Embassy Compound in a United States. Furthermore, the RP-US Military Bases Agreement was never advised for ratification by
room of approximately 10 x 12 square feet. He will be guarded round the clock by U.S. military the United States Senate, a disparity in treatment, because the Philippines regarded it as a treaty and
personnel. The Philippine police and jail authorities, under the direct supervision of the Philippine had it concurred in by our Senate.
Department of Interior and Local Government (DILG) will have access to the place of detention to
ensure the United States is in compliance with the terms of the VFA. Subsequently, the United States agreed to turn over these bases to the Philippines; and with the
expiration of the RP-US Military Bases Agreement in 1991, the territory covered by these bases were
The matter was brought before the Court of Appeals which decided on January 2, 2007, as follows: finally ceded to the Philippines.
To prevent a recurrence of this experience, the provision in question was adopted in the 1987 Desiring to declare publicly and formally their sense of unity and their common determination to defend
Constitution. themselves against external armed attack, so that no potential aggressor could be under the illusion
that either of them stands alone in the Pacific area.
The provision is thus designed to ensure that any agreement allowing the presence of foreign military
bases, troops or facilities in Philippine territory shall be equally binding on the Philippines and the Desiring further to strengthen their present efforts for collective defense for the preservation of peace
foreign sovereign State involved. The idea is to prevent a recurrence of the situation in which the terms and security pending the development of a more comprehensive system of regional security in the
and conditions governing the presence of foreign armed forces in our territory were binding upon us but Pacific area.
not upon the foreign State.
Agreeing that nothing in this present instrument shall be considered or interpreted as in any way or
Applying the provision to the situation involved in these cases, the question is whether or not the sense altering or diminishing any existing agreements or understandings between the Republic of the
presence of US Armed Forces in Philippine territory pursuant to the VFA is allowed "under a treaty duly Philippines and the United States of America.
concurred in by the Senate xxx and recognized as a treaty by the other contracting State."
Have agreed as follows:
This Court finds that it is, for two reasons.
Article I. The parties undertake, as set forth in the Charter of the United Nations, to settle any
5
First, as held in Bayan v. Zamora, the VFA was duly concurred in by the Philippine Senate and has international disputes in which they may be involved by peaceful means in such a manner that
been recognized as a treaty by the United States as attested and certified by the duly authorized international peace and security and justice are not endangered and to refrain in their international
representative of the United States government. relation from the threat or use of force in any manner inconsistent with the purposes of the United
Nations.
The fact that the VFA was not submitted for advice and consent of the United States Senate does not
detract from its status as a binding international agreement or treaty recognized by the said State. For Article II. In order more effectively to achieve the objective of this Treaty, the Parties separately and
this is a matter of internal United States law. Notice can be taken of the internationally known practice jointly by self-help and mutual aid will maintain and develop their individual and collective capacity to
by the United States of submitting to its Senate for advice and consent agreements that are resist armed attack.
policymaking in nature, whereas those that carry out or further implement these policymaking
agreements are merely submitted to Congress, under the provisions of the so-called Case–Zablocki Article III. The Parties, through their Foreign Ministers or their deputies, will consult together from time
6 to time regarding the implementation of this Treaty and whenever in the opinion of either of them the
Act, within sixty days from ratification.
territorial integrity, political independence or security of either of the Parties is threatened by external
The second reason has to do with the relation between the VFA and the RP-US Mutual Defense Treaty armed attack in the Pacific.
of August 30, 1951. This earlier agreement was signed and duly ratified with the concurrence of both
the Philippine Senate and the United States Senate. Article IV. Each Party recognizes that an armed attack in the Pacific area on either of the parties would
be dangerous to its own peace and safety and declares that it would act to meet the common dangers
7
The RP-US Mutual Defense Treaty states: in accordance with its constitutional processes.

MUTUAL DEFENSE TREATY BETWEEN THE REPUBLIC OF THE PHILIPPINES AND THE UNITED Any such armed attack and all measures taken as a result thereof shall be immediately reported to the
STATES OF AMERICA. Signed at Washington, August 30, 1951. Security Council of the United Nations. Such measures shall be terminated when the Security Council
has taken the measures necessary to restore and maintain international peace and security.
The Parties of this Treaty
Article V. For the purpose of Article IV, an armed attack on either of the Parties is deemed to include an
Reaffirming their faith in the purposes and principles of the Charter of the United Nations and their armed attack on the metropolitan territory of either of the Parties, or on the island territories under its
desire to live in peace with all peoples and all governments, and desiring to strengthen the fabric of jurisdiction in the Pacific Ocean, its armed forces, public vessels or aircraft in the Pacific.
peace in the Pacific area.
Article VI. This Treaty does not affect and shall not be interpreted as affecting in any way the rights and
Recalling with mutual pride the historic relationship which brought their two peoples together in a obligations of the Parties under the Charter of the United Nations or the responsibility of the United
common bond of sympathy and mutual ideals to fight side-by-side against imperialist aggression during Nations for the maintenance of international peace and security.
the last war.
Article VII. This Treaty shall be ratified by the Republic of the Philippines and the United Nations of Considering that cooperation between the United States and the Republic of the Philippines promotes
America in accordance with their respective constitutional processes and will come into force when their common security interests;
instruments of ratification thereof have been exchanged by them at Manila.
Recognizing the desirability of defining the treatment of United States personnel visiting the Republic of
Article VIII. This Treaty shall remain in force indefinitely. Either Party may terminate it one year after the Philippines;
notice has been given to the other party.
9
Have agreed as follows:
In withness whereof the undersigned Plenipotentiaries have signed this Treaty.
Accordingly, as an implementing agreement of the RP-US Mutual Defense Treaty, it was not necessary
Done in duplicate at Washington this thirtieth day of August, 1951. to submit the VFA to the US Senate for advice and consent, but merely to the US Congress under the
Case–Zablocki Act within 60 days of its ratification. It is for this reason that the US has certified that it
For the Republic of the Philippines: recognizes the VFA as a binding international agreement, i.e., a treaty, and this substantially complies
10
with the requirements of Art. XVIII, Sec. 25 of our Constitution.
(Sgd.) Carlos P. Romulo
The provision of Art. XVIII, Sec. 25 of the Constitution, is complied with by virtue of the fact that the
(Sgd.) Joaquin M. Elizalde presence of the US Armed Forces through the VFA is a presence "allowed under" the RP-US Mutual
Defense Treaty. Since the RP-US Mutual Defense Treaty itself has been ratified and concurred in by
(Sgd.) Vicente J. Francisco
both the Philippine Senate and the US Senate, there is no violation of the Constitutional provision
(Sgd.) Diosdado Macapagal resulting from such presence.

For the United States of America: The VFA being a valid and binding agreement, the parties are required as a matter of international law
to abide by its terms and provisions.
(Sgd.) Dean Acheson
The VFA provides that in cases of offenses committed by the members of the US Armed Forces in the
(Sgd.) John Foster Dulles Philippines, the following rules apply:

(Sgd.) Tom Connally Article V

8 Criminal Jurisdiction
(Sgd.) Alexander Wiley

Clearly, therefore, joint RP-US military exercises for the purpose of developing the capability to resist an xxx
armed attack fall squarely under the provisions of the RP-US Mutual Defense Treaty. The VFA, which is
the instrument agreed upon to provide for the joint RP-US military exercises, is simply an implementing 6. The custody of any United States personnel over whom the Philippines is to exercise jurisdiction shall
agreement to the main RP-US Military Defense Treaty. The Preamble of the VFA states: immediately reside with United States military authorities, if they so request, from the commission of the
offense until completion of all judicial proceedings. United States military authorities shall, upon formal
The Government of the United States of America and the Government of the Republic of the notification by the Philippine authorities and without delay, make such personnel available to those
Philippines, authorities in time for any investigative or judicial proceedings relating to the offense with which the
person has been charged. In extraordinary cases, the Philippine Government shall present its position
Reaffirming their faith in the purposes and principles of the Charter of the United Nations and their to the United States Government regarding custody, which the United States Government shall take
desire to strengthen international and regional security in the Pacific area; into full account. In the event Philippine judicial proceedings are not completed within one year, the
United States shall be relieved of any obligations under this paragraph. The one year period will not
Reaffirming their obligations under the Mutual Defense Treaty of August 30, 1951; include the time necessary to appeal. Also, the one year period will not include any time during which
scheduled trial procedures are delayed because United States authorities, after timely notification by
Noting that from time to time elements of the United States armed forces may visit the Republic of the Philippine authorities to arrange for the presence of the accused, fail to do so.
Philippines;
Petitioners contend that these undertakings violate another provision of the Constitution, namely, that
providing for the exclusive power of this Court to adopt rules of procedure for all courts in the
Philippines (Art. VIII, Sec. 5[5]). They argue that to allow the transfer of custody of an accused to a of the accused in the United States Embassy, are not in accord with the VFA itself because such
foreign power is to provide for a different rule of procedure for that accused, which also violates the detention is not "by Philippine authorities."
equal protection clause of the Constitution (Art. III, Sec. 1.).
Respondents should therefore comply with the VFA and negotiate with representatives of the United
Again, this Court finds no violation of the Constitution. States towards an agreement on detention facilities under Philippine authorities as mandated by Art. V,
Sec. 10 of the VFA.
The equal protection clause is not violated, because there is a substantial basis for a different treatment
11
of a member of a foreign military armed forces allowed to enter our territory and all other accused. Next, the Court addresses the recent decision of the United States Supreme Court in Medellin v. Texas
( 552 US ___ No. 06-984, March 25, 2008), which held that treaties entered into by the United States
The rule in international law is that a foreign armed forces allowed to enter one’s territory is immune are not automatically part of their domestic law unless these treaties are self-executing or there is an
from local jurisdiction, except to the extent agreed upon. The Status of Forces Agreements involving implementing legislation to make them enforceable.1avvphi1
foreign military units around the world vary in terms and conditions, according to the situation of the
parties involved, and reflect their bargaining power. But the principle remains, i.e., the receiving State On February 3, 2009, the Court issued a Resolution, thus:
can exercise jurisdiction over the forces of the sending State only to the extent agreed upon by the
12 "G.R. No. 175888 (Suzette Nicolas y Sombilon v. Alberto Romulo, et al.); G.R. No. 176051 (Jovito R.
parties.
Salonga, et al. v. Daniel Smith, et al.); and G.R. No. 176222 (Bagong Alyansang Makabayan [BAYAN],
As a result, the situation involved is not one in which the power of this Court to adopt rules of procedure et al. v. President Gloria Macapagal-Arroyo, et al.).
is curtailed or violated, but rather one in which, as is normally encountered around the world, the laws
(including rules of procedure) of one State do not extend or apply – except to the extent agreed upon – The parties, including the Solicitor General, are required to submit within three (3) days a
to subjects of another State due to the recognition of extraterritorial immunity given to such bodies as Comment/Manifestation on the following points:
visiting foreign armed forces.
1. What is the implication on the RP-US Visiting Forces Agreement of the recent US Supreme Court
Nothing in the Constitution prohibits such agreements recognizing immunity from jurisdiction or some decision in Jose Ernesto Medellin v. Texas, dated March 25, 2008, to the effect that treaty stipulations
aspects of jurisdiction (such as custody), in relation to long-recognized subjects of such immunity like that are not self-executory can only be enforced pursuant to legislation to carry them into effect; and
Heads of State, diplomats and members of the armed forces contingents of a foreign State allowed to that, while treaties may comprise international commitments, they are not domestic law unless
enter another State’s territory. On the contrary, the Constitution states that the Philippines adopts the Congress has enacted implementing statutes or the treaty itself conveys an intention that it be "self-
generally accepted principles of international law as part of the law of the land. (Art. II, Sec. 2). executory" and is ratified on these terms?

Applying, however, the provisions of VFA, the Court finds that there is a different treatment when it 2. Whether the VFA is enforceable in the US as domestic law, either because it is self-executory or
comes to detention as against custody. The moment the accused has to be detained, e.g., after because there exists legislation to implement it.
conviction, the rule that governs is the following provision of the VFA:
3. Whether the RP-US Mutual Defense Treaty of August 30, 1951 was concurred in by the US Senate
Article V and, if so, is there proof of the US Senate advice and consent resolution? Peralta, J., no part."

Criminal Jurisdiction After deliberation, the Court holds, on these points, as follows:

xxx First, the VFA is a self-executing Agreement, as that term is defined in Medellin itself, because the
parties intend its provisions to be enforceable, precisely because the Agreement is intended to carry out
Sec. 10. The confinement or detention by Philippine authorities of United States personnel shall be obligations and undertakings under the RP-US Mutual Defense Treaty. As a matter of fact, the VFA has
carried out in facilities agreed on by appropriate Philippines and United States authorities. United States been implemented and executed, with the US faithfully complying with its obligation to produce L/CPL
personnel serving sentences in the Philippines shall have the right to visits and material assistance. Smith before the court during the trial.

It is clear that the parties to the VFA recognized the difference between custody during the trial and Secondly, the VFA is covered by implementing legislation, namely, the Case-Zablocki Act, USC Sec.
detention after conviction, because they provided for a specific arrangement to cover detention. And this 112(b), inasmuch as it is the very purpose and intent of the US Congress that executive agreements
specific arrangement clearly states not only that the detention shall be carried out in facilities agreed on registered under this Act within 60 days from their ratification be immediately implemented. The parties
by authorities of both parties, but also that the detention shall be "by Philippine authorities." Therefore, to these present cases do not question the fact that the VFA has been registered under the Case-
the Romulo-Kenney Agreements of December 19 and 22, 2006, which are agreements on the detention Zablocki Act.1avvphi1
In sum, therefore, the VFA differs from the Vienna Convention on Consular Relations and the Avena negotiate with the United States representatives for the appropriate agreement on detention facilities
decision of the International Court of Justice (ICJ), subject matter of the Medellin decision. The under Philippine authorities as provided in Art. V, Sec. 10 of the VFA, pending which the status quo
Convention and the ICJ decision are not self-executing and are not registrable under the Case-Zablocki shall be maintained until further orders by this Court.
Act, and thus lack legislative implementing authority.
The Court of Appeals is hereby directed to resolve without delay the related matters pending therein,
Finally, the RP-US Mutual Defense Treaty was advised and consented to by the US Senate on March namely, the petition for contempt and the appeal of L/CPL Daniel Smith from the judgment of conviction.
20, 1952, as reflected in the US Congressional Record, 82nd Congress, Second Session, Vol. 98 – Part
2, pp. 2594-2595. No costs.

The framers of the Constitution were aware that the application of international law in domestic courts SO ORDERED.
varies from country to country.
ADOLFO S. AZCUNA
As Ward N. Ferdinandusse states in his Treatise, DIRECT APPLICATION OF INTERNATIONAL Associate Justice
CRIMINAL LAW IN NATIONAL COURTS, some countries require legislation whereas others do not.

It was not the intention of the framers of the 1987 Constitution, in adopting Article XVIII, Sec. 25, to
require the other contracting State to convert their system to achieve alignment and parity with ours. It
was simply required that the treaty be recognized as a treaty by the other contracting State. With that, it
becomes for both parties a binding international obligation and the enforcement of that obligation is left
to the normal recourse and processes under international law.
13
Furthermore, as held by the US Supreme Court in Weinberger v. Rossi, an executive agreement is a
"treaty" within the meaning of that word in international law and constitutes enforceable domestic law
vis-à-vis the United States. Thus, the US Supreme Court in Weinberger enforced the provisions of the
executive agreement granting preferential employment to Filipinos in the US Bases here.

Accordingly, there are three types of treaties in the American system:

1. Art. II, Sec. 2 treaties – These are advised and consented to by the US Senate in accordance with
Art. II, Sec. 2 of the US Constitution.

2. Executive–Congressional Agreements: These are joint agreements of the President and Congress
and need not be submitted to the Senate.

3. Sole Executive Agreements. – These are agreements entered into by the President. They are to be
submitted to Congress within sixty (60) days of ratification under the provisions of the Case-Zablocki
Act, after which they are recognized by the Congress and may be implemented.

As regards the implementation of the RP-US Mutual Defense Treaty, military aid or assistance has
been given under it and this can only be done through implementing legislation. The VFA itself is
another form of implementation of its provisions.

WHEREFORE, the petitions are PARTLY GRANTED, and the Court of Appeals’ Decision in CA-G.R.
SP No. 97212 dated January 2, 2007 is MODIFIED. The Visiting Forces Agreement (VFA) between the
Republic of the Philippines and the United States, entered into on February 10, 1998, is UPHELD as
constitutional, but the Romulo-Kenney Agreements of December 19 and 22, 2006 are DECLARED not
in accordance with the VFA, and respondent Secretary of Foreign Affairs is hereby ordered to forthwith
ASSISTANT SECRETARY FOR STRATEGIC ASSESSMENTS RAYMUND JOSE QUILOP AS
CHAIRPERSON AND MEMBERS, RESPECTIVELY, OF THE NEGOTIATING PANEL FOR THE
PHILIPPINES ON EDCA, Respondents.

x-----------------------x

KILUSANG MAYO UNO, REPRESENTED BY ITS CHAIRPERSON, ELMER LABOG,


CONFEDERATION FOR UNITY, RECOGNITION AND ADVANCEMENT OF GOVERNMENT
EMPLOYEES (COURAGE), REPRESENTED BY ITS NATIONAL PRESIDENT FERDINAND GAITE,
NATIONAL FEDERATION OF LABOR UNIONS-KILUSANG MAYO UNO, REPRESENTED BY ITS
EN BANC
NATIONAL PRESIDENT JOSELITO USTAREZ, NENITA GONZAGA, VIOLETA ESPIRITU, VIRGINIA
January 12, 2016 FLORES, AND ARMANDO TEODORO, JR., Petitioners-in-Intervention,
RENE A.Q. SAGUISAG, JR., Petitioner-in-Intervention.
G.R. No. 212426
DECISION
RENE A.V. SAGUISAG, WIGBERTO E. TAÑADA, FRANCISCO "DODONG" NEMENZO, JR., SR.
MARY JOHN MANANZAN, PACIFICO A. AGABIN, ESTEBAN "STEVE" SALONGA, H. HARRY L. SERENO, J.:
ROQUE, JR., EVALYN G. URSUA, EDRE U. OLALIA, DR. CAROL PAGADUAN-ARAULLO, DR. 1
The petitions before this Court question the constitutionality of the Enhanced Defense Cooperation
ROLAND SIMBULAN, AND TEDDY CASIÑO, Petitioners,
Agreement (EDCA) between the Republic of the Philippines and the United States of America (U.S.).
vs.
Petitioners allege that respondents committed grave abuse of discretion amounting to lack or excess of
EXECUTIVE SECRETARY PAQUITO N. OCHOA, JR., DEPARTMENT OF NATIONAL DEFENSE 2
SECRETARY VOLTAIRE GAZMIN, DEPARTMENT OF FOREIGN AFFAIRS SECRETARY ALBERT jurisdiction when they entered into EDCA with the U.S., claiming that the instrument violated multiple
3
DEL ROSARIO, JR., DEPARTMENT OF BUDGET AND MANAGEMENT SECRETARY FLORENCIO constitutional provisions. In reply, respondents argue that petitioners lack standing to bring the suit. To
ABAD, AND ARMED FORCES OF THE PHILIPPINES CHIEF OF STAFF GENERAL EMMANUEL T. support the legality of their actions, respondents invoke the 1987 Constitution, treaties, and judicial
4
precedents.
BAUTISTA, Respondents.
A proper analysis of the issues requires this Court to lay down at the outset the basic parameters of the
x-----------------------x
constitutional powers and roles of the President and the Senate in respect of the above issues. A more
G.R. No. 212444 detailed discussion of these powers and roles will be made in the latter portions.

BAGONG ALYANSANG MAKABAYAN (BAYAN), REPRESENTED BY ITS SECRETARY GENERAL I. BROAD CONSTITUTIONAL CONTEXT OF THE POWERS OF THE PRESIDENT: DEFENSE,
RENATO M. REYES, JR., BAYAN MUNA PARTY-LIST REPRESENTATIVES NERI J. FOREIGN RELATIONS, AND EDCA
COLMENARES AND CARLOS ZARATE, GABRIELA WOMEN'S PARTY-LIST REPRESENTATIVES
A. The Prime Duty of the State and the Consolidation of Executive Power in the President
LUZ ILAGAN AND EMERENCIANA DE JESUS, ACT TEACHERS PARTY-LIST REPRESENTATIVE
ANTONIO L. TINIO, ANAKPAWIS PARTY-LIST REPRESENTATIVE FERNANDO HICAP,
Mataimtim kong pinanunumpaan (o pinatotohanan) na tutuparin ko nang buong katapatan at sigasig
KABATAAN PARTY-LIST REPRESENTATIVE TERRY RIDON, MAKABAYANG KOALISYON NG
ang aking mga tungkulin bilang Pangulo (o Pangalawang Pangulo o Nanunungkulang Pangulo) ng
MAMAMAYAN (MAKABAYAN), REPRESENTED BY SATURNINO OCAMPO AND LIZA MAZA,
Pilipinas, pangangalagaan at ipagtatanggol ang kanyang Konstitusyon, ipatutupad ang mga batas nito,
BIENVENIDO LUMBERA, JOEL C. LAMANGAN, RAFAEL MARIANO, SALVADOR FRANCE,
magiging makatarungan sa bawat tao, at itatalaga ang aking sarili sa paglilingkod sa Bansa. Kasihan
ROGELIO M. SOLUTA, AND CLEMENTE G. BAUTISTA, Petitioners,
nawa aka ng Diyos.
vs.
DEPARTMENT OF NATIONAL DEFENSE (DND) SECRETARY VOLTAIRE GAZMIN, DEPARTMENT - Panunumpa sa Katungkulan ng Pangulo ng Pilipinas ayon sa Saligang Batas
5

OF FOREIGN AFFAIRS SECRETARY ALBERT DEL ROSARIO, EXECUTIVE SECRETARY


PAQUITO N. OCHOA, JR., ARMED FORCES OF THE PHILIPPINES CHIEF OF STAFF GENERAL The 1987 Constitution has "vested the executive power in the President of the Republic of the
EMMANUEL T. BAUTISTA, DEFENSE UNDERSECRETARY PIO LORENZO BATINO, 6
Philippines." While the vastness of the executive power that has been consolidated in the person of the
AMBASSADOR LOURDES YPARRAGUIRRE, AMBASSADOR J. EDUARDO MALAYA, President cannot be expressed fully in one provision, the Constitution has stated the prime duty of the
DEPARTMENT OF JUSTICE UNDERSECRETARY FRANCISCO BARAAN III, AND DND government, of which the President is the head:
The prime duty of the Government is to serve and protect the people. The Government may call government which can act on the basis of the best available information and can decide with
upon the people to defend the State and, in the fulfillment thereof, all citizens may be required, under decisiveness. x x x It is also the President who possesses the most comprehensive and the most
7
conditions provided by law, to render personal military or civil service. (Emphases supplied) confidential information about foreign countries for our diplomatic and consular officials regularly brief
him on meaningful events all over the world. He has also unlimited access to ultra-sensitive military
B. The duty to protect the territory and the citizens of the Philippines, the power to call upon the intelligence data. In fine, the presidential role in foreign affairs is dominant and the President is
people to defend the State, and the President as Commander-in-Chief traditionally accorded a wider degree of discretion in the conduct of foreign affairs. The regularity, nay,
validity of his actions are adjudged under less stringent standards, lest their judicial repudiation lead to
The duty to protect the State and its people must be carried out earnestly and effectively throughout the breach of an international obligation, rupture of state relations, forfeiture of confidence, national
whole territory of the Philippines in accordance with the constitutional provision on national territory. embarrassment and a plethora of other problems with equally undesirable consequences.
17

Hence, the President of the Philippines, as the sole repository of executive power, is the guardian of the
Philippine archipelago, including all the islands and waters embraced therein and all other territories The role of the President in foreign affairs is qualified by the Constitution in that the Chief Executive
over which it has sovereignty or jurisdiction. These territories consist of its terrestrial, fluvial, and aerial must give paramount importance to the sovereignty of the nation, the integrity of its territory, its interest,
domains; including its territorial sea, the seabed, the subsoil, the insular shelves, and other submarine 18
and the right of the sovereign Filipino people to self-determination. In specific provisions, the
areas; and the waters around, between, and connecting the islands of the archipelago, regardless of President's power is also limited, or at least shared, as in Section 2 of Article II on the conduct of war;
8
their breadth and dimensions. Sections 20 and 21 of Article VII on foreign loans, treaties, and international agreements; Sections 4(2)
and 5(2)(a) of Article VIII on the judicial review of executive acts; Sections 4 and 25 of Article XVIII on
To carry out this important duty, the President is equipped with authority over the Armed Forces of the treaties and international agreements entered into prior to the Constitution and on the presence of
9
Philippines (AFP), which is the protector of the people and the state. The AFP's role is to secure the foreign military troops, bases, or facilities.
10
sovereignty of the State and the integrity of the national territory. In addition, the Executive is
constitutionally empowered to maintain peace and order; protect life, liberty, and property; and promote D. The relationship between the two major presidential functions and the role of the Senate
11
the general welfare.
Clearly, the power to defend the State and to act as its representative in the international sphere
In recognition of these powers, Congress has specified that the President must oversee, ensure, and inheres in the person of the President. This power, however, does not crystallize into absolute discretion
12
reinforce our defensive capabilities against external and internal threats and, in the same vein, ensure to craft whatever instrument the Chief Executive so desires. As previously mentioned, the Senate has a
that the country is adequately prepared for all national and local emergencies arising from natural and role in ensuring that treaties or international agreements the President enters into, as contemplated in
13
man-made disasters. Section 21 of Article VII of the Constitution, obtain the approval of two-thirds of its members.

To be sure, this power is limited by the Constitution itself. To illustrate, the President may call out the Previously, treaties under the 1973 Constitution required ratification by a majority of the Batasang
14 19
AFP to prevent or suppress instances of lawless violence, invasion or rebellion, but not suspend the Pambansa, except in instances wherein the President "may enter into international treaties or
privilege of the writ of habeas corpus for a period exceeding 60 days, or place the Philippines or any 20
agreements as the national welfare and interest may require." This left a large margin of discretion
part thereof under martial law exceeding that same span. In the exercise of these powers, the President that the President could use to bypass the Legislature altogether. This was a departure from the 1935
is also duty-bound to submit a report to Congress, in person or in writing, within 48 hours from the Constitution, which explicitly gave the President the power to enter into treaties only with the
proclamation of martial law or the suspension of the privilege of the writ of habeas corpus; and 21
concurrence of two-thirds of all the Members of the Senate. The 1987 Constitution returned the
Congress may in turn revoke the proclamation or suspension. The same provision provides for the 22 23
Senate's power and, with it, the legislative's traditional role in foreign affairs.
Supreme Court's review of the factual basis for the proclamation or suspension, as well as the
promulgation of the decision within 30 days from filing. The responsibility of the President when it comes to treaties and international agreements under the
present Constitution is therefore shared with the Senate. This shared role, petitioners claim, is
C. The power and duty to conduct foreign relations bypassed by EDCA.

The President also carries the mandate of being the sole organ in the conduct of foreign II. HISTORICAL ANTECEDENTS OF EDCA
15
relations. Since every state has the capacity to interact with and engage in relations with other
16
sovereign states, it is but logical that every state must vest in an agent the authority to represent its A. U.S. takeover of Spanish colonization and its military bases, and the transition to Philippine
interests to those other sovereign states. independence

The conduct of foreign relations is full of complexities and consequences, sometimes with life and death The presence of the U.S. military forces in the country can be traced to their pivotal victory in the 1898
24
significance to the nation especially in times of war. It can only be entrusted to that department of Battle of Manila Bay during the Spanish-American War. Spain relinquished its sovereignty over the
25 43
Philippine Islands in favor of the U.S. upon its formal surrender a few months later. By 1899, the U.S. military bases in the Philippines for 99 years or until the year 2046. The treaty also obliged the
26
Americans had consolidated a military administration in the archipelago. Philippines to negotiate with the U.S. to allow the latter to expand the existing bases or to acquire new
44
ones as military necessity might require.
When it became clear that the American forces intended to impose colonial control over the Philippine
45
Islands, General Emilio Aguinaldo immediately led the Filipinos into an all-out war against the A number of significant amendments to the 1947 MBA were made. With respect to its duration, the
27
U.S. The Filipinos were ultimately defeated in the Philippine-American War, which lasted until 1902 parties entered into the Ramos-Rusk Agreement of 1966, which reduced the term of the treaty from 99
28 46
and led to the downfall of the first Philippine Republic. The Americans henceforth began to strengthen years to a total of 44 years or until 1991. Concerning the number of U.S. military bases in the country,
29
their foothold in the country. They took over and expanded the former Spanish Naval Base in Subic the Bohlen-Serrano Memorandum of Agreement provided for the return to the Philippines of 17 U.S.
47
Bay, Zambales, and put up a cavalry post called Fort Stotsenberg in Pampanga, now known as Clark military bases covering a total area of 117,075 hectares. Twelve years later, the U.S. returned
30 48
Air Base. Sangley Point in Cavite City through an exchange of notes. Then, through the Romulo-Murphy
Exchange of Notes of 1979, the parties agreed to the recognition of Philippine sovereignty over Clark
When talks of the eventual independence of the Philippine Islands gained ground, the U.S. manifested 49
and Subic Bases and the reduction of the areas that could be used by the U.S. military. The
31 50
the desire to maintain military bases and armed forces in the country. The U.S. Congress later agreement also provided for the mandatory review of the treaty every five years. In 1983, the parties
enacted the Hare-Hawes-Cutting Act of 1933, which required that the proposed constitution of an 51
revised the 1947 MBA through the Romualdez-Armacost Agreement. The revision pertained to the
independent Philippines recognize the right of the U.S. to maintain the latter's armed forces and military operational use of the military bases by the U.S. government within the context of Philippine
32 52
bases. The Philippine Legislature rejected that law, as it also gave the U.S. the power to unilaterally sovereignty, including the need for prior consultation with the Philippine government on the former' s
designate any part of Philippine territory as a permanent military or naval base of the U.S. within two 53
use of the bases for military combat operations or the establishment of long-range missiles.
33
years from complete independence.
54
Pursuant to the legislative authorization granted under Republic Act No. 9, the President also entered
The U.S. Legislature subsequently crafted another law called the Tydings-McDuffie Act or the Philippine 55
into the 1947 Military Assistance Agreement with the U.S. This executive agreement established the
Independence Act of 1934. Compared to the old Hare-Hawes-Cutting Act, the new law provided for the 56
conditions under which U.S. military assistance would be granted to the Philippines, particularly the
surrender to the Commonwealth Government of "all military and other reservations" of the U.S. provision of military arms, ammunitions, supplies, equipment, vessels, services, and training for the
34 57
government in the Philippines, except "naval reservations and refueling stations." Furthermore, the latter's defense forces. An exchange of notes in 1953 made it clear that the agreement would remain
law authorized the U.S. President to enter into negotiations for the adjustment and settlement of all 58
in force until terminated by any of the parties.
questions relating to naval reservations and fueling stations within two years after the Philippines would
35 59
have gained independence. Under the Tydings-McDuffie Act, the U.S. President would proclaim the To further strengthen their defense and security relationship, the Philippines and the U.S. next entered
60 61
American withdrawal and surrender of sovereignty over the islands 10 years after the inauguration of into the MDT in 1951. Concurred in by both the Philippine and the U.S. Senates, the treaty has two
36
the new government in the Philippines. This law eventually led to the promulgation of the 1935 main features: first, it allowed for mutual assistance in maintaining and developing their individual and
62
Philippine Constitution. collective capacities to resist an armed attack; and second, it provided for their mutual self-defense in
63
the event of an armed attack against the territory of either party. The treaty was premised on their
37
The original plan to surrender the military bases changed. At the height of the Second World War, the recognition that an armed attack on either of them would equally be a threat to the security of the
Philippine and the U.S. Legislatures each passed resolutions authorizing their respective Presidents to 64
other.
negotiate the matter of retaining military bases in the country after the planned withdrawal of the
38
U.S. Subsequently, in 1946, the countries entered into the Treaty of General Relations, in which the C. Current legal regime on the presence of U.S. armed forces in the country
U.S. relinquished all control and sovereignty over the Philippine Islands, except the areas that would be
39 In view of the impending expiration of the 1947 MBA in 1991, the Philippines and the U.S. negotiated for
covered by the American military bases in the country. This treaty eventually led to the creation of the
65
post-colonial legal regime on which would hinge the continued presence of U.S. military forces until a possible renewal of their defense and security relationship. Termed as the Treaty of Friendship,
1991: the Military Bases Agreement (MBA) of 1947, the Military Assistance Agreement of 1947, and the Cooperation and Security, the countries sought to recast their military ties by providing a new
40 66
Mutual Defense Treaty (MDT) of 1951. framework for their defense cooperation and the use of Philippine installations. One of the proposed
provisions included an arrangement in which U.S. forces would be granted the use of certain
67
B. Former legal regime on the presence of U.S. armed forces in the territory of an independent installations within the Philippine naval base in Subic. On 16 September 1991, the Senate rejected the
68
Philippines (1946-1991) proposed treaty.

Soon after the Philippines was granted independence, the two countries entered into their first military The consequent expiration of the 1947 MBA and the resulting paucity of any formal agreement dealing
41
arrangement pursuant to the Treaty of General Relations - the 1947 MBA. The Senate concurred on with the treatment of U.S. personnel in the Philippines led to the suspension in 1995 of large-scale joint
42 69 70
the premise of "mutuality of security interest," which provided for the presence and operation of 23 military exercises. In the meantime, the respective governments of the two countries agreed to hold
71
joint exercises at a substantially reduced level. The military arrangements between them were revived On 10 November 2015, months after the oral arguments were concluded and the parties ordered to file
72 91
in 1999 when they concluded the first Visiting Forces Agreement (VFA). their respective memoranda, the Senators adopted Senate Resolution No. (SR) 105. The resolution
92
expresses the "strong sense" of the Senators that for EDCA to become valid and effective, it must first
73
As a "reaffirm[ation] [of the] obligations under the MDT," the VFA has laid down the regulatory be transmitted to the Senate for deliberation and concurrence.
74
mechanism for the treatment of U.S. military and civilian personnel visiting the country. It contains
provisions on the entry and departure of U.S. personnel; the purpose, extent, and limitations of their III. ISSUES
activities; criminal and disciplinary jurisdiction; the waiver of certain claims; the importation and
exportation of equipment, materials, supplies, and other pieces of property owned by the U.S. Petitioners mainly seek a declaration that the Executive Department committed grave abuse of
government; and the movement of U.S. military vehicles, vessels, and aircraft into and within the discretion in entering into EDCA in the form of an executive agreement. For this reason, we cull the
75 issues before us:
country. The Philippines and the U.S. also entered into a second counterpart agreement (VFA II),
76
which in turn regulated the treatment of Philippine military and civilian personnel visiting the U.S. The
Philippine Senate concurred in the first VFA on 27 May 1999.
77 A. Whether the essential requisites for judicial review are present

Beginning in January 2002, U.S. military and civilian personnel started arriving in Mindanao to take part B. Whether the President may enter into an executive agreement on foreign military bases,
78
in joint military exercises with their Filipino counterparts. Called Balikatan, these exercises involved troops, or facilities
79
trainings aimed at simulating joint military maneuvers pursuant to the MDT.
C. Whether the provisions under EDCA are consistent with the Constitution, as well as with
In the same year, the Philippines and the U.S. entered into the Mutual Logistics Support Agreement to existing laws and treaties
80
"further the interoperability, readiness, and effectiveness of their respective military forces" in
81 IV. DISCUSSION
accordance with the MDT, the Military Assistance Agreement of 1953, and the VFA. The new
agreement outlined the basic terms, conditions, and procedures for facilitating the reciprocal provision A. Whether the essential requisites for judicial review have been satisfied
82
of logistics support, supplies, and services between the military forces of the two countries. The
phrase "logistics support and services" includes billeting, operations support, construction and use of Petitioners are hailing this Court's power of judicial review in order to strike down EDCA for violating the
temporary structures, and storage services during an approved activity under the existing military Constitution. They stress that our fundamental law is explicit in prohibiting the presence of foreign
83 84
arrangements. Already extended twice, the agreement will last until 2017. military forces in the country, except under a treaty concurred in by the Senate. Before this Court may
begin to analyze the constitutionality or validity of an official act of a coequal branch of government,
D. The Enhanced Defense Cooperation Agreement however, petitioners must show that they have satisfied all the essential requisites for judicial review.
93

EDCA authorizes the U.S. military forces to have access to and conduct activities within certain "Agreed Distinguished from the general notion of judicial power, the power of judicial review specially refers to
Locations" in the country. It was not transmitted to the Senate on the executive's understanding that to both the authority and the duty of this Court to determine whether a branch or an instrumentality of
85
do so was no longer necessary. Accordingly, in June 2014, the Department of Foreign Affairs (DFA) 94
government has acted beyond the scope of the latter's constitutional powers. As articulated in Section
and the U.S. Embassy exchanged diplomatic notes confirming the completion of all necessary internal 1, Article VIII of the Constitution, the power of judicial review involves the power to resolve cases in
86
requirements for the agreement to enter into force in the two countries. which the questions concern the constitutionality or validity of any treaty, international or executive
agreement, law, presidential decree, proclamation, order, instruction, ordinance, or
According to the Philippine government, the conclusion of EDCA was the result of intensive and 95
87 regulation. In Angara v. Electoral Commission, this Court exhaustively discussed this "moderating
comprehensive negotiations in the course of almost two years. After eight rounds of negotiations, the
power" as part of the system of checks and balances under the Constitution. In our fundamental law,
Secretary of National Defense and the U.S. Ambassador to the Philippines signed the agreement on 28
88 89 the role of the Court is to determine whether a branch of government has adhered to the specific
April 2014. President Benigno S. Aquino III ratified EDCA on 6 June 2014. The OSG clarified during 96
90 restrictions and limitations of the latter's power:
the oral arguments that the Philippine and the U.S. governments had yet to agree formally on the
specific sites of the Agreed Locations mentioned in the agreement. The separation of powers is a fundamental principle in our system of government. It obtains not through
express provision but by actual division in our Constitution. Each department of the government has
Two petitions for certiorari were thereafter filed before us assailing the constitutionality of EDCA. They
exclusive cognizance of matters within its jurisdiction, and is supreme within its own
primarily argue that it should have been in the form of a treaty concurred in by the Senate, not an
sphere. But it does not follow from the fact that the three powers are to be kept separate and distinct
executive agreement.
that the Constitution intended them to be absolutely unrestrained and independent of each other. The
Constitution has provided for an elaborate system of checks and balances to secure coordination
in the workings of the various departments of the government. x x x. And the judiciary in turn, with the
Supreme Court as the final arbiter, effectively checks the other departments in the exercise of its Even as we are left with no recourse but to bare our power to check an act of a coequal branch of
power to determine the law, and hence to declare executive and legislative acts void if violative government - in this case the executive - we must abide by the stringent requirements for the exercise
101
of the Constitution. of that power under the Constitution. Demetria v. Alba and Francisco v. House of
102
Representatives cite the "pillars" of the limitations on the power of judicial review as enunciated in the
xxxx concurring opinion of U.S. Supreme Court Justice Brandeis in Ashwander v. Tennessee Valley
103 104
Authority. Francisco redressed these "pillars" under the following categories:
As any human production, our Constitution is of course lacking perfection and perfectibility, but as
much as it was within the power of our people, acting through their delegates to so provide, that 1. That there be absolute necessity of deciding a case
instrument which is the expression of their sovereignty however limited, has established a
republican government intended to operate and function as a harmonious whole, under a 2. That rules of constitutional law shall be formulated only as required by the facts of the case
system of checks and balances, and subject to specific limitations and restrictions provided in
the said instrument. The Constitution sets forth in no uncertain language the restrictions and 3. That judgment may not be sustained on some other ground
limitations upon governmental powers and agencies. If these restrictions and limitations are
transcended it would be inconceivable if the Constitution had not provided for a mechanism by 4. That there be actual injury sustained by the party by reason of the operation of the statute
which to direct the course of government along constitutional channels, for then the distribution
5. That the parties are not in estoppel
of powers would be mere verbiage, the bill of rights mere expressions of sentiment, and the
principles of good government mere political apothegms. Certainly, the limitations and restrictions 6. That the Court upholds the presumption of constitutionality
embodied in our Constitution are real as they should be in any living constitution. x x x. In our case, this
moderating power is granted, if not expressly, by clear implication from section 2 of article VIII of [the (Emphases supplied)
1935] Constitution.
These are the specific safeguards laid down by the Court when it exercises its power of judicial
The Constitution is a definition of the powers of government. Who is to determine the nature, scope and 105
review. Guided by these pillars, it may invoke the power only when the following four stringent
extent of such powers? The Constitution itself has provided for the instrumentality of the judiciary as the requirements are satisfied: (a) there is an actual case or controversy; (b) petitioners possess locus
rational way. And when the judiciary mediates to allocate constitutional boundaries, it does not standi; (c) the question of constitutionality is raised at the earliest opportunity; and (d) the issue of
assert any superiority over the other departments; it does not in reality nullify or invalidate an 106
constitutionality is the lis mota of the case. Of these four, the first two conditions will be the focus of
act of the legislature, but only asserts the solemn and sacred obligation assigned to it by the our discussion.
Constitution to determine conflicting claims of authority under the Constitution and to establish
for the parties in an actual controversy the rights which that instrument secures and guarantees 1. Petitioners have shown the presence of an actual case or controversy.
to them. This is in truth all that is involved in what is termed "judicial supremacy" which properly is the
107
power of judicial review under the Constitution. x x x x. (Emphases supplied) The OSG maintains that there is no actual case or controversy that exists, since the Senators have
not been deprived of the opportunity to invoke the privileges of the institution they are representing. It
The power of judicial review has since been strengthened in the 1987 Constitution. The scope of that contends that the nonparticipation of the Senators in the present petitions only confirms that even they
power has been extended to the determination of whether in matters traditionally considered to be believe that EDCA is a binding executive agreement that does not require their concurrence.
within the sphere of appreciation of another branch of government, an exercise of discretion has been
97 108
attended with grave abuse. The expansion of this power has made the political question doctrine "no It must be emphasized that the Senate has already expressed its position through SR 105. Through
longer the insurmountable obstacle to the exercise of judicial power or the impenetrable shield that the Resolution, the Senate has taken a position contrary to that of the OSG. As the body tasked to
98
protects executive and legislative actions from judicial inquiry or review." participate in foreign affairs by ratifying treaties, its belief that EDCA infringes upon its constitutional role
indicates that an actual controversy - albeit brought to the Court by non-Senators, exists.
This moderating power, however, must be exercised carefully and only if it cannot be completely
avoided. We stress that our Constitution is so incisively designed that it identifies the spheres of Moreover, we cannot consider the sheer abstention of the Senators from the present proceedings as
expertise within which the different branches of government shall function and the questions of policy basis for finding that there is no actual case or controversy before us. We point out that the focus of this
99
that they shall resolve. Since the power of judicial review involves the delicate exercise of examining requirement is the ripeness for adjudication of the matter at hand, as opposed to its being merely
109
the validity or constitutionality of an act of a coequal branch of government, this Court must continually conjectural or anticipatory. The case must involve a definite and concrete issue involving real parties
exercise restraint to avoid the risk of supplanting the wisdom of the constitutionally appointed actor with with conflicting legal rights and legal claims admitting of specific relief through a decree conclusive in
100 110
that of its own. nature. It should not equate with a mere request for an opinion or advice on what the law would be
111
upon an abstract, hypothetical, or contingent state of facts. As explained in Angara v. Electoral In assailing the constitutionality of a governmental act, petitioners suing as citizens may dodge the
112
Commission: requirement of having to establish a direct and personal interest if they show that the act affects a public
120 121
right. In arguing that they have legal standing, they claim that the case they have filed is a
[The] power of judicial review is limited to actual cases and controversies to be exercised after concerned citizen's suit. But aside from general statements that the petitions involve the protection of a
full opportunity of argument by the parties, and limited further to the constitutional question raised or public right, and that their constitutional rights as citizens would be violated, they fail to make any
the very lis mota presented. Any attempt at abstraction could only lead to dialectics and barren specific assertion of a particular public right that would be violated by the enforcement of EDCA. For
legal questions and to sterile conclusions of wisdom, justice or expediency of legislation. More their failure to do so, the present petitions cannot be considered by the Court as citizens' suits
than that, courts accord the presumption of constitutionality to legislative enactments, not only because that would justify a disregard of the aforementioned requirements.
the legislature is presumed to abide by the Constitution but also because the judiciary in the
122
determination of actual cases and controversies must reflect the wisdom and justice of the In claiming that they have legal standing as taxpayers, petitioners aver that the implementation of
people as expressed through their representatives in the executive and legislative departments EDCA would result in the unlawful use of public funds. They emphasize that Article X(1) refers to an
of the government. (Emphases supplied) appropriation of funds; and that the agreement entails a waiver of the payment of taxes, fees, and
rentals. During the oral arguments, however, they admitted that the government had not yet
We find that the matter before us involves an actual case or controversy that is already ripe for 123
appropriated or actually disbursed public funds for the purpose of implementing the agreement. The
adjudication. The Executive Department has already sent an official confirmation to the U.S. Embassy 124
OSG, on the other hand, maintains that petitioners cannot sue as taxpayers. Respondent explains
113
that "all internal requirements of the Philippines x x x have already been complied with." By this that EDCA is neither meant to be a tax measure, nor is it directed at the disbursement of public funds.
exchange of diplomatic notes, the Executive Department effectively performed the last act required
under Article XII(l) of EDCA before the agreement entered into force. Section 25, Article XVIII of the A taxpayer's suit concerns a case in which the official act complained of directly involves the illegal
125
Constitution, is clear that the presence of foreign military forces in the country shall only be allowed by disbursement of public funds derived from taxation. Here, those challenging the act must specifically
virtue of a treaty concurred in by the Senate. Hence, the performance of an official act by the Executive show that they have sufficient interest in preventing the illegal expenditure of public money, and that
126
Department that led to the entry into force of an executive agreement was sufficient to satisfy the actual they will sustain a direct injury as a result of the enforcement of the assailed act. Applying that
case or controversy requirement. principle to this case, they must establish that EDCA involves the exercise by Congress of its taxing or
127
spending powers.
2. While petitioners Saguisag et. al., do not have legal standing, they nonetheless raise issues
involving matters of transcendental importance. We agree with the OSG that the petitions cannot qualify as taxpayers' suits. We emphasize that a
taxpayers' suit contemplates a situation in which there is already an appropriation or a disbursement of
The question of locus standi or legal standing focuses on the determination of whether those assailing 128
public funds. A reading of Article X(l) of EDCA would show that there has been neither an
the governmental act have the right of appearance to bring the matter to the court for appropriation nor an authorization of disbursement of funds. The cited provision reads:
114
adjudication. They must show that they have a personal and substantial interest in the case, such
that they have sustained or are in immediate danger of sustaining, some direct injury as a consequence All obligations under this Agreement are subject to the availability of appropriated funds authorized
115
of the enforcement of the challenged governmental act. Here, "interest" in the question involved must for these purposes. (Emphases supplied)
be material - an interest that is in issue and will be affected by the official act - as distinguished from
116 This provision means that if the implementation of EDCA would require the disbursement of public
being merely incidental or general. Clearly, it would be insufficient to show that the law or any
117 funds, the money must come from appropriated funds that are specifically authorized for this purpose.
governmental act is invalid, and that petitioners stand to suffer in some indefinite way. They must
show that they have a particular interest in bringing the suit, and that they have been or are about to be Under the agreement, before there can even be a disbursement of public funds, there must first be a
denied some right or privilege to which they are lawfully entitled, or that they are about to be subjected legislative action. Until and unless the Legislature appropriates funds for EDCA, or unless
118 petitioners can pinpoint a specific item in the current budget that allows expenditure under the
to some burden or penalty by reason of the act complained of. The reason why those who challenge
the validity of a law or an international agreement are required to allege the existence of a personal agreement, we cannot at this time rule that there is in fact an appropriation or a disbursement of
stake in the outcome of the controversy is "to assure the concrete adverseness which sharpens the funds that would justify the filing of a taxpayers' suit.
presentation of issues upon which the court so largely depends for illumination of difficult constitutional 129
questions."
119 Petitioners Bayan et al. also claim that their co-petitioners who are party-list representatives have the
standing to challenge the act of the Executive Department, especially if it impairs the constitutional
The present petitions cannot qualify as citizens', taxpayers', or legislators' suits; the Senate as a body prerogatives, powers, and privileges of their office. While they admit that there is no incumbent Senator
has the requisite standing, but considering that it has not formally filed a pleading to join the suit, as it who has taken part in the present petition, they nonetheless assert that they also stand to sustain a
merely conveyed to the Supreme Court its sense that EDCA needs the Senate's concurrence to be derivative but substantial injury as legislators. They argue that under the Constitution, legislative power
valid, petitioners continue to suffer from lack of standing. is vested in both the Senate and the House of Representatives; consequently, it is the entire Legislative
Department that has a voice in determining whether or not the presence of foreign military should be Nevertheless, this Court finds that there is basis for it to review the act of the Executive for the following
allowed. They maintain that as members of the Legislature, they have the requisite personality to bring reasons.
a suit, especially when a constitutional issue is raised.
In any case, petitioners raise issues involving matters of transcendental importance.
130
The OSG counters that petitioners do not have any legal standing to file the suits concerning the lack
138
of Senate concurrence in EDCA. Respondent emphasizes that the power to concur in treaties and Petitioners argue that the Court may set aside procedural technicalities, as the present petition
international agreements is an "institutional prerogative" granted by the Constitution to the Senate. tackles issues that are of transcendental importance. They point out that the matter before us is about
Accordingly, the OSG argues that in case of an allegation of impairment of that power, the injured party the proper exercise of the Executive Department's power to enter into international agreements in
would be the Senate as an institution or any of its incumbent members, as it is the Senate's relation to that of the Senate to concur in those agreements. They also assert that EDCA would cause
constitutional function that is allegedly being violated. grave injustice, as well as irreparable violation of the Constitution and of the Filipino people's rights.
139
The legal standing of an institution of the Legislature or of any of its Members has already been The OSG, on the other hand, insists that petitioners cannot raise the mere fact that the present
131 petitions involve matters of transcendental importance in order to cure their inability to comply with the
recognized by this Court in a number of cases. What is in question here is the alleged impairment of
the constitutional duties and powers granted to, or the impermissible intrusion upon the domain of, the constitutional requirement of standing. Respondent bewails the overuse of "transcendental importance"
132 as an exception to the traditional requirements of constitutional litigation. It stresses that one of the
Legislature or an institution thereof. In the case of suits initiated by the legislators themselves, this
Court has recognized their standing to question the validity of any official action that they claim infringes purposes of these requirements is to protect the Supreme Court from unnecessary litigation of
133 constitutional questions.
the prerogatives, powers, and privileges vested by the Constitution in their office. As aptly explained
134
by Justice Perfecto in Mabanag v. Lopez Vito: 140
In a number of cases, this Court has indeed taken a liberal stance towards the requirement of legal
Being members of Congress, they are even duty bound to see that the latter act within the bounds standing, especially when paramount interest is involved. Indeed, when those who challenge the official
of the Constitution which, as representatives of the people, they should uphold, unless they are to act are able to craft an issue of transcendental significance to the people, the Court may exercise its
commit a flagrant betrayal of public trust. They are representatives of the sovereign people and it is sound discretion and take cognizance of the suit. It may do so in spite of the inability of the petitioners to
their sacred duty to see to it that the fundamental law embodying the will of the sovereign show that they have been personally injured by the operation of a law or any other government act.
people is not trampled upon. (Emphases supplied)
While this Court has yet to thoroughly delineate the outer limits of this doctrine, we emphasize that not
We emphasize that in a legislators' suit, those Members of Congress who are challenging the official act every other case, however strong public interest may be, can qualify as an issue of transcendental
have standing only to the extent that the alleged violation impinges on their right to participate in the importance. Before it can be impelled to brush aside the essential requisites for exercising its power of
135 judicial review, it must at the very least consider a number of factors: (1) the character of the funds or
exercise of the powers of the institution of which they are members. Legislators have the standing "to
maintain inviolate the prerogatives, powers, and privileges vested by the Constitution in their office and other assets involved in the case; (2) the presence of a clear case of disregard of a constitutional or
are allowed to sue to question the validity of any official action, which they claim infringes their statutory prohibition by the public respondent agency or instrumentality of the government; and (3) the
136 141
prerogatives as legislators." As legislators, they must clearly show that there was a direct injury to lack of any other party that has a more direct and specific interest in raising the present questions.
137
their persons or the institution to which they belong.
An exhaustive evaluation of the memoranda of the parties, together with the oral arguments, shows that
As correctly argued by respondent, the power to concur in a treaty or an international agreement is an petitioners have presented serious constitutional issues that provide ample justification for the Court to
institutional prerogative granted by the Constitution to the Senate, not to the entire Legislature. set aside the rule on standing. The transcendental importance of the issues presented here is rooted in
In Pimentel v. Office of the Executive Secretary, this Court did not recognize the standing of one of the the Constitution itself. Section 25, Article XVIII thereof, cannot be any clearer: there is a much stricter
petitioners therein who was a member of the House of Representatives. The petition in that case sought mechanism required before foreign military troops, facilities, or bases may be allowed in the country.
to compel the transmission to the Senate for concurrence of the signed text of the Statute of the The DFA has already confirmed to the U.S. Embassy that "all internal requirements of the Philippines x
142
International Criminal Court. Since that petition invoked the power of the Senate to grant or withhold its x x have already been complied with." It behooves the Court in this instance to take a liberal stance
concurrence in a treaty entered into by the Executive Department, only then incumbent Senator towards the rule on standing and to determine forthwith whether there was grave abuse of discretion on
Pimentel was allowed to assert that authority of the Senate of which he was a member. the part of the Executive Department.

Therefore, none of the initial petitioners in the present controversy has the standing to maintain We therefore rule that this case is a proper subject for judicial review.
the suits as legislators.
B. Whether the President may enter into an executive agreement on foreign military bases,
troops, or facilities
155 156
C. Whether the provisions under EDCA are consistent with the Constitution, as well as with orders; the authority to deport undesirable aliens; the conferment of national awards under the
157 158
existing laws and treaties President's jurisdiction; and the overall administration and control of the executive department.

Issues B and C shall be discussed together infra. These obligations are as broad as they sound, for a President cannot function with crippled hands, but
must be capable of securing the rule of law within all territories of the Philippine Islands and be
1. The role of the President as the executor of the law includes the duty to defend the State, for empowered to do so within constitutional limits. Congress cannot, for instance, limit or take over the
which purpose he may use that power in the conduct of foreign relations 159
President's power to adopt implementing rules and regulations for a law it has enacted.

Historically, the Philippines has mirrored the division of powers in the U.S. government. When the 160
More important, this mandate is self-executory by virtue of its being inherently executive in nature. As
Philippine government was still an agency of the Congress of the U.S., it was as an agent entrusted 161
Justice Antonio T. Carpio previously wrote,
with powers categorized as executive, legislative, and judicial, and divided among these three great
143
branches. By this division, the law implied that the divided powers cannot be exercised except by the [i]f the rules are issued by the President in implementation or execution of self-executory constitutional
144
department given the power. powers vested in the President, the rule-making power of the President is not a delegated legislative
power. The most important self-executory constitutional power of the President is the President's
This divide continued throughout the different versions of the Philippine Constitution and specifically constitutional duty and mandate to "ensure that the laws be faithfully executed." The rule is that the
145
vested the supreme executive power in the Governor-General of the Philippines, a position inherited President can execute the law without any delegation of power from the legislature.
by the President of the Philippines when the country attained independence. One of the principal
functions of the supreme executive is the responsibility for the faithful execution of the laws as The import of this characteristic is that the manner of the President's execution of the law, even
146
embodied by the oath of office. The oath of the President prescribed by the 1987 Constitution reads if not expressly granted by the law, is justified by necessity and limited only by law, since the
162
thus: President must "take necessary and proper steps to carry into execution the law." Justice
163
George Malcolm states this principle in a grand manner:
I do solemnly swear (or affirm) that I will faithfully and conscientiously fulfill my duties as
President (or Vice-President or Acting President) of the Philippines, preserve and defend its The executive should be clothed with sufficient power to administer efficiently the affairs of state. He
Constitution, execute its laws, do justice to every man, and consecrate myself to the service of the should have complete control of the instrumentalities through whom his responsibility is discharged. It is
147
Nation. So help me God. (In case of affirmation, last sentence will be omitted.) (Emphases supplied) still true, as said by Hamilton, that "A feeble executive implies a feeble execution of the government. A
feeble execution is but another phrase for a bad execution; and a government ill executed, whatever it
This Court has interpreted the faithful execution clause as an obligation imposed on the President, and may be in theory, must be in practice a bad government." The mistakes of State governments need not
148
not a separate grant of power. Section 1 7, Article VII of the Constitution, expresses this duty in no be repeated here.
uncertain terms and includes it in the provision regarding the President's power of control over the
executive department, viz: xxxx

The President shall have control of all the executive departments, bureaus, and offices. He shall ensure Every other consideration to one side, this remains certain - The Congress of the United States clearly
that the laws be faithfully executed. intended that the Governor-General's power should be commensurate with his responsibility. The
Congress never intended that the Governor-General should be saddled with the responsibility of
The equivalent provisions in the next preceding Constitution did not explicitly require this oath from the administering the government and of executing the laws but shorn of the power to do so. The interests
President. In the 1973 Constitution, for instance, the provision simply gives the President control over of the Philippines will be best served by strict adherence to the basic principles of constitutional
149
the ministries. A similar language, not in the form of the President's oath, was present in the 1935 government.
150
Constitution, particularly in the enumeration of executive functions. By 1987, executive power was
151
codified not only in the Constitution, but also in the Administrative Code: In light of this constitutional duty, it is the President's prerogative to do whatever is legal and necessary
for Philippine defense interests. It is no coincidence that the constitutional provision on the faithful
SECTION 1. Power of Control. - The President shall have control of all the executive departments, 164
execution clause was followed by that on the President's commander-in-chief powers, which are
bureaus, and offices. He shall ensure that the laws be faithfully executed. (Emphasis supplied) specifically granted during extraordinary events of lawless violence, invasion, or rebellion. And this duty
of defending the country is unceasing, even in times when there is no state of lawlesss violence,
Hence, the duty to faithfully execute the laws of the land is inherent in executive power and is intimately invasion, or rebellion. At such times, the President has full powers to ensure the faithful execution of the
related to the other executive functions. These functions include the faithful execution of the law in laws.
152 153
autonomous regions; the right to prosecute crimes; the implementation of transportation
154
projects; the duty to ensure compliance with treaties, executive agreements and executive
It would therefore be remiss for the President and repugnant to the faithful-execution clause of the question whether the Philippine government should espouse claims of its nationals against a foreign
Constitution to do nothing when the call of the moment requires increasing the military's defensive government is a foreign relations matter, the authority for which is demonstrably committed by our
capabilities, which could include forging alliances with states that hold a common interest with the Constitution not to the courts but to the political branches. In this case, the Executive Department has
Philippines or bringing an international suit against an offending state. already decided that it is to the best interest of the country to waive all claims of its nationals for
reparations against Japan in the Treaty of Peace of 1951. The wisdom of such decision is not for the
The context drawn in the analysis above has been termed by Justice Arturo D. Brion's Dissenting courts to question. Neither could petitioners herein assail the said determination by the Executive
165
Opinion as the beginning of a "patent misconception." His dissent argues that this approach taken in Department via the instant petition for certiorari.
analyzing the President's role as executor of the laws is preceded by the duty to preserve and defend
166
the Constitution, which was allegedly overlooked. In the seminal case of US v. Curtiss-Wright Export Corp., the US Supreme Court held that "[t]he
President is the sole organ of the nation in its external relations, and its sole representative with foreign
In arguing against the approach, however, the dissent grossly failed to appreciate the nuances of the relations."
analysis, if read holistically and in context. The concept that the President cannot function with crippled
167
hands and therefore can disregard the need for Senate concurrence in treaties was never expressed It is quite apparent that if, in the maintenance of our international relations, embarrassment - perhaps
or implied. Rather, the appropriate reading of the preceding analysis shows that the point being serious embarrassment - is to be avoided and success for our aims achieved, congressional legislation
elucidated is the reality that the President's duty to execute the laws and protect the Philippines is which is to be made effective through negotiation and inquiry within the international field must often
inextricably interwoven with his foreign affairs powers, such that he must resolve issues imbued with accord to the President a degree of discretion and freedom from statutory restriction which
both concerns to the full extent of his powers, subject only to the limits supplied by law. In other words, would not be admissible where domestic affairs alone involved. Moreover, he, not Congress, has
apart from an expressly mandated limit, or an implied limit by virtue of incompatibility, the manner of the better opportunity of knowing the conditions which prevail in foreign countries, and especially is this
execution by the President must be given utmost deference. This approach is not different from that true in time of war. He has his confidential sources of information. He has his agents in the form of
taken by the Court in situations with fairly similar contexts. diplomatic, consular and other officials ....

Thus, the analysis portrayed by the dissent does not give the President authority to bypass This ruling has been incorporated in our jurisprudence through Bavan v. Executive
constitutional safeguards and limits. In fact, it specifies what these limitations are, how these limitations Secretary and Pimentel v. Executive Secretary; its overreaching principle was, perhaps, best
are triggered, how these limitations function, and what can be done within the sphere of constitutional articulated in (now Chief) Justice Puno's dissent in Secretary of Justice v. Lantion:
duties and limitations of the President.
. . . The conduct of foreign relations is full of complexities and consequences, sometimes with life and
Justice Brion's dissent likewise misinterprets the analysis proffered when it claims that the foreign death significance to the nation especially in times of war. It can only be entrusted to that department of
168
relations power of the President should not be interpreted in isolation. The analysis itself government which can act on the basis of the best available information and can decide with
demonstrates how the foreign affairs function, while mostly the President's, is shared in several decisiveness .... It is also the President who possesses the most comprehensive and the most
instances, namely in Section 2 of Article II on the conduct of war; Sections 20 and 21 of Article VII on confidential information about foreign countries for our diplomatic and consular officials regularly brief
foreign loans, treaties, and international agreements; Sections 4(2) and 5(2)(a) of Article VIII on the him on meaningful events all over the world. He has also unlimited access to ultra-sensitive military
judicial review of executive acts; Sections 4 and 25 of Article XVIII on treaties and international intelligence data. In fine, the presidential role in foreign affairs is dominant and the President is
agreements entered into prior to the Constitution and on the presence of foreign military troops, bases, traditionally accorded a wider degree of discretion in the conduct of foreign affairs. The
or facilities. regularity, nay, validity of his actions are adjudged under less stringent standards, lest their
judicial repudiation lead to breach of an international obligation, rupture of state relations,
In fact, the analysis devotes a whole subheading to the relationship between the two major presidential forfeiture of confidence, national embarrassment and a plethora of other problems with equally
functions and the role of the Senate in it. 169
undesirable consequences. (Emphases supplied)

This approach of giving utmost deference to presidential initiatives in respect of foreign affairs is not Understandably, this Court must view the instant case with the same perspective and understanding,
novel to the Court. The President's act of treating EDCA as an executive agreement is not the principal knowing full well the constitutional and legal repercussions of any judicial overreach.
power being analyzed as the Dissenting Opinion seems to suggest. Rather, the preliminary analysis is
in reference to the expansive power of foreign affairs. We have long treated this power as something 2. The plain meaning of the Constitution prohibits the entry of foreign military bases, troops or
the Courts must not unduly restrict. As we stated recently in Vinuya v. Romulo: facilities, except by way of a treaty concurred in by the Senate - a clear limitation on the
President's dual role as defender of the State and as sole authority in foreign relations.
To be sure, not all cases implicating foreign relations present political questions, and courts certainly
possess the authority to construe or invalidate treaties and executive agreements. However, the
Despite the President's roles as defender of the State and sole authority in foreign relations, the 1987 Constitution clearly requires submission of EDCA to the Senate. Two specific provisions versus one
Constitution expressly limits his ability in instances when it involves the entry of foreign military bases, general provision means that the specific provisions prevail. The term "executive agreement" is "a term
troops or facilities. The initial limitation is found in Section 21 of the provisions on the Executive wandering alone in the Constitution, bereft of provenance and an unidentified constitutional mystery."
Department: "No treaty or international agreement shall be valid and effective unless concurred in by at
least two-thirds of all the Members of the Senate." The specific limitation is given by Section 25 of the The author of SR 105, Senator Miriam Defensor Santiago, upon interpellation even added that the
Transitory Provisions, the full text of which reads as follows: MDT, which the Executive claims to be partly implemented through EDCA, is already obsolete.

SECTION 25. After the expiration in 1991 of the Agreement between the Republic of the Philippines There are two insurmountable obstacles to this Court's agreement with SR 105, as well as with the
and the United States of America concerning Military Bases, foreign military bases, troops, or facilities comment on interpellation made by Senator Santiago.
shall not be allowed in the Philippines except under a treaty duly concurred in by the Senate and, when
the Congress so requires, ratified by a majority of the votes cast by the people in a national referendum First, the concept of "executive agreement" is so well-entrenched in this Court's pronouncements on the
held for that purpose, and recognized as a treaty by the other contracting State. powers of the President. When the Court validated the concept of "executive agreement," it did so with
full knowledge of the Senate's role in concurring in treaties. It was aware of the problematique of
It is quite plain that the Transitory Provisions of the 1987 Constitution intended to add to the basic distinguishing when an international agreement needed Senate concurrence for validity, and when it did
requirements of a treaty under Section 21 of Article VII. This means that both provisions must be read not; and the Court continued to validate the existence of "executive agreements" even after the 1987
172
as additional limitations to the President's overarching executive function in matters of defense and Constitution. This follows a long line of similar decisions upholding the power of the President to enter
173
foreign relations. into an executive agreement.
174
3. The President, however, may enter into an executive agreement on foreign military bases, Second, the MDT has not been rendered obsolescent, considering that as late as 2009, this Court
troops, or facilities, if (a) it is not the instrument that allows the presence of foreign military continued to recognize its validity.
bases, troops, or facilities; or (b) it merely aims to implement an existing law or treaty.
Third, to this Court, a plain textual reading of Article XIII, Section 25, inevitably leads to the conclusion
Again we refer to Section 25, Article XVIII of the Constitution: that it applies only to a proposed agreement between our government and a foreign government,
whereby military bases, troops, or facilities of such foreign government would be "allowed" or would
SECTION 25. After the expiration in 1991 of the Agreement between the Republic of the Philippines "gain entry" Philippine territory.
and the United States of America concerning Military Bases, foreign military bases, troops, or
facilities shall not be allowed in the Philippines except under a treaty duly concurred in by the Note that the provision "shall not be allowed" is a negative injunction. This wording signifies that the
Senate and, when the Congress so requires, ratified by a majority of the votes cast by the people in a President is not authorized by law to allow foreign military bases, troops, or facilities to enter the
national referendum held for that purpose, and recognized as a treaty by the other contracting State. Philippines, except under a treaty concurred in by the Senate. Hence, the constitutionally restricted
(Emphases supplied) authority pertains to the entry of the bases, troops, or facilities, and not to the activities to be done after
entry.
170
In view of this provision, petitioners argue that EDCA must be in the form of a "treaty" duly concurred
in by the Senate. They stress that the Constitution is unambigous in mandating the transmission to the Under the principles of constitutional construction, of paramount consideration is the plain meaning of
175
Senate of all international agreements concluded after the expiration of the MBA in 1991 - agreements the language expressed in the Constitution, or the verba legis rule. It is presumed that the provisions
176
that concern the presence of foreign military bases, troops, or facilities in the country. Accordingly, have been carefully crafted in order to express the objective it seeks to attain. It is incumbent upon
petitioners maintain that the Executive Department is not given the choice to conclude agreements like the Court to refrain from going beyond the plain meaning of the words used in the Constitution. It is
EDCA in the form of an executive agreement. presumed that the framers and the people meant what they said when they said it, and that this
understanding was reflected in the Constitution and understood by the people in the way it was meant
177
This is also the view of the Senate, which, through a majority vote of 15 of its members - with 1 against to be understood when the fundamental law was ordained and promulgated. As this Court has often
171
and 2 abstaining - says in SR 105 that EDCA must be submitted to the Senate in the form of a treaty said:
for concurrence by at least two-thirds of all its members.
We look to the language of the document itself in our search for its meaning. We do not of course stop
The Senate cites two constitutional provisions (Article VI, Section 21 and Article XVIII, Section 25) to there, but that is where we begin. It is to be assumed that the words in which constitutional provisions
support its position. Compared with the lone constitutional provision that the Office of the Solicitor are couched express the objective sought to be attained. They are to be given their ordinary
General (OSG) cites, which is Article XVIII, Section 4(2), which includes the constitutionality of meaning except where technical terms are employed in which case the significance thus attached to
"executive agreement(s)" among the cases subject to the Supreme Court's power of judicial review, the them prevails. As the Constitution is not primarily a lawyer's document, it being essential for the
rule of law to obtain that it should ever be present in the people's consciousness, its language as and intent of the Mutual Defense Treaty and the VFA support the conclusion that combat-related
much as possible should be understood in the sense they have in common use. What it says activities -as opposed to combat itself-such as the one subject of the instant petition, are indeed
184
according to the text of the provision to be construed compels acceptance and negates the power of the authorized. (Emphasis supplied)
courts to alter it, based on the postulate that the framers and the people mean what they say.
Thus, these are the cases where the need for construction is reduced to a Moreover, the Court indicated that the Constitution continues to govern the conduct of foreign military
178 185
minimum. (Emphases supplied) troops in the Philippines, readily implying the legality of their initial entry into the country.

It is only in those instances in which the constitutional provision is unclear, ambiguous, or silent that The OSG emphasizes that EDCA can be in the form of an executive agreement, since it merely
179 186
further construction must be done to elicit its meaning. In Ang Bagong Bayani-OFW v. Commission involves "adjustments in detail" in the implementation of the MDT and the VFA. It points out that there
180 are existing treaties between the Philippines and the U.S. that have already been concurred in by the
on Elections, we reiterated this guiding principle:
Philippine Senate and have thereby met the requirements of the Constitution under Section 25.
it [is] safer to construe the Constitution from what appears upon its face. The proper interpretation Because of the status of these prior agreements, respondent emphasizes that EDCA need not be
therefore depends more on how it was understood by the people adopting it than in the framers' transmitted to the Senate.
understanding thereof. (Emphases supplied)
The aforecited Dissenting Opinion of Justice Brion disagrees with the ponencia's application of verba
187
The effect of this statement is surprisingly profound, for, if taken literally, the phrase "shall not be legis construction to the words of Article XVIII, Section 25. It claims that the provision is "neither plain,
188
allowed in the Philippines" plainly refers to the entry of bases, troops, or facilities in the country. nor that simple." To buttress its disagreement, the dissent states that the provision refers to a
189
The Oxford English Dictionary defines the word "allow" as a transitive verb that means "to permit, historical incident, which is the expiration of the 1947 MBA. Accordingly, this position requires
enable"; "to give consent to the occurrence of or relax restraint on (an action, event, or activity)"; "to questioning the circumstances that led to the historical event, and the meaning of the terms under
consent to the presence or attendance of (a person)"; and, when with an adverbial of place, "to permit Article XVIII, Section 25.
181
(a person or animal) to go, come, or be in, out, near, etc." Black's Law Dictionary defines the term as
182 This objection is quite strange. The construction technique of verba legis is not inapplicable just
one that means "[t]o grant, approve, or permit."
because a provision has a specific historical context. In fact, every provision of the Constitution has a
The verb "allow" is followed by the word "in," which is a preposition used to indicate "place or position in specific historical context. The purpose of constitutional and statutory construction is to set tiers of
space or anything having material extension: Within the limits or bounds of, within (any place or interpretation to guide the Court as to how a particular provision functions. Verba legis is of paramount
183
thing)." That something is the Philippines, which is the noun that follows. consideration, but it is not the only consideration. As this Court has often said:

It is evident that the constitutional restriction refers solely to the initial entry of the foreign military bases, We look to the language of the document itself in our search for its meaning. We do not of course
troops, or facilities. Once entry is authorized, the subsequent acts are thereafter subject only to the stop there, but that is where we begin. It is to be assumed that the words in which constitutional
limitations provided by the rest of the Constitution and Philippine law, and not to the Section 25 provisions are couched express the objective sought to be attained. They are to be given their
requirement of validity through a treaty. ordinary meaning except where technical terms are employed in which case the significance thus
attached to them prevails. As the Constitution is not primarily a lawyer's document, it being essential for
The VFA has already allowed the entry of troops in the Philippines. This Court stated in Lim v. the rule of law to obtain that it should ever be present in the people's consciousness, its language as
Executive Secretary: much as possible should be understood in the sense they have in common use. What it says
according to the text of the provision to be construed compels acceptance and negates the power of the
After studied reflection, it appeared farfetched that the ambiguity surrounding the meaning of the word courts to alter it, based on the postulate that the framers and the people mean what they say.
"activities" arose from accident. In our view, it was deliberately made that way to give both parties a Thus, these are the cases where the need for construction is reduced to a
certain leeway in negotiation. In this manner, visiting US forces may sojourn in Philippine territory 190
minimum. (Emphases supplied)
for purposes other than military. As conceived, the joint exercises may include training on new
techniques of patrol and surveillance to protect the nation's marine resources, sea search-and-rescue As applied, verba legis aids in construing the ordinary meaning of terms. In this case, the phrase being
operations to assist vessels in distress, disaster relief operations, civic action projects such as the construed is "shall not be allowed in the Philippines" and not the preceding one referring to "the
building of school houses, medical and humanitarian missions, and the like. expiration in 1991 of the Agreement between the Republic of the Philippines and the United States of
America concerning Military Bases, foreign military bases, troops, or facilities." It is explicit in the
Under these auspices, the VFA gives legitimacy to the current Balikatan exercises. It is only logical to wording of the provision itself that any interpretation goes beyond the text itself and into the discussion
assume that "Balikatan 02-1," a "mutual anti- terrorism advising, assisting and training exercise," falls of the framers, the context of the Constitutional Commission's time of drafting, and the history of the
under the umbrella of sanctioned or allowable activities in the context of the agreement. Both the history 1947 MBA. Without reference to these factors, a reader would not understand those terms. However,
for the phrase "shall not be allowed in the Philippines," there is no need for such reference. The law is (a) All cases in which the constitutionality or validity of any treaty, international or executive
clear. No less than the Senate understood this when it ratified the VFA. agreement, law, presidential decree, proclamation, order, instruction, ordinance, or regulation is in
question. (Emphases supplied)
4. The President may generally enter into executive agreements subject to limitations defined by
the Constitution and may be in furtherance of a treaty already concurred in by the Senate. In Commissioner of Customs v. Eastern Sea Trading, executive agreements are defined as
"international agreements embodying adjustments of detail carrying out well-established national
We discuss in this section why the President can enter into executive agreements. policies and traditions and those involving arrangements of a more or less temporary
204
nature." In Bayan Muna v. Romulo, this Court further clarified that executive agreements can cover a
It would be helpful to put into context the contested language found in Article XVIII, Section 25. Its more 205
wide array of subjects that have various scopes and purposes. They are no longer limited to the
exacting requirement was introduced because of the previous experience of the country when its traditional subjects that are usually covered by executive agreements as identified in Eastern Sea
191
representatives felt compelled to consent to the old MBA. They felt constrained to agree to the MBA Trading. The Court thoroughly discussed this matter in the following manner:
192
in fulfilment of one of the major conditions for the country to gain independence from the U.S. As a
result of that experience, a second layer of consent for agreements that allow military bases, troops and The categorization of subject matters that may be covered by international
facilities in the country is now articulated in Article XVIII of our present Constitution. agreementsmentioned in Eastern Sea Trading is not cast in stone. x x x.

This second layer of consent, however, cannot be interpreted in such a way that we completely ignore As may be noted, almost half a century has elapsed since the Court rendered its decision in Eastern
the intent of our constitutional framers when they provided for that additional layer, nor the vigorous Sea Trading. Since then, the conduct of foreign affairs has become more complex and the
statements of this Court that affirm the continued existence of that class of international agreements domain of international law wider, as to include such subjects as human rights, the environment, and
called "executive agreements." the sea. In fact, in the US alone, the executive agreements executed by its President from 1980 to 2000
covered subjects such as defense, trade, scientific cooperation, aviation, atomic energy,
The power of the President to enter into binding executive agreements without Senate concurrence is environmental cooperation, peace corps, arms limitation, and nuclear safety, among others.
193
already well-established in this jurisdiction. That power has been alluded to in our present and past Surely, the enumeration in Eastern Sea Trading cannot circumscribe the option of each state on
194 195 196
Constitutions, in various statutes, in Supreme Court decisions, and during the deliberations of the matter of which the international agreement format would be convenient to serve its best
197
the Constitutional Commission. They cover a wide array of subjects with varying scopes and interest. As Francis Sayre said in his work referred to earlier:
198 199
purposes, including those that involve the presence of foreign military forces in the country.
200
. . . It would be useless to undertake to discuss here the large variety of executive agreements as
As the sole organ of our foreign relations and the constitutionally assigned chief architect of our such concluded from time to time. Hundreds of executive agreements, other than those entered into
201
foreign policy, the President is vested with the exclusive power to conduct and manage the country's under the trade-agreement act, have been negotiated with foreign governments. . . . They cover such
interface with other states and governments. Being the principal representative of the Philippines, the subjects as the inspection of vessels, navigation dues, income tax on shipping profits, the admission of
Chief Executive speaks and listens for the nation; initiates, maintains, and develops diplomatic relations civil air craft, custom matters and commercial relations generally, international claims, postal matters,
with other states and governments; negotiates and enters into international agreements; promotes the registration of trademarks and copyrights, etc .... (Emphases Supplied)
trade, investments, tourism and other economic relations; and settles international disputes with other
202
states. One of the distinguishing features of executive agreements is that their validity and effectivity are not
206
affected by a lack of Senate concurrence. This distinctive feature was recognized as early as
As previously discussed, this constitutional mandate emanates from the inherent power of the President in Eastern Sea Trading (1961), viz:
to enter into agreements with other states, including the prerogative to conclude binding executive
agreements that do not require further Senate concurrence. The existence of this presidential Treaties are formal documents which require ratification with the approval of two-thirds of the
203
power is so well-entrenched that Section 5(2)(a), Article VIII of the Constitution, even provides for a Senate. Executive agreements become binding through executive action without the need of a
check on its exercise. As expressed below, executive agreements are among those official vote by the Senate or by Congress.
governmental acts that can be the subject of this Court's power of judicial review:
xxxx
(2) Review, revise, reverse, modify, or affirm on appeal or certiorari, as the law or the Rules of Court
may provide, final judgments and orders of lower courts in: [T]he right of the Executive to enter into binding agreements without the necessity of
subsequent Congressional approval has been confirmed by long usage. From the earliest days of
our history we have entered into executive agreements covering such subjects as commercial and
consular relations, most-favored-nation rights, patent rights, trademark and copyright protection, postal
and navigation arrangements and the settlement of claims. The validity of these has never been FR. BERNAS: I wonder if a quotation from the Supreme Court decision [in Eastern Sea Trading]
seriously questioned by our courts. (Emphases Supplied) might help clarify this:

That notion was carried over to the present Constitution. In fact, the framers specifically deliberated on The right of the executive to enter into binding agreements without the necessity of subsequent
whether the general term "international agreement" included executive agreements, and whether it was Congressional approval has been confirmed by long usage. From the earliest days of our history,
necessary to include an express proviso that would exclude executive agreements from the requirement we have entered into executive agreements covering such subjects as commercial and consular
of Senate concurrence. After noted constitutionalist Fr. Joaquin Bernas quoted the Court's ruling relations, most favored nation rights, patent rights, trademark and copyright protection, postal and
in Eastern Sea Trading, the Constitutional Commission members ultimately decided that the term navigation arrangements and the settlement of claims. The validity of this has never been seriously
"international agreements" as contemplated in Section 21, Article VII, does not include executive questioned by our Courts.
207
agreements, and that a proviso is no longer needed. Their discussion is reproduced below:
Agreements with respect to the registration of trademarks have been concluded by the executive of
MS. AQUINO: Madam President, first I would like a clarification from the Committee. We have retained various countries under the Act of Congress of March 3, 1881 (21 Stat. 502) . . . International
the words "international agreement" which I think is the correct judgment on the matter because an agreements involving political issues or changes of national policy and those involving
international agreement is different from a treaty. A treaty is a contract between parties which is in the international agreements of a permanent character usually take the form of treaties. But international
nature of international agreement and also a municipal law in the sense that the people are bound. So agreements embodying adjustments of detail, carrying out well established national policies and
there is a conceptual difference. However, I would like to be clarified if the international agreements traditions and those involving arrangements of a more or less temporary nature usually take the
include executive agreements. form of executive agreements.

MR. CONCEPCION: That depends upon the parties. All parties to these international negotiations MR. ROMULO: Is the Commissioner, therefore, excluding the executive agreements?
stipulate the conditions which are necessary for the agreement or whatever it may be to become valid
or effective as regards the parties. FR. BERNAS: What we are referring to, therefore, when we say international agreements which
need concurrence by at least two-thirds are those which are permanent in nature.
MS. AQUINO: Would that depend on the parties or would that depend on the nature of the executive
agreement? According to common usage, there are two types of executive agreement: one is purely MS. AQUINO: And it may include commercial agreements which are executive agreements essentially
proceeding from an executive act which affects external relations independent of the but which are proceeding from the authorization of Congress. If that is our understanding, then I am
legislative and the other is an executive act in pursuance of legislative authorization. The first willing to withdraw that amendment.
kind might take the form of just conventions or exchanges of notes or protocol while the other,
which would be pursuant to the legislative authorization, may be in the nature of commercial FR. BERNAS: If it is with prior authorization of Congress, then it does not need subsequent
agreements. concurrence by Congress.

MR. CONCEPCION: Executive agreements are generally made to implement a treaty already MS. AQUINO: In that case, I am withdrawing my amendment.
enforced or to determine the details for the implementation of the treaty. We are speaking of
MR. TINGSON: Madam President.
executive agreements, not international agreements.
THE PRESIDENT: Is Commissioner Aquino satisfied?
MS. AQUINO: I am in full agreement with that, except that it does not cover the first kind of executive
agreement which is just protocol or an exchange of notes and this would be in the nature of MS. AQUINO: Yes. There is already an agreement among us on the definition of "executive
reinforcement of claims of a citizen against a country, for example. agreements" and that would make unnecessary any explicit proviso on the matter.
MR. CONCEPCION: The Commissioner is free to require ratification for validity insofar as the xxx
Philippines is concerned.
MR. GUINGONA: I am not clear as to the meaning of "executive agreements" because I heard that
MS. AQUINO: It is my humble submission that we should provide, unless the Committee explains to these executive agreements must rely on treaties. In other words, there must first be treaties.
us otherwise, an explicit proviso which would except executive agreements from the requirement
of concurrence of two-thirds of the Members of the Senate. Unless I am enlightened by the MR. CONCEPCION: No, I was speaking about the common use, as executive agreements being the
Committee I propose that tentatively, the sentence should read. "No treaty or international agreement implementation of treaties, details of which do not affect the sovereignty of the State.
EXCEPT EXECUTIVE AGREEMENTS shall be valid and effective."
213
MR. GUINGONA: But what about the matter of permanence, Madam President? Would 99 years be form. Consequently, under international law, the distinction between a treaty and an international
considered permanent? What would be the measure of permanency? I do not conceive of a treaty that agreement or even an executive agreement is irrelevant for purposes of determining international rights
is going to be forever, so there must be some kind of a time limit. and obligations.

MR. CONCEPCION: I suppose the Commissioner's question is whether this type of agreement should However, this principle does not mean that the domestic law distinguishing treaties, international
be included in a provision of the Constitution requiring the concurrence of Congress. agreements, and executive agreements is relegated to a mere variation in form, or that the
constitutional requirement of Senate concurrence is demoted to an optional constitutional directive.
MR. GUINGONA: It depends on the concept of the executive agreement of which I am not clear. If the There remain two very important features that distinguish treaties from executive agreements and
executive agreement partakes of the nature of a treaty, then it should also be included. translate them into terms of art in the domestic setting.

MR. CONCEPCION: Whether it partakes or not of the nature of a treaty, it is within the power of the First, executive agreements must remain traceable to an express or implied authorization under the
Constitutional Commission to require that. Constitution, statutes, or treaties. The absence of these precedents puts the validity and effectivity of
executive agreements under serious question for the main function of the Executive is to enforce the
MR. GUINGONA: Yes. That is why I am trying to clarify whether the words "international Constitution and the laws enacted by the Legislature, not to defeat or interfere in the performance of
agreements" would include executive agreements. 214
these rules. In turn, executive agreements cannot create new international obligations that are not
expressly allowed or reasonably implied in the law they purport to implement.
MR. CONCEPCION: No, not necessarily; generally no.
Second, treaties are, by their very nature, considered superior to executive agreements. Treaties are
xxx 215
products of the acts of the Executive and the Senate unlike executive agreements, which are solely
216
MR. ROMULO: I wish to be recognized first. I have only one question. Do we take it, therefore, that as executive actions. Because of legislative participation through the Senate, a treaty is regarded as
217
far as the Committee is concerned, the term "international agreements" does not include the being on the same level as a statute. If there is an irreconcilable conflict, a later law or treaty takes
218
term "executive agreements" as read by the Commissioner in that text? precedence over one that is prior. An executive agreement is treated differently. Executive
219
agreements that are inconsistent with either a law or a treaty are considered ineffective. Both types of
220
FR. BERNAS: Yes. (Emphases Supplied) international agreement are nevertheless subject to the supremacy of the Constitution.

The inapplicability to executive agreements of the requirements under Section 21 was again recognized This rule does not imply, though, that the President is given carte blanche to exercise this discretion.
in Bayan v. Zamora and in Bayan Muna v. Romulo. These cases, both decided under the aegis of the Although the Chief Executive wields the exclusive authority to conduct our foreign relations, this power
present Constitution, quoted Eastern Sea Trading in reiterating that executive agreements are valid and must still be exercised within the context and the parameters set by the Constitution, as well as by
binding even without the concurrence of the Senate. existing domestic and international laws. There are constitutional provisions that restrict or limit the
President's prerogative in concluding international agreements, such as those that involve the following:
Executive agreements may dispense with the requirement of Senate concurrence because of the legal
221
mandate with which they are concluded. As culled from the afore-quoted deliberations of the a. The policy of freedom from nuclear weapons within Philippine territory
208
Constitutional Commission, past Supreme Court Decisions, and works of noted scholars, executive
agreements merely involve arrangements on the implementation of existing policies, rules, laws, or b. The fixing of tariff rates, import and export quotas, tonnage and wharfage dues, and other duties or
222
209
agreements. They are concluded (1) to adjust the details of a treaty; (2) pursuant to or upon imposts, which must be pursuant to the authority granted by Congress
210
confirmation by an act of the Legislature; or (3) in the exercise of the President's independent powers
211 c. The grant of any tax exemption, which must be pursuant to a law concurred in by a majority of all the
under the Constitution. The raison d'etre of executive agreements hinges on prior constitutional or 223
Members of Congress
legislative authorizations.
d. The contracting or guaranteeing, on behalf of the Philippines, of foreign loans that must be previously
The special nature of an executive agreement is not just a domestic variation in international 224
concurred in by the Monetary Board
agreements. International practice has accepted the use of various forms and designations of
international agreements, ranging from the traditional notion of a treaty - which connotes a formal, e. The authorization of the presence of foreign military bases, troops, or facilities in the country must be
solemn instrument - to engagements concluded in modem, simplified forms that no longer necessitate in the form of a treaty duly concurred in by the Senate.
225
212
ratification. An international agreement may take different forms: treaty, act, protocol,
agreement, concordat, compromis d'arbitrage, convention, covenant, declaration, exchange of notes, f. For agreements that do not fall under paragraph 5, the concurrence of the Senate is required, should
statute, pact, charter, agreed minute, memorandum of agreement, modus vivendi, or some other the form of the government chosen be a treaty.
5. The President had the choice to enter into EDCA by way of an executive agreement or a Petitioner parlays the notion that the Agreement is of dubious validity, partaking as it does of the nature
treaty. of a treaty; hence, it must be duly concurred in by the Senate. x x x x. Pressing its point, petitioner
submits that the subject of the Agreement does not fall under any of the subject-categories that xx x
No court can tell the President to desist from choosing an executive agreement over a treaty to embody may be covered by an executive agreement, such as commercial/consular relations, most-favored
an international agreement, unless the case falls squarely within Article VIII, Section 25. nation rights, patent rights, trademark and copyright protection, postal and navigation arrangements and
settlement of claims.
As can be gleaned from the debates among the members of the Constitutional Commission, they were
aware that legally binding international agreements were being entered into by countries in forms other The categorization of subject matters that may be covered by international agreements mentioned
than a treaty. At the same time, it is clear that they were also keen to preserve the concept of "executive in Eastern Sea Trading is not cast in stone. There are no hard and fast rules on the propriety of
agreements" and the right of the President to enter into such agreements. entering, on a given subject, into a treaty or an executive agreement as an instrument of
international relations. The primary consideration in the choice of the form of agreement is
What we can glean from the discussions of the Constitutional Commissioners is that they understood the parties' intent and desire to craft an international agreement in the form they so wish to
the following realities: further their respective interests. Verily, the matter of form takes a back seat when it comes to
effectiveness and binding effect of the enforcement of a treaty or an executive agreement, as the
1. Treaties, international agreements, and executive agreements are all constitutional manifestations of
parties in either international agreement each labor under the pacta sunt servanda principle.
the conduct of foreign affairs with their distinct legal characteristics.
xxxx
a. Treaties are formal contracts between the Philippines and other States-parties, which are in the
226
nature of international agreements, and also of municipal laws in the sense of their binding nature. But over and above the foregoing considerations is the fact that - save for the situation and matters
contemplated in Sec. 25, Art. XVIII of the Constitution - when a treaty is required, the Constitution
b. International agreements are similar instruments, the provisions of which may require the ratification
does not classify any subject, like that involving political issues, to be in the form of, and ratified
of a designated number of parties thereto. These agreements involving political issues or changes in
as, a treaty. What the Constitution merely prescribes is that treaties need the concurrence of the
national policy, as well as those involving international agreements of a permanent character, usually
Senate by a vote defined therein to complete the ratification process.
take the form of treaties. They may also include commercial agreements, which are executive
agreements essentially, but which proceed from previous authorization by Congress, thus dispensing xxxx
227
with the requirement of concurrence by the Senate.
x x x. As the President wields vast powers and influence, her conduct in the external affairs of the nation
c. Executive agreements are generally intended to implement a treaty already enforced or to determine is, as Bayan would put it, "executive altogether." The right of the President to enter into or ratify
228
the details of the implementation thereof that do not affect the sovereignty of the State. binding executive agreements has been confirmed by long practice.

2. Treaties and international agreements that cannot be mere executive agreements must, by In thus agreeing to conclude the Agreement thru E/N BF0-028-03, then President Gloria Macapagal-
constitutional decree, be concurred in by at least two-thirds of the Senate. Arroyo, represented by the Secretary of Foreign Affairs, acted within the scope of the authority and
discretion vested in her by the Constitution. At the end of the day, the President - by ratifying,
3. However, an agreement - the subject of which is the entry of foreign military troops, bases, or
thru her deputies, the non-surrender agreement - did nothing more than discharge a
facilities - is particularly restricted. The requirements are that it be in the form of a treaty concurred in by
constitutional duty and exercise a prerogative that pertains to her office. (Emphases supplied)
the Senate; that when Congress so requires, it be ratified by a majority of the votes cast by the people
in a national referendum held for that purpose; and that it be recognized as a treaty by the other Indeed, in the field of external affairs, the President must be given a larger measure of authority and
contracting State. wider discretion, subject only to the least amount of checks and restrictions under the
229
Constitution. The rationale behind this power and discretion was recognized by the Court in Vinuya v.
4. Thus, executive agreements can continue to exist as a species of international agreements. 230
Executive Secretary, cited earlier.
That is why our Court has ruled the way it has in several cases.
Section 9 of Executive Order No. 459, or the Guidelines in the Negotiation of International Agreements
In Bayan Muna v. Romulo, we ruled that the President acted within the scope of her constitutional and its Ratification, thus, correctly reflected the inherent powers of the President when it stated that the
authority and discretion when she chose to enter into the RP-U.S. Non-Surrender Agreement in the DFA "shall determine whether an agreement is an executive agreement or a treaty."
form of an executive agreement, instead of a treaty, and in ratifying the agreement without Senate
concurrence. The Court en banc discussed this intrinsic presidential prerogative as follows:
Accordingly, in the exercise of its power of judicial review, the Court does not look into whether an 3. The executive agreement must not go beyond the parameters, limitations, and standards set by the
international agreement should be in the form of a treaty or an executive agreement, save in cases in law and/or treaty that the former purports to implement; and must not unduly expand the international
which the Constitution or a statute requires otherwise. Rather, in view of the vast constitutional powers obligation expressly mentioned or necessarily implied in the law or treaty.
and prerogatives granted to the President in the field of foreign affairs, the task of the Court is to
determine whether the international agreement is consistent with the applicable limitations. 4. The executive agreement must be consistent with the Constitution, as well as with existing laws and
treaties.
6. Executive agreements may cover the matter of foreign military forces if it merely involves
detail adjustments. In light of the President's choice to enter into EDCA in the form of an executive agreement, respondents
carry the burden of proving that it is a mere implementation of existing laws and treaties concurred in by
The practice of resorting to executive agreements in adjusting the details of a law or a treaty that the Senate. EDCA must thus be carefully dissected to ascertain if it remains within the legal parameters
already deals with the presence of foreign military forces is not at all unusual in this jurisdiction. In fact, of a valid executive agreement.
231
the Court has already implicitly acknowledged this practice in Lim v. Executive Secretary. In that
case, the Court was asked to scrutinize the constitutionality of the Terms of Reference of the Balikatan 7. EDCA is consistent with the content, purpose, and framework of the MDT and the VFA
02-1 joint military exercises, which sought to implement the VFA. Concluded in the form of an executive
agreement, the Terms of Reference detailed the coverage of the term "activities" mentioned in the The starting point of our analysis is the rule that "an executive agreement xx x may not be used to
234
treaty and settled the matters pertaining to the construction of temporary structures for the U.S. troops amend a treaty." In Lim v. Executive Secretary and in Nicolas v. Romulo, the Court approached the
during the activities; the duration and location of the exercises; the number of participants; and the question of the validity of executive agreements by comparing them with the general framework and the
extent of and limitations on the activities of the U.S. forces. The Court upheld the Terms of Reference specific provisions of the treaties they seek to implement.
as being consistent with the VFA. It no longer took issue with the fact that the Balikatan Terms of
In Lim, the Terms of Reference of the joint military exercises was scrutinized by studying "the
Reference was not in the form of a treaty concurred in by the Senate, even if it dealt with the regulation 235
framework of the treaty antecedents to which the Philippines bound itself," i.e., the MDT and the VFA.
of the activities of foreign military forces on Philippine territory. 236
The Court proceeded to examine the extent of the term "activities" as contemplated in Articles 1 and
237
232
In Nicolas v. Romulo, the Court again impliedly affirmed the use of an executive agreement in an II of the VFA. It later on found that the term "activities" was deliberately left undefined and ambiguous
attempt to adjust the details of a provision of the VFA. The Philippines and the U.S. entered into the in order to permit "a wide scope of undertakings subject only to the approval of the Philippine
238 239
Romulo-Kenney Agreement, which undertook to clarify the detention of a U.S. Armed Forces member, government" and thereby allow the parties "a certain leeway in negotiation." The Court eventually
whose case was pending appeal after his conviction by a trial court for the crime of rape. In testing the ruled that the Terms of Reference fell within the sanctioned or allowable activities, especially in the
validity of the latter agreement, the Court precisely alluded to one of the inherent limitations of an context of the VFA and the MDT.
executive agreement: it cannot go beyond the terms of the treaty it purports to implement. It was
The Court applied the same approach to Nicolas v. Romulo. It studied the provisions of the VFA on
eventually ruled that the Romulo-Kenney Agreement was "not in accord" with the VFA, since the former 240
custody and detention to ascertain the validity of the Romulo-Kenney Agreement. It eventually found
was squarely inconsistent with a provision in the treaty requiring that the detention be "by Philippine
that the two international agreements were not in accord, since the Romulo-Kenney Agreement had
authorities." Consequently, the Court ordered the Secretary of Foreign Affairs to comply with the VFA
stipulated that U.S. military personnel shall be detained at the U.S. Embassy Compound and guarded
and "forthwith negotiate with the United States representatives for the appropriate agreement on
233 by U.S. military personnel, instead of by Philippine authorities. According to the Court, the parties
detention facilities under Philippine authorities as provided in Art. V, Sec. 10 of the VFA. " 241
"recognized the difference between custody during the trial and detention after conviction." Pursuant
Culling from the foregoing discussions, we reiterate the following pronouncements to guide us in to Article V(6) of the VFA, the custody of a U.S. military personnel resides with U.S. military authorities
resolving the present controversy: during trial. Once there is a finding of guilt, Article V(l0) requires that the confinement or detention be
"by Philippine authorities."
1. Section 25, Article XVIII of the Constitution, contains stringent requirements that must be fulfilled by
the international agreement allowing the presence of foreign military bases, troops, or facilities in the Justice Marvic M.V.F. Leonen's Dissenting Opinion posits that EDCA "substantially modifies or amends
242
Philippines: (a) the agreement must be in the form of a treaty, and (b) it must be duly concurred in by the VFA" and follows with an enumeration of the differences between EDCA and the VFA. While
the Senate. these arguments will be rebutted more fully further on, an initial answer can already be given to each of
the concerns raised by his dissent.
2. If the agreement is not covered by the above situation, then the President may choose the form of the
agreement (i.e., either an executive agreement or a treaty), provided that the agreement dealing with The first difference emphasized is that EDCA does not only regulate visits as the VFA does, but allows
foreign military bases, troops, or facilities is not the principal agreement that first allows their entry or temporary stationing on a rotational basis of U.S. military personnel and their contractors in physical
presence in the Philippines. locations with permanent facilities and pre-positioned military materiel.
This argument does not take into account that these permanent facilities, while built by U.S. forces, are authority derived from the treaty, but are not necessarily expressed word-for-word in the mother treaty.
243
to be owned by the Philippines once constructed. Even the VFA allowed construction for the benefit This concern shall be further elucidated in this Decision.
of U.S. forces during their temporary visits.
The fifth difference highlighted by the Dissenting Opinion is that the VFA does not have provisions that
The second difference stated by the dissent is that EDCA allows the prepositioning of military materiel, may be construed as a restriction on or modification of obligations found in existing statues, including
which can include various types of warships, fighter planes, bombers, and vessels, as well as land and the jurisdiction of courts, local autonomy, and taxation. Implied in this argument is that EDCA contains
244 249
amphibious vehicles and their corresponding ammunition. such restrictions or modifications.

However, the VFA clearly allows the same kind of equipment, vehicles, vessels, and aircraft to be This last argument cannot be accepted in view of the clear provisions of EDCA. Both the VFA and
brought into the country. Articles VII and VIII of the VFA contemplates that U.S. equipment, materials, EDCA ensure Philippine jurisdiction in all instances contemplated by both agreements, with the
supplies, and other property are imported into or acquired in the Philippines by or on behalf of the U.S. exception of those outlined by the VFA in Articles III-VI. In the VFA, taxes are clearly waived whereas in
Armed Forces; as are vehicles, vessels, and aircraft operated by or for U.S. forces in connection with EDCA, taxes are assumed by the government as will be discussed later on. This fact does not,
activities under the VFA. These provisions likewise provide for the waiver of the specific duties, taxes, therefore, produce a diminution of jurisdiction on the part of the Philippines, but rather a recognition of
charges, and fees that correspond to these equipment. sovereignty and the rights that attend it, some of which may be waived as in the cases under Articles III-
VI of the VFA.
The third difference adverted to by the Justice Leonen's dissent is that the VFA contemplates the entry
of troops for training exercises, whereas EDCA allows the use of territory for launching military and Taking off from these concerns, the provisions of EDCA must be compared with those of the MDT and
245
paramilitary operations conducted in other states. The dissent of Justice Teresita J. Leonardo-De the VFA, which are the two treaties from which EDCA allegedly draws its validity.
Castro also notes that VFA was intended for non-combat activides only, whereas the entry and activities
of U.S. forces into Agreed Locations were borne of military necessity or had a martial character, and "Authorized presence" under the VFA versus "authorized activities" under EDCA: (1) U.S.
246 personnel and (2) U.S. contractors
were therefore not contemplated by the VFA.
250
This Court's jurisprudence however established in no uncertain terms that combat-related activities, as The OSG argues that EDCA merely details existing policies under the MDT and the VFA. It explains
opposed to actual combat, were allowed under the MDT and VFA, viz: that EDCA articulates the principle of defensive preparation embodied in Article II of the MDT; and
seeks to enhance the defensive, strategic, and technological capabilities of both parties pursuant to the
Both the history and intent of the Mutual Defense Treaty and the VFA support the conclusion that objective of the treaty to strengthen those capabilities to prevent or resist a possible armed attack.
combat-related activities as opposed to combat itself such as the one subject of the instant petition, are Respondent also points out that EDCA simply implements Article I of the VFA, which already allows the
247
indeed authorized. entry of U.S. troops and personnel into the country. Respondent stresses this Court's recognition in Lim
v. Executive Secretary that U.S. troops and personnel are authorized to conduct activities that promote
Hence, even if EDCA was borne of military necessity, it cannot be said to have strayed from the intent the goal of maintaining and developing their defense capability.
of the VFA since EDCA's combat-related components are allowed under the treaty.
251
Petitioners contest the assertion that the provisions of EDCA merely implement the MDT. According
Moreover, both the VFA and EDCA are silent on what these activities actually are. Both the VFA and to them, the treaty does not specifically authorize the entry of U.S. troops in the country in order to
EDCA deal with the presence of U.S. forces within the Philippines, but make no mention of being maintain and develop the individual and collective capacities of both the Philippines and the U.S. to
platforms for activity beyond Philippine territory. While it may be that, as applied, military operations resist an armed attack. They emphasize that the treaty was concluded at a time when there was as yet
under either the VFA or EDCA would be carried out in the future the scope of judicial review does not no specific constitutional prohibition on the presence of foreign military forces in the country.
cover potential breaches of discretion but only actual occurrences or blatantly illegal provisions. Hence,
we cannot invalidate EDCA on the basis of the potentially abusive use of its provisions. Petitioners also challenge the argument that EDCA simply implements the VFA. They assert that the
agreement covers only short-term or temporary visits of U.S. troops "from time to time" for the specific
The fourth difference is that EDCA supposedly introduces a new concept not contemplated in the VFA purpose of combined military exercises with their Filipino counterparts. They stress that, in contrast,
248
or the MDT: Agreed Locations, Contractors, Pre-positioning, and Operational Control. U.S. troops are allowed under EDCA to perform activities beyond combined military exercises, such as
those enumerated in Articles 111(1) and IV(4) thereof. Furthermore, there is some degree of
As previously mentioned, these points shall be addressed fully and individually in the latter analysis of permanence in the presence of U.S. troops in the country, since the effectivity of EDCA is continuous
EDCA's provisions. However, it must already be clarified that the terms and details used by an until terminated. They proceed to argue that while troops have a "rotational" presence, this scheme in
implementing agreement need not be found in the mother treaty. They must be sourced from the fact fosters their permanent presence.
a. Admission of U.S. military and civilian personnel into Philippine territory is already allowed under the Article II of EDCA must then be read with Article III of the VFA, which provides for the entry
VFA accommodations to be accorded to U.S. military and civilian personnel:

We shall first deal with the recognition under EDCA of the presence in the country of three distinct 1. The Government of the Philippines shall facilitate the admission of United States personnel and
classes of individuals who will be conducting different types of activities within the Agreed Locations: (1) their departure from the Philippines in connection with activities covered by this agreement.
U.S. military personnel; (2) U.S. civilian personnel; and (3) U.S. contractors. The agreement refers to
them as follows: 2. United States military personnel shall be exempt from passport and visa regulations upon
enteringand departing the Philippines.
"United States personnel" means United States military and civilian personnel temporarily in the
territory of the Philippines in connection with activities approved by the Philippines, as those terms 3. The following documents only, which shall be required in respect of United States military personnel
252 who enter the Philippines; xx xx.
are defined in the VFA.

"United States forces" means the entity comprising United States personnel and all property, 4. United States civilian personnel shall be exempt from visa requirements but shall present, upon
equipment, and materiel of the United States Armed Forces present in the territory of the demand, valid passports upon entry and departure of the Philippines. (Emphases Supplied)
253
Philippines.
By virtue of Articles I and III of the VFA, the Philippines already allows U.S. military and civilian
"United States contractors" means companies and firms, and their employees, under contract or personnel to be "temporarily in the Philippines," so long as their presence is "in connection with
subcontract to or on behalf of the United States Department of Defense. United States contractors activities approved by the Philippine Government." The Philippines, through Article III, even guarantees
are not included as part of the definition of United States personnel in this Agreement, including within that it shall facilitate the admission of U.S. personnel into the country and grant exemptions from
254 passport and visa regulations. The VFA does not even limit their temporary presence to specific
the context of the VFA.
locations.
United States forces may contract for any materiel, supplies, equipment, and services (including
construction) to be furnished or undertaken in the territory of the Philippines without restriction as to Based on the above provisions, the admission and presence of U.S. military and civilian personnel
choice of contractor, supplier, or person who provides such materiel, supplies, equipment, in Philippine territory are already allowed under the VFA, the treaty supposedly being
or services. Such contracts shall be solicited, awarded, and administered in accordance with the laws implemented by EDCA. What EDCA has effectively done, in fact, is merely provide the mechanism to
255 identify the locations in which U.S. personnel may perform allowed activities pursuant to the VFA. As
and regulations of the United States. (Emphases Supplied)
the implementing agreement, it regulates and limits the presence of U.S. personnel in the country.
A thorough evaluation of how EDCA is phrased clarities that the agreement does not deal with the
256 b. EDCA does not provide the legal basis for admission of U.S. contractors into Philippine territory; their
entry into the country of U.S. personnel and contractors per se. While Articles I(l)(b) and
257 entry must be sourced from extraneous Philippine statutes and regulations for the admission of alien
II(4) speak of "the right to access and use" the Agreed Locations, their wordings indicate the
presumption that these groups have already been allowed entry into Philippine territory, for which, employees or business persons.
unlike the VFA, EDCA has no specific provision. Instead, Article II of the latter simply alludes to the VFA
in describing U.S. personnel, a term defined under Article I of the treaty as follows: Of the three aforementioned classes of individuals who will be conducting certain activities within the
Agreed Locations, we note that only U.S. contractors are not explicitly mentioned in the VFA. This does
As used in this Agreement, "United States personnel" means United States military and civilian not mean, though, that the recognition of their presence under EDCA is ipso facto an amendment of the
personnel temporarily in the Philippines in connection with activities approved by the Philippine treaty, and that there must be Senate concurrence before they are allowed to enter the country.
Government. Within this definition:
Nowhere in EDCA are U.S. contractors guaranteed immediate admission into the Philippines. Articles III
1. The term "military personnel" refers to military members of the United States Army, Navy, and IV, in fact, merely grant them the right of access to, and the authority to conduct certain activities
Marine Corps, Air Force, and Coast Guard. within the Agreed Locations. Since Article II(3) of EDCA specifically leaves out U.S. contractors from the
coverage of the VFA, they shall not be granted the same entry accommodations and privileges as those
2. The term "civilian personnel" refers to individuals who are neither nationals of nor ordinarily enjoyed by U.S. military and civilian personnel under the VFA.
resident in the Philippines and who are employed by the United States armed forces or who are
accompanying the United States armed forces, such as employees of the American Red Cross and Consequently, it is neither mandatory nor obligatory on the part of the Philippines to admit U.S.
258 259
the United Services Organization. contractors into the country. We emphasize that the admission of aliens into Philippine territory is "a
matter of pure permission and simple tolerance which creates no obligation on the part of the
260
government to permit them to stay." Unlike U.S. personnel who are accorded entry accommodations,
261
U.S. contractors are subject to Philippine immigration laws. The latter must comply with our visa and xxx
262
passport regulations and prove that they are not subject to exclusion under any provision of
263 Article I - Definitions
Philippine immigration laws. The President may also deny them entry pursuant to his absolute and
unqualified power to prohibit or prevent the admission of aliens whose presence in the country would be
inimical to public interest.
264 As used in this Agreement, "United States personnel" means United States military and civilian
personnel temporarily in the Philippines in connection with activities approved by the Philippine
In the same vein, the President may exercise the plenary power to expel or deport U.S. Government. Within this definition: xx x
265
contractors as may be necessitated by national security, public safety, public health, public morals,
266
and national interest. They may also be deported if they are found to be illegal or undesirable aliens Article II - Respect for Law
267 268
pursuant to the Philippine Immigration Act and the Data Privacy Act. In contrast, Article 111(5) of
It is the duty of United States personnel to respect the laws of the Republic of the
the VFA requires a request for removal from the Philippine government before a member of the U.S.
Philippines and to abstain from any activity inconsistent with the spirit of this agreement, and, in
personnel may be "dispos[ed] xx x outside of the Philippines."
particular, from any political activity in the Philippines. The Government of the United States shall take
c. Authorized activities of U.S. military and civilian personnel within Philippine territory are in furtherance all measures within its authority to ensure that this is done.
of the MDT and the VFA
Article VII - Importation and Exportation
We begin our analysis by quoting the relevant sections of the MDT and the VFA that pertain to the
1. United States Government equipment, materials, supplies, and other property imported into or
activities in which U.S. military and civilian personnel may engage:
acquired in the Philippines by or on behalf of the United States armed forces in connection with
MUTUAL DEFENSE TREATY activities to which this agreement applies, shall be free of all Philippine duties, taxes and other
similar charges. Title to such property shall remain with the United States, which may remove such
Article II property from the Philippines at any time, free from export duties, taxes, and other similar charges. x x
x.
In order more effectively to achieve the objective of this Treaty, the Parties separately and jointly
byself-help and mutual aid will maintain and develop their individual and collective capacity to Article VIII - Movement of Vessels and Aircraft
resist armed attack.
1. Aircraft operated by or for the United States armed forces may enter the Philippines upon
Article III approval of the Government of the Philippines in accordance with procedures stipulated
in implementing arrangements.
The Parties, through their Foreign Ministers or their deputies, will consult together from time to
time regarding the implementation of this Treaty and whenever in the opinion of either of them the 2. Vessels operated by or for the United States armed forces may enter the Philippines upon
territorial integrity, political independence or security of either of the Parties is threatened by external approval of the Government of the Philippines. The movement of vessels shall be in accordance
armed attack in the Pacific. with international custom and practice governing such vessels, and such agreed implementing
arrangements as necessary. x x x (Emphases Supplied)
VISITING FORCES AGREEMENT
Manifest in these provisions is the abundance of references to the creation of further "implementing
Preamble arrangements" including the identification of "activities [to be] approved by the Philippine Government."
To determine the parameters of these implementing arrangements and activities, we referred to the
xxx content, purpose, and framework of the MDT and the VFA.

Reaffirming their obligations under the Mutual Defense Treaty of August 30, 1951; By its very language, the MDT contemplates a situation in which both countries shall engage
in joint activities, so that they can maintain and develop their defense capabilities. The wording itself
Noting that from time to time elements of the United States armed forces may visit the Republic of the evidently invites a reasonable construction that the joint activities shall involve joint military trainings,
Philippines; 269 270
maneuvers, and exercises. Both the interpretation and the subsequent practice of the parties show
that the MDT independently allows joint military exercises in the country. Lim v. Executive
Considering that cooperation between the United States and the Republic of the Philippines promotes 271 272
Secretary and Nicolas v. Romulo recognized that Balikatan exercises, which are activities that
their common security interests;
seek to enhance and develop the strategic and technological capabilities of the parties to resist an
273
armed attack, "fall squarely under the provisions of the RP-US MDT." In Lim, the Court especially Under these auspices, the VFA gives legitimacy to the current Balikatan exercises. It is only logical to
noted that the Philippines and the U.S. continued to conduct joint military exercises even after the assume that "Balikatan 02-1," a "mutual anti-terrorism advising, assisting and training exercise,"
274
expiration of the MBA and even before the conclusion of the VFA. These activities presumably falls under the umbrella of sanctioned or allowable activities in the context of the
related to the Status of Forces Agreement, in which the parties agreed on the status to be accorded to agreement. Both the history and intent of the Mutual Defense Treaty and the VFA support the
U.S. military and civilian personnel while conducting activities in the Philippines in relation to the conclusion that combat-related activities - as opposed to combat itself- such as the one subject of the
275
MDT. instant petition, are indeed authorized. (Emphases Supplied)

Further, it can be logically inferred from Article V of the MDT that these joint activities may be conducted The joint report of the Senate committees on foreign relations and on national defense and security
on Philippine or on U.S. soil. The article expressly provides that the term armed attack includes "an further explains the wide range and variety of activities contemplated in the VFA, and how these
277
armed attack on the metropolitan territory of either of the Parties, or on the island territories under activities shall be identified:
its jurisdiction in the Pacific or on its armed forces, public vessels or aircraft in the Pacific." Surely,
in maintaining and developing our defense capabilities, an assessment or training will need to be These joint exercises envisioned in the VFA are not limited to combat-related activities; they have
performed, separately and jointly by self-help and mutual aid, in the territories of the contracting parties. a wide range and variety. They include exercises that will reinforce the AFP's ability to acquire new
It is reasonable to conclude that the assessment of defense capabilities would entail understanding the techniques of patrol and surveillance to protect the country's maritime resources; sea-search and
terrain, wind flow patterns, and other environmental factors unique to the Philippines. rescue operations to assist ships in distress; and disaster-relief operations to aid the civilian victims
of natural calamities, such as earthquakes, typhoons and tidal waves.
It would also be reasonable to conclude that a simulation of how to respond to attacks in vulnerable
areas would be part of the training of the parties to maintain and develop their capacity to resist an xxxx
actual armed attack and to test and validate the defense plan of the Philippines. It is likewise
reasonable to imagine that part of the training would involve an analysis of the effect of the weapons Joint activities under the VFA will include combat maneuvers; training in aircraft maintenance and
that may be used and how to be prepared for the eventuality. This Court recognizes that all of this may equipment repair; civic-action projects; and consultations and meetings of the Philippine-U.S. Mutual
require training in the area where an armed attack might be directed at the Philippine territory. Defense Board. It is at the level of the Mutual Defense Board-which is headed jointly by the Chief of
Staff of the AFP and the Commander in Chief of the U.S. Pacific Command-that the VFA exercises
The provisions of the MDT must then be read in conjunction with those of the VFA. are planned. Final approval of any activity involving U.S. forces is, however, invariably given by the
Philippine Government.
Article I of the VFA indicates that the presence of U.S. military and civilian personnel in the Philippines
is "in connection with activities approved by the Philippine Government." While the treaty does not xxxx
expressly enumerate or detail the nature of activities of U.S. troops in the country, its Preamble makes
explicit references to the reaffirmation of the obligations of both countries under the MDT. These Siazon clarified that it is not the VFA by itself that determines what activities will be
obligations include the strengthening of international and regional security in the Pacific area and the conductedbetween the armed forces of the U.S. and the Philippines. The VFA regulates and
promotion of common security interests. provides the legal framework for the presence, conduct and legal status of U.S. personnel while
they are in the country for visits, joint exercises and other related activities. (Emphases Supplied)
The Court has already settled in Lim v. Executive Secretary that the phrase "activities approved by the
Philippine Government" under Article I of the VFA was intended to be ambiguous in order to afford the What can be gleaned from the provisions of the VFA, the joint report of the Senate committees
276
parties flexibility to adjust the details of the purpose of the visit of U.S. personnel. In ruling that the on foreign relations and on national defense and security, and the ruling of this Court in Lim is
Terms of Reference for the Balikatan Exercises in 2002 fell within the context of the treaty, this Court that the "activities" referred to in the treaty are meant to be specified and
explained: identified infurther agreements. EDCA is one such agreement.

After studied reflection, it appeared farfetched that the ambiguity surrounding the meaning of the EDCA seeks to be an instrument that enumerates the Philippine-approved activities of U.S. personnel
word "activities" arose from accident. In our view, it was deliberately made that way to give both referred to in the VFA. EDCA allows U.S. military and civilian personnel to perform "activities approved
278
parties a certain leeway in negotiation. In this manner, visiting US forces may sojourn in by the Philippines, as those terms are defined in the VFA" and clarifies that these activities include
Philippine territory for purposes other than military. As conceived, the joint exercises may include those conducted within the Agreed Locations:
training on new techniques of patrol and surveillance to protect the nation's marine resources, sea
1. Security cooperation exercises; joint and combined training activities; humanitarian assistance and
search-and-rescue operations to assist vessels in distress, disaster relief operations, civic action 279
disaster relief activities; and such other activities as may be agreed upon by the Parties
projects such as the building of school houses, medical and humanitarian missions, and the like.
2. Training; transit; support and related activities; refueling of aircraft; bunkering of vessels; temporary The Exercise is a mutual counter-terrorism advising, assisting and training Exercise relative to
maintenance of vehicles, vessels, and aircraft; temporary accommodation of personnel; Philippine efforts against the ASG, and will be conducted on the Island of Basilan. Further advising,
communications; prepositioning of equipment, supplies, and materiel; deployment of forces and assisting and training exercises shall be conducted in Malagutay and the Zamboanga area. Related
280
materiel; and such other activities as the Parties may agree activities in Cebu will be for support of the Exercise.

3. Exercise of operational control over the Agreed Locations for construction activities and other types xx xx.
281
of activity, including alterations and improvements thereof
US exercise participants shall not engage in combat, without prejudice to their right of self-
4. Exercise of all rights and authorities within the Agreed Locations that are necessary for their defense.
operational control or defense, including the adoption of apfropriate measures to protect U.S. forces and
282 These terms of Reference are for purposes of this Exercise only and do not create additional legal
contractors
obligations between the US Government and the Republic of the Philippines.
283
5. Use of water, electricity, and other public utilities
II. EXERCISE LEVEL
6. Operation of their own telecommunication systems, including the utilization of such means and
services as are required to ensure the full ability to operate telecommunication systems, as well as the 1. TRAINING
284
use of the necessary radio spectrum allocated for this purpose
a. The Exercise shall involve the conduct of mutual military assisting, advising and trainingof RP
According to Article I of EDCA, one of the purposes of these activities is to maintain and develop, jointly and US Forces with the primary objective of enhancing the operational capabilities of both forces to
and by mutual aid, the individual and collective capacities of both countries to resist an armed attack. It combat terrorism.
further states that the activities are in furtherance of the MDT and within the context of the VFA.
b. At no time shall US Forces operate independently within RP territory.
We note that these planned activities are very similar to those under the Terms of
285
Reference mentioned in Lim. Both EDCA and the Terms of Reference authorize the U.S. to perform c. Flight plans of all aircraft involved in the exercise will comply with the local air traffic regulations.
the following: (a) participate in training exercises; (b) retain command over their forces; (c) establish
2. ADMINISTRATION & LOGISTICS
temporary structures in the country; (d) share in the use of their respective resources, equipment and
other assets; and (e) exercise their right to self-defense. We quote the relevant portion of the Terms and xxxx
286
Conditions as follows:
a. RP and US participating forces may share, in accordance with their respective laws and regulations,
I. POLICY LEVEL in the use of their resources, equipment and other assets. They will use their respective logistics
channels. x x x. (Emphases Supplied)
xxxx
After a thorough examination of the content, purpose, and framework of the MDT and the VFA, we find
No permanent US basing and support facilities shall be established. Temporary structures such as
that EDCA has remained within the parameters set in these two treaties. Just like the Terms of
those for troop billeting, classroom instruction and messing may be set up for use by RP and US
Reference mentioned in Lim, mere adjustments in detail to implement the MDT and the VFA can be in
Forces during the Exercise.
the form of executive agreements.
The Exercise shall be implemented jointly by RP and US Exercise Co-Directors under the authority of 287
Petitioners assert that the duration of the activities mentioned in EDCA is no longer consistent with
the Chief of Staff, AFP. In no instance will US Forces operate independently during field training
the temporary nature of the visits as contemplated in the VFA. They point out that Article XII(4) of EDCA
exercises (FTX). AFP and US Unit Commanders will retain command over their respective forces
has an initial term of 10 years, a term automatically renewed unless the Philippines or the U.S.
under the overall authority of the Exercise Co-Directors. RP and US participants shall comply with
terminates the agreement. According to petitioners, such length of time already has a badge of
operational instructions of the AFP during the FTX.
permanency.
The exercise shall be conducted and completed within a period of not more than six months, with the
In connection with this, Justice Teresita J. Leonardo-De Castro likewise argues in her Concurring and
projected participation of 660 US personnel and 3,800 RP Forces. The Chief of Staff, AFP shall direct
Dissenting Opinion that the VFA contemplated mere temporary visits from U.S. forces, whereas EDCA
the Exercise Co-Directors to wind up and terminate the Exercise and other activities within the six 288
allows an unlimited period for U.S. forces to stay in the Philippines.
month Exercise period.
However, the provisions of EDCA directly contradict this argument by limiting itself to 10 years of 3. This Agreement shall have an initial term of 25 years and thereafter shall continue in force, but
effectivity. Although this term is automatically renewed, the process for terminating the agreement is may be terminated by either Party at any time upon one year's written notice to the other Party through
unilateral and the right to do so automatically accrues at the end of the 10 year period. Clearly, this diplomatic channels. (emphasis supplied)
method does not create a permanent obligation.
The phrasing in EDCA is similar to that in the U.S.-Australia treaty but with a term less than half of that
Drawing on the reasoning in Lim, we also believe that it could not have been by chance that the VFA is provided in the latter agreement. This means that EDCA merely follows the practice of other states in
does not include a maximum time limit with respect to the presence of U.S. personnel in the country. not specifying a non-extendible maximum term. This practice, however, does not automatically grant a
We construe this lack of specificity as a deliberate effort on the part of the Philippine and the U.S. badge of permanency to its terms. Article XII(4) of EDCA provides very clearly, in fact, that its effectivity
governments to leave out this aspect and reserve it for the "adjustment in detail" stage of the is for an initial term of 10 years, which is far shorter than the terms of effectivity between the U.S. and
implementation of the treaty. We interpret the subsequent, unconditional concurrence of the Senate in other states. It is simply illogical to conclude that the initial, extendible term of 10 years somehow gives
the entire text of the VFA as an implicit grant to the President of a margin of appreciation in determining EDCA provisions a permanent character.
the duration of the "temporary" presence of U.S. personnel in the country.
The reasoning behind this interpretation is rooted in the constitutional role of the President who, as
Justice Brion's dissent argues that the presence of U.S. forces under EDCA is "more permanent" in Commander-in-Chief of our armed forces, is the principal strategist of the nation and, as such, duty-
289 291
nature. However, this argument has not taken root by virtue of a simple glance at its provisions on the bound to defend our national sovereignty and territorial integrity; who, as chief architect of our foreign
effectivity period. EDCA does not grant permanent bases, but rather temporary rotational access to relations, is the head policymaker tasked to assess, ensure, and protect our national security and
292
facilities for efficiency. As Professor Aileen S.P. Baviera notes: interests; who holds the most comprehensive and most confidential information about foreign
293
countries that may affect how we conduct our external affairs; and who has unrestricted access to
The new EDCA would grant American troops, ships and planes rotational access to facilities of the 294
highly classified military intelligence data that may threaten the life of the nation. Thus, if after a
Armed Forces of the Philippines – but not permanent bases which are prohibited under the Philippine geopolitical prognosis of situations affecting the country, a belief is engendered that a much longer
Constitution - with the result of reducing response time should an external threat from a common period of military training is needed, the President must be given ample discretion to adopt necessary
290
adversary crystallize. measures including the flexibility to set an extended timetable.

EDCA is far from being permanent in nature compared to the practice of states as shown in other Due to the sensitivity and often strict confidentiality of these concerns, we acknowledge that the
defense cooperation agreements. For example, Article XIV(l) of the U.S.-Romania defense agreement President may not always be able to candidly and openly discuss the complete situation being faced by
provides the following: the nation. The Chief Executive's hands must not be unduly tied, especially if the situation calls for
crafting programs and setting timelines for approved activities. These activities may be necessary for
This Agreement is concluded for an indefinite period and shall enter into force in accordance with the maintaining and developing our capacity to resist an armed attack, ensuring our national sovereignty
internal laws of each Party x x x. (emphasis supplied) and territorial integrity, and securing our national interests. If the Senate decides that the President is in
the best position to define in operational terms the meaning of temporary in relation to the visits,
Likewise, Article 36(2) of the US-Poland Status of Forces Agreement reads:
considered individually or in their totality, the Court must respect that policy decision. If the Senate feels
This Agreement has been concluded for an indefinite period of time. It may be terminated by written that there is no need to set a time limit to these visits, neither should we.
notification by either Party and in that event it terminates 2 years after the receipt of the notification.
Evidently, the fact that the VFA does not provide specificity in regard to the extent of the "temporary"
(emphasis supplied)
nature of the visits of U.S. personnel does not suggest that the duration to which the President may
Section VIII of US.-Denmark Mutual Support Agreement similarly provides: agree is unlimited. Instead, the boundaries of the meaning of the term temporary in Article I of the treaty
295
must be measured depending on the purpose of each visit or activity. That purpose must be analyzed
8.1 This Agreement, which consists of a Preamble, SECTIONs I-VIII, and Annexes A and B, shall on a case-by-case basis depending on the factual circumstances surrounding the conclusion of the
become effective on the date of the last signature affixed below and shall remain in force until implementing agreement. While the validity of the President's actions will be judged under less stringent
terminated by the Parties, provided that it may be terminated by either Party upon 180 days written standards, the power of this Court to determine whether there was grave abuse of discretion remains
notice of its intention to do so to the other Party. (emphasis supplied) unimpaired.

On the other hand, Article XXI(3) of the US.-Australia Force Posture Agreement provides a longer initial d. Authorized activities performed by US. contractors within Philippine territory - who were legitimately
term: permitted to enter the country independent of EDCA - are subject to relevant Philippine statutes and
regulations and must be consistent with the MDT and the VFA
296
Petitioners also raise concerns about the U.S. government's purported practice of hiring private the U.S. position that these contractors are bound by the local laws of their host state. This stance was
307
security contractors in other countries. They claim that these contractors - one of which has already echoed by other U.S. Navy representatives.
been operating in Mindanao since 2004 - have been implicated in incidents or scandals in other parts of
the globe involving rendition, torture and other human rights violations. They also assert that these This incident simply shows that the Senate was well aware of the presence of U.S. contractors for the
contractors employ paramilitary forces in other countries where they are operating. purpose of fulfilling the terms of the VFA. That they are bound by Philippine law is clear to all, even to
the U.S.
Under Articles III and IV of EDCA, U.S. contractors are authorized to perform only the following
activities: As applied to EDCA, even when U.S. contractors are granted access to the Agreed Locations, all their
activities must be consistent with Philippine laws and regulations and pursuant to the MDT and the VFA.
1. Training; transit; support and related activities; refueling of aircraft; bunkering of vessels; temporary
maintenance of vehicles, vessels, and aircraft; temporary accommodation of personnel; While we recognize the concerns of petitioners, they do not give the Court enough justification to strike
communications; prepositioning of equipment, supplies, and materiel; deployment of forces and down EDCA. In Lim v. Executive Secretary, we have already explained that we cannot take judicial
297 notice of claims aired in news reports, "not because of any issue as to their truth, accuracy, or
materiel; and such other activities as the Parties may agree
impartiality, but for the simple reason that facts must be established in accordance with the rules of
308
2. Prepositioning and storage of defense equipment, supplies, and materiel, including delivery, evidence." What is more, we cannot move one step ahead and speculate that the alleged illegal
298
management, inspection, use, maintenance, and removal of such equipment, supplies and materiel activities of these contractors in other countries would take place in the Philippines with certainty. As
can be seen from the above discussion, making sure that U.S. contractors comply with Philippine laws
3. Carrying out of matters in accordance with, and to the extent permissible under, U.S. laws, is a function of law enforcement. EDCA does not stand in the way of law enforcement.
299
regulations, and policies
Nevertheless, we emphasize that U.S. contractors are explicitly excluded from the coverage of the VFA.
EDCA requires that all activities within Philippine territory be in accordance with Philippine law. This As visiting aliens, their entry, presence, and activities are subject to all laws and treaties applicable
means that certain privileges denied to aliens are likewise denied to foreign military contractors. within the Philippine territory. They may be refused entry or expelled from the country if they engage in
300 301
Relevantly, providing security and carrying, owning, and possessing firearms are illegal for foreign illegal or undesirable activities. There is nothing that prevents them from being detained in the country
civilians. 309 310
or being subject to the jurisdiction of our courts. Our penal laws, labor laws, and immigrations
311
laws apply to them and therefore limit their activities here. Until and unless there is another law or
The laws in place already address issues regarding the regulation of contractors. In the 2015 Foreign treaty that specifically deals with their entry and activities, their presence in the country is subject to
302
Investment Negative list, the Executive Department has already identified corporations that have unqualified Philippine jurisdiction.
equity restrictions in Philippine jurisdiction. Of note is No. 5 on the list - private security agencies that
303
cannot have any foreign equity by virtue of Section 4 of Republic Act No. 5487; and No. 15, which EDCA does not allow the presence of U.S.-owned or -controlled military facilities and bases in
regulates contracts for the construction of defense-related structures based on Commonwealth Act No. the Philippines
541.
Petitioners Saguisag et al. claim that EDCA permits the establishment of U.S. military bases through the
Hence, any other entity brought into the Philippines by virtue of EDCA must subscribe to corporate and 312
"euphemistically" termed "Agreed Locations. " Alluding to the definition of this term in Article II(4) of
civil requirements imposed by the law, depending on the entity's corporate structure and the nature of EDCA, they point out that these locations are actually military bases, as the definition refers to facilities
its business. and areas to which U.S. military forces have access for a variety of purposes. Petitioners claim that
there are several badges of exclusivity in the use of the Agreed Locations by U.S. forces. First, Article
That Philippine laws extraneous to EDCA shall govern the regulation of the activities of U.S. contractors V(2) of EDCA alludes to a "return" of these areas once they are no longer needed by U.S. forces,
has been clear even to some of the present members of the Senate. indicating that there would be some transfer of use. Second, Article IV(4) ofEDCA talks about American
forces' unimpeded access to the Agreed Locations for all matters relating to the prepositioning and
For instance, in 2012, a U.S. Navy contractor, the Glenn Marine, was accused of spilling fuel in the
304 storage of U.S. military equipment, supplies, and materiel. Third, Article VII of EDCA authorizes U.S.
waters off Manila Bay. The Senate Committee on Foreign Relations and the Senate Committee on
forces to use public utilities and to operate their own telecommunications system.
Environment and Natural Resources chairperson claimed environmental and procedural violations by
305
the contractor. The U.S. Navy investigated the contractor and promised stricter guidelines to be a. Preliminary point on badges of exclusivity
306
imposed upon its contractors. The statement attributed to Commander Ron Steiner of the public
affairs office of the U.S. Navy's 7th Fleet - that U.S. Navy contractors are bound by Philippine laws - is
of particular relevance. The statement acknowledges not just the presence of the contractors, but also
319
As a preliminary observation, petitioners have cherry-picked provisions of EDCA by presenting so- Petitioners also point out that EDCA is strongly reminiscent of and in fact bears a one-to-one
called "badges of exclusivity," despite the presence of contrary provisions within the text of the correspondence with the provisions of the 1947 MBA. They assert that both agreements (a) allow
agreement itself. similar activities within the area; (b) provide for the same "species of ownership" over facilities; and (c)
320
grant operational control over the entire area. Finally, they argue that EDCA is in fact an
First, they clarify the word "return" in Article V(2) of EDCA. However, the use of the word "return" is implementation of the new defense policy of the U.S. According to them, this policy was not what was
within the context of a lengthy provision. The provision as a whole reads as follows: originally intended either by the MDT or by the VFA.

The United States shall return to the Philippines any Agreed Locations, or any portion thereof, including On these points, the Court is not persuaded.
non-relocatable structures and assemblies constructed, modified, or improved by the United States,
321
once no longer required by United States forces for activities under this Agreement. The Parties or the The similar activities cited by petitioners simply show that under the MBA, the U.S. had the right to
Designated Authorities shall consult regarding the terms of return of any Agreed Locations, including construct, operate, maintain, utilize, occupy, garrison, and control the bases. The so-called parallel
possible compensation for improvements or construction. provisions of EDCA allow only operational control over the Agreed Locations specifically for
construction activities. They do not allow the overarching power to operate, maintain, utilize, occupy,
The context of use is "required by United States forces for activities under this Agreement." Therefore, garrison, and control a base with full discretion. EDCA in fact limits the rights of the U.S. in respect of
the return of an Agreed Location would be within the parameters of an activity that the Mutual Defense every activity, including construction, by giving the MDB and the SEB the power to determine the details
Board (MDB) and the Security Engagement Board (SEB) would authorize. Thus, possession by the of all activities such as, but not limited to, operation, maintenance, utility, occupancy, garrisoning, and
U.S. prior to its return of the Agreed Location would be based on the authority given to it by a joint body 322
control.
co-chaired by the "AFP Chief of Staff and Commander, U.S. PACOM with representatives from the
Philippines' Department of National Defense and Department of Foreign Affairs sitting as The "species of ownership" on the other hand, is distinguished by the nature of the property. For
313
members." The terms shall be negotiated by both the Philippines and the U.S., or through their immovable property constructed or developed by the U.S., EDCA expresses that ownership will
323
Designated Authorities. This provision, seen as a whole, contradicts petitioners' interpretation of the automatically be vested to the Philippines. On the other hand, for movable properties brought into the
return as a "badge of exclusivity." In fact, it shows the cooperation and partnership aspect of EDCA in Philippines by the U.S., EDCA provides that ownership is retained by the latter. In contrast, the MBA
full bloom. dictates that the U.S. retains ownership over immovable and movable properties.

Second, the term "unimpeded access" must likewise be viewed from a contextual perspective. Article To our mind, both EDCA and the MBA simply incorporate what is already the law of the land in the
IV(4) states that U.S. forces and U.S. contractors shall have "unimpeded access to Agreed Locations Philippines. The Civil Code's provisions on ownership, as applied, grant the owner of a movable
324
for all matters relating to the prepositioning and storage of defense equipment, supplies, and materiel, property full rights over that property, even if located in another person's property.
including delivery, management, inspection, use, maintenance, and removal of such equipment,
supplies and materiel." The parallelism, however, ends when the situation involves facilities that can be considered immovable.
325
Under the MBA, the U.S. retains ownership if it paid for the facility. Under EDCA, an immovable is
326
At the beginning of Article IV, EDCA states that the Philippines gives the U.S. the authority to bring in owned by the Philippines, even if built completely on the back of U.S. funding. This is consistent with
327
these equipment, supplies, and materiel through the MDB and SEB security mechanism. These items the constitutional prohibition on foreign land ownership.
314 315
are owned by the U.S., are exclusively for the use of the U.S. and, after going through the joint
316 Despite the apparent similarity, the ownership of property is but a part of a larger whole that must be
consent mechanisms of the MDB and the SEB, are within the control of the U.S. More importantly,
before these items are considered prepositioned, they must have gone through the process of prior considered before the constitutional restriction is violated. Thus, petitioners' points on operational
317 control will be given more attention in the discussion below. The arguments on policy are, however,
authorization by the MDB and the SEB and given proper notification to the AFP.
outside the scope of judicial review and will not be discussed
Therefore, this "unimpeded access" to the Agreed Locations is a necessary adjunct to the ownership,
use, and control of the U.S. over its own equipment, supplies, and materiel and must have first been Moreover, a direct comparison of the MBA and EDCA will result in several important distinctions that
allowed by the joint mechanisms in play between the two states since the time of the MDT and the VFA. would allay suspicion that EDCA is but a disguised version of the MBA.
It is not the use of the Agreed Locations that is exclusive per se; it is mere access to items in order to
exercise the rights of ownership granted by virtue of the Philippine Civil Code.
318 b. There are substantial matters that the US. cannot do under EDCA, but which it was authorized to do
under the 1947 MBA
As for the view that EDCA authorizes U.S. forces to use public utilities and to operate their own
telecommunications system, it will be met and answered in part D, infra. The Philippine experience with U.S. military bases under the 1947 MBA is simply not possible under
EDCA for a number of important reasons.
First, in the 1947 MBA, the U.S. retained all rights of jurisdiction in and over Philippine territory
with the Republic of the Philippines may deem improved by United States forces, remain the
occupied by American bases. In contrast, the U.S. under EDCA does not enjoy any such right over any
necessary to retain for the mutual protection of the property of the Philippines.Permanent buildings
part of the Philippines in which its forces or equipment may be found. Below is a comparative table
Republic of the Philippines and of the United constructed by United States forces become the
between the old treaty and EDCA:
States of America. x x x. property of the Philippines, once constructed, but
shall be used by United States forces until no
1947 MBA/ 1946 Treaty of General Relations EDCA longer required by United States forces.

1947 MBA, Art. I(1): EDCA, preamble:

The Government of the Republic of Affirming that the Parties share an understanding Second, in the bases agreement, the U.S. and the Philippines were visibly not on equal footing when it
the Philippines (hereinafter referred to as the for the United States not to establish a came to deciding whether to expand or to increase the number of bases, as the Philippines may be
Philippines) grants to the Government of permanent military presence or base in the compelled to negotiate with the U.S. the moment the latter requested an expansion of the existing
the United States of America (hereinafter referred territory of the Philippines; bases or to acquire additional bases. In EDCA, U.S. access is purely at the invitation of the Philippines.
to as the United States) the right to retain the
use of the bases in the Philippines listed in xxxx 1947 MBA/ 1946 Treaty of General Relations EDCA
Annex A attached hereto.
Recognizing that all United States access to and
1947 MBA, Art. XVII(2): use of facilities and areas will be at the invitation 1947 MBA, Art.I(3): EDCA, preamble:
of the Philippines and with full respect for the
All buildings and structures which are erected Philippine Constitution and Philippine laws; The Philippines agree to enter into Recognizing that all United States access to and
by the United States in the bases shall be negotiations with the United States at the use of facilities and areas will be at the invitation of
the property of the United States and may be xxxx latter's request, to permit the United States to the Philippines and with full respect for the
removed by it before the expiration of this expand such bases, to exchange such bases Philippine Constitution and Philippine laws;
Agreement or the earlier relinquishment of the EDCA, Art. II(4): for other bases, to acquire additional bases,
base on which the structures are situated. There or relinquish rights to bases, as any of such xxxx
"Agreed Locations" means facilities and
shall be no obligation on the part of the exigencies may be required by military
areas that are provided by the Government of EDCA. Art. II(4):
Philippines or of the United States to rebuild or necessity.
the Philippines through the AFP and that United
repair any destruction or damage inflicted from
States forces, United States contractors, and 1946 Treaty of Gen. Relations, Art. I: "Agreed Locations" means facilities and
any cause whatsoever on any of the said
others as mutually agreed, shall have the right to areas that are provided by the Government of the
buildings or structures owned or used by the
access and use pursuant to this Agreement. Such The United States of America agrees to Philippines through the AFP and that United States
United States in the bases. x x x x.
Agreed Locations may be listed in an annex to be withdraw and surrender, and does hereby forces, United States contractors, and others as
appended to this Agreement, and may be further withdraw and surrender, all rights of mutually agreed, shall have the right to access and
1946 Treaty of Gen. Relations, Art. I:
described in implementing arrangements. possession, supervision, jurisdiction, use pursuant to this Agreement. Such Agreed
The United States of America agrees to withdraw control or sovereignty existing and exercised Locations may be listed in an annex to be appended
and surrender, and does hereby withdraw and EDCA, Art. V: by the United States of America in and over to this Agreement, and may be further described in
surrender, all rights of possession, the territory and the people of the Philippine implementing arrangements.
1. The Philippines shall retain ownership of and Islands, except the use of such
supervision, jurisdiction, control or
title to Agreed Locations. bases, necessary appurtenances to such
sovereignty existing and exercised by the United
States of America in and over the territory and bases, and the rights incident thereto, as the
xxxx
the people of the Philippine Islands, except the United States of America, by agreement with
use of such bases, necessary appurtenances 4. All buildings, non-relocatable structures, and the Republic of the Philippines may deem
to such bases, and the rights incident thereto, assemblies affixed to the land in the Agreed necessary to retain for the mutual protection of
as the United States of America, by agreement Locations, including ones altered or the Republic of the Philippines and of the
United States of America. x x x. or appropriate for the control
thereof and all the rights, power and
Third, in EDCA, the Philippines is- guaranteed access over the entire area of the Agreed Locations. On authority within the limits of territorial
the other hand, given that the U.S. had complete control over its military bases under the 1947 MBA, waters and air space adjacent to, or
the treaty did not provide for any express recognition of the right of access of Philippine authorities. in the vicinity of, the bases which
Without that provision and in light of the retention of U.S. sovereignty over the old military bases, the are necessary to provide access to
U.S. could effectively prevent Philippine authorities from entering those bases. them, or appropriate for their control.

1947 MBA EDCA Fifth, the U.S. under the bases agreement was given the authority to use Philippine territory for
additional staging areas, bombing and gunnery ranges. No such right is given under EDCA, as seen
below:
No EDCA, Art. III(5):
equivalent
1947 MBA EDCA
provision. The Philippine Designated Authority and its authorized representative shall have
access to the entire area of the Agreed Locations. Such access shall be provided
promptly consistent with operational safety and security requirements in accordance 1947 MBA, Art. VI: EDCA, Art. III(1):
with agreed procedures developed by the Parties.
The United States shall, subject to With consideration of the views of the Parties,
Fourth, in the bases agreement, the U.S. retained the right, power, and authority over the previous agreement with the the Philippines hereby authorizes and agrees that United
establishment, use, operation, defense, and control of military bases, including the limits of territorial Philippines, have the right to use land States forces, United States contractors, and vehicles,
waters and air space adjacent to or in the vicinity of those bases. The only standard used in determining and coastal sea areas of appropriate vessels, and aircraft operated by or for United States forces
the extent of its control was military necessity. On the other hand, there is no such grant of power or size and location for periodic may conduct the following activities with respect to Agreed
authority under EDCA. It merely allows the U.S. to exercise operational control over the construction of maneuvers, for additional staging Locations: training; transit; support and related activities;
Philippine-owned structures and facilities: areas, bombing and gunnery refueling of aircraft; bunkering of vessels; temporary
ranges, and for such intermediate maintenance of vehicles, vessels, and aircraft; temporary
airfields as may be required for safe accommodation of personnel; communications; prepositioning
1947 MBA EDCA
and efficient air operations. Operations of equipment, supplies, and materiel; deploying forces and
in such areas shall be carried on with materiel; and such other activities as the Parties may agree.
1947 MBA, Art.I(2): EDCA, Art. III(4): due regard and safeguards for the
public safety.
The Philippines agrees to permit the The Philippines hereby grants to the United
United States, upon notice to the States, through bilateral security mechanisms, such as 1947 MBA, Art.I(2):
Philippines, to use such of those the MDB and SEB, operational control of Agreed
bases listed in Annex B as the United Locations for construction The Philippines agrees to permit the
States determines to be required by activities and authority to undertake such activities on, United States, upon notice to the
military necessity. and make alterations and improvements to, Agreed Philippines, to use such of those
Locations. United States forces shall consult on issues bases listed in Annex B as the United
1947 MBA, Art. III(1): regarding such construction, alterations, and States determines to be required by
improvements based on the Parties' shared intent that the military necessity.
It is mutually agreed that the United technical requirements and construction standards of any
Statesshall have the rights, power and such projects undertaken by or on behalf of United States Sixth, under the MBA, the U.S. was given the right, power, and authority to control and prohibit the
authority within the bases which forces should be consistent with the requirements and movement and operation of all types of vehicles within the vicinity of the bases. The U.S. does not have
are necessary for the establishment, standards of both Parties. any right, power, or authority to do so under EDCA.
use, operation and defense thereof
1947 MBA EDCA bridges, viaducts, canals, lakes, rivers and streams in (including roads, ports, and airfields),
the Philippines under conditions no less favorable including those owned or controlled by local
than those that may be applicablefrom time to time to governments, and to other land and facilities
1947 MBA, Art. 111(2)(c) No
the military forces of the Philippines. (including roads, ports, and airfields).
equivalent
Such rights, power and authority shall include, inter alia, the right, power and provision.
authority: x x x x to control (including the right to prohibit) in so far as may be required Ninth, under EDCA, the U.S. no longer has the right, power, and authority to construct, install, maintain,
for the efficient operation and safety of the bases, and within the limits of military and employ any type of facility, weapon, substance, device, vessel or vehicle, or system unlike in the
necessity, anchorages, moorings, landings, takeoffs, movements and operation of old treaty. EDCA merely grants the U.S., through bilateral security mechanisms, the authority to
ships and water-borne craft, aircraft and other vehicles on water, in the air or on undertake construction, alteration, or improvements on the Philippine-owned Agreed Locations.
land comprising
1947 MBA EDCA
Seventh, under EDCA, the U.S. is merely given temporary access to land and facilities (including
roads, ports, and airfields). On the other hand, the old treaty gave the U.S. the right to improve and
1947 MBA, Art. III(2)(e): EDCA, Art. III(4):
deepen the harbors, channels, entrances, and anchorages; and to construct or maintain necessary
roads and bridges that would afford it access to its military bases. Such rights, power and authority shall The Philippines hereby grants to the United
include, inter alia, the right, power and States, through bilateral security mechanisms,
1947 MBA EDCA authority: x x x x to construct, install, maintain, such as the MDB and SEB, operational control of
and employ on any base any type of facilities, Agreed Locations for construction activities
weapons, substance, device, vessel or and authority to undertake such activities on,
1947 MBA, Art. III(2)(b): EDCA, Art. III(2):
vehicle on or under the ground, in the air or on or and make alterations and improvements to,
under the water that may be requisite or Agreed Locations. United States forces shall
Such rights, power and authority shall When requested, the Designated Authority of the
appropriate, including meteorological systems, consult on issues regarding such construction,
include, inter alia, the right, power and Philippines shall assist in facilitating transit or
aerial and water navigation lights, radio and radar alterations, and improvements based on the
authority: x x x x to improve and deepen temporary access by United States forces to public
apparatus and electronic devices, of any desired Parties' shared intent that the technical
the harbors, channels, entrances and land and facilities (including roads, ports, and
power, type of emission and frequency. requirements and construction standards of any
anchorages, and to construct or maintain airfields), including those owned or controlled by local
such projects undertaken by or on behalf of United
necessary roadsand bridges affording access governments, and to other land and facilities (including
States forces should be consistent with the
to the bases. roads, ports, and airfields).
requirements and standards of both Parties.

Eighth, in the 1947 MBA, the U.S. was granted the automatic right to use any and all public utilities,
Tenth, EDCA does not allow the U.S. to acquire, by condemnation or expropriation proceedings, real
services and facilities, airfields, ports, harbors, roads, highways, railroads, bridges, viaducts, canals,
property belonging to any private person. The old military bases agreement gave this right to the U.S.
lakes, rivers, and streams in the Philippines in the same manner that Philippine military forces enjoyed
as seen below:
that right. No such arrangement appears in EDCA. In fact, it merely extends to U.S. forces temporary
access to public land and facilities when requested:
1947 MBA EDCA
1947 MBA EDCA
1947 MBA, Art. XXII(l): No
equivalent
1947 MBA, Art. VII: EDCA, Art. III(2):
Whenever it is necessary to acquire by provision.
It is mutually agreed that the United States may When requested, the Designated Authority
condemnation or expropriation proceedings real property belonging to any private
employ and use for United States military forces any of the Philippines shall assist in facilitating
persons, associations or corporations located in bases named in Annex A and Annex B
and all public utilities, other services and facilities, transit or temporary access by United
in order to carry out the purposes of this Agreement, the Philippines will institute and
airfields, ports, harbors, roads, highways, railroads, States forces to public land and facilities
prosecute such condemnation or expropriation proceedings in accordance with the laws 1947 MBA, Art. XIII(l)(a): No
of the Philippines. The United States agrees to reimburse the Philippines for all the equivalent
reasonable expenses, damages and costs therebv incurred, including the value of the The Philippines consents that the United provision.
property as determined by the Court. In addition, subject to the mutual agreement of the
two Governments, the United States will reimburse the Philippines for the reasonable States shall have the right to exercise jurisdiction over the following offenses:
costs of transportation and removal of any occupants displaced or ejected by reason of (a) Any offense committed by any person within any base except where the offender
the condemnation or expropriation. and offended parties are both Philippine citizens (not members of the armed forces of
the United States on active duty) or the offense is against the security of the Philippines.

Eleventh, EDCA does not allow the U.S. to unilaterally bring into the country non-Philippine nationals
who are under its employ, together with their families, in connection with the construction, maintenance, Thirteenth, EDCA does not allow the U.S. to operate military post exchange (PX) facilities, which is
or operation of the bases. EDCA strictly adheres to the limits under the VFA. free of customs duties and taxes, unlike what the expired MBA expressly allowed. Parenthetically, the
PX store has become the cultural icon of U.S. military presence in the country.

1947 MBA EDCA


1947 MBA EDCA

1947 MBA, Art. XI(l): EDCA, Art. II:


1947 MBA, Art. XVIII(l): No
It is mutually agreed that the United States shall 1. "United States personnel" means United equivalent
have the right to bring into the States military and civilian It is mutually agreed that the United States provision.
Philippines members of the United States military personneltemporarily in the territory of the
shall have the right to establish on bases, free of all licenses; fees; sales, excise or
forces and the United States nationals employed Philippines in connection with activities approved
other taxes, or imposts; Government agencies, including concessions, such
by or under a contract with the United States by the Philippines, as those terms are defined
as sales commissaries and post exchanges; messes and social clubs, for the
together with their families, and technical in the VFA.
exclusive use of the United States military forces and authorized civilian personnel
personnel of other nationalities (not being
x xx x and their families. The merchandise or services sold or dispensed by such agencies
persons excluded by the laws of the Philippines) in
shall be free of all taxes, duties and inspection by the Philippine
connection with the construction, maintenance, or
3. "United States contractors" means authorities. Administrative measures shall be taken by the appropriate authorities of the
operation of the bases. The United States shall
companies and firms, and their employees, United States to prevent the resale of goods which are sold under the provisions of this
make suitable arrangements so that such persons
under contract or subcontract to or on behalf of Article to persons not entitled to buy goods at such agencies and, generally, to prevent
may be readily identified and their status
the United States Department of Defense. United abuse of the privileges granted under this Article. There shall be cooperation between
established when necessary by the Philippine
States contractors are not includedas part of such authorities and the Philippines to this end.
authorities. Such persons, other than members of
the United States armed forces in uniform, shall the definition of United States personnel in
present their travel documents to the appropriate this Agreement, including within the context of In sum, EDCA is a far cry from a basing agreement as was understood by the people at the time that
Philippine authorities for visas, it being understood the VFA. the 1987 Constitution was adopted.
that no objection will be made to their travel to
the Philippines as non-immigrants. Nevertheless, a comprehensive review of what the Constitution means by "foreign military bases" and
"facilities" is required before EDCA can be deemed to have passed judicial scrutiny.

Twelfth, EDCA does not allow the U.S. to exercise jurisdiction over any offense committed by any c. The meaning of military facilities and bases
person within the Agreed Locations, unlike in the former military bases:
An appreciation of what a military base is, as understood by the Filipino people in 1987, would be vital
1947 MBA EDCA in determining whether EDCA breached the constitutional restriction.
Prior to the drafting of the 1987 Constitution, the last definition of "military base" was provided under Notably, nearly 30 years have passed since then, and the ever-evolving world of military technology and
328
Presidential Decree No. (PD) 1227. Unlawful entry into a military base is punishable under the decree geopolitics has surpassed the understanding of the Philippine people in 1986. The last direct military
as supported by Article 281 of the Revised Penal Code, which itself prohibits the act of trespass. action of the U.S. in the region was the use of Subic base as the staging ground for Desert Shield and
331
Desert Storm during the Gulf War. In 1991, the Philippine Senate rejected the successor treaty of the
Section 2 of the law defines the term in this manner: "'[M]ilitary base' as used in this decree means any 1947 MBA that would have allowed the continuation of U.S. bases in the Philippines.
military, air, naval, or coast guard reservation, base, fort, camp, arsenal, yard, station, or installation in
the Philippines." Henceforth, any proposed entry of U.S. forces into the Philippines had to evolve likewise, taking into
consideration the subsisting agreements between both parties, the rejection of the 1991 proposal, and a
Commissioner Tadeo, in presenting his objections to U.S. presence in the Philippines before the 1986 concrete understanding of what was constitutionally restricted. This trend birthed the VFA which, as
Constitutional Commission, listed the areas that he considered as military bases: discussed, has already been upheld by this Court.

1,000 hectares Camp O'Donnel The latest agreement is EDCA, which proposes a novel concept termed "Agreed Locations."

20,000 hectares Crow Valley Weapon's Range By definition, Agreed Locations are

55,000 hectares Clark Air Base facilities and areas that are provided by the Government of the Philippines through the AFP and that
United States forces, United States contractors, and others as mutually agreed, shall have the right to
150 hectares Wallace Air Station access and use pursuant to this Agreement. Such Agreed Locations may be listed in an annex to be
332
appended to this Agreement, and may be further described in implementing arrangements.
400 hectares John Hay Air Station
Preliminarily, respondent already claims that the proviso that the Philippines shall retain ownership of
15,000 hectares Subic Naval Base
and title to the Agreed Locations means that EDCA is "consistent with Article II of the VFA which
333
1,000 hectares San Miguel Naval Communication recognizes Philippine sovereignty and jurisdiction over locations within Philippine territory.

750 hectares Radio Transmitter in Capas, Tarlac By this interpretation, respondent acknowledges that the contention of petitioners springs from an
understanding that the Agreed Locations merely circumvent the constitutional restrictions. Framed
329 differently, the bone of contention is whether the Agreed Locations are, from a legal perspective, foreign
900 hectares Radio Bigot Annex at Bamban, Tarlac
military facilities or bases. This legal framework triggers Section 25, Article XVIII, and makes Senate
The Bases Conversion and Development Act of 1992 described its coverage in its Declaration of concurrence a sine qua non.
Policies:
Article III of EDCA provides for Agreed Locations, in which the U.S. is authorized by the Philippines to
Sec. 2. Declaration of Policies. - It is hereby declared the policy of the Government to accelerate the "conduct the following activities: "training; transit; support and related activities; refueling of aircraft;
sound and balanced conversion into alternative productive uses of the Clark and Subic military bunkering of vessels; temporary maintenance of vehicles, vessels and aircraft; temporary
reservations and their extensions (John Hay Station, Wallace Air Station, O'Donnell Transmitter Station, accommodation of personnel; communications; prepositioning of equipment, supplies and materiel;
San Miguel Naval Communications Station and Capas Relay Station), to raise funds by the sale of deploying forces and materiel; and such other activities as the Parties may agree."
portions of Metro Manila military camps, and to apply said funds as provided herein for the development
and conversion to productive civilian use of the lands covered under the 194 7 Military Bases This creation of EDCA must then be tested against a proper interpretation of the Section 25 restriction.
330
Agreement between the Philippines and the United States of America, as amended.
d. Reasons for the constitutional requirements and legal standards for constitutionally compatible
The result of the debates and subsequent voting is Section 25, Article XVIII of the Constitution, which military bases and facilities
specifically restricts, among others, foreign military facilities or bases. At the time of its crafting of the
Constitution, the 1986 Constitutional Commission had a clear idea of what exactly it was restricting. Section 25 does not define what is meant by a "foreign military facility or base." While it specifically
While the term "facilities and bases" was left undefined, its point of reference was clearly those areas alludes to U.S. military facilities and bases that existed during the framing of the Constitution, the
covered by the 1947 MBA as amended. provision was clearly meant to apply to those bases existing at the time and to any future facility or
base. The basis for the restriction must first be deduced from the spirit of the law, in order to set a
standard for the application of its text, given the particular historical events preceding the agreement.
Once more, we must look to the 1986 Constitutional Commissioners to glean, from their collective winter of political discontent. Hence, if I may join Commissioner Laurel, I also invoke a caveat not only
336
wisdom, the intent of Section 25. Their speeches are rich with history and wisdom and present a clear against the tyranny of labels but also the tyranny of slogans.
picture of what they considered in the crafting the provision.
xxxx
334
SPEECH OF COMMISSIONER REGALADO
337
SPEECH OF COMMISSIONER SUAREZ
xxxx
MR. SUAREZ: Thank you, Madam President.
We have been regaled here by those who favor the adoption of the anti-bases provisions with what
purports to be an objective presentation of the historical background of the military bases in the I am quite satisfied that the crucial issues involved in the resolution of the problem of the removal of
Philippines. Care appears, however, to have been taken to underscore the inequity in their inception foreign bases from the Philippines have been adequately treated by previous speakers. Let me,
as well as their implementation, as to seriously reflect on the supposed objectivity of the report. therefore, just recapitulate the arguments adduced in favor of a foreign bases-free Philippines:
Pronouncements of military and civilian officials shortly after World War II are quoted in support of the
proposition on neutrality; regrettably, the implication is that the same remains valid today, as if the 1. That every nation should be free to shape its own destiny without outside interference;
world and international activity stood still for the last 40 years.
2. That no lasting peace and no true sovereignty would ever be achieved so long as there are foreign
We have been given inspired lectures on the effect of the presence of the military bases on our military forces in our country;
sovereignty - whether in its legal or political sense is not clear - and the theory that any country
3. That the presence of foreign military bases deprives us of the very substance of national
with foreign bases in its territory cannot claim to be fully sovereign or completely independent. I
sovereigntyand this is a constant source of national embarrassment and an insult to our national
was not aware that the concepts of sovereignty and independence have now assumed the totality
dignity and selfrespect as a nation;
principle, such that a willing assumption of some delimitations in the exercise of some aspects thereof
would put that State in a lower bracket of nationhood. 4. That these foreign military bases unnecessarily expose our country to devastating nuclear
attacks;
xxxx
5. That these foreign military bases create social problems and are designed to perpetuate the strangle-
We have been receiving a continuous influx of materials on the pros and cons on the advisability of
hold of United States interests in our national economy and development;
having military bases within our shores. Most of us who, only about three months ago, were just mulling
the prospects of these varying contentions are now expected, like armchair generals, to decide not only 6. That the extraterritorial rights enjoyed by these foreign bases operate to deprive our country of
on the geopolitical aspects and contingent implications of the military bases but also on their political, jurisdiction over civil and criminal offenses committed within our own national territory and against
social, economic and cultural impact on our national life. We are asked to answer a plethora of Filipinos;
questions, such as: 1) whether the bases are magnets of nuclear attack or are deterrents to such
attack; 2) whether an alliance or mutual defense treaty is a derogation of our national sovereignty; 3) 7. That the bases agreements are colonial impositions and dictations upon our helpless country; and
whether criticism of us by Russia, Vietnam and North Korea is outweighed by the support for us of the
ASEAN countries, the United States, South Korea, Taiwan, Australia and New Zealand; and 4) whether 8. That on the legal viewpoint and in the ultimate analysis, all the bases agreements are null and
the social, moral and legal problems spawned by the military bases and their operations can be void ab initio, especially because they did not count the sovereign consent and will of the Filipino
338
compensated by the economic benefits outlined in papers which have been furnished recently to all of people.
335
us.
xxxx
xxxx
In the real sense, Madam President, if we in the Commission could accommodate the provisions I have
Of course, one side of persuasion has submitted categorical, unequivocal and forceful assertions of cited, what is our objection to include in our Constitution a matter as priceless as the nationalist values
their positions. They are entitled to the luxury of the absolutes. We are urged now to adopt the we cherish? A matter of the gravest concern for the safety and survival of this nation indeed
proposed declaration as a "golden," "unique" and "last" opportunity for Filipinos to assert their deserves a place in our Constitution.
sovereign rights. Unfortunately, I have never been enchanted by superlatives, much less for the
applause of the moment or the ovation of the hour. Nor do I look forward to any glorious summer after a xxxx
x x x Why should we bargain away our dignity and our self-respect as a nation and the future of namely: It is ideal for us as an independent and sovereign nation to ultimately abrogate the RP-
339 347
generations to come with thirty pieces of silver? US military treaty and, at the right time, build our own air and naval might.
340
SPEECH OF COMMISSIONER BENNAGEN xxxx

xxxx Allow me to say in summation that I am for the retention of American military bases in the
Philippines provided that such an extension from one period to another shall be concluded
The underlying principle of military bases and nuclear weapons wherever they are found and whoever upon concurrence of the parties, and such extension shall be based on justice, the historical
owns them is that those are for killing people or for terrorizing humanity. This objective by itself at amity of the people of the Philippines and the United States and their common defense
any point in history is morally repugnant. This alone is reason enough for us to constitutionalize the ban 348
interest.
341
on foreign military bases and on nuclear weapons.
349
SPEECH OF COMMISSIONER ALONTO
342
SPEECH OF COMMISSIONER BACANI
xxxx
xxxx
Madam President, sometime ago after this Commission started with this task of framing a constitution, I
x x x Hence, the remedy to prostitution does not seem to be primarily to remove the read a statement of President Aquino to the effect that she is for the removal of the U.S. military bases
bases because even if the bases are removed, the girls mired in poverty will look for their clientele in this country but that the removal of the U.S. military bases should not be done just to give way to
elsewhere. The remedy to the problem of prostitution lies primarily elsewhere - in an alert and other foreign bases. Today, there are two world superpowers, both vying to control any and all countries
343
concerned citizenry, a healthy economy and a sound education in values. which have importance to their strategy for world domination. The Philippines is one such country.
344
SPEECH OF COMMISSIONER JAMIR Madam President, I submit that I am one of those ready to completely remove any vestiges of the
days of enslavement, but not prepared to erase them if to do so would merely leave a vacuum to be
xxxx occupied by a far worse type.
350

One of the reasons advanced against the maintenance of foreign military bases here is that they SPEECH OF COMMISSIONER GASCON
351

impair portions of our sovereignty. While I agree that our country's sovereignty should not be
impaired, I also hold the view that there are times when it is necessary to do so according to the xxxx
imperatives of national interest. There are precedents to this effect. Thus, during World War II, England
leased its bases in the West Indies and in Bermuda for 99 years to the United States for its use as naval Let us consider the situation of peace in our world today. Consider our brethren in the Middle East, in
and air bases. It was done in consideration of 50 overaged destroyers which the United States gave to Indo-China, Central America, in South Africa - there has been escalation of war in some of these areas
England for its use in the Battle of the Atlantic. because of foreign intervention which views these conflicts through the narrow prism of the East-West
conflict. The United States bases have been used as springboards for intervention in some of
A few years ago, England gave the Island of Diego Garcia to the United States for the latter's use as a these conflicts. We should not allow ourselves to be party to the warlike mentality of these
naval base in the Indian Ocean. About the same time, the United States obtained bases in Spain, Egypt foreign interventionists. We must always be on the side of peace – this means that we should not
and Israel. In doing so, these countries, in effect, contributed to the launching of a preventive defense 352
always rely on military solution.
345
posture against possible trouble in the Middle East and in the Indian Ocean for their own protection.
xxxx
346
SPEECH OF COMMISSIONER TINGSON
x x x The United States bases, therefore, are springboards for intervention in our own internal
xxxx affairs and in the affairs of other nations in this region.

In the case of the Philippines and the other Southeast Asian nations, the presence of American troops xxxx
in the country is a projection of America's security interest. Enrile said that nonetheless, they also serve,
although in an incidental and secondary way, the security interest of the Republic of the Philippines and Thus, I firmly believe that a self-respecting nation should safeguard its fundamental freedoms which
the region. Yes, of course, Mr. Enrile also echoes the sentiments of most of us in this Commission, should logically be declared in black and white in our fundamental law of the land - the Constitution. Let
us express our desire for national sovereignty so we may be able to achieve national self-
determination. Let us express our desire for neutrality so that we may be able to follow active First, the failure of the Constitutional Commission to decisively respond to the continuing violation of
nonaligned independent foreign policies. Let us express our desire for peace and a nuclear-free zone our territorial integrity via the military bases agreement which permits the retention of U.S.
so we may be able to pursue a healthy and tranquil existence, to have peace that is autonomous and facilities within the Philippine soil over which our authorities have no exclusive jurisdiction
353
not imposed. contrary to the accepted definition of the exercise of sovereignty.

xxxx Second, consent by this forum, this Constitutional Commission, to an exception in the application of a
provision in the Bill of Rights that we have just drafted regarding equal application of the laws of the
354
SPEECH OF COMMISSIONER TADEO land to all inhabitants, permanent or otherwise, within its territorial boundaries.

Para sa magbubukid, ano ha ang kahulugan ng U.S. military bases? Para sa magbubukid, ang Third, the continued exercise by the United States of extraterritoriality despite the condemnations
kahulugan nito ay pagkaalipin. Para sa magbubukid, ang pananatili ng U.S. military bases ay tinik sa of such practice by the world community of nations in the light of overwhelming international approval of
dibdib ng sambayanang Pilipinong patuloy na nakabaon. Para sa sambayanang magbubukid, ang 358
eradicating all vestiges of colonialism.
ibig sabihin ng U.S. military bases ay batong pabigat na patuloy na pinapasan ng sambayanang
Pilipino. Para sa sambayanang magbubukid, ang pananatili ng U.S. military bases ay isang xxxx
nagdudumilat na katotohanan ng patuloy na paggahasa ng imperyalistang Estados Unidos sa
ating Inang Bayan - economically, politically and culturally. Para sa sambayanang magbubukid Sixth, the deification of a new concept called pragmatic sovereignty, in the hope that such can be
ang U.S. military bases ay kasingkahulugan ng nuclear weapon - ang kahulugan ay magneto ng wielded to force the United States government to concede to better terms and conditions concerning the
isang nuclear war. Para sa sambayanang magbubukid, ang kahulugan ng U.S. military bases ay military bases agreement, including the transfer of complete control to the Philippine government
355 of the U.S. facilities, while in the meantime we have to suffer all existing indignities and disrespect
isang salot.
towards our rights as a sovereign nation.
356
SPEECH OF COMMISSIONER QUESADA
xxxx
xxxx
Eighth, the utter failure of this forum to view the issue of foreign military bases as essentially a
The drift in the voting on issues related to freeing ourselves from the instruments of domination question of sovereignty which does not require in-depth studies or analyses and which this forum has,
and subservience has clearly been defined these past weeks. as a constituent assembly drafting a constitution, the expertise and capacity to decide on except that it
lacks the political will that brought it to existence and now engages in an elaborate scheme of buck-
xxxx passing.

So for the record, Mr. Presiding Officer, I would like to declare my support for the committee's position xxxx
to enshrine in the Constitution a fundamental principle forbidding foreign military bases, troops or
facilities in any part of the Philippine territory as a clear and concrete manifestation of our inherent Without any doubt we can establish a new social order in our country, if we reclaim, restore, uphold and
right to national self-determination, independence and sovereignty. defend our national sovereignty. National sovereignty is what the military bases issue is all
about. It is only the sovereign people exercising their national sovereignty who can design an
Mr. Presiding Officer, I would like to relate now these attributes of genuine nationhood to the social cost 359
independent course and take full control of their national destiny.
of allowing foreign countries to maintain military bases in our country. Previous speakers have dwelt on
360
this subject, either to highlight its importance in relation to the other issues or to gloss over its SPEECH OF COMMISSIONER P ADILLA
357
significance and !llake this a part of future negotiations.
xxxx
xxxx
Mr. Presiding Officer, in advocating the majority committee report, specifically Sections 3 and 4 on
Mr. Presiding Officer, I feel that banning foreign military bases is one of the solutions and is the neutrality, nuclear and bases-free country, some views stress sovereignty of the Republic and even
361
response of the Filipino people against this condition and other conditions that have already been invoke survival of the Filipino nation and people.
clearly and emphatically discussed in past deliberations. The deletion, therefore, of Section 3 in the
362
Constitution we are drafting will have the following implications: REBUTTAL OF COMMISSIONER NOLLEDO

xxxx
The anachronistic and ephemeral arguments against the provisions of the committee report to In this case, EDCA explicitly provides that ownership of the Agreed Locations remains with the
368
dismantle the American bases after 1991 only show the urgent need to free our country from the Philippine govemment. What U.S. personnel have a right to, pending mutual agreement, is access to
363 369
entangling alliance with any power bloc. and use of these locations.

xxxx The right of the owner of the property to allow access and use is consistent with the Civil Code, since
the owner may dispose of the property in whatever way deemed fit, subject to the limits of the law. So
xx x Mr. Presiding Officer, it is not necessary for us to possess expertise to know that the so-called RP- long as the right of ownership itself is not transferred, then whatever rights are transmitted by
US Bases Agreement will expire in 1991, that it infringes on our sovereignty and jurisdiction as well agreement does not completely divest the owner of the rights over the property, but may only limit them
as national dignity and honor, that it goes against the UN policy of disarmament and that it in accordance with law.
364
constitutes unjust intervention in our internal affairs. (Emphases Supplied)
Hence, even control over the property is something that an owner may transmit freely. This act does not
The Constitutional Commission eventually agreed to allow foreign military bases, troops, or facilities, translate into the full transfer of ownership, but only of certain rights. In Roman Catholic Apostolic
subject to the provisions of Section 25. It is thus important to read its discussions carefully. From these Administrator of Davao, Inc. v. Land Registration Commission, we stated that the constitutional
discussions, we can deduce three legal standards that were articulated by the Constitutional proscription on property ownership is not violated despite the foreign national's control over the
Commission Members. These are characteristics of any agreement that the country, and by extension 370
property.
this Court, must ensure are observed. We can thereby determine whether a military base or facility in
the Philippines, which houses or is accessed by foreign military troops, is foreign or remains a EDCA, in respect of its provisions on Agreed Locations, is essentially a contract of use and access.
Philippine military base or facility. The legal standards we find applicable are: independence from Under its pertinent provisions, it is the Designated Authority of the Philippines that shall, when
371
foreign control, sovereignty and applicable law, and national security and territorial integrity. requested, assist in facilitating transit or access to public land and facilities. The activities carried out
372
within these locations are subject to agreement as authorized by the Philippine govemment. Granting
i. First standard: independence from foreign control the U.S. operational control over these locations is likewise subject to EDCA' s security mechanisms,
373
which are bilateral procedures involving Philippine consent and cooperation. Finally, the Philippine
Very clearly, much of the opposition to the U.S. bases at the time of the Constitution's drafting was Designated Authority or a duly designated representative is given access to the Agreed Locations.
374

aimed at asserting Philippine independence from the U.S., as well as control over our country's territory
and military. To our mind, these provisions do not raise the spectre of U.S. control, which was so feared by the
Constitutional Commission. In fact, they seem to have been the product of deliberate negotiation from
Under the Civil Code, there are several aspects of control exercised over property. the point of view of the Philippine government, which balanced constitutional restrictions on foreign
365 military bases and facilities against the security needs of the country. In the 1947 MBA, the U.S. forces
Property is classified as private or public. It is public if "intended for public use, such as roads, canals,
had "the right, power and authority x x x to construct (including dredging and filling), operate, maintain,
rivers, torrents, ports and bridges constructed by the State, banks, shores, roadsteads, and others of 375
utilize, occupy, garrison and control the bases." No similarly explicit provision is present in EDCA.
similar character[,]" or "[t]hose which belong to the State, without being for public use, and are intended
366
for some public service or for the development of the national wealth. " Nevertheless, the threshold for allowing the presence of foreign military facilities and bases has been
raised by the present Constitution. Section 25 is explicit that foreign military bases, troops, or facilities
Quite clearly, the Agreed Locations are contained within a property for public use, be it within a
shall not be allowed in the Philippines, except under a treaty duly concurred in by the Senate. Merely
government military camp or property that belongs to the Philippines.1avvphi1
stating that the Philippines would retain ownership would do violence to the constitutional requirement if
Once ownership is established, then the rights of ownership flow freely. Article 428 of the Civil Code the Agreed Locations were simply to become a less obvious manifestation of the U.S. bases that were
provides that "[t]he owner has the right to enjoy and dispose of a thing, without other limitations than rejected in 1991.
those established by law." Moreover, the owner "has also a right of action against the holder and
When debates took place over the military provisions of the Constitution, the committee rejected a
possessor of the thing in order to recover it."
specific provision proposed by Commissioner Sarmiento. The discussion illuminates and provides
Philippine civil law therefore accords very strong rights to the owner of property, even against those who context to the 1986 Constitutional Commission's vision of control and independence from the U.S., to
hold the property. Possession, after all, merely raises a disputable presumption of ownership, which can wit:
367
be contested through normal judicial processes.
MR. SARMIENTO: Madam President, my proposed amendment reads as follows: "THE STATE SHALL
ESTABLISH AND MAINTAIN AN INDEPENDENT AND SELF-RELIANT ARMED FORCES OF THE
PHILIPPINES." Allow me to briefly explain, Madam President. The Armed Forces of the Philippines is a
vital component of Philippine society depending upon its training, orientation and support. It will either in Australia, England and in Paris. So, it does not mean that when we send military officers to United
be the people's protector or a staunch supporter of a usurper or tyrant, local and foreign interest. The States schools or to other military schools, we will be under the control of that country. We also have
Armed Forces of the Philippines' past and recent experience shows it has never been foreign officers in our schools, we in the Command and General Staff College in Fort Bonifacio and in
377
independent and self-reliant. Facts, data and statistics will show that it has been substantially our National Defense College, also in Fort Bonifacio. (Emphases supplied)
dependent upon a foreign power. In March 1968, Congressman Barbero, himself a member of the
Armed Forces of the Philippines, revealed top secret documents showing what he described as U.S. This logic was accepted in Tañada v. Angara, in which the Court ruled that independence does not
dictation over the affairs of the Armed Forces of the Philippines. He showed that under existing mean the absence of foreign participation:
arrangements, the United States unilaterally determines not only the types and quantity of arms
and equipments that our armed forces would have, but also the time when these items are to be Furthermore, the constitutional policy of a "self-reliant and independent national economy" does not
made available to us. It is clear, as he pointed out, that the composition, capability and schedule necessarily rule out the entry of foreign investments, goods and services. It contemplates neither
of development of the Armed Forces of the Philippines is under the effective control of the U.S. "economic seclusion" nor "mendicancy in the international community." As explained by Constitutional
376
government. (Emphases supplied) Commissioner Bernardo Villegas, sponsor of this constitutional policy:

Commissioner Sarmiento proposed a motherhood statement in the 1987 Constitution that would assert Economic self reliance is a primary objective of a developing country that is keenly aware of
"independent" and "self-reliant" armed forces. This proposal was rejected by the committee, overdependence on external assistance for even its most basic needs. It does not mean autarky or
however. As Commissioner De Castro asserted, the involvement of the Philippine military with economic seclusion; rather, it means avoiding mendicancy in the international
the U.S. did not, by itself, rob the Philippines of its real independence. He made reference to the community. Independence refers to the freedom from undue foreign control of the national
context of the times: that the limited resources of the Philippines and the current insurgency at that time economy, especially in such strategic industries as in the development of natural resources and public
378
necessitated a strong military relationship with the U.S. He said that the U.S. would not in any way utilities. (Emphases supplied)
control the Philippine military despite this relationship and the fact that the former would furnish military
The heart of the constitutional restriction on foreign military facilities and bases is therefore the assertion
hardware or extend military assistance and training to our military. Rather, he claimed that the proposal
of independence from the U.S. and other foreign powers, as independence is exhibited by the degree of
was in compliance with the treaties between the two states.
foreign control exerted over these areas.1âwphi1 The essence of that independence is self-governance
379
MR. DE CASTRO: If the Commissioner will take note of my speech on U.S. military bases on 12 and self-control. Independence itself is "[t]he state or condition of being free from dependence,
380
September 1986, I spoke on the selfreliance policy of the armed forces. However, due to very limited subjection, or control. "
resources, the only thing we could do is manufacture small arms ammunition. We cannot blame the
Petitioners assert that EDCA provides the U.S. extensive control and authority over Philippine facilities
armed forces. We have to blame the whole Republic of the Philippines for failure to provide the 381
and locations, such that the agreement effectively violates Section 25 of the 1987 Constitution.
necessary funds to make the Philippine Armed Forces self-reliant. Indeed that is a beautiful dream. And
I would like it that way. But as of this time, fighting an insurgency case, a rebellion in our country - Under Article VI(3) of EDCA, U.S. forces are authorized to act as necessary for "operational control and
insurgency - and with very limited funds and very limited number of men, it will be quite impossible for defense." The term "operational control" has led petitioners to regard U.S. control over the Agreed
the Philippines to appropriate the necessary funds therefor. However, if we say that the U.S. 382
Locations as unqualified and, therefore, total. Petitioners contend that the word "their" refers to the
government is furnishing us the military hardware, it is not control of our armed forces or of our
subject "Agreed Locations."
government. It is in compliance with the Mutual Defense Treaty. It is under the military assistance
program that it becomes the responsibility of the United States to furnish us the necessary hardware in This argument misreads the text, which is quoted below:
connection with the military bases agreement. Please be informed that there are three (3) treaties
connected with the military bases agreement; namely: the RP-US Military Bases Agreement, the Mutual United States forces are authorized to exercise all rights and authorities within Agreed Locations that
Defense Treaty and the Military Assistance Program. are necessary for their operational control or defense, including taking appropriate measure to protect
United States forces and United States contractors. The United States should coordinate such
My dear Commissioner, when we enter into a treaty and we are furnished the military hardware measures with appropriate authorities of the Philippines.
pursuant to that treaty, it is not in control of our armed forces nor control of our
government. True indeed, we have military officers trained in the U.S. armed forces school. This is part A basic textual construction would show that the word "their," as understood above, is a possessive
of our Military Assistance Program, but it does not mean that the minds of our military officers are for pronoun for the subject "they," a third-person personal pronoun in plural form. Thus, "their" cannot be
the U.S. government, no. I am one of those who took four courses in the United States schools, but I used for a non-personal subject such as "Agreed Locations." The simple grammatical conclusion is that
assure you, my mind is for the Filipino people. Also, while we are sending military officers to train or to "their" refers to the previous third-person plural noun, which is "United States forces." This conclusion is
study in U.S. military schools, we are also sending our officers to study in other military schools such as in line with the definition of operational control.
392
a. U.S. operational control as the exercise of authority over U.S. personnel, and not over the Agreed from operational control in U.S. parlance. Citing the Doctrine for the Armed Forces of the United
Locations States, Joint Publication 1, "command and control (C2)" is defined as "the exercise of authority and
direction by a properly designated commander over assigned and attached forces in the
Operational control, as cited by both petitioner and respondents, is a military term referring to 393
accomplishment of the mission x x x." Operational control, on the other hand, refers to "[t]hose
functions of command over assigned forces involving the composition of subordinate forces, the
[t]he authority to perform those functions of command over subordinate forces involving organizing and assignment of tasks, the designation of objectives, the overall control of assigned resources, and the full
employing commands and forces, assigning tasks, designating objective, and giving authoritative authoritative direction necessary to accomplish the mission."
394
383
direction necessary to accomplish the mission.
Two things demonstrate the errors in petitioners' line of argument.
At times, though, operational control can mean something slightly different. In JUSMAG Philippines v.
National Labor Relations Commission, the Memorandum of Agreement between the AFP and JUSMAG Firstly, the phrase "consistent with operational safety and security requirements in accordance with
384
Philippines defined the term as follows: agreed procedures developed by the Parties" does not add any qualification beyond that which is
already imposed by existing treaties. To recall, EDCA is based upon prior treaties, namely the VFA and
The term "Operational Control" includes, but is not limited to, all personnel administrative actions, such 395
the MDT. Treaties are in themselves contracts from which rights and obligations may be claimed or
as: hiring recommendations; firing recommendations; position classification; discipline; nomination and 396
waived. In this particular case, the Philippines has already agreed to abide by the security
approval of incentive awards; and payroll computation. mechanisms that have long been in place between the U.S. and the Philippines based on the
397
implementation of their treaty relations.
Clearly, traditional standards define "operational control" as personnel control. Philippine law, for
instance, deems operational control as one exercised by police officers and civilian authorities over their Secondly, the full document cited by petitioners contradicts the equation of "operational control" with
subordinates and is distinct from the administrative control that they also exercise over police "effective command and control," since it defines the terms quite differently, viz:
398
385
subordinates. Similarly, a municipal mayor exercises operational control over the police within the
386
municipal government, just as city mayor possesses the same power over the police within the city Command and control encompasses the exercise of authority, responsibility, and direction by a
387
government. commander over assigned and attached forces to accomplish the mission. Command at all levels is the
art of motivating and directing people and organizations into action to accomplish missions. Control is
Thus, the legal concept of operational control involves authority over personnel in a commander- inherent in command. To control is to manage and direct forces and functions consistent with a
subordinate relationship and does not include control over the Agreed Locations in this particular case. commander's command authority. Control of forces and functions helps commanders and staffs
Though not necessarily stated in EDCA provisions, this interpretation is readily implied by the reference compute requirements, allocate means, and integrate efforts. Mission command is the preferred method
to the taking of "appropriate measures to protect United States forces and United States contractors." of exercising C2. A complete discussion of tenets, organization, and processes for effective C2 is
provided in Section B, "Command and Control of Joint Forces," of Chapter V "Joint Command and
It is but logical, even necessary, for the U.S. to have operational control over its own forces, in much the
Control."
same way that the Philippines exercises operational control over its own units.
399
Operational control is defined thus:
For actual operations, EDCA is clear that any activity must be planned and pre-approved by the MDB-
388
SEB. This provision evinces the partnership aspect of EDCA, such that both stakeholders have a say OPCON is able to be delegated from a lesser authority than COCOM. It is the authority to perform those
on how its provisions should be put into effect. functions of command over subordinate forces involving organizing and employing commands and
forces, assigning tasks, designating objectives, and giving authoritative direction over all aspects of
b. Operational control vis-à-vis effective command and control
military operations and joint training necessary to accomplish the mission. It should be delegated to and
Petitioners assert that beyond the concept of operational control over personnel, qualifying access to exercised by the commanders of subordinate organizations; normally, this authority is exercised through
the Agreed Locations by the Philippine Designated Authority with the phrase "consistent with subordinate JFCs, Service, and/or functional component commanders. OPCON provides authority to
operational safety and security requirements in accordance with agreed procedures developed by the organize and employ commands and forces as the commander considers necessary to accomplish
Parties" leads to the conclusion that the U.S. exercises effective control over the Agreed assigned missions. It does not include authoritative direction for logistics or matters of administration,
389
Locations. They claim that if the Philippines exercises possession of and control over a given area, its discipline, internal organization, or unit training. These elements of COCOM must be specifically
representative should not have to be authorized by a special provision.
390 delegated by the CCDR. OPCON does include the authority to delineate functional responsibilities and
operational areas of subordinate JFCs.
For these reasons, petitioners argue that the "operational control" in EDCA is the "effective command
391
and control" in the 1947 MBA. In their Memorandum, they distinguish effective command and control
Operational control is therefore the delegable aspect of combatant command, while command and that "all United States access to and use of facilities and areas will be at the invitation of the Philippines
control is the overall power and responsibility exercised by the commander with reference to a mission. and with full respect for the Philippine Constitution and Philippine laws."
Operational control is a narrower power and must be given, while command and control is plenary and
vested in a commander. Operational control does not include the planning, programming, budgeting, The sensitivity of EDCA provisions to the laws of the Philippines must be seen in light of Philippine
and execution process input; the assignment of subordinate commanders; the building of relationships sovereignty and jurisdiction over the Agreed Locations.
with Department of Defense agencies; or the directive authority for logistics, whereas these factors are 406
included in the concept of command and control.
400 Sovereignty is the possession of sovereign power, while jurisdiction is the conferment by law of
407
power and authority to apply the law. Article I of the 1987 Constitution states:
This distinction, found in the same document cited by petitioners, destroys the very foundation of the
arguments they have built: that EDCA is the same as the MBA. The national territory comprises the Philippine archipelago, with all the islands and waters embraced
therein, and all other territories over which the Philippines has sovereignty or jurisdiction, consisting
c. Limited operational control over the Agreed Locations only for construction activitites of its terrestrial, fluvial, and aerial domains, including its territorial sea, the seabed, the subsoil, the
insular shelves, and other submarine areas. The waters around, between, and connecting the islands of
As petitioners assert, EDCA indeed contains a specific provision that gives to the U.S. operational the archipelago, regardless of their breadth and dimensions, form part of the internal waters of the
401
control within the Agreed Locations during construction activities. This exercise of operational control Philippines. (Emphasis supplied)
is premised upon the approval by the MDB and the SEB of the construction activity through consultation
and mutual agreement on the requirements and standards of the construction, alteration, or From the text of EDCA itself, Agreed Locations are territories of the Philippines that the U.S. forces are
402 408
improvement. allowed to access and use. By withholding ownership of these areas and retaining unrestricted
access to them, the government asserts sovereignty over its territory. That sovereignty exists so long as
409
Despite this grant of operational control to the U.S., it must be emphasized that the grant is only for the Filipino people exist.
construction activities. The narrow and limited instance wherein the U.S. is given operational control
within an Agreed Location cannot be equated with foreign military control, which is so abhorred by the Significantly, the Philippines retains primary responsibility for security with respect to the Agreed
410
Constitution. Locations. Hence, Philippine law remains in force therein, and it cannot be said that jurisdiction has
been transferred to the U.S. Even the previously discussed necessary measures for operational control
411
The clear import of the provision is that in the absence of construction activities, operational control over and defense over U.S. forces must be coordinated with Philippine authorities.
the Agreed Location is vested in the Philippine authorities. This meaning is implicit in the specific grant
of operational control only during construction activities. The principle of constitutional Jurisprudence bears out the fact that even under the former legal regime of the MBA, Philippine laws
412
construction, "expressio unius est exclusio alterius," means the failure to mention the thing becomes the continue to be in force within the bases. The difference between then and now is that EDCA retains
403 the primary jurisdiction of the Philippines over the security of the Agreed Locations, an important
ground for inferring that it was deliberately excluded. Following this construction, since EDCA
mentions the existence of U.S. operational control over the Agreed Locations for construction activities, provision that gives it actual control over those locations. Previously, it was the provost marshal of the
then it is quite logical to conclude that it is not exercised over other activities. U.S. who kept the peace and enforced Philippine law in the bases. In this instance, Philippine forces act
413
as peace officers, in stark contrast to the 1947 MBA provisions on jurisdiction.
Limited control does not violate the Constitution. The fear of the commissioners was total control, to the
404 iii. Third standard: must respect national security and territorial integrity
point that the foreign military forces might dictate the terms of their acts within the Philippines. More
important, limited control does not mean an abdication or derogation of Philippine sovereignty and legal
jurisdiction over the Agreed Locations. It is more akin to the extension of diplomatic courtesies and The last standard this Court must set is that the EDCA provisions on the Agreed Locations must not
405
rights to diplomatic agents, which is a waiver of control on a limited scale and subject to the terms of impair or threaten the national security and territorial integrity of the Philippines.
the treaty.
This Court acknowledged in Bayan v. Zamora that the evolution of technology has essentially rendered
This point leads us to the second standard envisioned by the framers of the Constitution: that the the prior notion of permanent military bases obsolete.
Philippines must retain sovereignty and jurisdiction over its territory.
Moreover, military bases established within the territory of another state is no longer viable because of
ii. Second standard: Philippine sovereignty and applicable law the alternatives offered by new means and weapons of warfare such as nuclear weapons, guided
missiles as well as huge sea vessels that can stay afloat in the sea even for months and years without
EDCA states in its Preamble the "understanding for the United States not to establish a permanent returning to their home country. These military warships are actually used as substitutes for a land-
military presence or base in the territory of the Philippines." Further on, it likewise states the recognition home base not only of military aircraft but also of military personnel and facilities. Besides, vessels are
414
mobile as compared to a land-based military headquarters.
428 429
The VFA serves as the basis for the entry of U.S. troops in a limited scope. It does not allow, for an armed conflict to distinguish between lawful targets and protected targets. In an actual armed
instance, the re-establishment of the Subic military base or the Clark Air Field as U.S. military conflict between the U.S. and a third state, the Agreed Locations cannot be considered U.S. territory,
430
reservations. In this context, therefore, this Court has interpreted the restrictions on foreign bases, since ownership of territory even in times of armed conflict does not change.
troops, or facilities as three independent restrictions. In accord with this interpretation, each restriction
must have its own qualification. Hence, any armed attack by forces of a third state against an Agreed Location can only be legitimate
under international humanitarian law if it is against a bona fide U.S. military base, facility, or installation
415
Petitioners quote from the website http://en.wikipedia.org to define what a military base is. While the that directly contributes to the military effort of the U.S. Moreover, the third state's forces must take all
source is not authoritative, petitioners make the point that the Agreed Locations, by granting access and measures to ensure that they have complied with the principle of distinction (between combatants and
416
use to U.S. forces and contractors, are U.S. bases under a different name. More important, they non-combatants).
417
claim that the Agreed Locations invite instances of attack on the Philippines from enemies of the U.S.
There is, then, ample legal protection for the Philippines under international law that would ensure its
We believe that the raised fear of an attack on the Philippines is not in the realm of law, but of politics territorial integrity and national security in the event an Agreed Location is subjected to attack. As EDCA
and policy. At the very least, we can say that under international law, EDCA does not provide a legal stands, it does not create the situation so feared by petitioners - one in which the Philippines, while not
431
basis for a justified attack on the Philippines. participating in an armed conflict, would be legitimately targeted by an enemy of the U.S.

In the first place, international law disallows any attack on the Agreed Locations simply because of the In the second place, this is a policy question about the wisdom of allowing the presence of U.S.
presence of U.S. personnel. Article 2(4) of the United Nations Charter states that "All Members shall personnel within our territory and is therefore outside the scope of judicial review.
refrain in their international relations from the threat or use of force against the territorial integrity or
political independence of any state, or in any other manner inconsistent with the Purposes of the United Evidently, the concept of giving foreign troops access to "agreed" locations, areas, or facilities within the
418 military base of another sovereign state is nothing new on the international plane. In fact, this
Nations." Any unlawful attack on the Philippines breaches the treaty, and triggers Article 51 of the
same charter, which guarantees the inherent right of individual or collective self-defence. arrangement has been used as the framework for several defense cooperation agreements, such as in
the following:
Moreover, even if the lawfulness of the attack were not in question, international humanitarian law
432
standards prevent participants in an armed conflict from targeting non-participants. International 1. 2006 U.S.-Bulgaria Defense Cooperation Agreement
humanitarian law, which is the branch of international law applicable to armed conflict, expressly limits 433
419
allowable military conduct exhibited by forces of a participant in an armed conflict. Under this legal 2. 2009 U.S.-Colombia Defense Cooperation Agreement
regime, participants to an armed conflict are held to specific standards of conduct that require them to 434
420 3. 2009 U.S.-Poland Status of Forces Agreement
distinguish between combatants and non-combatants, as embodied by the Geneva Conventions and
421
their Additional Protocols. 4. 2014 U.S.-Australia Force Posture Agreement
435

Corollary to this point, Professor John Woodcliffe, professor of international law at the University of 5. 2014 U.S.-Afghanistan Security and Defense Cooperation Agreement
436

Leicester, noted that there is no legal consensus for what constitutes a base, as opposed to other terms
422 437
such as "facilities" or "installation." In strategic literature, "base" is defined as an installation "over In all of these arrangements, the host state grants U.S. forces access to their military bases. That
423
which the user State has a right to exclusive control in an extraterritorial sense." Since this definition 438
access is without rental or similar costs to the U.S. Further, U.S. forces are allowed to undertake
would exclude most foreign military installations, a more important distinction must be made. construction activities in, and make alterations and improvements to, the agreed locations, facilities, or
439 440
areas. As in EDCA, the host states retain ownership and jurisdiction over the said bases.
For Woodcliffe, a type of installation excluded from the definition of "base" is one that does not fulfill a
combat role. He cites an example of the use of the territory of a state for training purposes, such as to In fact, some of the host states in these agreements give specific military-related rights to the U.S. For
424
obtain experience in local geography and climactic conditions or to carry out joint exercises. Another example, under Article IV(l) of the US.-Bulgaria Defense Cooperation Agreement, "the United States
example given is an advanced communications technology installation for purposes of information forces x x x are authorized access to and may use agreed facilities and areas x x x for staging and
425
gathering and communication. Unsurprisingly, he deems these non-combat uses as borderline deploying of forces and materiel, with the purpose of conducting x x x contingency operations and other
situations that would be excluded from the functional understanding of military bases and missions, including those undertaken in the framework of the North Atlantic Treaty." In some of these
426
installations. agreements, host countries allow U.S. forces to construct facilities for the latter’s exclusive use.
441

By virtue of this ambiguity, the laws of war dictate that the status of a building or person is presumed to Troop billeting, including construction of temporary structures, is nothing new. In Lim v. Executive
427
be protected, unless proven otherwise. Moreover, the principle of distinction requires combatants in Secretary, the Court already upheld the Terms of Reference of Balikatan 02-1, which authorized U.S.
452
forces to set up "[t]emporary structures such as those for troop billeting, classroom instruction and "humanitarian assistance and disaster relief equipment, supplies, and materiel. " Nuclear weapons
messing x x x during the Exercise." Similar provisions are also in the Mutual Logistics Support are specifically excluded from the materiel that will be prepositioned.
Agreement of 2002 and 2007, which are essentially executive agreements that implement the VFA, the
MDT, and the 1953 Military Assistance Agreement. These executive agreements similarly tackle the Therefore, there is no basis to invalidate EDCA on fears that it increases the threat to our national
442 security. If anything, EDCA increases the likelihood that, in an event requiring a defensive response, the
"reciprocal provision of logistic support, supplies, and services," which include "[b ]illeting, x x x
operations support (and construction and use of temporary structures incident to operations support), Philippines will be prepared alongside the U.S. to defend its islands and insure its territorial integrity
443 pursuant to a relationship built on the MDT and VFA.
training services, x x x storage services, x x x during an approved activity." These logistic supplies,
support, and services include temporary use of "nonlethal items of military equipment which are not
designated as significant military equipment on the U.S. Munitions List, during an approved 8. Others issues and concerns raised
444
activity." The first Mutual Logistics Support Agreement has lapsed, while the second one has been
A point was raised during the oral arguments that the language of the MDT only refers to mutual help
extended until 2017 without any formal objection before this Court from the Senate or any of its 453
and defense in the Pacific area. We believe that any discussion of the activities to be undertaken
members.
under EDCA vis-a-vis the defense of areas beyond the Pacific is premature. We note that a proper
The provisions in EDCA dealing with Agreed Locations are analogous to those in the aforementioned petition on that issue must be filed before we rule thereon. We also note that none of the petitions or
executive agreements. Instead of authorizing the building of temporary structures as previous memoranda has attempted to discuss this issue, except only to theorize that the U.S. will not come to
agreements have done, EDCA authorizes the U.S. to build permanent structures or alter or improve our aid in the event of an attack outside of the Pacific. This is a matter of policy and is beyond the scope
445
existing ones for, and to be owned by, the Philippines. EDCA is clear that the Philippines retains of this judicial review.
ownership of altered or improved facilities and newly constructed permanent or non-relocatable
446 In reference to the issue on telecommunications, suffice it to say that the initial impression of the facility
structures. Under EDCA, U.S. forces will also be allowed to use facilities and areas for "training; x x x;
adverted to does appear to be one of those that require a public franchise by way of congressional
support and related activities; x x x; temporary accommodation of personnel; communications" and
447 action under Section 11, Article XII of the Constitution. As respondents submit, however, the system
agreed activities.
referred to in the agreement does not provide telecommunications services to the public for
454
Concerns on national security problems that arise from foreign military equipment being present in the compensation. It is clear from Article VIl(2) of EDCA that the telecommunication system is solely for
Philippines must likewise be contextualized. Most significantly, the VFA already authorizes the the use of the U.S. and not the public in general, and that this system will not interfere with that which
presence of U.S. military equipment in the country. Article VII of the VFA already authorizes the local operators use. Consequently, a public franchise is no longer necessary.
U.S. to import into or acquire in the Philippines "equipment, materials, supplies, and other property" that
Additionally, the charge that EDCA allows nuclear weapons within Philippine territory is entirely
will be used "in connection with activities" contemplated therein. The same section also recognizes that
speculative. It is noteworthy that the agreement in fact specifies that the prepositioned materiel shall not
"[t]itle to such property shall remain" with the US and that they have the discretion to "remove such 455
include nuclear weapons. Petitioners argue that only prepositioned nuclear weapons are prohibited by
property from the Philippines at any time."
EDCA; and that, therefore, the U.S. would insidiously bring nuclear weapons to Philippine
456
There is nothing novel, either, in the EDCA provision on the prepositioning and storing of "defense territory. The general prohibition on nuclear weapons, whether prepositioned or not, is already
457
448
equipment, supplies, and materiel," since these are sanctioned in the VFA. In fact, the two countries expressed in the 1987 Constitution. It would be unnecessary or superfluous to include all prohibitions
have already entered into various implementing agreements in the past that are comparable to the already in the Constitution or in the law through a document like EDCA.
present one. The Balikatan 02-1 Terms of Reference mentioned in Lim v. Executive
Finally, petitioners allege that EDCA creates a tax exemption, which under the law must originate from
Secretary specifically recognizes that Philippine and U.S. forces "may share x x x in the use of their
Congress. This allegation ignores jurisprudence on the government's assumption of tax liability. EDCA
resources, equipment and other assets." Both the 2002 and 2007 Mutual Logistics Support Agreements
simply states that the taxes on the use of water, electricity, and public utilities are for the account of the
speak of the provision of support and services, including the "construction and use of temporary 458
449 Philippine Government. This provision creates a situation in which a contracting party assumes the
structures incident to operations support" and "storage services" during approved activities. These 459
tax liability of the other. In National Power Corporation v. Province of Quezon, we distinguished
logistic supplies, support, and services include the "temporary use of x x x nonlethal items of military
between enforceable and unenforceable stipulations on the assumption of tax liability. Afterwards, we
equipment which are not designated as significant military equipment on the U.S. Munitions List, during
450 concluded that an enforceable assumption of tax liability requires the party assuming the liability to have
an approved activity." Those activities include "combined exercises and training, operations and other 460
actual interest in the property taxed. This rule applies to EDCA, since the Philippine Government
deployments" and "cooperative efforts, such as humanitarian assistance, disaster relief and rescue
451 stands to benefit not only from the structures to be built thereon or improved, but also from the joint
operations, and maritime anti-pollution operations" within or outside Philippine territory. Under EDCA, 461
training with U.S. forces, disaster preparation, and the preferential use of Philippine suppliers. Hence,
the equipment, supplies, and materiel that will be prepositioned at Agreed Locations include
the provision on the assumption of tax liability does not constitute a tax exemption as petitioners have
posited.
Additional issues were raised by petitioners, all relating principally to provisions already sufficiently SO ORDERED.
addressed above. This Court takes this occasion to emphasize that the agreement has been construed
herein as to absolutely disauthorize the violation of the Constitution or any applicable statute. On the MARIA LOURDES P. A. SERENO
contrary, the applicability of Philippine law is explicit in EDCA. Chief Justice

EPILOGUE

The fear that EDCA is a reincarnation of the U.S. bases so zealously protested by noted personalities in
Philippine history arises not so much from xenophobia, but from a genuine desire for self-determination,
nationalism, and above all a commitment to ensure the independence of the Philippine Republic from
any foreign domination.

Mere fears, however, cannot curtail the exercise by the President of the Philippines of his Constitutional
prerogatives in respect of foreign affairs. They cannot cripple him when he deems that additional
security measures are made necessary by the times. As it stands, the Philippines through the
Department of Foreign Affairs has filed several diplomatic protests against the actions of the People's
462
Republic of China in the West Philippine Sea; initiated arbitration against that country under the
463
United Nations Convention on the Law of the Sea; is in the process of negotiations with the Moro
464
Islamic Liberation Front for peace in Southern Philippines, which is the subject of a current case
before this Court; and faces increasing incidents of kidnappings of Filipinos and foreigners allegedly by
465
the Abu Sayyaf or the New People's Army. The Philippine military is conducting reforms that seek to
466
ensure the security and safety of the nation in the years to come. In the future, the Philippines must
navigate a world in which armed forces fight with increasing sophistication in both strategy and
technology, while employing asymmetric warfare and remote weapons.

Additionally, our country is fighting a most terrifying enemy: the backlash of Mother Nature. The
Philippines is one of the countries most directly affected and damaged by climate change. It is no
coincidence that the record-setting tropical cyclone Yolanda (internationally named Haiyan), one of the
most devastating forces of nature the world has ever seen hit the Philippines on 8 November 2013 and
467 468
killed at least 6,000 people. This necessitated a massive rehabilitation project. In the aftermath, the
U.S. military was among the first to extend help and support to the Philippines.

That calamity brought out the best in the Filipinos as thousands upon thousands volunteered their help,
their wealth, and their prayers to those affected. It also brought to the fore the value of having friends in
the international community.

In order to keep the peace in its archipelago in this region of the world, and to sustain itself at the same
time against the destructive forces of nature, the Philippines will need friends. Who they are, and what
form the friendships will take, are for the President to decide. The only restriction is what the
Constitution itself expressly prohibits. It appears that this overarching concern for balancing
constitutional requirements against the dictates of necessity was what led to EDCA.

As it is, EDCA is not constitutionally infirm. As an executive agreement, it remains consistent with
existing laws and treaties that it purports to implement.

WHEREFORE, we hereby DISMISS the petitions.


SECRETARY FOR STRATEGIC ASSESSMENTS RAYMUND JOSE QUILOP AS CHAIRPERSON
AND MEMBERS, RESPECTIVELY, OF THE NEGOTIATING PANEL FOR THE PHILIPPINES ON
EDCA, Respondents.

x-----------------------x

KILUSANG MAYO UNO, REPRESENTED BY ITS CHAIRPERSON, ELMER LABOG,


CONFEDERATION FOR UNITY, RECOGNITION AND ADVANCEMENT OF GOVERNMENT
EMPLOYEES (COURAGE), REPRESENTED BY ITS NATIONAL PRESIDENT FERDINAND GAITE,
NATIONAL FEDERATION OF LABOR UNIONS-KILUSANG MAYO UNO, REPRESENTED BY ITS
EN BANC
NATIONAL PRESIDENT JOSELITO USTAREZ, NENITA GONZAGA, VIOLETA ESPIRITU, VIRGINIA
July 26, 2016 FLORES, and ARMANDO TEODORO, JR., Petitioners-in-Intervention,
RENE A.Q. SAGUISAG, JR., Petitioner-in-Intervention.
G.R. No. 212426
RESOLUTION
RENE A.V. SAGUISAG, WIGBERTO E. TAÑADA, FRANCISCO "DODONG" NEMENZO, JR., SR.
MARY JOHN MANANZAN, PACIFICO A. AGABIN, ESTEBAN "STEVE" SALONGA, H. HARRY L. SERENO, CJ:
ROQUE, JR., EVALYN G. URSUA, EDRE U. OLALIA, DR. CAROL PAGADUAN-ARAULLO, DR.
The Motion for Reconsideration before us seeks to reverse the Decision of this Court in Saguisag et. al.,
ROLAND SIMBULAN, and TEDDY CASIÑO, Petitioners, 1 2
v. Executive Secretary dated 12 January 2016. The petitions in Sasguisag, et. al. had questioned the
vs
constitutionality of the Enhanced Defense Cooperation Agreement (EDCA) between the Republic of the
EXECUTIVE SECRETARY PAQUITO N. OCHOA, JR., DEPARTMENT OF NATIONAL DEFENSE
SECRETARY VOLTAIRE GAZMIN, DEPARTMENT OF FOREIGN AFFAIRS SECRETARY ALBERT Philippines and the United States of America (U.S.). There, this Court ruled that the petitions be
3
DEL ROSARIO, JR., DEPARTMENT OF BUDGET AND MANAGEMENT SECRETARY FLORENCIO dismissed.
ABAD, and ARMED FORCES OF THE PHILIPPINES CHIEF OF STAFF GENERAL EMMANUEL T.
On 3 February 2016, petitioners in the Decision filed the instant Motion, asking for a reconsideration of
BAUTISTA, Respondents.
the Decision in Saguisag, et. al., questioning the ruling of the Court on both procedural and substantive
grounds, viz:
x-----------------------x
WHEREFORE, premises considered, petitioners respectfully pray that the Honorable Court
G.R. No. 212444
RECONSIDER, REVERSE, AND SET- ASIDE its Decision dated January 12, 2016, and issue a new
BAGONG ALY ANSANG MAKABAYAN (BAYAN), REPRESENTED BY ITS SECRETARY GENERAL Decision GRANTING the instant consolidated petitions by declaring the Enhanced Defense
RENATO M. REYES, JR., BAYAN MUNA PARTY-LIST REPRESENTATIVES NERI J. Cooperation Agreement (EDCA) entered into by the respondents for the Philippine government, with the
COLMENARES, and CARLOS ZARATE, GABRIELA WOMEN'S PARTY-LIST REPRESENTATIVES United States of America, UNCONSTITUTIONAL AND INVALID and to permanently enjoin its
LUZ ILAGAN AND EMERENCIANA DE JESUS, ACT TEACHERS PARTY-LIST REPRESENTATIVE implementation.
ANTONIO L. TINIO, ANAKPAWIS PARTY-LIST REPRESENTATIVE FERNANDO HICAP,
KABATAAN PARTY-LIST REPRESENTATIVE TERRY RIDON, MAKABAYANG KOALISYON NG Other fonns of relief just and equitable under the premises are likewise prayed for.
MAMAMAYAN (MAKABAYAN), REPRESENTED BY SATURNINO OCAMPO, and LIZA MAZA,
At the outset, petitioners questioned the procedural findings of the Court despite acknowledging the fact
BIENVENIDO LUMBERA, JOEL C. LAMANGAN, RAFAEL MARIANO, SALVADOR FRANCE, 4
that the Court had given them standing to sue. Therefore this issue is now irrelevant and academic,
ROGELIO M. SOLUTA, and CLEMENTE G. BAUTISTA, Petitioners,
and deserves no reconsideration.
vs.
DEPARTMENT OF NATIONAL DEFENSE (DND) SECRETARY VOLTAIRE GAZMIN, DEPARTMENT As for the substantive grounds, petitioners claim this Court erred when it ruled that EDCA was not a
OF FOREIGN AFFAIRS SECRETARY ALBERT DEL ROSARIO, EXECUTIVE SECRETARY 5
treaty. In connection to this, petitioners move that EDCA must be in the form of a treaty in order to
PAQUITO N. OCHOA, JR., ARMED FORCES OF THE PHILIPPINES CHIEF OF STAFF GENERAL comply with the constitutional restriction under Section 25, Article· XVIII of the 1987 Constitution on
EMMANUEL T. BAUTISTA, DEFENSE UNDERSECRETARY PIO LORENZO BATINO, 6
foreign military bases, troops, and facilities. Additionally, they reiterate their arguments on the issues of
AMBASSADOR LOURDES YPARRAGUIRRE, AMBASSADOR J. EDUARDO MALAYA, telecommunications, taxation, and nuclear weapons.
7

DEPARTMENT OF JUSTICE UNDERSECRETARY FRANCISCO BARAAN III, and DND ASSISTANT


We deny the Motion for Reconsideration. On strict construction of an exception
16
Petitioners do not present new arguments to buttress their claims of error on the part of this Court. They This Court agrees with petitioners' cited jurisprudence that exceptions are strictly construed. However,
have rehashed their prior arguments and made them responsive to the structure of the Decision their patent misunderstanding of the Decision and the confusion this creates behooves this Court to
in Saguisag, yet the points being made are the same. address this argument.

However, certain claims made by petitioners must be addressed. To be clear, the Court did not add an exception to Section 25 Article XVIII. The general rule is that
17
foreign bases, troops, and facilities are not allowed in the Philippines. The exception to this is authority
On verba legis interpretation 18
granted to the foreign state in the form of a treaty duly concurred in by the Philippine Senate.

Petitioners assert that this Court contradicted itself when it interpreted the word "allowed in" to refer to It is in the operation of this exception that the Court exercised its power of review. The lengthy legal
the initial entry of foreign bases, troops, and facilities, based on the fact that the plain meaning of the analysis resulted in a proper categorization of EDCA: an executive cagreement authorized by treaty.
provision in question referred to prohibiting the return of foreign bases, troops, and facilities except This Court undeniably considered the arguments asserting that EDCA was, in fact, a treaty and not an
8
under a treaty concurred in by the Senate. executive agreement, but these arguments fell flat before the stronger legal position that EDCA merely
implemented the VFA and MDT. As we stated in the Decision:
This argument fails to consider the function and application of the verba legis rule.
9
x x x [I]t must already be clarified that the terms and details used by an implementing agreement need
Firstly, verba legis is a mode of construing the provisions of law as they stand. This takes into account not be found in the mother treaty. They must be sourced from the authority derived from the treaty, but
the language of the law, which is in English, and therefore includes reference to the meaning of the are not necessarily expressed word-for-word in the mother treaty.
19

words based on the actual use of the word in the language.


Hence, the argument that the Court added an exception to the law is erroneous and potentially
Secondly, by interpreting "allowed in" as referring to an initial entry, the Court has simply applied the misleading. The parties, both petitioners and respondents must therefore read the Decision carefully in
10
plain meaning of the words in the particular provision. Necessarily, once entry has been established order to fully comply with its disposition.
by a subsisting treaty, latter instances of entry need not be embodied by a separate treaty. After all, the
Constitution did not state that foreign military bases, troops, and facilities shall not subsist or exist in the On EDCA as a treaty
Philippines.
The principal reason for the Motion for Reconsideration is evidently petitioners' disagreement with the
Petitioners' own interpretation and application of the verba legis rule will in fact result in an absurdity, Decision that EDCA implements the VFA
11
which legal construction strictly abhors. If this Court accept the essence of their argument that every
instance of entry by foreign bases, troops, and facilities must be set out in detail in a new treaty, then and MDT. They reiterate their arguments that EDCA's provisions fall outside the allegedly limited scope
the resulting bureaucratic impossibility of negotiating a treaty for the entry of a head of State's or military of the VFA and MDT because it provides a wider arrangement than the VFA for military bases, troops,
20
officer's security detail, meetings of foreign military officials in the country, and indeed military exercises and facilities, and it allows the establishment of U.S. military bases.
such as Balikatan will occupy much of, if not all of the official working time by various government
21
agencies. This is precisely the reason why any valid mode of interpretation must take into account how Specifically, petitioners cite the terms of the VFA referring to "joint exercises," such that arrangements
12 involving the individual States-parties such as exclusive use of prepositioned materiel are not covered
the law is exercised and its goals effected. Ut res magis valeat quam pereat.
by the VFA. More emphatically, they state that prepositioning itself as an activity is not allowed under
22
The Constitution cannot be viewed solely as a list of prohibitions and limitations on governmental the VFA.
power, but rather as an instrument providing the process of structuring government in order that it may
13 Evidently, petitioners left out of their quote the portion of the Decision which cited the Senate report on
effectively serve the people. It is not simply a set of rules, but an entire legal framework for Philippine
society. the VFA. The full quote reads as follows:

In this particular case, we find that EDCA did not go beyond the framework. The entry of US troops has Siazon clarified that it is not the VFA by itself that determines what activities will be conducted between
long been authorized under a valid and subsisting treaty, which is the Visiting Forces Agreement the armed forces of the U.S. and the Philippines. The VFA regulates and provides the legal framework
14 15 for the presence, conduct and legal status of U.S. personnel while they are in the country for visits,
(VFA). Reading the VFA along with the longstanding Mutual Defense Treaty (MDT) led this Court to
23
the conclusion that an executive agreement such as the EDCA was well within the bounds of the joint exercises and other related activities.
obligations imposed by both treaties.
Quite clearly, the VFA contemplated activities beyond joint exercises, which this Court had already We ruled in Saguisag, et. al. that the EDCA is not a treaty despite the presence of these provisions. The
24
recognized and alluded to in Lim v. Executive Secretary, even though the Court in that case was faced very nature of EDCA, its provisions and subject matter, indubitably categorize it as an executive
with a challenge to the Terms of Reference of a specific type of joint exercise, the Balikatan Exercise. agreement - a class of agreement that is not covered by the Article XVIII Section 25 restriction - in
32
painstaking detail. To partially quote the Decision:
One source petitioners used to make claims on the limitation of the VFA to joint exercises is the alleged
Department of Foreign Affairs (DFA) Primer on the VFA, which they claim states that: Executive agreements may dispense with the requirement of Senate concurrence because of the legal
mandate with which they are concluded. As culled from the afore-quoted deliberations of the
Furthermore, the VFA does not involve access arrangements for United States armed forces or the pre- Constitutional Commission, past Supreme Court Decisions, and works of noted scholars, executive
positioning in the country of U.S. armaments and war materials. The agreement is about personnel and agreements merely involve arrangements on the implementation of existing policies, rules, laws, or
25
not equipment or supplies. agreements. They are concluded (1) to adjust the details of a treaty; (2) pursuant to or upon
confirmation by an act of the Legislature; or (3) in the exercise of the President's independent powers
Unfortunately, the uniform resource locator link cited by petitioners is inaccessible. However, even if we under the Constitution. The raison d'etre of executive agreements hinges on prior constitutional or
grant its veracity, the text of the VFA itself belies such a claim. Article I of the VFA states that "[a ]s used legislative authorizations.
in this Agreement, "United States personnel" means United States military and civilian personnel
temporarily in the Philippines in connection with activities approved by the Philippine The special nature of an executive agreement is not just a domestic variation in international
26
Government." These "activities" were, as stated in Lim, left to further implementing agreements. It is agreements. International practice has accepted the use of various forms and designations of
true that Article VII on Importation did not indicate pre-positioned materiel, since it referred to "United international agreements, ranging from the traditional notion of a treaty - which connotes a formal,
States Government equipment, materials, supplies, and other property imported into or acquired in the solemn instrument - to engagements concluded in modern, simplified forms that no longer necessitate
Philippines by or on behalf of the United States armed forces in connection with activities to which this ratification. An international agreement may take different forms: treaty, act, protocol,
27
agreement applies[.]" agreement, concordat, compromis d'arbitrage, convention, covenant, declaration, exchange of notes,
statute, pact, charter, agreed minute, memorandum of agreement, modus vivendi, or some other form.
Nonetheless, neither did the text of the VFA indicate "joint exercises" as the only activity, or even as Consequently, under international law, the distinction between a treaty and an international agreement
one of those activities authorized by the treaty. In fact, the Court had previously noted that or even an executive agreement is irrelevant for purposes of determining international rights and
obligations.
[n]ot much help can be had therefrom [VFA], unfortunately, since the terminology employed is itself the
source of the problem. The VFA permits United States personnel to engage, on an impermanent basis, However, this principle does not mean that the domestic law distinguishing treaties, international
in "activities," the exact meaning of which was left undefined. The expression is ambiguous, permitting a agreements, and executive agreements is relegated to a mere variation in form, or that the
wide scope of undertakings subject only to the approval of the Philippine government. The sole constitutional requirement of Senate concurrence is demoted to an optional constitutional directive.
encumbrance placed on its definition is couched in the negative, in that United States personnel must There remain two very important features that distinguish treaties from executive agreements and
"abstain from any activity inconsistent with the spirit of this agreement, and in particular, from any translate them into terms of art in the domestic setting.
28
political activity." All other activities, in other words, are fair game.
First, executive agreements must remain traceable to an express or implied authorization under
Moreover, even if the DFA Primer was accurate, properly cited, and offered as evidence, it is quite clear the Constitution, statutes, or treaties. The absence of these precedents puts the validity and
29
that the DFA's opinion on the VFA is not legally binding nor conclusive. It is the exclusive duty of the effectivity of executive agreements under serious question for the main function of the Executive is to
30
Court to interpret with finality what the VFA can or cannot allow according to its provisions. enforce the Constitution and the laws enacted by the Legislature, not to defeat or interfere in the
performance of these rules. In turn, executive agreements cannot create new international obligations
In addition to this, petitioners detail their objections to EDCA in a similar way to their original petition,
that are not expressly allowed or reasonably implied in the law they purport to implement.
claiming that the VFA and MDT did not allow EDCA to contain the following provisions:
Second, treaties are, by their very nature, considered superior to executive agreements. Treaties are
1. Agreed Locations
products of the acts of the Executive and the Senate unlike executive agreements, which are solely
2. Rotational presence of personnel executive actions. Because of legislative participation through the Senate, a treaty is regarded as being
on the same level as a statute. If there is an irreconcilable conflict, a later law or treaty takes
3. U.S. contractors precedence over one that is prior. An executive agreement is treated differently. Executive agreements
that are inconsistent with either a law or a treaty are considered ineffective. Both types of international
31 33
4. Activities of U.S. contractors agreement are nevertheless subject to the supremacy of the Constitution. (Emphasis supplied,
citations omitted)
Subsequently, the Decision goes to great lengths to illustrate the source of EDCA's validity, in that as an While this Motion for Reconsideration was pending resolution, the United Nations Permanent Court of
executive agreement it fell within the parameters of the VFA and MDT, and seamlessly merged with the Arbitration tribunal constituted under the Convention on the Law of the Sea (UNCLOS) in Republic of
whole web of Philippine law. We need not restate the arguments here. It suffices to state that this Court the Philippines v. People's Republic of China released its monumental decision on the afternoon of 12
42
remains unconvinced that EDCA deserves treaty status under the law. July 2016. The findings and declarations in this decision contextualizes the security requirements of
the Philippines, as they indicate an alarming degree of international law violations committed against
On EDCA as basing agreement the Philippines' sovereign rights over its exclusive economic zone (EEZ).

Petitioners claim that the Decision did not consider the similarity of EDCA to the previous Military Bases Firstly, the tribunal found China's claimed nine-dash line, which included sovereign claims over most of
34
Agreement (MBA) as grounds to declare it unconstitutional. the West Philippine, invalid under the UNCLOS for exceeding the limits of China's maritime zones
43
granted under the convention.
Firstly, the Court has discussed this issue in length and there is no need to rehash the analysis leading
towards the conclusion that EDCA is different from the MBA or any basing agreement for that matter. Secondly, the tribunal found that the maritime features within the West Philippine Sea/South China Sea
that China had been using as basis to claim sovereign rights within the Philippines' EEZ were not
Secondly, the new issues raised by petitioners are not weighty enough to overturn the legal distinction entitled to independent maritime zones.
44

between EDCA and the MBA.


Thirdly, the tribunal found that the actions of China within the EEZ of the Philippines, namely: forcing a
In disagreeing with the Court in respect of the MBA's jurisdictional provisions, petitioners cite an 45
Philippine vessel to cease-and-desist from survey operations, the promulgation of a fishing
exchange of notes categorized as an "amendment" to the MBA, as if to say it operated as a new treaty 46
moratorium in 2012, the failure to exercise due diligence in preventing Chinese fishing vessels from
35
and should be read into the MBA. 47
fishing in the Philippines' EEZ without complying with Philippine regulations, the failure to prevent
48
Chinese fishing vessels from harvesting endangered species, the prevention of Filipino fishermen
This misleadingly equates an exchange of notes with an amendatory treaty. Diplomatic exchanges of 49
from fishing in traditional fishing grounds in Scarborough Shoal, and the island-building operations in
notes are not treaties but rather formal communication tools on routine agreements, akin to private law 50
36 37 various reefs, all violate its obligations to respect the rights of the Philippines over its EEZ.
contracts, for the executive branch. This cannot truly amend or change the terms of the treaty, but
merely serve as private contracts between the executive branches of government. They cannot ipso Fourthly, the tribunal rejected Chinese claims of sovereignty over features within the Philippine's
38
facto amend treaty obligations between States, but may be treaty-authorized or treaty-implementing. 51
EEZ, and found that its construction of installations and structures, and later on the creation of an
52
artificial island, violated its international obligations.
Hence, it is correct to state that the MBA as the treaty did not give the Philippines jurisdiction over the
bases because its provisions on U.S. jurisdiction were explicit. What the exchange of notes did provide Fifthly, the tribunal found that the behaviour of Chinese law enforcement vessels breached safe
was effectively a contractual waiver of the jurisdictional rights granted to the U.S. under the MBA, but navigation provisions of the UNCLOS in respect of near-collision instances within Scarborough Shoal.
53

did not amend the treaty itself.


Finally, the tribunal found that since the arbitration was initiated in 2013, China has aggravated the
Petitioners reassert that EDCA provisions on operational control, access to Agreed Locations, various dispute by building a large artificial island on a low-tide elevation located in the EEZ of the Philippines
rights and authorities granted to the US "ensures, establishes, and replicates what MBA had aggravated the Parties' dispute concerning the protection and preservation of the marine environment at
39
provided." However, as thoroughly and individually discussed in Saguisag, et. al., the significant Mischief Reef by inflicting permanent, irreparable harm to the coral reef habitat of that feature, extended
differences taken as a whole result in a very different instrument, such that EDCA has not re-introduced the dispute concerning the protection and preservation of the marine environment by commencing
40
the military bases so contemplated under Article XVIII Section 25 of the Constitution. large-scale island-building and construction works at Cuarteron Reef: Fiery Cross Reef, Gaven Reef
(North), Johnson Reef, Hughes Reef, and Subi Reef, aggravated the dispute concerning the status of
On policy matters
maritime features in the Spratly Islands and their capacity to generate entitlements to maritime zones by
Petitioners have littered their motion with alleged facts on U.S. practices, ineffective provisions, or even permanently destroying evidence of the natural condition of Mischief Reef, Cuarteron Reef, Fiery Cross
54
41
absent provisions to bolster their position that EDCA is invalid. In this way, petitioners essentially ask Reef, Gaven Reef (North), Johnson Reef, Hughes Reef, and Subi Reef.
this Court to replace the prerogative of the political branches and rescind the EDCA because it not a
Taken as a whole, the arbitral tribunal has painted a harrowing picture of a major world power unlawfully
good deal for the Philippines. Unfortunately, the Court's only concern is the legality of EDCA and not its
imposing its might against the Philippines, There are clear indications that these violations of the
wisdom or folly. Their remedy clearly belongs to the executive or legislative branches of government.
Philippines' sovereign rights over its EEZ are continuing. The Philippine state is constitutionally-bound
EPILOGUE to defend its sovereignty, and must thus prepare militarily.
No less than the 1987 Constitution demands that the "State shall protect the nation's marine wealth in This Motion for Reconsideration has not raised any additional legal arguments that warrant revisiting the
its archipelagic waters, territorial sea, and exclusive economic zone, and reserve its use and enjoyment Decision.
55
exclusively to Filipino citizens. "
WHEREFORE, we hereby DENY the Motion for Reconsideration.
No less than the 1987 Constitution states that the principal role of the military under the President as
commander-in-chief shall be as protector of the people and the State to secure the sovereignty of the SO ORDERED.
56
State and the integrity of the national territory.
MARIA LOURDES P. A. SERENO
57 Chief Justice
To recall, the Philippines and the U.S. entered into the MDT in 1951 with two things in mind, first, it
allowed for mutual assistance in maintaining and developing their individual and collective capacities to
58
resist an armed attack; and second, it provided for their mutual self-defense in the event of an armed
59
attack against the ten-itory of either party. The treaty was premised on their recognition that an anned
60
attack on either of them would equally be a threat to the security of the other.

The EDCA embodies this very purpose. It puts into greater effect a treaty entered into more than 50
years ago in order to safeguard the sovereignty of the Philippines, and cement the military friendship of
the U.S. and Philippines that has thrived for decades through multiple presidents and multiple treaties.
While it is a fact that our country is now independent, and that the 1987 Constitution requires Senate
consent for foreign military bases, troops, and facilities, the EDCA as envisioned by the executive and
as formulated falls within the legal regime of the MDT and the VFA.

In the context of recent developments, the President is bound to defend the EEZ of the Philippines and
ensure its vast maritime wealth for the exclusive enjoyment of Filipinos. In this light, he is obligated to
equip himself with all resources within his power to command. With the MDT and VFA as a blueprint
and guide, EDCA strengthens the Armed Forces of the Philippines and through them, the President's
ability to respond to any potential military crisis with sufficient haste and greater strength.

The Republic of Indonesia is strengthening its military presence and defences in the South China
61 62
Sea. Vietnam has lent its voice in support of the settlement of disputes by peaceful means but still
63
strongly asserts its sovereignty over the Paracel islands against China. The international community
64
has given its voice in support of the tribunal's decision in the UNCLOS arbitration.
65
Despite all this, China has rejected the ruling. Its ships have continued to drive off Filipino fishermen
66
from areas within the Philippines' EEZ. Its military officials have promised to continue its artificial
67
island-building in the contested areas despite the ruling against these activities.

In this light, the Philippines must continue to ensure its ability to prevent any military aggression that
violates its sovereign rights. Whether the threat is internal or external is a matter for the proper
authorities to decide. President Rodrigo Roa Duterte has declared, in his inaugural speech, that the
68
threats pervading society are many: corruption, crime, drugs, and the breakdown of law and order. He
69
has stated that the Republic of the Philippines will honor treaties and international obligations. He has
70
also openly supported EDCA's continuation.

Thus, we find no reason for EDCA to be declared unconstitutional. It fully conforms to the Philippines'
legal regime through the MDT and VFA. It also fully conforms to the government's continued policy to
enhance our military capability in the face of various military and humanitarian issues that may arise.
[1]
complementary to the national criminal jurisdictions. Its jurisdiction covers the crime of genocide,
[2]
crimes against humanity, war crimes and the crime of aggression as defined in the Statute. The
Statute was opened for signature by all states in Rome on July 17, 1998 and had remained open for
EN BANC signature until December 31, 2000 at the United Nations Headquarters in New York. The Philippines
signed the Statute on December 28, 2000 through Charge d Affairs Enrique A. Manalo of the Philippine
[3]
Mission to the United Nations. Its provisions, however, require that it be subject to ratification,
SENATOR AQUILINO PIMENTEL, JR., G.R. No. 158088 acceptance or approval of the signatory states.
[4]

REP. ETTA ROSALES, PHILIPPINE


COALITION FOR THE ESTABLISHMENT Petitioners filed the instant petition to compel the respondents the Office of the Executive Secretary and
OF THE INTERNATIONAL Present: the Department of Foreign Affairs to transmit the signed text of the treaty to the Senate of the
CRIMINAL COURT, TASK FORCE Philippines for ratification.
DETAINEES OF THE PHILIPPINES, Davide, Jr., C.J.,
FAMILIES OF VICTIMS OF Puno, It is the theory of the petitioners that ratification of a treaty, under both domestic law and international
INVOLUNTARY DISAPPEARANCES, Panganiban, law, is a function of the Senate. Hence, it is the duty of the executive department to transmit the signed
BIANCA HACINTHA R. ROQUE, Quisumbing, copy of the Rome Statute to the Senate to allow it to exercise its discretion with respect to ratification of
HARRISON JACOB R. ROQUE, Ynares-Santiago, treaties. Moreover, petitioners submit that the Philippines has a ministerial duty to ratify the Rome
*
AHMED PAGLINAWAN, RON P. SALO, Sandoval-Gutierrez, Statute under treaty law and customary international law. Petitioners invoke the Vienna Convention on
LEAVIDES G. DOMINGO, EDGARDO *Carpio, the Law of Treaties enjoining the states to refrain from acts which would defeat the object and purpose
CARLO VISTAN, NOEL VILLAROMAN, Austria-Martinez, of a treaty when they have signed the treaty prior to ratification unless they have made their intention
[5]
CELESTE CEMBRANO, LIZA ABIERA, *Corona, clear not to become parties to the treaty.
JAIME ARROYO, MARWIL LLASOS, Carpio Morales,
CRISTINA ATENDIDO, ISRAFEL Callejo, Sr., The Office of the Solicitor General, commenting for the respondents, questioned the standing of the
FAGELA, and ROMEL BAGARES, Azcuna, petitioners to file the instant suit. It also contended that the petition at bar violates the rule on hierarchy
Petitioners, Tinga, of courts. On the substantive issue raised by petitioners, respondents argue that the executive
Chico-Nazario, and department has no duty to transmit the Rome Statute to the Senate for concurrence.
- versus - Garcia, JJ.
A petition for mandamus may be filed when any tribunal, corporation, board, officer or person unlawfully
OFFICE OF THE EXECUTIVE neglects the performance of an act which the law specifically enjoins as a duty resulting from an office,
[6]
SECRETARY, represented by Promulgated: trust, or station. We have held that to be given due course, a petition for mandamus must have been
HON. ALBERTO ROMULO, and the instituted by a party aggrieved by the alleged inaction of any tribunal, corporation, board or person
DEPARTMENT OF FOREIGN which unlawfully excludes said party from the enjoyment of a legal right. The petitioner in every case
AFFAIRS, represented by HON. BLAS OPLE, July 6, 2005 must therefore be an aggrieved party in the sense that he possesses a clear legal right to be enforced
[7]
Respondents. and a direct interest in the duty or act to be performed. The Court will exercise its power of judicial
x- - - - - - - - - - - - - - - - - - - - - - - - - - - - - - - - - - - - - - - - - - - - - - - - - - -x review only if the case is brought before it by a party who has the legal standing to raise the
constitutional or legal question. Legal standing means a personal and substantial interest in the case
DECISION such that the party has sustained or will sustain direct injury as a result of the government act that is
being challenged. The term interest is material interest, an interest in issue and to be affected by the
[8]
PUNO J.: decree, as distinguished from mere interest in the question involved, or a mere incidental interest.

This is a petition for mandamus filed by petitioners to compel the The petition at bar was filed by Senator Aquilino Pimentel, Jr. who asserts his legal standing to file the
Office of the Executive Secretary and the Department of Foreign Affairs to transmit the signed copy of suit as member of the Senate; Congresswoman Loretta Ann Rosales, a member of the House of
the Rome Statute of the International Criminal Court to the Senate of the Philippines for its concurrence Representatives and Chairperson of its Committee on Human Rights; the Philippine Coalition for the
in accordance with Section 21, Article VII of the 1987 Constitution. Establishment of the International Criminal Court which is composed of individuals and corporate
entities dedicated to the Philippine ratification of the Rome Statute; the Task Force Detainees of the
The Rome Statute established the International Criminal Court which shall have the power to exercise Philippines, a juridical entity with the avowed purpose of promoting the cause of human rights and
its jurisdiction over persons for the most serious crimes of international concern xxx and shall be human rights victims in the country; the Families of Victims of Involuntary Disappearances, a juridical
[13]
entity duly organized and existing pursuant to Philippine Laws with the avowed purpose of promoting otherwise transact the business of foreign relations. In the realm of treaty-making, the President has
the cause of families and victims of human rights violations in the country; Bianca Hacintha Roque and the sole authority to negotiate with other states.
Harrison Jacob Roque, aged two (2) and one (1), respectively, at the time of filing of the instant petition,
and suing under the doctrine of inter-generational rights enunciated in the case of Oposa vs. Factoran, Nonetheless, while the President has the sole authority to negotiate and enter into treaties, the
[9] Constitution provides a limitation to his power by requiring the concurrence of 2/3 of all the members of
Jr.; and a group of fifth year working law students from the University of the Philippines College of
Law who are suing as taxpayers. the Senate for the validity of the treaty entered into by him. Section 21, Article VII of the 1987
Constitution provides that no treaty or international agreement shall be valid and effective unless
The question in standing is whether a party has alleged such a personal stake in the outcome of the concurred in by at least two-thirds of all the Members of the Senate. The 1935 and the 1973
controversy as to assure that concrete adverseness which sharpens the presentation of issues upon Constitution also required the concurrence by the legislature to the treaties entered into by the
[10]
which the court so largely depends for illumination of difficult constitutional questions. executive. Section 10 (7), Article VII of the 1935 Constitution provided:

We find that among the petitioners, only Senator Pimentel has the legal standing to file the instant suit. Sec. 10. (7) The President shall have the power, with the concurrence of two-thirds of all the Members
The other petitioners maintain their standing as advocates and defenders of human rights, and as of the Senate, to make treaties xxx
citizens of the country. They have not shown, however, that they have sustained or will sustain a direct
injury from the non-transmittal of the signed text of the Rome Statute to the Senate. Their contention Section 14 (1) Article VIII of the 1973 Constitution stated:
that they will be deprived of their remedies for the protection and enforcement of their rights does not
persuade. The Rome Statute is intended to complement national criminal laws and courts. Sufficient Sec. 14. (1) Except as otherwise provided in this Constitution, no treaty shall be valid and effective
remedies are available under our national laws to protect our citizens against human rights violations unless concurred in by a majority of all the Members of the Batasang Pambansa.
and petitioners can always seek redress for any abuse in our domestic courts.
The participation of the legislative branch in the treaty-making process was deemed essential to provide
[14]
As regards Senator Pimentel, it has been held that to the extent the powers of Congress are impaired, a check on the executive in the field of foreign relations. By requiring the concurrence of the
so is the power of each member thereof, since his office confers a right to participate in the exercise of legislature in the treaties entered into by the President, the Constitution ensures a healthy system of
[15]
[11]
the powers of that institution. Thus, legislators have the standing to maintain inviolate the checks and balance necessary in the nations pursuit of political maturity and growth.
prerogatives, powers and privileges vested by the Constitution in their office and are allowed to sue to
In filing this petition, the petitioners interpret Section 21, Article VII of the 1987 Constitution to mean that
question the validity of any official action which they claim infringes their prerogatives as legislators. The
the power to ratify treaties belongs to the Senate.
petition at bar invokes the power of the Senate to grant or withhold its concurrence to a treaty entered
into by the executive branch, in this case, the Rome Statute. The petition seeks to order the executive We disagree.
branch to transmit the copy of the treaty to the Senate to allow it to exercise such authority. Senator
Pimentel, as member of the institution, certainly has the legal standing to assert such authority of the Justice Isagani Cruz, in his book on International Law, describes the treaty-making process in this wise:
Senate.
The usual steps in the treaty-making process are: negotiation, signature, ratification, and exchange of
We now go to the substantive issue. the instruments of ratification. The treaty may then be submitted for registration and publication under
the U.N. Charter, although this step is not essential to the validity of the agreement as between the
The core issue in this petition for mandamus is whether the Executive Secretary and the Department of parties.
Foreign Affairs have a ministerial duty to transmit to the Senate the copy of the Rome Statute signed
by a member of the Philippine Mission to the United Nations even without the signature of the Negotiation may be undertaken directly by the head of state but he now usually assigns this task to his
President. authorized representatives. These representatives are provided with credentials known as full powers,
which they exhibit to the other negotiators at the start of the formal discussions. It is standard practice
We rule in the negative. for one of the parties to submit a draft of the proposed treaty which, together with the counter-proposals,
becomes the basis of the subsequent negotiations. The negotiations may be brief or protracted,
In our system of government, the President, being the head of state, is regarded as the sole organ and
[12] depending on the issues involved, and may even collapse in case the parties are unable to come to an
authority in external relations and is the countrys sole representative with foreign nations. As the
agreement on the points under consideration.
chief architect of foreign policy, the President acts as the countrys mouthpiece with respect to
international affairs. Hence, the President is vested with the authority to deal with foreign states and
governments, extend or withhold recognition, maintain diplomatic relations, enter into treaties, and
If and when the negotiators finally decide on the terms of the treaty, the same is opened for signature. i. All executive agreements shall be transmitted to the Department of Foreign Affairs after their signing
This step is primarily intended as a means of authenticating the instrument and for the purpose of for the preparation of the ratification papers. The transmittal shall include the highlights of the
symbolizing the good faith of the parties; but, significantly, it does not indicate the final consent of agreements and the benefits which will accrue to the Philippines arising from them.
the state in cases where ratification of the treaty is required. The document is ordinarily signed in
accordance with the alternat, that is, each of the several negotiators is allowed to sign first on the copy ii. The Department of Foreign Affairs, pursuant to the endorsement by the concerned agency, shall
which he will bring home to his own state. transmit the agreements to the President of the Philippines for his ratification. The original signed
instrument of ratification shall then be returned to the Department of Foreign Affairs for appropriate
Ratification, which is the next step, is the formal act by which a state confirms and accepts the action.
provisions of a treaty concluded by its representatives. The purpose of ratification is to enable the
contracting states to examine the treaty more closely and to give them an opportunity to refuse B. Treaties.
to be bound by it should they find it inimical to their interests. It is for this reason that most
treaties are made subject to the scrutiny and consent of a department of the government other i. All treaties, regardless of their designation, shall comply with the requirements provided in sub-
than that which negotiated them. paragraph[s] 1 and 2, item A (Executive Agreements) of this Section. In addition, the Department of
Foreign Affairs shall submit the treaties to the Senate of the Philippines for concurrence in the
xxx ratification by the President. A certified true copy of the treaties, in such numbers as may be required by
the Senate, together with a certified true copy of the ratification instrument, shall accompany the
The last step in the treaty-making process is the exchange of the instruments of ratification, which submission of the treaties to the Senate.
usually also signifies the effectivity of the treaty unless a different date has been agreed upon by the
parties. Where ratification is dispensed with and no effectivity clause is embodied in the treaty, the ii. Upon receipt of the concurrence by the Senate, the Department of Foreign Affairs shall comply with
[16] the provision of the treaties in effecting their entry into force.
instrument is deemed effective upon its signature. [emphasis supplied]

Petitioners arguments equate the signing of the treaty by the Philippine representative with ratification. It Petitioners submission that the Philippines is bound under treaty law and international law to ratify the
should be underscored that the signing of the treaty and the ratification are two separate and distinct treaty which it has signed is without basis. The signature does not signify the final consent of the state
steps in the treaty-making process. As earlier discussed, the signature is primarily intended as a means to the treaty. It is the ratification that binds the state to the provisions thereof. In fact, the Rome Statute
of authenticating the instrument and as a symbol of the good faith of the parties. It is usually performed itself requires that the signature of the representatives of the states be subject to ratification,
by the states authorized representative in the diplomatic mission. Ratification, on the other hand, is the acceptance or approval of the signatory states. Ratification is the act by which the provisions of a treaty
formal act by which a state confirms and accepts the provisions of a treaty concluded by its are formally confirmed and approved by a State. By ratifying a treaty signed in its behalf, a state
representative. It is generally held to be an executive act, undertaken by the head of the state or of the expresses its willingness to be bound by the provisions of such treaty. After the treaty is signed by the
[17] states representative, the President, being accountable to the people, is burdened with the
government. Thus, Executive Order No. 459 issued by President Fidel V. Ramos on November 25,
1997 provides the guidelines in the negotiation of international agreements and its ratification. It responsibility and the duty to carefully study the contents of the treaty and ensure that they are not
mandates that after the treaty has been signed by the Philippine representative, the same shall be inimical to the interest of the state and its people. Thus, the President has the discretion even after the
transmitted to the Department of Foreign Affairs. The Department of Foreign Affairs shall then prepare signing of the treaty by the Philippine representative whether or not to ratify the same. The Vienna
the ratification papers and forward the signed copy of the treaty to the President for ratification. After the Convention on the Law of Treaties does not contemplate to defeat or even restrain this power of the
President has ratified the treaty, the Department of Foreign Affairs shall submit the same to the Senate head of states. If that were so, the requirement of ratification of treaties would be pointless and futile. It
for concurrence. Upon receipt of the concurrence of the Senate, the Department of Foreign Affairs shall has been held that a state has no legal or even moral duty to ratify a treaty which has been signed by its
[18]
comply with the provisions of the treaty to render it effective. Section 7 of Executive Order No. 459 plenipotentiaries. There is no legal obligation to ratify a treaty, but it goes without saying that the
reads: refusal must be based on substantial grounds and not on superficial or whimsical reasons. Otherwise,
[19]
the other state would be justified in taking offense.
Sec. 7. Domestic Requirements for the Entry into Force of a Treaty or an Executive
Agreement. The domestic requirements for the entry into force of a treaty or an executive agreement, It should be emphasized that under our Constitution, the power to ratify is vested in the President,
or any amendment thereto, shall be as follows: subject to the concurrence of the Senate. The role of the Senate, however, is limited only to giving or
[20]
withholding its consent, or concurrence, to the ratification. Hence, it is within the authority of the
A. Executive Agreements. President to refuse to submit a treaty to the Senate or, having secured its consent for its ratification,
[21]
refuse to ratify it. Although the refusal of a state to ratify a treaty which has been signed in its behalf
[22]
is a serious step that should not be taken lightly, such decision is within the competence of the
President alone, which cannot be encroached by this Court via a writ of mandamus. This Court has no
[23]
jurisdiction over actions seeking to enjoin the President in the performance of his official duties. The
Court, therefore, cannot issue the writ of mandamus prayed for by the petitioners as it is beyond its
jurisdiction to compel the executive branch of the government to transmit the signed text of Rome
Statute to the Senate.

IN VIEW WHEREOF, the petition is DISMISSED.

SO ORDERED.

Vous aimerez peut-être aussi